You are on page 1of 309

SURDS

INDICES AND LOGARITHMS


x5 is a short way of writing x  x  x x x 3  2 
4 2 2

y3 is a short way of writing y  y  y


3   2 
4 2 2 2
24 is a short way of writing 2 x 2 x 2 x 2
 34  2  22  2
Generally;
a  a  a  a ........................... a  38  24
n (v) Any number raised to an index zero is equivalent
(product of n a’s ) can be written as a , where a to one. For example
is the base n is the index, power or exponent of a. x 0  1 for example 1000  1
 30 
 1, 40  1, 10000  1
0
LAWS OF INDICES
Consider x, m and n where x  0 (vi) One raised to any index n whether positive or
(i) In multiplication, powers of terms with the same negative always gives 1. For example
base are added. n
m n 1n  1 1 1 for example 1 4  1
x  x
m n
x
(vii) A number with a negative power is the same as
For example:
the reciprocal raised to the opposite power
32  34 1
24
xn 
3 xn
For example
 36 1
23 
(ii) For division, powers of terms with the same base 23
are subtracted 1

xm  xn  xm  n 8
For example: 1
34 
2 4  2 6 34
(viii) A number raised to an index a half is always
2 4 

 6
equivalent to the square root of that number. A
 2 4  6 number raised to an index a third is always
 22 equivalent to the cube root of that number.
1
(iii) A number raised to two indices is always
equivalent to the same number raised to the x2  x
product of indices. 1

a 
m n
 am  n x3
1
 3
x

 a mn x4  4
x
e.g.  3 2 3
 23
 3  3
6
1
(iv) When a product is raised to a power, the factors of Generally x n  n
x
that product are also raised to that power. (xm × For example:
an)y = (xm)y × (an)y
64 2  64  8
1

= xmy × any
27 3  3 27  3
1
For example:
16 4  4 16  2
1

(ix) Any function raised to a negative index is


equivalent to the reciprocal of the same function
raised to the positive index.

169
SURDS
m m  25   7  1
x  y
      21
 y x
1
For example: 
21
23  32
4 4
 1

2
(ii) 92  2  24
Example I  3 
2 2
 21  24
Write the product below in terms of powers  34  21 4
(i) 3 x 34 x 32
(ii) 5 x 52 x 54  34  25
5 7
(iii) 2 x 2 x 22 x 2 x 23 (iii) 2 2 2 1

Solution  25  7  1
(i) 3  34  3  32  211 
1
.
 31  34  31  32 211
Example III
 31 4 1 2 Evaluate the following:
 38 36 22 a5
(i) (ii) (iii)
(ii) 5  52  54 93 8 a3
Solution
 51  52  54
36 36
1 2  4 
5
3 
(i)
93 2 3
5 7
36
(iii) 2 2 2 2 2 3 3  6
3
 21  21  23  21  23
 36  6
 21  1  3 1  3
 30
2 9

Example II
 1
Simplify the following:-
(i) 25  27  21 (ii)
22
(ii) 9 2  2
2 4 8
(iii) 25  27  21

Solution
(i) 25  27  21

170
SURDS


22
23
ii 3  2 
4 2 2


22 3   2 
4 2 2 2

23
34  2  2 2  2
 22  3
 21 38  2 4


1

1 iii 3  2 
3 2 2

3   2 
21 2
3 2 2 2
5
a
(iii) 33  2  22  2
a3
 a5  3 3 6  24

 a 2 i v  x y  x y 
2 5 2 1 2 4

  x   yy  xx   y y  
2 2 4 4
22 2 55 2 
11 4 214
a5
 3  a2
a   x   y  x
2 2
 y 5 2 1 4 2  4

 x   y  x   y 
4 10 4 8

 x  x   y  y 
4 4 10 8

 x 4  4
  y 10  8

Example IV x  y
0 2

Simplify the following:-  1  y2


i   2  3 
3 2 2 2
 y2
 ii   3  2 
4 2 2
 v 4  3 
2
3  4 
1 4 4 2

 iii   3  2 
3 2 2   (2 )  3  3  (2 ) 
2 2 1 4 2 2 2

iv   x y   x 5 2
y 2    2  3  3  2 
22 8 2
2 1 4 1 4

 4  3  3  44    2  3  3    2  4 2 8 2
2
 v 2 1 4 4 1

 24  31  38  216
 vi  10  10  1 2 3

 24  16  31  8
1 1
 vii   3 5  3 5 

2 2  212  37
Solution vi 10  10 1 2 3

i  2  3 
3 2 2 2
 10  1  2 2

2  3 
3 2 2 2
 10  1 2

2  3 
6 4
 102
26  34 1 1
 viii  3  5 2 3  5

2

171
SURDS
1

1
Simplify the following:
 15 2  15 2
i  0.125 3
2

1 1

 15 2 2
 ii  100
11
2
 4
1 1
2

 15 0
 iii 
1 1
4 2
 4 6

1
 1  12
Example VI  iv  5 
 16 
Simplify the following:
 v
1
2
25
22  32  31  3
a   vi   8a  3  3
2

32  24  2
4a 3  3b 2
34  33  32  vii 
b 22 a 4  22 b
33  35
 64a 
1
6 2

x  x
4 2  viii 
c  64a  6  3
1

x 2  x 6
Solution:
Solution
0.125
2
22  32  31  3 (1) 3
a  2
32  24  2  125  3
  
 22   32  31  3   1000 
  4 1    2
2  2   32   53 
 
3
  103 
2 
2
3 
4
 
  5   2
 
2 3    5 3 
2
3
  
 2  25
 3 42 
  10  
 
 23  32 3 2
5 3
3 2   
3 3 3
4
b  10 
33  35 5
2

  
34  3   2  10 

33  5 52 25
1
 2

3 10 100

32
 1 1
100 2  4
11
 31 2
(ii) 2

 31 3 3
 100 2  4 2
x 4  x 2
c
x 2  x 6  10   2  2
3
2 2  2
3

x 4  2 2 3 2  3

x 2  6  10 2  2 2


x2  103  23

4
1013 3 23 16
x

 x 2  4 (iii) 4  4
 x6
Example VII

172
SURDS

 2  2
1
3 2  2
1
6
(vii)
4a3  3b 2
 2
2 1
3 2
2 1
6 22 a 4  22 b
2 1 2 2 a 3  3b 2
 2 3 2 3 
22 a 4  22 b
2  1
 2 3 3  22   a3   b2 
  2    4      3
2  2
2
 21  a   b 
 5 161  2
1
(iv)  22   a3   b2 
  4    4    1   3
1 2  a  b 
 81  2
    2 2 4  a 3  4  b 21  3
 16 
1  2  2 a 1 b1 3
9  2 2
  2  3b  2 1
1
4  2 a

 
1
 9 2 2 3b
  4  
  4a

 9 4
2 1
2

(vii)
64a  6
1
2

9 4 64a  6  3
1
1

  2a  
1
9

6 2

4 
 
3  3
1

(v) 25
1
2  4a 2 
 5  2 1
2

 2a 
6 1
2

 4a  2 3  3
1
2  1
 5 2

 2a 
3
 51 

1  4a  2 1

5 23 a 3

(vi) 8a  3 2
3 2 2
a2 
1

 2a   3  3
2

2  a 
2 1
23 a 3
2 1

2a 3  
2
 3 23 a 3

 2a  2 22 a 2

1

2a 2
1

4a 2

173
SURDS

 23   a 3 
2  2 
1 1
  2   2 
3 6
 2 3

2  a  
2   2 
1 1
5 6 4 2

     a3 
23  2  2 
a 
1
2 2 2
2
3

2  a 
5
 3  2 3  2
2 6
 22
7
6
 25 a 5 2
 17

6
32a 5 2
7  17
 2 6 6

Example VII 10


 2 6
Simplify
5
1  2 3
(a) 2  64
6 3
1
33  81  5
3
2
1 1
(b) 8 6 4 3

22  42  125
1 1 2
3
32 6  16 2
(c)
22  42  125 43
(c) 2
22  (22 ) 2  (53 ) 3
43 
Solution: (22 )3
1 22  24  52
(a) 26  64 3 
26
33  81  22  4 
  2
  26   5
26   2 
6  3
1
 
 3
3  34 
26 1

6  1 26 52
26  2 3
1
  26  6  2
33  34 5
1 1
26  22  20  2 
 5 25
33  34
26   2 Example VIII
 2 x  8 x 1
33  4 Simplify
16 x 1
24
 Solution:
31 2 x  8 x 1
1 1
8 6 4 3 16 x 1
2 x   23 
(b) 1 1
x 1

32 6  16 2 
2  4 x 1

174
SURDS

2x  2 
3 x  1

2 
4 x 1
Example IX
2 x  23 x  3 Without using calculators. Find the value of

24 x  4 3 1
2x  3x  3 12 2  16 8

27 18
1 1
24 x  4 6 2
24 x  3

24 x  4 Solution:
 2 4 x  3   4 x  4  12
3
2  16
1
8
 24 x  3  4 x  4
27 18 2
1 1
6
 2 1

3  4   2 
3 1
2 4 8


3    2  3 
Example IX: 3
1
6 2
1
2
Without using tables or calculator, evaluate
2  5  3  2   2
3 4 1
 8 
2 2 8
3
    1  2 
 125   2 1
 2 2  31
1

8 
2
3
3 1
Solution: 3 2
 23  2 2

2 2 
 8  3  5  1
3 2  2 2 3
1

    1 
 125  8 2  3
3
2
 2 2  23
1

2 
 
3 1
2 3 2
 2 2
2  3 3
  
5
 3  3 12   33 2
  2 1 2  23 
 2 
5    3  
 3 2  2 12 
2 2
  
  2 3  3
 5   3 2
0 3
       3 
 5   2 2   23
   8
2
2
3
3  52 
     2 

 
Simplify
5  2 3 2 
 
1
3
1
0.027 3  18 91  27 3
1
 
2
2 52 Solution:
    3 1
5 1  27  1
 18     33  3
3
2 1
 
2 3  1000  9
5 23
    1  3 
3

1
3
18 3
1
2 52     3 3
3   10   9
52 23
 
22 52
23 5 2
 
2 2 52
 2

175
SURDS
31 1
1 3  3
   2  31 3x  2 
3  10  34
1
 6
3x  2  3 4
10 x2  4
59
 x  4  2
10
x  2
Example X
Solve for x
(a) 81 3x   32 
2
x 1
 27 3 x (d) x
1
(b) 32 x  1  647 x  233
4
3x  3 x
 30
8 4
x  0
x2 1 x
(c) 3  x2  4  0
81
2 x2  4
(d) 3  9 1
x x
x  2
(e) 4  23 x 1  2 x
 x  2, x 2

(f) 3x 
2
27
4 x  20 3  (e) 42 3 x  1
2 x

Solution
2   2
2 3 x 1
 2 x

(a) 81x 1  27 3 x 22  23 x  3  2 x

3 4 x 1
 3 
3 3 x
22  3 x  3  2 x
34 x 1  39 x 2 3 x 1  2  x
 4x  4  9 x 2 x4 x11
13x  4 11
xx  4
4 2
x   4 x  20 3 

2
13 (f) 3x 27
(b) 32x 
1
 647 x  233 3x
2
 
33 4 x  20
3 
8
x  12 x  20
2

2  1
  26   233
5 x 7 x
 3 x 2  12 x  20  0
2
x  2, x  10
25 x  23  256 x  233
STUDENTS’ EXERCISE
2  1
  226 5x 3
5 x 7 x 56 x33
 3
 2  233 (1) Simplify the following:
2
25 x
2 51x51
3 x
 23  256 x  233
 233 36 a  10  10
1 4

 51x  3  3336 b  4  3 3  4  4 2


25 x 356 x  2x33 51
2 1 4

51x  36 c  x y 
2 4
1
2

x
36 d  a b 
2 1 3

51 e  3 
3
1
3
1
(c) 3x  2  f  5 
5
1
2
81 (2) Evaluate the following

176
SURDS

a  23  3 23   5  2  x
1 1
21 2 2 6. Solve for
1
b  24  2 2 a 4 x 1 
25  21 256
34  33  31 1
c
32  3 4
b 27 x   243
3
d   9 3  4 12  c 34 x 1
 27 x  5
2
2 1
1 (d) 913 x 
 56 
5 
3
  3
 23  5 2  23
2 729
 53 
e  
52
3. Simplify the following
2 x  4 x 1 7. Solve for the unknown
a 
16 x 1 a  32 x  1  22 p  8  648
16 x 1  4 x  2
b  b 7 4 x 1  83k 1  21952
64 x
c
1
 81  2 x 1  34 k 1  108
c  1 
 27 3 
d 3 x 1  2 2 k  3  576
d 27
2
3
 8  2
3
8
4
3
 8. Simplify the following
4. Evaluate the following 24  21
2 a 
a   1 
 
3
 81
3
4 2 2
 27 
3x  9 x  1
b 
1 b 
27 x 1
16 4

1
c 2 x  8 x 1
c
27 3

d  16
3
4 16 x 1
1 1
1  1 
d 
2 3
5. Simplify the following     
x
5
6
 x
2
3  16   64 
a  1 9. Without using tables or calculator, evaluate;

 125
6
x 1 1 2
2  64 3  8 3
 27 x  6  3 16
1
b
10. Simplify the following:
 c   0.09 
1
2 3 1
12 2
 16 8
a 
d  2916
1256
11
(d)
1 1
22 27 6
 18 2

 e  16
1 1
4
33  92  125 3
b
33  75  7 2 93
f  11. Without using tables or calculator, evaluate;
 4  38
3
2
1  8 
1 3
16  28  7
2 4 4
16 2  64 3   
g  125 
82  256

177
SURDS
12. Without using tables or calculator evaluate: 17. 2
0.008
1
3
13. Without using table or calculator solve for x
32x  1  4 x  3  224
8
14. Without using a calculator or tables simplify:

40.04
1
28 4  16
1
3
4


15. Simplify
1 1
 8  3
 64  2
      2 1
 27   9 
16. Find the value of x
152 x  4  3 2 x  4
2  x 
 1 
1
x
17. 4 x  1     16 2

 32 
Solutions
12 7 y2
1. (a) 1; (b) 2  3 (c)
x
5
(d) a6b−3 (e) 3 (f) 5 2
27 1
2. (a) 66 (b) 26 (c) 32 (d) (e)
2 5
2
1 3
3. (a) 2-x+2 (b) 1 (c) (d)
3 22
1
4. (a) 36 (b) 2 (c) (d) 8
3
4 1
5. (a) x 3
(b) (c) 0.3 (d) 54
3x 2
73
(e) 2 (f) 11 (g) 214 × 78
3  26
2
6. (a) x = –3 (b) x = 2 (c) x = - 2 (d) x 
3
7. (a) x = 1, p = 6; (b) x = 1; k = 1
(c) k = 1, x = 1
8. (a) 25 (b) 35 (c) 21 (d) 24
9. 4.41
10. (a) 23 (b) 3× 5
11. 4.41
12. 0.2
13. x = 3
14. 4
15. 2
16. 2

178
SURDS
The term logarithm was first introduced by a Scottish Evaluate the following:
Mathematician John Napier (1550-1617) in reference (a) log2 64
to the power or index of a real number. The name
therefore suggests that he must have used ratios to (b) log12 1728
obtain the power or index of a real number. Its (c) log3 729
believed to have come from two Greek words logo
meaning ratios and arithmos which means a number. (d) log2 3.2  log2 10
The logarithms of number to base x is the power to (e) log10 40  log10 50  log10 20
which x must be raised to give the number that is if:
Solutions
m = xa then
a = logx m e.g. 100 = 102 (a) log2 64  log2 26
 log10100 = 2  6log 2 2
Similarly 100 =103  6 1
 log10 1000  3
 6
Laws of Logarithms  log 2 64  6
(1) loga x + loga y (b) log12 1728  log12 123
For example:
 3log12 12
log2 3 + log2 8 = log2 (3 x 8)
= log2 24  3 1
log103 + log102 = log10(3 x 2)  3
=log10 6
(c) log3 729
 log3 36
(2) log a x  log a y  log a  x 
 y  6 log3 3
For example
 61
 10 
log2 10  log2 5  log2    6
5
(d) log2 3.2  log2 10
 log2 2
 log2 32
 2
log10 8  log10 2  log10 8
 log2 25
 log10 4
 5 log2 2
3. loga x n  n loga x  5 1
log 2 32  2log 2 3  5
2
log3 8  2log3 8  log 2 3.2  log 2 10  5

4. log a a  1 e) log10 40  log10 50  log10 20


e.g. log1010 = 1, log22 = 1, log33 = 1,  log10  40  50   log10 20
log44 = 1 etc.  log10 2000  log10 20
2000
5. loga 1 = 0  log10
e.g. log10 1  0, log2 1  0 20
 log10 100

Example I

179
SURDS

 log10 10 2  log10 x 5  log10 5  1  log10 42


 2 log10 10  log10 x 5  log10 5  log10 10  log10 16
2  1  log10 5 x 5  log10 160
 log10 40  log10 50  log10 20 5 x5  160
2 x 5
 32
Example II
x 5
 25
Simplify:
1  x 2
log10 25  2 log10 x  log10 2 x 2
2
Solution: Example V
1 Solve the equation
log10 25  2 log10 x  log10 2 x 2 1
2 log125 x  log125 5x 
1
3
 log10 25 2  log10 x 2  log10 2 x 2
1
 log10 5  log10 x 2   log10 2 x 2  log125 x  5x  
 x   log
3
 log10 5 2 10 2x 2 log125 5 x 2 1
5 x  2 x 
3
 log10 2
2
125
1
3
 5x2
 log10 10 5  5x2
1 1  x2
1 x2 1
 log10 25  log10 x  log10 2 x 2  1
2
x 1
Example III
If log10 y +3log10 x = 2  x  1 x  1
Express y in terms of x
Solution Example VI
log10 y  3log10 x  2 Solve the equations
5x  2
log10 y  log10 x3  2
log10 yx3  2 Solution:
 log10 yx 3
2 5x  2
 yx3  102 x log10 5  log10 2
100 log10 2
y  3 x 
x log105
Example IV 0.30103
Solve the equation x 
0.69897
5 log10 x  log10 5  1  2 log10 4 x  0.431

Example VII
Solve the equation

180
SURDS

3x 1  5 x 1 (1) log10 32
log10 3x 1  log10 5x 1 (2) log10 40
(3) log10 5
 x 1 log10 3   x 1 log10 5 Solution:
x log10 3  log10 3  x log10 5  log10 5 log10 32  log10 25
log10 3  log10 5  x log10 5  x log10 3
 5 log10 2
log10 3  log10 5  x  log10 5  log10 3
log10 3  log10 5  50.30103
x 
log10 5  log10 3  1.50515
x  5.301 log10 40  log10 4  log10 10
x  5.301  log10 22  log10 10
 2log10 2  1
Example VIII
 2  0.30103  1
Given that log10 2  0.3010,  1.60206
log10 3  0.4771 find without using tables or log10 5  log10 102
calculator the value of a if  log10 10  log10 2
log10 a  1.6020  1  0.30103
 0.69897
Solution:
log10 a  1.6020
Example XI
log10 a  1  0.6020
log10 3  0.4771, log10 2  0.3010
Evaluate:
log10 a  1  2  0.3010
(a) log 108
log10 a  log10 10  log10 22 (b) log10 30
log10 a  log10 10  log10 4 (c) log10 1.5
log10 a  log10 40 (d) log10 30
a  40 Solution
log10 8  log10 2 3
Example IX
Given that log10 7  0.8451, log10 2  0.3010  3 log10 2
 3  0.3010
Use this information to find:-
 0.903
log10 6449 log10 6  log10 3  log10 2
Solution:  0.4771  0.3010
 log10 64  log10 49  0.7781
 log10 26  log10 7 2 log10 1.5  log10 3
2
 6 log10 2  2 log10 7  log10 3  log10 2
 6  0.3010  20.8451  0.4771  0.3010
 0.1158  0.1761

Example X
Given that log10 2  0.30103
Find :

181
SURDS

log10 30  log10 10  log10 3


 1  log10 3
 1  0.4771 Example III
 1.4771 If log x  a, log y  b, log z  c
Express in terms of a, b and c.
Example XI
If log x  p, log y  q , log z  r  x2 y3   1 
Express the following in terms of log  log 2 2  log x 3 y 2 z 5 
p and q  z  x y 
2
(a) log x y Solution

logxyz  x2 y3   x2 y3 
(b) log   log  1

 x  z   z   z 
2
2

(c) log
 log x 2  log y 3  log z
1
2
y
 x2 z  1
(d) log   2 log x  3 log y  log z

 y  2
Solution:
1
log x 2 y  log x 2  log y  2a  3b  c
 2log x  log y 2
 1 
2p q
log  2 2  
 log1  log x 2  log y 2 
 
 log x y 2
2pq x y 
log  xyz   log x  log y  log z  log1   2log x  2log y 
 pqr  log1  2log x  2log y
 0  2a  2b
 log  xyz   pqr
   2a  2b 
 x z 2
 1 
log
y  log 2 2    2a  b 
x y 
  log 
x  log z 2  log y
logx y 2 z 5 
3
1
 log x 2  log z 2  log y
 log x 3  log y 2  log z 5
1
 log x  2log z  log y  3 log x  2 log y  5 log z
2

1
p  2r  q  3a  2b  5c
 logx 3 y 2 z 5   3a  2b  5c
2
 x z2  1
 log    p  2q  r
 y  2 Example XIII
x z 2 Simplify
  log x  log z  log y
2
log 
 y  1
log10 16  2 log10 a  log10 a 2
 2log x  log z  log y 2 5
2pr  q Solution:

182
SURDS
1 a
log10 16 2  2 log10 5   log10 a 2
1 2
a
log10 16  log10
2  
5  log10 a 2
a2
log10 4  log10 25  log10 a 2
 
 4 
 2 
a 
log10  25 
 log10 a 2
100
log10 a  log10 a 2
2

 100 
 2  a 2 
 log10  a 

 log10 100
 log10 10 2
 2 log10 10  2

183
SURDS
LOGARITHS THAT ACQUIRE USE OF (d) 4.2781  2
TABLES
The logarithm of a number consists of two parts the (e) 2.8  3
characteristics and the matrix. The mantissa is the part (f) 2.671  3
that comes after the decimal point and it usually kept
positive. It can be read from the table of common (g) 1.68  2.66
logarithms. The characteristics is a whole number to
(h) 5 .4416  2
obtain the characteristics the number is first written in
standard form. The power of 10 is the characteristics. Solutions:
Example (a) 3. 6862  2.5352
Use mathematical tables to find the logarithms of the
3.6862
following numbers.

(a) 3600 (b) 0.074 (c) 23.8 2.5352
(d) 0.00027 (e) 1.62 (d) 81.3
(f) (0.02621 (g) 0.32 (h) 0.32 5.1510
(l) 0.009 (j) 0.009  3.6862  2.5352
(k) 0.00000823 (l) 0.00624
(m) 0.00624  5.1510
NOTE the table below shows the steps involved when
finding the logarithm of a number. (b) 1. 6518  3.8304
Solution
Number Step I Step II Logarithm 1.6518

x Standard (Characteristic) of a 3.8304
form + Mantissa number x
3600 3.6 x 103 3 + 0.5563 3.5563 3.4822
0.074 7.4 x 10-2 2 + 0.8692 2 .8692  1.6518  3.8304  3.4822
23.8 2.38 x 101 1 + 0.3766 1.3766
0.00027 2.7 x 10-4 4  0.4314 4.4314
(c) 1.2814 3
1.62 1.62 x 100 0  0.2095 0.2095

81.3 8.13 x 101 1 + 0.9101 1.9101 1. 2 8 1 4


0.0621 6.21 x 10- 2  0.7931 2 .7931  3
2

0.32 3.2 x 10-1 1 0.5051 1.5051 3.8 4 4 2


0.009 9.0 x 10-3 3  0.9542 3.9542  1.2814  3  3.8442
0.00000823 8.23 x 10- 6  0.9154 6 .9154
6

0.00000062 6.2 x 10-7 7  0.7924 7 .7924 (d) 4.2781  2


0.00624 6.24 x 10- 3  0.7952 3 .7952
3
4.2781
2
4 0.2781
 
2 2
Example II  2  0.13905
Simplify the following:  2 .13905
(a) 3.6862  2.5352
(b) 1.6518  3.8304 (e) 2.80  3
(c) 1.2814  3

184
SURDS

2.80 3  1.80  Antilog  + 1.5658


   x 10 3
3.1838
3 3  0.1838 
3 1.80 1.5269 x 103
  1526.9 1526.9
3 3
 41.5  36.8  1526.9
 1  0.6
Example IV
 1 .6 Use the mathematical tables to evaluate;
5.32 x 0.375
2.80 Solution
  1 .6
3 5.32 x 0.375
Number Standard form of Logarithm of
(f) 2.671  3 x x
5.32 5.32 x 100 0.7259
2.671 0.375 3.75 x 10 -1
 1.5740
3 5.32 x 0.375 0.7259
3.  1.671  Antilog  
   10 1.5740
0

3  0.2999  0.2999
0
3 1.671 1.9948 1.9948 x 10
 
3 3
 1  0.557
 1 .557
 2.671  3  1 .557

(g) 1.68  2.66


 1.68  2.66  3.02

(h) 5.4416  2
5.4416
2
6  1.4416

2
 3  0.7208
 3 .7208

Example III
Use tables to evaluate 41.5 x 36.8
Solution
Number Standard form Logarithm of x
(x)
41.5 4.15 x 101 1.6180
36.8 3.68 x 101 1.5658
41.5 x 36.8 1.6180

185
SURDS
Example V Example
If x  4.096
3
use tables to find the value of x Use mathematical tables to evaluate:
5
Solution: 6.973
x 3  4.096 Solution
Number x Standard form of Logarithm of x
x 3  4.096 x
x  4.096 6.973 x 100
1
3 6.973 0.8424
6.973  0.8424
1
5
Number Standard form Logarithm of x
(x) 5
4.096 4.096 x 100 0.6124 antilog x 100 0.1685
0.1687
4.096 
1
3 1
0.6124  1.4747 x 100
3 1.4747
  6.973 5  1.4747
1

0.6124
3  4dec places tab 
 Antilog  0.2041
  10
0
 Example
 0.2041  Use mathematical tables
1.5999 1.5999 x 100
0.00243 
2
5

 4.096  1.60
1
3

Solution
2 dec placestab . Number x Standard form of Logarithm of x
x
Example VI 0.00243 2.43 x 10-3 3.3856
Use mathematical tables to evaluate:
19.76  (8.27)3 0.00243
2
5
3.3856  2
Solution 5
19.76  (8.27)3
6.7712
Number Standard form Logarithm of x 
(x) 5
19.76 1.976 x 101 1.2958 10  4.7712
8.27 8.27 x 100 0.9175 
5
(8.27)3 0.9175
x 3 Antilog x 10-2  2  0.9542
2.7525 0.9542 2 .9542
19.76 1.2958 8.999 x 10-2
(8.27)3 -
2.7525 0.08999
2.5433
 Antilog  2 2.5433
   10 0.08999
 0.5433 
 0.002435  0.09
2

0.03494 3.4938 x 10-2


2 dec places tab 
19.76
  0.3494
8.273

186
SURDS

STUDENTS’ EXERCISE

1 . Simplify the following  a  log10  4 y  3  log10 3  log10  2 y 1


(a)log10 5  log10 16
 b  log10  7 y  2   log10 y 1  1
(b)log10 72  log10 24
1
 c  log 100  25  2log  y  4 
(c) log10 400  log10 20
2
8. Without using tables or calculator
(d) log10 4  log10 0.4
3log10 5  log10 20  log10 0.25   6 
(e) log10 1000
9. Simplify the following
(f) log 2 4  3log 2 2  log 2 4  a  2log10 5  3log10 2  log10 0.2
2. Evaluate the following
a  log9 81 (b) log2 8
 b  log3 9  log 3 21  log 3 7

(c) log2 32 (d) log9 6561


 c  log 7 14  2log 7 7
1 1
(e) log2 128 (f) log10 100000 d log81  log 25  log 49
4 2
3. Solve the following equations
   e  log 250  1.6  2log 2
a  log10 x 2  9  log10 x  1
10. If log x  a, log y  b and log z  c
(b) log10 x  6  1
c  log10 x 4   1
express the following in terms of a, b and c
i  log y 3 (ii) log x 2  
4. Simplify:
 2
y 3 
a 
log10 900 iii log z iv log  x
log10 3  log10 10  z 

 b
log10 512 100 x 2
log10 64  log10 8
v  log xy 2 vi log
y
c log12 9  log12 16  3 
d log a a  log a a 2 vii log  3 10002x 
 y 
 
5. Prove that:
11. Solve the equations
1 1  a  52 x  2  2 6 x 1
a  log5 36  log 5 14  log 5 21  log 5 400  1
2 2  b  log10 7 x  2  log10  x  1  1
 b  log10 4  log10 0.025   1  c  log10 3m  4  log10 m  2  3log 2
c 2log10 10  log10 5  log10 50 1
 d  log10 3 y 1  log10  2 y 1  log10 4
6. Without using tables or calculator, evaluate the  e  log10 8 x3  3
following expressions
1

 a  log 6 6  log 6 36
log3 81
 b  log3 27
log3 9
7. Solve the equations

170
SURDS

 f  log10 y  1  log10 y  log10 20  a  2.6 268  1.5352


 g  2log10 x  log10  x  6   b 1. 6518  3. 8304
 h  2  log10 x  log10 4   1 log10 2.5  c 1. 6141  2. 7332
 i  log10 2 y  11  log10 3  log10 3  log10 y  d  5. 3215  4.8328
 j log10  x 2  2   log10 3x  6  0  e  2.68  1. 66
 f  7. 7  3
12. Find x if  g   2. 5  2   1.6
18. Use logarithm tables to evaluate the following:
4 7 5
1760  7.718  43.8
2 log10 4  log10 x  log10  7   log10  7   log10 14 a 
19.5
0.4045  283
b
94.32  3.283
72.01
c
94.37  3.283
14. Simplify the following d 3
0.6765

a 
log10 4
b
log 27 e 5
0.09056
log10 8 log81
0.0782  34.39
log1000 f 
c  d  log 50  log 2  log10 100 4.836
log100
62.58  0.3216
 e  3log10 5  log10 10  log10 625  log10 2 g 3
41.58
1
 f  log120  log 27  21log 6 0.6327  3 2.834
3 h
2.918
15. If log10 2  0.30103 and
 0.0072   81.32
log10 3  0.47712 Find without using tables i  3
23140
or calculator correct to 5 decimal place the values
3
0.0357
of :
 j
 a  log10 4.5  b  log10 60
3
5.12  33.17
 c  log10 8  d  log10 1.5
ANSWERS
 e  log10 18 1. (a) log1080 (b) log103
16. Express the following in form log10 x 16
a  log10 2  log10 5 (c) 0 (d) log10
10
b  log10 25  log10 12  log10 3 (e) 3 (f) 3
c log10 12.5  3 log10 2
2. (a) 2 (b) 3
d 3 log10 x  1 log10 y (c) 5 (d) 4
2 (e) 7 (f) 5
e2 log10 p  log10 q
17. Simplify the following: 3. (a) x = 1, x = 9
(b)x = 16
(c) x = 40

4. (a) 2 (b) 3

170
SURDS

5 1
3y
c log10 100 d  log10 x
(c) 2 (d)
2 2

5.

6. (a) -1 (b) 5 e log10 p 2q


7. (a) y = 1 (b) y = -1, 12
(c) 54, -46
7
17. a  3. 0916 b  1.8214
8. 2
c  2.3473 d  8.1543

2 e  3.02 f 3. 9


9. (a) 3, (b) 3 (c) log7  
7
 15 
(d) log10  
7
(e) 2 g  5. 9
10. (i) 3b (ii) 2a (iii) ½ c
(iv)2a + 3b – ½ c (v) a + 2b
2
(vi) 1 + a - ½ b (vii) 1  a  b
3
11.(a) x = 1.1617 (b) x = 4
(c) m = 4 (d) y = ½
(e) x = 5 (f) y = 5, y = -4
(g) x = 3, x = -2 (h) x = + 8
(i) y = 6, y = - ½
(j) x = 4, x = -1
3
12. x  
2
13. x = 5
2
14. a   b 3
3 4
c 3 d  0
2
e 0 f  1
15. a  0.65321 b 0.59272
c 0.90309 (d) 0.17609
e 1.25527
16. a  log10 15 blog10 100

171
SURDS
A surd is an irrational root of a rational number for
example; 5, 7 and 11
Example I g 125
Simplify the following
 25  3
(a) 24 (b) 18 (c) 108
 25  3
(d) 72 (e) 98 (f)
 5  3
1800 h 162
(g) 125 (h) 162  81  2
Solution:
 81  2
a  24
 9 2
 4 6
 9 2
 4 6 Example II
 2  6 Simplify the following:
 2 6 a  20  5
b 18 b  4 3  12
 92 c 11  44  99
 9 2 d  3 27  48  2 75  108
 3 2 e 7 24  3 54  4 216
 3 2 Solution
a  20  5
 5 4  5
c 108
2 5 5
 36  3
2 5  5
 36  3
 5 2  1
 6 3  5 3
d 72  3 5
 36  2 (b) 4 3  12
 36  2
b 4 3  43
4 3 4 3
 6 2 
4 34  32 3 2 3
e 98   3 4 324  2 
 49  2 
2 32 3

 49  2 c 11  44  99
 7 2 11  11 4  9  11
 7 2
 11  2 11  3 11
f  1800
 11 1  2  3
 900  2
 11  0 
 900  2
 30 2
 0
ADDITION & SUBTRACTION OF
SURDS
172
Example I  a  b c
Simplify the following: For example:
a  5 2  7 2  4 2 3 2 5 2
b  8 3  5 3  4 3
 3  5 2
c 11 2  9 3  7 2  4 3
Solution:  8 2
(a) 5 2 7 24 2 (2) a c  b c  a  b  c
 2 5  7  4 For example:
 2 12  4 5 32 3
 2 8
 8 2
 5  2 3
(b) 8 35 34 3  3 3
 3 8  5  4   
(3) a b c  ab c
 3 7  For example

 7 3
5 4 5   5  4 5
(c) 11 2  9 3  7 2  4 3  20 5
 11 2  7 2  9 3  4 3 (4) a b  ab
 2 11  7   3 9  4 For example
 18 2  5 3 2  3
d) 3 27  48  2 75  108  23
 3 9  3  16  3  2 25  3  36  3
 6
   
33 3  4 3  25 3  6 3
 9 3 4 3  25 3   6 3 3 3
 9 3  4 3  10 3  6 3  9
 3 9  4  10  6 
3
 3 1
(v)
    a 2   b 2
 3
(e) 7 24  3 54  4 216 a b a b 
7 4  6  3 9  6  4 36  6
Example:
     
     2   3 2
7 2 6  33 6 46 6
14 6  9 6  24 6
5  3 5  3  5
6 14  9  24  2
6  1
 6 (vi) a b  a2  b

PROPERTIES OF SURDS Example


(1) a c  b c  c a  b  Simplify the following as far as possible

173
(a) 3  27 (c) 4  6 8 2 3 6 3 2  
(b) 18  8 
(d) 3  2 2 5  3 2  
(c) 27  5 Solution:
(d)  3 6

(e) 3 2 3
Solution
a  3  27
 3  27
 81
 9 a   5 3 2  5 3 2 
 3  27  9  5  53 2 3 2   53 2 
b 18  8  5  3 10  3 10  9  2 
 18  8  5  18
 144   13
 12 Alternatively
 5  3 2  5  3 2 
c 27  5
  5   3 2 
2 2

 27  5
5  18
 135   13
 9  15  b 3 3  75  2 12 

3  25  3  2 4  3 
3 15
3
d   3 6

3 3 5 3  4 3 
 3 3 3 3 3 3
3  3  3 3 3  3
 27  9

  3 6
 27 (c)  4 6  8 2  3 6  3 2 
6 3 6  3 2   8 2 3 6  3 2 
e 3 2 
3
4

 3 2   3 2  2  2 
 3 2   3 2   3 2
 72  12 12  24 12  48
 72  48  12 12
 3 3 3 2  2  2
 24 12 12
 27 2 2    24  12 4  3
 54 2  24  24 3
Example
Simplify the following:
 4 6 8 2  3 
6  3 2  24  12 12

(a)  53 2  5 3 2 
(b) 3 3  75  2 12 
174
d  3  2 2  5  3 2  a 
2

2 3

35  3 2   2 2 5  3 2 
3 3. 3
2 3
 15  9 2  10 2  12 
3
 3 2  2
3
3

Two 
 3surds 
CONJUGATE SURDS
 2 are 
5 to3 bebconjugate
2 said  3  to2 each b
1
1 2
other. If their product is a rational number.
For example;

1 1  2 
3 and 3 because 1 2 1  2 
3 3  33 1  2

1  2
2

2
 9
3 1  2

5  2  and 5  2 are
1 2
Similarly
1 2

conjugate to each other because 1
5  2  5  2   1  2
2  6 
 52   2 
2 2
c
2  6 2  6 
52 4 2 6

3  2
2
  6
2

4  2 6
RATIONALIZING THE 
46
DENOMINATOR
Rationalising the denominator of a fraction 42 6

means making the denominator a rational 2
number. To a rationalize the denominator of  2  6
a fraction, we multiply the denominator and 1
numerator by the conjugate surd of the d
3 2 2 3
denominator.

1 3 22 3 
Example:
Rationalise the following
3 22 3  3 22 3

(a) 2 (b) 1 3 22 3



1 2 3 2    2 3 
2 2
3
(c) 2 (d) 1 (e) 3 22 3
2 6 3 22 3 
18  12
1 5 3 22 3
1 5 
6
Solution: 3 2 2 3

6

(e) 1  5 
1  5 

175
1  5  1  5   1 1  5   5 1  5  
6 2

1  5  1  5   3   2 
2 2

1   5 
2 2

62
1 5  5  5 
 32
15
62
6 2 5 
 1
4
2  6
3  5 
 1
2 2
 2  6
  3  1 5 3 2
2 2
Example II Comparing  2  6 with a  b
27
Express in the form gives a   2, b  6
5 2

a b c  where a, b and c are
Example IV

integers. Simplify 63  28 as far as possible.


Solution 175  63
27 Solution:
5 2  63  28  175  63 

27  5 2   175  63   175  63 
 5 2  5 2  63  175  63   28  175  63 

27  5  2 
 175 
2
  63 
2

 5 2   2 2
27  5  2 
11025  63  4900  1764

 175  63
52 105  63  70  42
9 5 2   
112

 
280
27 
 9 5  2 112
5 2 5

2
Example III Alternatively;
2 63  28 97  47
If  a  b find the values 
3 2 175  63 25  7  9  7
of a and b 3 7  2 7
Solution 
5 7 3 7
2

2  3 2 7 3  2
3 2 3 2  3 2

7  5  3
5

2
Example V

176
1 2 2 32 33
Express  in form 
5 3 5 3 43
a b c 
1  3
of hence state the values of 1
2
a, b and c . 11 b3

   1  3
Solution: 22  33
1

2 Since 3  1.732
5 3 5 3
1  3
  1  1.732

1  5 3 2   5  3 2 3
 5 3  5  3  0.732
Example VII
 5 3  2 5  2 3
 12 7
 5   3  p  q r find
2 2
If
11  7


2 5  5   3 3 the values of p, q and r .
5  3 Solution:
55  33 23

12 7

12 7  11  7 
22 11  7 11  7  11  7
3 33 3 5 5

Comparing with 12 77  84
2 2 
11  7
a b  c 84  12 77
gives 
2 4
a  3, b  3, c  5  21  3 77
pq r
Example VI  p  21, q  3, r  77
1 3
Express in the form Example VIII
2 3 2 3 4  3 in the
Express 
1  3 2 3 2 3
a b 3 hence evaluate
2 3 form a  b c hence find the
correct to 3 significant figures if
values of a, b and c .
3  1.732 .
Solution: Solution

1  3 2  3 2 3 4 3

2 3 2  3 2 3 2 3

=
2  3  3 2  3
 2   3 
2 2

177
2 3 4 3

2 3 2 b

2  3  4  3 2  3
2


2  3 2  3
4  4 3  3  8  4 3  2 3  3

 2  3
2

2

4 4 3 38 4 3  2 3 3

43
12  2 3

1
12  2 3  a  b c
a  12, b   2 c  3

STUDENTS’ EXERCISE
(1) Simplify the following
a  54 b  99 c 578
d  5488 e 1575 d  432
2. Express each of the following as a square root
of a single number.
a  12  3
a  3 2, b  9 3, c 1 7
2 b  5  20  45
d  5 3 e 4 7 f  9 6 c 3 45  80  2 125  3125
g  12 3 d  6  4 3  6  4 3 
3. Simplify the following by rationalizing the
denominator e 4  3 2  5  3 2 
a 
8
b
9
c
21 f  5 6  3 5  4 15  3 2 
8 3 7 6. Rationalize the following
1 4 1 3
d
10 2
e
2 5
f 
20
a   b c
12 10 45 5 2 3 1 3
24 4 5 6 7 4 3 5 3
g h d (e) f 
25 3 10 5 7 4 3 5 3
3 15  2 22 4 6 5 3
i g h
2
7 2  165 6 5 5 2
4. Simplify the following
3 54 3 2 5  3 12
a  8 2  3 3  10 2 i  k
3 54 3 4 5 2
b  5 3  4 5  15 5  6 3 7. Simplify the following
c  7 11  8 5  3 11 a 
2
3 2

3
3 2
5. Simplify the following
4 1
b 
3 5 3 5
2 33 2
8. If a 6  , find the value
3
of a

178
5 4.  a  18 2  3 3
9. Given that  20  a 5
5  b  11 3  19 5
determine the value of a.  c  4 11 8 5
10. Given that
6 5. (a) 3 3 (b) 0 (c) 20 5
 a 22 3 (d) -12 (e) 2 + 3 2 (f)
3 22 3
90 3  69 10
30 90
11. Simplify  5
10 30 6 (a) (b) 8  4 3 (c)
2 3 5
12. Simplify 
1 3 32 3 2  3
Solution 23  7 19  8 3
(d) (e) (f)
1.  a  3 6  b  3 11  c  17 2 18 13
 d  28 7  e  15 7  f  12 3 14  5 3
11
2.  a  18 , b 243 c 7 (g) 19  3 30 (h)
4
15  6  10  2
d 75 e 112 f  486
3
g 432
(i) 31  8 15 (j)
40  2 10  24 15  6 6
3.  a  2 2 , b3 3 c 3 7
78
4 5
d 5 3 6 e 2, f  7 (a) 5 3  2 (b)
15  3 5
3
4
2 2
g 6 h 2 i  6 7 8 a=2
5 3 9 a=3
10 a = 3; b = 2
11. 2 3
12. 4  3

CIRCLES
A circle is defined by two things i.e. the radius
and the center. The fixed point is called the Terms used in circles
center. The constant distance from a fixed point 1. Circumference
to any point on the circumference is called the It is the length of the boundary showing the
radius. circle. In other words, it is the distance around
the circle.

r = radius O = center

179
The circumference of the circle is given by the found if the radius and the angle subtended at
formula the center are known.
C D
C   (2r )
C  2 r
2. Arc of the circle
It’s the proportion of the circumference.

AB = Chord
CD = Chord
AF = Chord
DE = Chord
When the circle is divided into two parts
The line joining the center of a circle to the mid
(unequal parts) the longer part is known as the
major arc while the shorter part is known as the point of a chord is perpendicular.
minor arc.

3. Center
It is a fixed point which is equidistant from all
points on the circumference.

4. Diameter
It is a line joining the two points on the AB = Chord
circumference and passes through the center.
Two chords are said to be equal if they lie at the
same distance from the centre.

AC = diameter
DC = diameter

5. Radius Chords AB and BC are equal if their


It is a line joining the center to any point on the perpendicular distances from the center O are
circumference. equal

7. Segment
This is a portion cut off by a chord. The large
portion is called the major segment while the
smaller portion is the minor segment.

6. Chord
It is a line joining the two points on the
circumference of a circle. Its length can be

180
Minor
O = Center
segment
R = radius
AT = Tangent

Major segment
Angle Properties of a Circle
1. Angles subtended at the circumference by a
common chord in the same segment are
8. Sector equal
This is the proportion of a circle found between
two radii. The larger portion is called the major
sector and smaller proportion is called the minor
sector.

Minor
sector

2. The angle subtended by the arc of the circle


Major sector at the center is twice the angle it subtends at
the circumference is illustrated below
O = Centre of the circle

y y

O
2y

O = Centre of the circle

9. Tangents to circles
A tangent is a line drawn from a point outside
the circle touching the circle at only one point.
A tangent to the circle always makes an angle of
900 with the radius at the point of contact. 3. The angle subtended by the diameter of the
circle at the circumference of a circle is 90
as illustrated below
90°

A C
B

90°

181
C = Centre,
AB = the diameter

4. Two opposite angles of a cyclic


quadrilateral inscribed in a circle are
supplementary

5. The angle which a tangent makes with the i.


chord at the point of contact is equal to the
angle in the alternate segment of the circle

ii.

6. tangents to a circle from a point outside the


circle are equal in length

Solution:
7. If two chords of a circle intersect inside or i.
outside the circle, the product of the
segments of the chord is equal to the
product of he segments of the other chord.

The angle subtended by the arc of a circle at the


Example I
center is twice the angle it subtends at the
Find the angles marked by the small letters
circumference
a  2(30)
 600

x 182
Example II
Find the angles marked with the letters where O
is the center.
i.

62  2 x
2 x  62
x  31
 x  310 ii.

ii.

iii.

y  2x  3
 1460

iii.
p z
iv

X Y

m  100  15  180
115  m  180
m  180  115
m  650 Solution:
0
Chord xy subtends an angle of m = 65 at the i.
circumference, xy also subtends an angle of b at
the circumference.
 b  650 , since equal chords subtend equal
angles at the circumference

Chord pz subtends an angle of 15° at the


circumference; pz also subtends an angle c at the Completing quadrilateral we have;
circumference.
 c  150

183
y  360  262
y  980
y y  2x
116 O t x 98  2 x
°
x  490

49  z  180
(Because two opposite angles of a cyclic
z  180  49
quadrilateral are supplementary)
116° + x = 180° z  1310
x  180  116
iv.
x  640

y  2x
 2(64)
 1280

t  (360  y )
t  360  128 y  130  180
t  232 0
y  180  130
ii. y  500
x  2y
x  2(50)
x  1000

Example III
Calculate the angles marked. O is the center
z  108  180
wherever given.
z  720
2z  x
x  1440
y  360  114
y  2160

iii.

Solution:

184
C
x 90 – x

D O
54° B
x
90 – x
A
AB = Diameter BC and AB are tangents to the circle
The diameter subtends an angle of 90 at the  AB and BC makes 90 with the radius of the
circumference. circle.
 35  a  90 90  x  90  x  54  180
234  2 x  180
234  180  2 x
2 x  54
x  27

a  90  35
a  55 0

CD  Diameter
x  90 0 27  27  y  180
50  b  x  160 54  y  180
50  90  b  180 y  180  54
b  180  (140) y  126
1
 40 0 ADC   126 
2
Example IV
 63
In the diagram below, O is the center of the
circle, AB and CD are tangents to the circle,   ADC  63 0
ABC = 54. Find  ADC.
Example V
C

D O
54° B

A
Solution:
In the diagram, TP is a tangent to the circle with
center C and angle PQC =40. Find the values of
x and y.

185
Solution: In the diagram above, TC is the tangent and O is
Considering triangle PQC the center of the circle. Calculate the values of x
and y.

Solution:

PC = QC (both radii)
 CPQ =  CQP
CPQ  40  40  180 Extracting triangle AOC, we have;
CPQ  180  80
 1000
Considering triangle PCT;

35  35  n  180
n  1100
90  80  x  180
n  x  180
x  180  170 110  x  180
x  10 0
x  700
70  90  y  180
160  y  180
y  200
 x  700 and y  200
Example VII
In the diagram, O is the center of the circle and
Angles on a straight line add up to 1800 ABC is a straight line where BD = CD and 
AOD = 100°. Find the size of;
y  40  90  180 i.  DBC
y  180  130 ii.  DBA
y  500
 x  100 and y  500

Example VI

186
40  75  AXY  180
Solution: 115  AXY  180
y AXY  650
65  YXW  180
O YXW  180  65
O
100° YXW  1150
D Since two opposite angles of a cyclic
A quadrilateral are supplementary;
x
 115  YZW  180
B
YZW  180  115
C YZW  650
1 75  XYZ  180  angles on the straight line 
y   100 
2 XYZ  180  75
y  50 0 XYZ  1050
x  y  180  105  XWZ  180
x  50  180 XWZ  750
x  130 0 65
let DBC  p
p  x  180
p  130  180
p  50 0
50  50  n  180
100  n  180 Example IX
In the diagram below, O is the center.
n  80 0
 DBC  p  50 0
DBA  x  130 0

Example VIII
WX and ZY produced meet at A. WXYZ is a z
cyclic quadrilateral.  WAZ = 40,  XYA =
75. Find the angle of he cyclic quadrilateral.

Find the values of x, y and z.


Solution:

Solution:

x  2  40
x  800

187
z  400 (Because equal chords subtend equal 108  AOC  180
angles at the circumference) AOC  180  108
x  BOA  180 AOC  720
 AOC +  COB = 180°
80  BOA  180 72° +  COB = 180°
BOA  1000  COB = 180° – 72°
1  COB = 108°
y  BOA Extracting out triangle COB;
2
1 O
y   100 
2
108°
y  50 0
 equal chords subtends equal 
z  400  
 angles at the circumference 
x C x
Example X C B
In the diagram below, AB is the diameter of the 2 x  108  180
circle. DA and DC are tangents to the circle at A 2 x  72
0
and C respectively. Given that  CAB = 54 , x  360
find the values of x and y. Extracting triangle DAC;
A

54°

D y O

C
x
B
Solution: y  36  36  180
A
y  1080
54°

Example XI
D y O In the diagram below, SCT is a tangent. ABT is
a straight line through O. Find the missing
C angles.
x
B
Extracting triangle AOC, we have;

 54  54  AOC  180 Solution:

188
90  p  90  p  50  180
230  2 p  180
67  OCA  90
p  25 0
OCA  230
25  25  AOB  180
Extracting out triangle OCA AOB  130 0
1
x  (130)
2
x  65 0
y  90  x
y  90  65
y  250
Example XIII
23  23  COA  180
COA  180  46
COA  1340 D
TOC  134  180
TOC  180  134
 460 C
46  90  y  180
136  y  180 In the diagram above, ABCD is a cyclic
quadrilateral. Find the values of x and y.
y  44 0

Solution:
Example XII
In the diagram below TA, TB are tangents, CB
is a diameter. D

DCB  60  180  angles on thestraight line 


DCB  1200
120  x  180
Find the values of x and y. x  600
Solution:

189
5 x 6
y  1 x  180 
6 6 9
11
y  x  180 9 x  36
6
36
11
y   60  180
x
6 9
y  110  180 x  4 cm
Alternatively;
y  700 AM x MB = CM x MD
 x  600 , y  700 9 x x = 6 x 6
9x = 36
Example XV x =4
Two chords AB and CD intersect at M. If AM =
9 cm, CM = 6 cm and MD = 6 cm, what is the Example XVI
length AB? Find the values of x and y.

Solution:
Let MB = x

Solution:
Triangles AMC and MDB are similar.

Triangles APO and BPC are similar.

190
8 y

16 10
8 10
y
16
y  5 cm
Alternatively;
10  8  16  y
x 8 80  16 y

18 12 y  5cm
12 x  18  8
12 x  144 Example XVII
x  12 cm Chords AB and DC intersect at T. AB = 8 cm,
Alternatively; BT = 6 cm and CD = 5 m. Find the length of
12  x  18  8 CT.
12 x  144
x  12

Solution:
Triangles TBC and TDA are similar.
AT .BT  DT .CT
Triangles YOW and ZOX are similar. (8  6)6  (5  x) x
84  5 x  x 2
x 2  5x  84  0
( x  7)( x  12)  0
x  7, x  12
 CT  7 cm

Example XVIII
NY is a tangent to the circle. Chord WX
intersects the tangent at N. If NY = 16 cm and
NX = 12 cm, what is XW?

191
Solution:

Using Pythogoras theorem


XW  t AM 2  MO 2  AO 2
(t  12)12  (0  16)16 x
2

   15  17
2 2
12t  144  256  
2
12t  112 x2
 225  289
112 4
t
12 x2
 289  225
t  9.33 cm 4
x2
Example XIX  64
In a circle of radius 17 cm. calculate the length 4
of the chord which is 15 cm from the center. x  16
 The length of the chord is 16 cm
Solution:
Let the length of the chord be x.
Example XX
In a circle of radius 13 cm, calculate the length
of the chord which is 5 cm from the center.

Solution:
Let the length of the chord be y

Extracting triangle AOM

Extracting triangle MOB, we have;

192
C
25°
20cm

A x M
2
Using Pythogoras theorem
MO 2  MB 2  OB 2 x
2 sin 25 = 2
0
 y 20
5     132
2

2 x
 sin 25=
y2 40
25   169
4 x  40sin 250
y2 x = 16.905°
 169  25
4
y 2  4(144) Important formulae used in circles
i. Area of the circle =  r
2
y  24
 The length of the chord is 24 cm ii. Area of the sector =   r 2
360
iii. Circumference of the circle  2 r
Example XXI D
Find the length of the chord that subtends an
angle of 500 at the center of a circle of radius 20 iv. Length of the arc =   2 r
360
cm.
Solution: Example I
Let the length of the chord be x
The diagram below shows a circle with an arc
which subtends an angle of 120 0 at the center of
the circle of radius 14 cm (   22 )
7

a) Find the area of the circle


b) Find the area of the minor sector AOB
c) Area of the major sector AOB
d) Circumference of the circle
e) Length of the minor arc AB and length
of the major arc AB
Extracting triangle AMC, we have; f) Area of the shaded segment
Solution:
a) Area of the circle =  r
2

193
22 88
  14  14 =88 
7 3
 616cm 2 176
 cm
b) Area of the sector =   r 2 3
360
 58.667 cm
120
  r2 f) Area of the shaded segment
360 = (Area of the minor sector) – (Area of the
120 triangle AOB)
  616  1
360    r 2  absin
360 2
616 2
 cm  120 22  1 
3 =  14 14    14 14sin120 
 360 7  2 
 205.33cm 2
c) Area of the major sector 616  3 
   196 
= (Area of the circle) – (Area of the minor 3  4 
sector)
616
616   49 3
 616  3
3  120.4628 cm 2
1232
 cm 2
3 Example
 410.667 cm 2 AT is a tangent to the circle with center O and
Alternatively; radius 12 cm. if OT = 13cm, calculate
Area of the major sector a) Length of AT
 360  120  b) Angle AOT
   616 c) Area of the shaded region
 360 
1232
 cm 2 (as before)
3 O
d) Circumference of the circle B
12cm
= D T
=2 r A
22 Solution:
=2  14 Note: A tangent to the circle makes 900 with the
7
=88 cm radius of the circle.
e) Length of the minor arc AB

=  2 r O
360 B
120 12cm
  88 T
360
A
88
 cm Extracting triangle AOT, we have;
3
 29.33 cm
Length of the major arc AB

Using Pythogoras theorem

194
AT 2  AO 2  OT 2 x  x  x  180
AT 2  122  132 3x  180
AT 2  144  169 x  600
AT 2  169  144 Let O be the center of the circle.
AT 2  25
AT  5
Let angle AOT be θ
12
cos  
13
 12 
  cos 1  
 13 
  22.6 0

Area of the shaded part BOC  2  60


1
 bh  Area of the sector  1200
2
1   22.6 22 
=  12  5     122 
2   360 7 
 1.589 cm2

Example
An equilateral triangle ABC is inscribed in a
circle of radius 10 cm. Calculate the length of
BC, area of the triangle ABC, and hence the Extracting triangle BOC, we have;
area of the shaded region.

BC  x cm
x
Solution: sin 60  2
Consider the equilateral triangle ABC; 10
x
sin 60 
20
x  20sin 60
BC  20sin 60
 10 3 cm

All the angles are equal since all the sides are
equal.

195
1
Area= 10 3 10 3 sin 60
2
1 3 Considering triangle KMB, we have;
 10 3 10 3 
2 2 MB
 129.9038 cm 2 tan60 
8.5
Area of the shaded region
MB  8.5tan60
= (area of the circle) – (area of the triangle
ABC)  14.7224 cm
1 BC  2 14.7224
  r 2  ab sin 
2  29.4448 cm
 22  Area of the shaded part
  10 10   129.9038
 7  = (area of the triangle) - (area of the circle)
 184.3819 cm 2

Example
The diagram below shows a circle with center O
and radius 8.5 cm inscribed in an equilateral
triangle ABC.

1   22 
   29.4448  29.4448sin 60     8.52 
 2   7 
2
=148.34896 cm
If AB, AC and BC are segments to the circle,
calculate the length of any of the tangents. Example
Calculate the area of the shaded part.
Solution:

Calculate the radius of the inscribed circle and


the area of the shaded part.
Solution:
Extracting OKMB;

196
Example
In the figure below PT and PS are tangents to
the circle with center O. if OT = 6 cm and angle
POT = 500, calculate the area of the shaded
region (  = 3.14)

O
Solution:
PT and PS are tangents implying that they make
900 with the radius of the circle.

Extracting triangle POT, we have;


h  62  32
h  36  9
h  27

PT
tan500 =
6
PT=6tan50
Area of the shaded part
1  Area of   Area of   Area of 
radius= 27      
3 =  the triangle    the triangle
   the 
 PTO   POS   sector 

1
93      
3 1  1    
1   bh    bh     r2 
 3 3  2   2   360 
3
1  1   100 
 3    6  6tan50     6  6tan50     3.14  62 
 2   2   360 
 1.7321 cm  11.503cm2
Example
Area of shaded part

=
 Area of the triangle    Area of the circle 
3
1   22
  
2
  6  6sin 60     3 
   7 
2
3
6.1594
  2.053 cm 2
3 In the diagram above, AB and AC are tangents
 The area of the shaded part is 2.053 cm 2
to the circle at points B and C respectively. O is

197
the center of the circle. Given that AB = BC = 6
r
AC = 12 cm, find cos 30
i. The obtuse angle BOC 3
ii. The radius of the circle r  6
2
iii. The area of the minor sector BOC, hence
12
the area of the shaded region r
Solution: 3
r4 3
r  6.928 cm
Area of the sector

  r2
360
120 22
 
2
   4 3
360 7
Triangle ABC is an equilateral triangle. All  50.286cm 2
angles are equal and they are 600. Area of the shaded part
Since AB and AC are tangents;  Area of   Area of   Area of 
 ABO and ACO are right angles     
=  the triangle    the triangle    the


 CBO  BCO  300  AOC   AOB   sector =
     
Considering triangle BOC 1  1 
   4 3 12     4 3 12   50.287
2  2 
32.8514 cm2
Example
The figure below shows two intersecting circles.
Determine the area of the intersecting region

30  30    180
  1200
Since BO = CO (both radii), a perpendicular
Solution:
bisector from O to BC bisects both the angle
BOC and the base BC.
30

Considering triangle AXB


30

6
cos 30 
r

198
X 6cm Considering circle 1
B Area of the shaded part
90 1
  r 2  absin
360 2
6cm
1 22 2 1
  6   6  6sin90
4 7 2
22  36 36

28 2
A Area of the shaded part = 10.2857 cm2
6  Area of the intersecting region
tan =
6 = 2 × 10.2857
tan  1 = 20.5714 cm
  tan 1 (1) Example
  450 P
 XAB  BAY  450

97.2

XY  62  62 The diagram above shows two circles, centers A


and B of radii 4 cm and 6 cm respectively,
 72
intersect as shown. If  PBQ = 30 and  PAQ
6 2 = 97.2, what is the area of the region common to
AB  62  62 the two circles?
Solution:
 6 2 cm
Separating the two circles
P

97.2

199
Solution:
R

M X Y N

10cm

40°
Q k O P
10cm

Length of the arc XRY


97.2 22 2 1
A1    4   4  4sin97.2 R
360 7 2
X Y
A1  5.6402 cm 2

30 22 2 1
A2    6   6  6sin30 10cm
360 7 2 100°
22  62
A2  9 O
84 
A2 = 0.4286 cm2.  2 r
360
Total area of the shaded part
100
 A1  A2  2  3.14  10
360
 5.6402  0.429 17.444 cm
 6.069 cm 2  The length of the arc XRY = 17.444 cm
MQ = XK
XK
Example sin 40 
In the figure below, QXRYP is a semi circle 10
with center O and radius 10 cm. MN is parallel XK = 10 sin 40°.
XK = 6.4279
to the diameter QP. Angle XOQ = 400.
MQ = XK = 6.42790
R
MX = QO = OK
M X Y N OK
cos 40 
10
10cm  MX = 10 – 10cos40°
MX = 2.33956
40° Perimeter:
Q 10cm O P
= QM + MX + XRY + YN + NP
Find the = 6.4279 + 2.33956 + 17.4444 + 2.3396 +
a. Length of 6.4279 + 10 + 10
i. The arc XRY = 54.9794 cm
ii. MQ
iii. MX
b. Perimeter of the above figure
 take  to be 3.14 
STUDENTS’ EXERCISE
1. Find the length of the chord that; 2. In a circle of radius 5 cm, calculate the
a) Subtends an angle of 1000 at the center length of the chord which is 3 c from the
of a circle of radius 20 cm center.
0
b) Subtends an angle of 120 at the center 3. A circle has a radius of 25 cm. How far
of a circle with radius 7.2 cm from the center is a chord 14 cm long?

200
4. Two parallel chords are in opposite sides
of the center of a circle of radius 13 cm. If
the chords are of lengths 15 cm and 8 cm,
how far apart are they?
5. Chords AB and CD of a circle are
produced to meet outside the center at T. If
AB = 5 cm, BT = 4 cm and CD = 9 cm,
calculate TD.
6.

d.
X

In the figure above, XB = 2 cm, CX = 6 cm,


and XD = 4 cm. Calculate the length of AB
7. Find the angles marked by small letters
below where O is the center wherever
given.

a. 8. Find the angles marked by letters in the


following figures;
i.

b.

ii.

c.

iii.

201
B

O 52° y
z A
z

C
10. In the figure below, O is the center of
the circle. ABCD is a cyclic quadrilateral.
The sides BA and DC are extended to P and
iv. Q respectively. If reflex  BOD=2200, 
OBC = 400 and  ODA = 500.

v. Find  DCB,  DAB,  DAP

11. Find the values of x and y in the diagram


below.

9. Given that AB and AC are tangents to the


circle with center O. Find the angles
12.
indicated by the letters;
B
a

O b
40° A
c
C
B In the diagram above MON is a diameter of a
x circle with center O ad  LON = 76. Find the
values of x, y.
O y
24° 13. The diagram shows a circle with center O.
A
c given that  PQR = 1400 and  POQ =
z 1200, find the angles of the triangle PQR.
C

202
T

30°

14.
z
x O y
20°

A
17. find the lettered angles in the figures,
where O is the center

The diagram above shows a circle with center


O circumscribing triangle ABC. Determine
i) The reflex angle AOB
ii) An expression for y in terms of x
15. Find the missing angles in the following
diagrams

18. In the diagram below, A is a diameter and


O is the center of the circle. Angle BAP =
62. Find  AOP and  APB.

16. Find the angles x, y and z in the figure


below, given that O is the center of the
circle and TA is a tangent to the circle.

19. Find the sizes of the angles marked a, b, c,


d and e in the figure below

203
P

6cm

O Q
6cm
Take π = 22/7
20. Find the values of x, y and z. 22. The figure below shows a flower bed
bonded by two concentric arcs. Given the
information in the figure below, calculate
the area of the flower bed.

21. The diagram below shows a right angled 23. In the diagram below TA is a tangent to
isosceles triangle bonded by two circular the circle. CD is a diameter of the circle
arcs. The outer arc is a semi circle whose and CAT is a straight line. Angle DAT =
diameter is PQ and the inner arc is a 420. Find CTA.
quarter of a circle with center O. Calculate
the area of the shaded part.

ANSWERS
1. (a) 30.642 cm i) e = 870 f = 880 10.  DCB = 700
(b) 12.471 cm ii) x = 800 y = 1000  DAB = 1100
2. 8 cm z = 990  DAP = 700
3. 24 cm iii) n = 960 m = 600 x = 400 y = 700
4. 22.988 cm iv) k = 600 l = 1100 11. x = 380 y = 520
12.  QPR = 500
0
5. TD = 3 cm v) z = 130 w = 1150
6. AB = 14 cm 9.  PQR = 700
7. (a) x = 820 i) a = 900 c = 900  PRQ = 600
(b) y = 520 b = 140 0
13. (i) 180 + 2y
(c) z = 700 ii) x = 900 y = 660 (ii) x = 90 - y
(d) x = 3200 z = 90 0
14. x = 580 d = 390
8. iii) z = 450 y = 480

204
THREE-DIMENSIONAL GEOMETRY
In order to understand three-dimensional geometry,  If a line is perpendicular to two different lines in
the phrase “three-dimensional” has to be fully a plane, then it is perpendicular to every other
understood and well differentiated from one line in the plane. See the figure below.
dimensional and two- dimensional geometry. Note: A plane is a surface such that the straight line
One-dimensional geometry: Here, there is only one joining any two points on it lies entirely in
direction to move along for example from up the surface of a line.
moving downwards along a straight line. Using the diagram below, there are many planes that
Two dimensional geometry: Here, there are two are visible, though so many others can be created.
directions to move along, that is up and down and, For example ABCD, DHGC, EFBA are all planes.
left and right, for example on a flat shape like a Among the planes that can be created include AGC,
rectangle. AHD and others. See the figure below.
Three-dimensional geometry: Here, there are three F G
directions to move along, that is up and down, left
E H
and right, and in and out. See the diagrams below for
clarity.
One dimensional B C

Two dimensional A D
In the figure below, line FB is perpendicular to AB
and BC. This means it is also perpendicular to BN
and BK and any other line on the plane that comes in
contact with it.
Three dimensional F

B C

In this chapter, we are interested in three- A N D


dimensional geometry; so let us turn all our attention Some special properties of three dimensional
to three-dimensional geometry. solids
The following are very essential and have to be 1. Cuboid
mastered by heart: X W
 Pythogoras’ theorem
 Trigonometry U V
Also, note the following:
S
 Any vertical line is perpendicular to any R
horizontal line, for example in the diagram
below, line EA is perpendicular to line AB, and P Q
line AD. Line HD is perpendicular to line AD  Here, all sides are not equal
and line DC.  All opposite faces are parallel e.g. PSXU is
F G parallel to QVWR, PQRS is parallel to UVWX,
and others
E H  All adjacent faces are perpendicular to each
other as seen in the diagram above. XUPS and
B C
XUVW, UPQV and QVWR

A D 2. Cube
205
 The properties of a cube are similar to those of a W
Cuboid, except that in a cube all sides are equal.
3. Right pyramid
In a right pyramid, the following are noted basing on S R
the diagram below;
V α

P Q

Note:
 The angle θ seen in the diagram above is
between a line and a line that is line XP and line
D C PS and not between line PX and plane PQRS.
See number 4b in the examples, the angle
O P required is FMB but not FAB.
A B  Line WR is called a normal to the plane, while
 All slanting heights have the same length. For line PR is called the projection of line PW on the
example VA = VB = VC = VD. plane.
 Planes VDA, VAB, VBC and VDC are known For the case of pyramids, the angle between lines
as slanting planes. and planes is obtained using the same technique as
 Only one vertical line (VO) can be drawn from in cuboids, that is a projection of the line in question
the vertex (V) to the center (O) of the base and the normal have to be identified.
V
ABCD.
 Other lines from the vertex (V) only make right
angles with the lines making the plane ABCD.
That is AB, BC, CD and DA, provided the
divide the individual triangles into equal parts.
D
Angles between a line and a plane C

If you are to find it easy in finding the angle between O β


a line and a plane, you must be in position to A
B
identify a convenient point on the line from which a
In this figure, the angle between plane ABCD and
line can be drawn to meet the plane at right angles.
the line VB is β. Note that the normal to the plane
See the diagrams below for clarity.
X VO and the projection of line VB to the plane is OB.
W Angle between a plane and a plane
U
To determine the angle between any two intersecting
V planes,
S  Take any point on the line of intersection
R
θ
 At the chosen point, draw two lines, one on each
α plane such that these lines are perpendicular to
P Q the line of intersection
In the figure, let us consider two lines and the angles  Find the angle between the two drawn lines, this
they make with the respective planes. is the angle between the two planes
 The angle between line PW and plane PQRS is α
W See the diagram below
(1) D P C

α R
P
θ
A K B

R M P

206
In the diagram above,  Extract the required planes and follow the
 the selected point is K on the line of steps discussed previously in this chapter.
intersection V
 PK is the perpendicular line on plane ABCD
 KM is the perpendicular line on the plane
ABPR
 angle θ is the angle between the two planes
(2) Consider the figure below;
H G D
C
θ
E F O

D C A B
Where O is the center and VDA is an isosceles
A B triangle
Angle between plane BEH and plane EFGH is θ is the angle between ABCD and plane
obtained as follows VAD.
 Extract out the planes of interest b) Angle between plane VAB and VDC
H  Extract the required planes but here the
G
K meeting plane is not a line but it is a single
θ M
E point. So we just bisect the angles of the
F triangles at the meeting point such that the
bisecting lines meet the bases of the triangles
D C at right angles as seen below.
V
A B
 Draw out the perpendicular lines KM and KB
as described before on the common line EH
 Then find the angle θ between the two lines θ
KM and KB. This is the angle between the
two planes. D
Note: Triangle KMB is right angled since KM is C
perpendicular to plane GFBC. It then follows
that KM is also perpendicular to any other
line in the plane just like KM and MB are A B
perpendicular to each other.
Note that this is true for only isosceles triangles. θ is
(3) Using the figure below, the angles between
the angle between plane VAB and VDC.
different planes can be obtained in the
following ways:
Examples:
V
1. The figure below is a cuboid with dimensions as
shown
H G

E F 6 cm
C
D
D C
8 cm
A B A 10 cm B
a) Angle between plane VAD and the base ABCD Calculate:
i) the length AC,

207
ii) length EB,  b2  e2  h2  2ehCosB
iii) angle EBH,
 b2  e2  h2  2ehCos
iv) the angle between ADGF and ABCD.
Solution b  8 cm, EB = BE = 11.66
i) Length AC Note that EB = BE since these are not vectors and
D so we don’t need to change signs
C
e : Using triangle HDB
8 cm  HD 2  DB 2  HB 2
 62  12.812  HB 2
A 10 cm B note that AC=DB=12.81
Using Pythagoras’ theorem  HB  200.0961
 AB 2  BC 2  AC 2
 HB  14.146 cm
 102  82  AC 2
 e = 14.146 cm
 100  64  AC 2
 from b 2  e2  h 2  2eh cos B
 AC 2  164
 82  14.1462  11.662  2 14.146  11.66cos 
 AC  164
 272.065  329.885cos 
 AC  12.81 cm
272.065
ii) length EB  cos  
E F
329.885
272.065
   cos 1
329.885
6 cm    34.4 (1dp)
0

Alternatively; EBH is a right angled triangle


A
10 cm B Since a line on the plane ABFE that comes in
contact is the point E is perpendicular to line EH.
Using Pythagoras’ theorem
Remember the properteis of a cuboid.
 FA2  AB 2  BE 2
H
 62  102  BE 2
8
 36  100  BE 2
 BE 2  136 E

 BE  136
 BE  11.66 cm 136
iii) angle EBH √136
H √ θ
G
B
E F HEB  90
8
D C  tan  
136
θ
8
   tan 1
A B 136
Let angle EBH be θ    34.40
 Note that in this triangle, no angle is given, iv) angle between ADGF and ABCD
therefore we can’t use the Sine rule, therefore
we are only left with the Cosine rule to use in
order to get θ.
Using the figure above

208
H G Let the angle between QW and PQRS be θ.
Using the above diagram, we can calculate angle θ.
6 cm WS
E
F
Using tan   ;
SQ
D C
α 9
8 cm
tan  
10
A
10 cm B 9
  tan 1
Let the angle between plane ADGF and ABCD be α 10
  41.987  420
 The angle between QW and PQRS is 420
6 cm
α
10 cm
(iii) angle between plane QTW and TWVU.
6
tan   W V
10 K
6 θ L
  tan 1 T U 9 cm
10
  30.96  31.00 R
S
Therefore, the angle between plane ABCD and
ADGF is 31.00 6 cm
2) The diagram below shows a cuboid with sides P 8 cm Q
8cm, 6cm and 9cm. 0
Why KLQ is 90 has been explained:
W V Let the angle be θ
Using Pythagoras’ theorem;
T U 9 cm LQ2 = LU2 + UQ2
2
6
LQ 2     92
S
R 2
6 cm LQ  32  92
2

P 8 cm Q
LQ  90
i) Find length SQ
LQ  9.487
ii) Angle between QW and PQRS
iii) Angle between plane QTW and TWVU Using the dotted triangle
9.487
tan  
Solution 8
i) length SQ   49.90
Using Pythogoras theorem  Angle between QTW and TWVU is 49.90
SQ 2  PQ 2  PS 2 3. The diagram below is a cuboid with dimensions
15cm by 20cm by 12 cm. Determine the angle
SQ  82  62 between the following planes:
SQ  64  36 S R
SQ  10 cm Q
P
(ii) W

N 20 cm
9 cm
K

12 cm L 15 cm M
S R
i) KLMN and LKRQ
6 cm ii) LQRK and MNRQ
θ
iii) LNRP and KNRS
P 8 cm Q iv) KNQP and LMQP
Solution:
209
(i) angle between KLMN and LKRQ  LN 2  LM 2  MN 2
R
LN 2  152  122
Q LN  152  122
LN  19.21 cm
N
20 cm  a = 19.21cm
K
θ b = 15 cm = AC
M c = 12 cm = AB
15 cm
122  19.212  152  2 19.2115cos 
L
Let the angle be θ
450.0241
Using the triangle in the dotted lines   cos 
20 576.3
tan  
15    cos 1 0.780885
20
  tan 1    38.65
15
 Angle between LNRP and KNRS is 38.70
  53.10 (1dp) Note that the dotted triangle is similar to KNL
(ii) angle between LQRK and MNRQ and SRP
R (iv) angle between KNQP and LMQP
S R
Q
θ
P Q
N 20 θ θ θ
K cm 20 cm
L 15 cm M
K N
Let the angle be θ 12 cm
15 L 15 cm M
tan  
20 Using the dotted triangle
15 Let the angle be θ
  tan 1
20 12
tan  
  36.90 (1dp) 20
Note that the dotted triangle is similar to triangles 12
  tan 1
RKN and LQM 20
(iii) angle between LNRP and KNRS   30.960
S R  The angle between plane KNQP and LMQP is
θ
31.00
Q
A
P Note that the dotted triangle is similar to triangle
θ C KPL and QNM
20 cm
N 4) The figure below is a container of the form of a
K B θ cuboid used to feed chicken in a given farm.
12 cm
L 15 cm M F G
Let the angle be θ
Since the triangle in the dotted lines is not right H
E
angled, and yet no angle is defined in this triangle,
we use the Cosine rule
C
Let the triangle in the dotted lines be ABC 20 cm B
 c 2  a 2  b 2  2ab cos C 15 cm
But angle at C =  A 15 cm D
 C 2  a 2  b 2  2abCos Find the angles between:
Note that a = BC = LN a) AHGB and ABCD
But LN is obtained using Pythagoras’ theorem b) FAC and ABCD

210
c) if M is the midpoint of EH and N is the midpoint BD 2  BA2  AD 2
of FG, find the angles between planes
i) MNBA and ABCD BD  152  152
ii) MNBA and MNCB BD  450
Solution
a) AHGB and ABCD BD  21.21320
Let the angle be θ 1
 BM  BD
G 2
1
20 cm   21.213  10.607 cm
2
B θ H C Also FB = EA = 20cm.
θ 20
15 cm tan  
10.607
θ
20
A 15 cm D
  tan 1
Note that the dotted triangle is similar to triangles 10.607
ADH and BCG   62.1 (1dp)
0

Using tan  
opposite
and triangle ADH  The angle between planes FAC and ABCD is
adjacent 62.10
20 c) (i)Angle between planes MNBA and ABCD
tan   F N G
15
20
  tan 1 M
15
  53.1 H
 Angle between AHGB and ABCD is 53.10 E
B C
b) FAC and ABCD
θ
F G

H
E A D
opp
20 cm B θ
C Using the dotted triangle, tan  
15 cm adj
M
A 15 cm D
Opp = GC = EA = 20cm
1 1
Since ABCD is a square, the diagonal lines that cross adj = AD  15  7.5 cm
from B to D and A to C meet at right angles hence 2 2
the lines on the plane ABCD that are perpendicular 20
tan  
to the line of contact of the planes are these diagonal 7.5
lines. 20
If a point is taken to be the midpoint of line AC, it   tan 1
7.5
means that line FM meets line AC (the line of
contact of the planes) at right angles.   69.4 0

Using the triangle in the dotted lines  The angle between MNBA and ABCD is 69.40
c (ii) Angle between MNBA and MNCD
FB
tan  
BM
1
But BM = BD
2

211
F N G
iv) the inclination of PR to the horizontal
Solution
θ i) QR
20 cm Using triangle QRX
M θ H R
E
B C
θ 10 cm
15
cm X

A 15 cm D 30 cm

Let the angle between the planes be θ Q


Since triangles BNC, AMD and the dotted triangles QX = BC = 30 cm
are completely similar, so we can use any of the Using Pythogoras theorem
triangles. QX 2  XP 2  QR 2
Using triangle AMD
AD = 15 cm 302  102  QR 2
Since M is a midpoint of EH, it means that AM = QR  302  102
MD
From triangle EMA and using Pythogoras’ theorem QR  31.62 cm (2dp)
EM 2  EA2  MA2 ii) Angle QRC
R
2
 15  θ
MA     202
 2 10 cm

MA  21.36
 MA = MD = 21.36cm
Using the Cosine rule Q 30 cm X

m  d  a  2daCos
2 2 2 Note that angle QRC = angle QRX. So we can use
triangle QRX to get angle QRC.
 152  21.362  21.362  2  21.362 Cos Let angle QRC = QRX = θ
687.4992 QX
 cos  tan  
912.4992 RX
cos   0.7534 tan  
30
10
  41.10 30
 The angle between MNBA and MNCB is 41.10   tan 1
10
5) The figure below shows the top of a desk. The
base ABCD is a rectangle 40cmX30cm. The   71.6 (1dp)
0

edges AP, BQ, CR and DS are perpendicular and  Angle QRC is 71.60
PQRS is a rectangle. AP = BQ = 8cm and DS = iii) Angle between the planes PQXY and PQRS
CR= 18 cm. S R
S R
Y 10 cm
10 cm
Q X
P 8 cm
D Y X
C θ θ
30 cm θ 30
A 40 cm B cm
P Q
Calculate
The line of contact is PQ
i) QR
Following the steps used in finding the angle
ii) angle QRC
between a plane and a plane
iii) the angle between the plane PQXY and PQRS
212
Let the angle between the two planes be θ
 Note that the dotted triangle, triangle PSY and SKEW LINES
QRX are all the same. So we can use any Ever since our primary school, we know that only
triangle to find θ parallel lines are the only ones that don’t meet. In
10 three-dimensional geometry, there is a possibility
tan  
30 that two non parallel lines can fail to meet.
1 See the diagram below.
  tan 1
3 D C
  18.40
 The angle between planes PQXY and PQRS is A
B
18.40

iv) Inclination of PR to the horizontal R


S
The ‘horizontal’ in the figure are planes ABCD and
QXYP. However, QXYP is a virtual horizontal
surface. P Q
In
The horizontal surface we consider is QXYP since a this diagram, line PS and DQ are skew lines
point P is located in this plane but not in plane meaning that they are not parallel but they can never
ABCD. meet.
R However, the good news is that we can get the angle
between them by using ‘translation.’ In translation,
10 cm
we use another line that behaves in the same way as
the line in question and which line can meet the
other line in question. For example, to find the angle
Y
X between PS and DQ, we translate PS upwards to line
AD and the angle required will be between line AD
θ 30 and line DQ.
cm Example
P 40 Q D C
Note that the cm
angle
between PR and plane PQXY is
the angle between the diagonal line PX in the B
A
plane and line PR.
RX is perpendicular to PX since RX is perpendicular R
to the plane. 7 cm S
5 cm
Let the angle between PX and PR be θ
RX P 6 cm Q In
tan   the figure below, find the angles between the skew
PX
But PX is got by using Pythogoras theorem lines given
i) PS and DQ
PQ 2  QX 2  PX 2
402  302  PX 2 Solution
i) PS and DQ
PX  402  302 You notice that PS and DQ are skew lines and so
PX  50cm translation will help us.
RX = 10cm Line PS is easily translated than DQ, so we translate
line PS to line AD such that the angle is between line
10
tan   AD and line DQ.
50
1
  tan 1
5
  11.3 (1dp)
0

 The angle between line PR and the horizontal is


11.30

213
D C 62  52  SQ 2
θ B SQ  36  25
A 7 cm
SQ  7.81cm
R  From triangle DSQ
S
5 cm
 DS2 + SQ2 = DQ2
7 2  7.812  DQ 2
P 6 cm Q
DQ  7 2  7.812
DQ  110
DQ  10.488cm
Now applying the Cosine rule on triangle AQD
d 2  a 2  q 2  2aq cos 
Method I
Note that triangle DAQ is right angled. This is hard But d = AQ = 9.22 cm
to believe but it is the truth. q = AD = 5cm
9.22
tan   a = DQ = 10.488 cm
5  9.222  10.4882  52  2 10.488  5cos 
9.22
  tan 1  9.222  (10.4882  52 )  2  10.488  5cos 
5
 49.9897  104.88cos 
  61.50 49.9897
 The angle between PS and DQ is 61.50  cos 
104.88
Method II cos   0.4766
This method is intended for those who don’t believe   cos 1 0.4766
that DAQ is right angled. We are going to use the
  61.5
Cosine rule. However, this approach is slightly
 The angle between skew lines PS and DQ is 61.50
longer than the one above.
AD = 5cm Now The Non Believers Are Contented And I Am
AQ: To get AQ, we use Pythogoras theorem. Sure They Now Believe That Triangle DAQ is Right
AP 2  PQ 2  AQ 2 Angled.
7 2  62  AQ 2 NETS AND SOLIDS
A net of a solid is like a map of that solid. In a net,
AQ  7 2  62
all faces of that solid are visible just the same way a
AQ  9.22cm mechanic disassembles a vehicle such that every part
DQ: We use triangle DSQ to get length DQ. is visible.
D
Can you imagine that a well constructed cuboid can
be made to appear like a flat object! This flat object
7 cm is called the net of a cuboid.

See the diagram below.


S H G
Q E
F
We get SQ using Pythogoras theorem on triangle
PQS
C
 PQ2 + PS2 = SQ2 D

A B

This is a well constructed cuboid

214
E F We extract triangle AGC

G
E F
H G

4 cm
A C B
D
D C

A B 9 cm

A 12 cm B
E F
This is a well layered net of the cuboid above. It is
flat! It can be assembled again to get the cuboid Using Pythagoras’ theorem; AG 2  AC 2  CA2
above. But we can get AC by applying Pythogoras theorem
As a MUST, the corners with similar letters in the on triangle ACB
solid have to come in contact during the process of AC 2  AB 2  BC 2
assembling the net to give a well constructed cuboid.
Example AC  122  92
The diagram below is a net of a cage used to keep AC  15cm
puppies on a given farm
E F  AG 2  152  42
AG  241
E F
AG  15.524cm
H G  The length of AG is 15.52cm.
b (ii) Length of AF
A E F
C B
D
9 cm 4 cm
A B
12 cm
4 cm
A B
12 cm
E F Using Pythagoras’ theorem
Construct a well labeled cuboid from the net above AF 2  AB 2  BF 2
a) Calculate:
i) the length AG AF  122  42
ii) the length AF
iii) angle GAF AF  160
iv) angle between planes AGF and EFGH AF  12.65cm
Solution  The length of AF is 12.65 cm.
a) It is possible to get a piece of paper and label it b (iii) Angle GAF
to look like the net above. Then fold it such that G
you get a cuboid and making sure that corners 9 cm
with similar letters come in contact. 15.52 cm
You will get a figure that looks like the one below.
F
H G
θ 12.65 cm
E F
A
4 cm
D
C
Let the angle be θ
9 cm
A 12 B
Using the Cosine rule
cm

b (i) Length AG

215
a 2  f 2  g 2  2GF cos  tan  
LA
LK
92  15.522  12.652  2 15.52 12.65cos 
Note: since KL is perpendicular to HE of plane
92  15.522  12.652   2 15.52 12.65cos  HEAD, it means that it is perpendicular to any other
line in the plane, thus triangle KLA is right angled at
319.8929
 cos  point L.
2 15.52 12.65 Getting LA using Pythogoras’ theorem on triangle
cos   0.8147 LAE
  cos 1 0.8147 LA2  LE 2  EA2
2
  35.40  HE 
LA2     EA
2

  GAF is 35.40  2 
2
Alternatively 9
Though it is not easy to believe, but triangle AGF is     42
2
right angled (see diagram below for explanation). If
 GFA is right angled, the angle θ can be got LA  4.52  42
cheaply using tan θ. LA  6.02 cm
GF
tan  
AF Example 8
9 The figure below is a right pyramid on a rectangular
tan   base which is 10 cm by 8 cm. if the length of the
12.65 slant edge PV is 13.60 cm, find the following;
9
  tan 1 V
12.65
  35.4 as before
0

Explanation why GAF is right angled at F


We said that a line perpendicular to any two lines in
13.6
the plane is perpendicular to all other lines in the
plane. cm
S
H G R
E F 8 cm

P 10 cm Q
D C i) The length PR
A B ii) The height of the pyramid
 Line GF is perpendicular to line FB and FG of iii) Angle between PV and the base PQRS
the plane BFEA. iv) Area of VPQ
v) Volume of the pyramid
 Therefore, line GF is perpendicular to any other
line in plane BFEA. Thus GF is perpendicular to Solution:
i) The length PR
line FA, hence triangle GAF is right angled at F
Using Pythogoras’ theorem;
iv) Angle between planes AGF and EFGH
Method I R
H G
K 8 cm
L E θ F
P 10 cm Q

4 cm
C
D
9 cm
A 12 cm B
Using the dotted triangle

216
PQ 2  QR 2  PR 2
V
102  82  PR 2
PR  102  82
PR  164
12 cm
PR  12.806 cm (3 dp)
ii) The height of the pyramid
V
R

θ O 8 cm
13.6 cm
P 10 cm Q

S R Let the angle be θ


Using Pythogoras theorem,
8 cm
P
O
Q
PQ 2  QR 2  PR 2
10 cm
1 102  82  PR 2
Note that SO  SQ
2 PR  102  82
But SQ2  SP 2  PQ 2 (using Pythogoras PR  12.806 cm
theorem) 1 1
PO  PR   12.806
SQ 2  82  102 2 2
PO  6.403 cm
SQ  82  10 2 VO
SQ  12.806 cm tan  
PO
1 12
SO  12.806 tan  
2 6.403
SO  6.403 cm   tan 1.874
1

Using Pythogoras theorem on triangle VSO


  61.90
VO2  SO2  VS 2
 Angle between PV and the
 VO 2  6.4032  13.62
base PQRS is 61.90.
 VO 2  13.62  6.4032
Alternatively;
 VO 2  143.962 V
 VO  11.998 12 cm
 The height of the pyramid
is 12 cm. 13.6 cm 12 cm
iii) Angle between PV and the base PQRS

θ
P O
12
We can use sin  
13.6

217
12 abc
  sin 1
where s 
13.6 2
  61.9 as before
0 Using triangle VPQ;
13.6  13.6  10
s
iv) Area of VPQ 2
Method I s  18.6 cm
Since this is a right pyramid, it means that VP and
VQ are equal, and so triangle VPQ is an isosceles  A  18.6(18.6  13.6)(18.6  13.6)(18.6  10)
triangle.
A  3999
V
A  63.24 cm 2 as before

Method III
13.6 cm 13.6 cm Using triangle VPQ to get angle VPQ;
A B Let  VPQ = θ
5
cos  
13.6
  68.430
P Q
5 cm M 5 cm 1
Area  vq sin 
2
Area of A = Area of B 1
So if we find the area of A, or the area of B, we =  10  13.6sin68.43
simply multiply by 2 to get the area of triangle VPQ. 2
1 Area = 63.24 cm 2 as before
Area of A  bh
2
1
A   5 h v) Volume of the pyramid
2 1
h is got using Pythogoras theorem. Volume =  Base area  height
3
VM 2  MQ 2  VQ 2 But Base area = l  w
VM 2  52  13.62 1
Volume =  l  w  height
VM 2  13.62  52 3
VM  12.648 cm 1
=  10  8  12
1 3
 A=  5  12.648
2  320 cm 3
A= 31.62 cm 2  Volume of pyramid is 320 cm3
 Area of triangle A is 31.62 Example 9
Area of triangle B is also 31.62 The diagram is a right pyramid with a rectangular
base ABCD. N is the midpoint of AD. O is the
Area of VPQ = area of A + area of B center of ABCD. Given that AD = 12 cm, AB = 18
= 31.62 + 31.62 cm and height VO = 15 cm, calculate the;
i) Length of VN and VC
 63.24 cm2
ii) Angle between VB and the base ABCD
 Area of VPQ is 63.24 cm 2 iii) Angle between VBC and the base
Solution:
Method II i) Length of VN and VC
We can use Heron’s formula which states that Extracting triangle VON
A  s( s  a)( s  b)( s  c) ,

218
A 18 cm B AC 2  AD 2  DC 2
12 cm AC 2  122  182
N O K
AC  468
D C
AC  21.633 cm
1
15 cm AO  AC
2
1
  21.633
2
AO  10.817 cm
But AO  OC  10.817 cm
V Using Pythogoras theorem
N O VC 2  VO 2  OC 2
VC 2  152  10.8172
15 cm VC  152  10.8172
VC  18.493 cm
 Length VC  18.493 cm
V
ii) Angle between VB and the base ABCD
1
Note that NO  NK A B
2
O θ
But NK  AB  18 cm
D C
1
 NO  18  9 cm
2 15 cm 18.493 cm
Using Pythogoras Theorem
NO 2  OV 2  NV 2
92  152  NV 2
NV  92  152 V
NV  17.493 cm
Note that VB  VC  18.493 cm
 Length VN = 17.493 cm
Note that AC  DB  21.633 cm from the
A 18 cm B
working above.
12 cm O 1
 OB  DB
D C 2
C 1
15 cm
  21.633
2
 10.817 cm
Let the angle be θ
15
 tan  
10.817
V 15
  tan -1
Length VC 10.817
Finding AC   54.2 0

Alternatively;

219
OV a) Construct a well labeled pyramid using the net
sin   above
BV
b) Find;
15
sin   i) Angle AEC
18.493
ii) The length EO and AG
15
  sin -1
iii) The angle which each of the slanting planes
18.493 makes with the base
  54.20 as before Solution:
iii) Angle between VBC and the base E
Let the angle be θ (a)
A 18 cm B
OO 9 cm 6 cm
θ
D C G

D
C
15 cm
6 cm
O
A 8 cm B
b) (i) angle AEC
V E

15
tan  
9
6 cm
15
  tan -1 6 cm
9
D
  59.040 C
 angle between VBC and the base ABCD is 59.0 . o
6 cm
A 8 cm B
Example 10
The diagram below is a net of pyramid with ABCD Let the angle be θ
as the base. Getting AC using Pythogoras Theorem
E AB 2  BC 2  AC 2
6 cm 82  62  AC 2

D C AC  100
AC  10 cm
E O 6 cm E Using the cosine rule;
e2  a 2  c 2  2ac cos
8 cm
A B 102  62  62  2  6  6 cos
100  (36  36)  2  36 cos
28
E  cos
72
AB=8cm,BC=6 m, AE  DE  BE  CE  6 cm .
28
O is the point of intersection of the diagonals of the   cos 1
72
rectangle. G is a point on EO such that
2   112.9 0

OG  OE .  Angle AEC = 112.90


3

220
i) The length EO and AG ii) The angle which each of the slanting planes
E makes with the base
Note that planes EAD and EBC make the same angle
with the plane ABCD (the base) and plane EAB and
EDC make the same angle with the plane ABCD
(the base). So we choose one plane from each case
6 cm as a representative
G
Now taking plane EBC and the base ABCD;
D C E

3.317 cm
5 cm 6 cm
O 6 cm
G
A 8 cm B
D C
Length EO
Note that AC = DB = 10 cm θ
O 4 cm
1 1 A
 BO  DB   10  5 cm 8 cm B
2 2 Let the angle be θ
Now using Pythogoras Theorem 3.317
EO 2  OB 2  BE 2 tan  
4
EO 2  52  62 3.317
  tan -1
EO 2  62  52 4
EO  11   39.667 0
39.7 0
 angle between plan EBC and the base ABCD is
EO  3.317 cm (3 dp) 39.7o
Length AG Angle between plane DEC and the base ABCD;
2 E
From OG  OE
3
2
 OG   3.317cm
3
 OG  2.211cm 3.317 cm
E
θ Z

3 cm
G O 6 cm
D C
8 cm
6 cm
O EO
A 8 cm B tan  
OZ
Since AC = 10 cm 3.317
1 1 tan  
AO  AC   10  5cm 3
2 2 3.317
Using Pythogoras Theorem   tan -1   47.90
3
AG 2  AO 2  OG 2  angle between plane EC and the base ABCD is
AG  5  2.211
2 2 2 47.9o
Example 11
AG  52  2.2112 A ring of radius 8 cm hangs from three strings of
length 20 cm each. The strings are tied at equally
AG  5.467 cm
spaced points round the circumference. The other
 Length AG  5.467 cm ends of the strings are joined together at the ceiling
at point C. Find;

221
a) The depth of the string below the ceiling C
b) The angle between each string and the
horizontal
c) If O is the center of the ring, M is a point on the
depth of the ring from the ceiling such that

18.33 cm
CM : MO  2: 3 . Given that A is a point on the 20 cm
ring, find the angle between line MA and the
base (horizontal)
Solution
a) We first sketch the diagram to summarize the θ θ
information in the question.
8 cm 8 cm
C Oθ

b) angle between each string and the horizontal


Let the angle be θ and using any of the strings;
8
Using cos  
M 20 20
cm 8
  cos -1
20
  66.40
8 cm
O  angle between any string
A and the horizontal is 66.40 (1 dp).
The depth of the string has to be at a right angle with c) calculating distance MO
the horizontal (we assume that the ring is perfectly From CM : MO  2: 3
horizontal). 3
Let one of the points of contact of the strings with MO  of CO
5
the ring be R such that we can be able to extract the
3
triangle below; MO  18.33
C 5
MO  10.998 cm

20 cm

10.998 cm
O
8 cm
R α
A 8 cm O
You notice that the depth of the ring below the
ceiling is CO.
 CO 2  OM 2  CM 2 Let the angle be α
 CO  8  20
2 2 2
tan  
10.998
 CO 2  202  82 8
10.998
 CO  336   tan -1
8
 CO  18.33 cm   53.96 0
540
 The depth of the ring below the ceiling is 18.33cm
 angle between MA and the horizontal is 540 .

Example 12

222
The diagram below is a regular octahedron, Note that this question is the same as “find the angle
ABCDEF, of side 20 cm. the midpoint of AD is M. between plane EAD and plane ABCD”
find; E
a) The length of EM
b) The angle between EM and ABCD
c) The angle between EAD and FAD
d) The angle between EA and ABCD
Solution
E

17.321 cm

20 cm

θ
O 20 cm
10 cm
B C

M
20 cm
Let the angle be θ
10
cos 
20 cm D
A
M
17.321
10
  cos-1
17.321
  54.70
F
 The angle between line EM and ABCD is 54.7o.
a) Length EM c) angle between EAD and FAD

20 cm 20 cm

A D
10 cm M 10 cm

Using Pythagoras’ Theorem


EA2  AM 2  EM 2
202  102  EM 2
EM 2  202  102
EM  202  102
EM  300
EM  17.321 cm (3 dp)
 Length EM  17.321 cm

b) angle between EM and ABCD

223
E AC 2  AD 2  DC 2
AC 2  202  202
AC  800
AC  28.284 cm
 AO  14.142 cm
Still Using Pythogoras theorem
AE 2  AO 2  OE 2
202  14.1422  OE 2
B C OE 2  202  14.1422
OE  200
20 cm
O OE  14.142 cm
 The height OE  14.142 cm
θ  length EF  14.142  2
A D
M  28.284 cm
Now going back to triangle EMF
E

17.321 cm

M θ 28.284 cm
F
Let
the angle between the two planes be  , Let O be
the centre of the octahedron 17.321 cm
 EO = OF
We have to get EO
Using triangle EAO; F
E Using the cosine rule;
m2  e2  f 2  2ef cos
28.282  17.3212  17.3212  2 17.32117.321 cos
100  (36  36)  2  36 cos
199.72
20 cm  cos
2 17.3212
B C   109.40
 angle between EAD and FAD is 109.4o.
d) angle between EA and ABCD
20 cm
O

A 20 cm D
Using Pythagoras’ theorem

224
E From the working above AO = 14.142 cm
14.142
cos 
20
cos   0.7071
20 cm   450
 . angle between EA and ABCD = 45o
B C

20 cm
O
θ 14.142 cm
A 20 cm D

225
ANGLES
You are likely to meet the following angles in
construction:
i) 300
- Acute angles: these are less than 900, that is they From AA + OA = 1800
lie between 00 and 900
- Obtuse angles: these are angles that lie between 300 + OA = 1800
90 and 1800
- Reflex angles: these are angles that are greater OA = 1800 – 300 = 1500
than 1800
90°  The corresponding obtuse angle for 300 is 1500

OBTUSE ACUTE
ANGLES ANGLES
180° 0° ii) 600
From AA + OA = 1800
REFLEX
ANGLES 600 + OA = 1800

270° OA = 1800 – 600

OA = 1200
Note:
 The corresponding obtuse angle for 600 is 1200
 To draw a full circle, it means that you have
moved through 3600 from the 00 mark.
 To draw a straight line, it means that you have
moved through 1800 from the 00 mark. iii) 500
 To construct an angle of 900, it means that you From AA + OA = 1800
have moved through an angle of 900 from the 00
mark. 500 + OA = 1800
 Construction of one acute angle automatically
gives you the corresponding obtuse angle. OA = 1800 – 500
 Construction of one obtuse angle automatically
gives you the corresponding reflex angle OA = 1300

 The corresponding obtuse angle for 500 is 1300


Please note that, for angles less than 3600

- Acute angle (AA) + Corresponding obtuse angle


(OA) gives us 1800 Likewise, if you are given the following obtuse
That is AA + OA = 1800 angles, 1300, 1600, and you are required to obtain the
acute angle, you proceed this way.
- Obtuse angle (OA) + Corresponding reflex angle
(RA) which is less than 2700 gives us 3600 i) 1300
OA + RA = 3600, where 1800 < RA < 2700 From AA + OA = 1800

For example, if you are asked to find the AA + 1300= 1800


corresponding obtuse angle for the following acute
angles; 300, 600, 500, you proceed this way. AA = 1800 – 1300 = 500

226
 The corresponding acute angle for 1300 is 500  The corresponding acute angle is 300

ii) 1600 So if you are asked to construct 3300, you get the
From AA + OA = 1800 corresponding acute angle, construct it and the
remaining angle will be 3300.
AA + 1600 = 1800

AA = 1800 – 1600 = 200


Bisection of angles
 The corresponding acute angle for 1600 is 200
Bisection means dividing a given angle into two
Note:
equal parts.
Whenever you are asked to construct any obtuse
angle, just calculate the corresponding acute angle
and construct it, the remaining bigger angle on the The procedure for bisection;
straight line will be the required obtuse angle.
- Let us consider angle AOB in the diagram below
A

The same principle applies for reflex angles. For


example, if you are asked to get the corresponding O
obtuse angles for the following reflex angles, 2100,
3300, you proceed like this;

i) 2100
From obtuse angle + reflex angle = 360°;
B
0
OA + RA = 360
- Using a pair of compasses with a pencil, place
0 0 the pointer at point O to draw arcs on line OA
OA + 210 = 360
and OB at points C and D
OA = 3600 – 2100 = 1500

 The corresponding obtuse angle for 2100 is 1500


So if you are asked to construct 2100, construct 1500
and the remainder will be 2100

ii) 3300
Since for angles greater than 2700, the formula OA +
RA = 3600 cannot give us a corresponding obtuse - Shift the pointer to point C to draw an arc in
angle, we use AA + RA = 3600 to get the front of angle AOB. Shift the pointer to point D
to draw an arc to intersect the first one at point E.
corresponding acute angle.

NB: Angles greater than 2700 give us the


corresponding acute angle.

i.e. AA + RA = 3600

AA + 3300 = 3600

AA = 3600 – 3300 = 300

227
By doing this, we have bisected  AOB in to 
AOE and  EOB, and  AOE =  EOB.
Let us construct the following angles.

1. 3600
How to draw a perpendicular bisector - Just draw a circle for this angle

1. Drawing a perpendicular bisector using the same


line;
- See the procedure for drawing an angle of
900 in example (3) on the next page
2. Drawing a perpendicular bisector on a line from
a point not on the line;
- Let us consider line AB and point O in the
diagram below.

- Place the compass pointer at point O and


estimate the midpoint of line AB and extend
the part of compass carrying the pencil to
slightly go past this point (the estimated
midpoint)
- Then draw two arcs that cut through line AB
on both sides of the estimated midpoint at
points C and D 2. 180°
- Shift the pointer to points C and D - Just draw a single straight line for this angle
respectively and draw arcs on the side where
O is located. Label the intersection point E.
then draw a line to pass through O and E to 3. 90°
- Here we bisect 180° to get 90°
meet the line. The line will cut line AB at 900
- Draw a straight line and choose one point on
(at a right angle)
it and call it O
Some angles are obtained by simply equally dividing - Adjust your compass to about 2 cm, place
some angles. This is called bisecting an example; the compass pointer at point O and draw two
arcs to meet the straight line at points A and
- 900 is obtained by dividing 1800 B.
- 300 is obtained by dividing 600 - Now adjust the compass to a distance greater
- 450 is obtained by dividing 900 than 2cm, place the pointer at point A and
- 600 can be obtained by dividing 1200 or it can be draw two arcs one at either side of the line.
constructed independently. - Without adjusting the compass again, shift
the pointer to point B, draw two arcs as for A
such that the arcs drawn from point A meet
We can now sharpen our pencils, prepare our pair of those from point B at point C and D.
compasses and rulers and start to construct. Do not - Then join points C, O and D
forget your protractors for proving our angles.

228
- Adjust the pair of compasses to a length
of about 3 cm
- Place the compass pointer at point O and
draw two arcs, one above point O and
the other to cut through line AB at point
C
- Without adjusting your compass, place
the pointer at point C and draw an arc to
intersect the arc that was drawn above
point O

 COB =  COA =  AOD =  BOD = 900

4. 45°
- Following the instructions in (3) above, we
draw 90° and then bisect (divide) it to get  BOC = 600
45°.
- After drawing 90°, place the compass pointer 6. 30°
at point O and extend the part of compass - We obtain an angle of 300 by bisecting
with the pencil to draw arcs at points E and F an angle of 600.
as seen in the diagram below without - Following the steps above, we construct
adjusting the compass. an angle of 600.
- Now place the pointer at point E and then to - Following the principles of bisection, we
F to draw two arcs meeting at point G bisect it to obtain an angle of 300.
without adjusting the compass. Then join G
and O.
- Note that since 900 is not the required
angle, we draw the line forming it in
dotted form

- Note that 600 is not the required angle,


so the line forming it has to be in dotted
lines.
- 300 is the required angle and so the line
forming it has to be bold.

7. 15°
- We construct 300 and bisect it to get 150.
 GOF =  GOC = 450 Following the step above, we can obtain
150.
5. 60°
- Draw a straight line AB
- Identify a point O on the line.

229
10. 75°
Note that this is an acute angle but cannot be directly
constructed so;

- We construct 600 and we add 150 to it OR


- We construct 900 and subtract (cut away) 150
from it

Procedure

 DOE =  EOB = 150 - In the same plane and on the same paper, we
construct both 900 and 600 on the same side
8. 120° of the paper
- 1200 is an obtuse angle, to construct it, we
first construct the corresponding acute angle
and then we take the remainder to be 1200.
From AA + OA = 1800

AA + 1200 = 1800

AA = 600

So we construct 600.

11. 165°
The corresponding acute angle for 165° is
 COA = 1200.
(180° – 165°) = 15°. (Recall AA + OA = 1800)
9. 150°
- Here the corresponding acute angle is So we construct 150 and we take the remaining angle
1800 – 1500 = 300. So we construct an on the same straight line to be 165°
angle of 300and we take the remaining
obtuse angle as 1500.

The different types of figures found in construction

230
1. Triangles - All sides are equal
There are several types of triangles and these - Every angle in a square is a right angle
include Having known the different figures, we can now go
on to construct some of them.
a) Equilateral triangles
These have all sides and angles equal. Every
angle in an equilateral triangle is 600.
Example 1
b) Isosceles triangles
These have only two sides equal. The base Construct an equilateral triangle ABC of side 6cm.
angles are also equal

c) Scalene triangles
In these types of triangles, the angles and the Solution
sides are not equal, i.e. nothing on the triangle
resembles the other. You have to draw a sketch first and as you draw
remember that every angle in an equilateral triangle
is 600.

2. Parallelograms
These are geometric figures which are formed by
parallel lines.

Remember that parallel lines have the same gradient.

All properties that act on parallel lines also act on a


parallelogram. In a parallelogram all sides are not
equal.
Now follow the following steps.
3. Rhombus
This is almost a parallelogram but here all sides are - Draw a straight line in the middle of your page
equal. Actually, it is a transformed square so as to - Using your ruler measure off 6 cm from the line
make opposite sides parallel. and label the demarcations A and B as seen in
the sketch above
Note that in a rhombus: - Now get your compass, adjust it to 6 cm
- Without adjusting your compass, place the
- Opposite angles are equal needle at point A and draw an arc above line AB
- The diagonals bisect each other at right angles. - Then shift the pointer without adjusting your
4. Kite compass to point B and draw another arc above
- A four sided figure with adjacent sides equal line AB such that the new arc meets the old arc
- Two of its angles are equal - Label the point of intersection C
- Diagonals bisect each other at right angles - Now join A to C and B to C
5. Trapezium - You can measure angle at A, B and C, are they
- Has two of its sides parallel but not equal. 600 each? If no, try again until they are 600 each.
- A trapezium can have all its sides and angles Accurate diagram
different.
6. Circles
- A figure formed by one complete curve
- Has no corners
7. Rectangles
- Two opposite sides are equal
- All angles in a rectangle are right angles
8. Squares

231
After drawing your sketch;

- Draw line AB in the middle of your paper such


hat AB = 7 cm.
- Construct an angle of 450 at point A following
the procedure in the previous examples. Then on
Example 2
the line forming 450, measure off 7 cm and call
the point where 7 cm end from A point D.
Construct a quadrilateral ABCD where AB =
- Construct an angle of 1350 at point B (note that
BC=CD=DA=7 cm and  ABC=  ADC = 1350 and we construct 450 at point B and then take the
 DAB =  DCB = 450. remainder as 1350, since 450 is the corresponding
acute angle of 1350. remember the formula AA +
OA = 1800). Then on the line forming 1350,
measure off 7 cm from B and where the 7 cm
Solution stop, call that point C.
- Then join point D and C
Sketch Note:

- Remember to use dotted lines where applicable


- Try and measure angles ADC and DCB, aren’t
they 1350 and 450 respectively?

Example 3 Construct a square PQRS of side 8 cm.

232
Solution Construct triangle PQR such that PQ = 7 cm and PR
= 9 cm and angle QPR = 120°. Measure angle PQR
We know that each of the angles in a square is 90°. and QRP and length QR.

Sketch Solution

Sketch

Accurate diagram

- Draw line PQ such that PQ = 8 cm


- Construct an angle of 90° at P and measure off 8
Accurate diagram
cm from P and where 8 cm stops on that line,
call that point S
- Draw line PQ such that PQ = 7 cm
- Do the same at Q as for P
- Construct an angle of 1200 at point P. Remember
- Then join points S and R to get your square
to first construct the corresponding acute angle
(600) at P in order to give the remaining obtuse
angle as 1200
- Then measure off 9 cm from point P along the
line forming 1200 and name the point where the
9 cm end R
- Then join R to Q.

Note that you don’t have to construct all the angles,


Length QR = 13.9 cm
for example, in the figure above, the right
angles at S and R come out automatically if (Always use a divider when you are measuring.
angles at P and Q were drawn properly. Avoid putting the ruler directly on the line)

 QRP = 260
Example 4  PQR = 340
Circumscribing and inscribing

233
In simple terms, draw two arcs, one below and one above B such
that the new arcs meet the old arcs
- Circumscribing means that you construct a circle  Call the points of intersection of the arcs D and
outside a given figure, usually a triangle such E, and then join D and E.
that it touches all the corners (vertices) of that - Do the same for line BC as for line AB
given figure. - You can do the same for line AC if you wish
- Inscribing: here the circle is constructed inside - You note that the perpendicular bisectors
the figure such that it touches the lines forming intersect each other. Call this point O.
the figure but not the vertices (corners) - Now place your pointer at point O and adjust
Note that the above two terms are always applied on your compass such that the pencil sits at any of
triangles. the corners of the triangle and then rotate your
compass to draw a complete circle
- Does the circle touch all the three corners of the
triangle? If yes, congratulations, if no try again.
How to circumscribe

Let us consider triangle ABC below

The radius of this circle is 3.2 cm.

How to inscribe:

Let us consider triangle ABC below


- We draw perpendicular bisector to all the lines
using the very lines not from a point. C
Note: only draw a perpendicular bisector from a
point to a line when if asked to do so.
8cm 7cm
- Let us consider lines AB and BC
- For line AB;
 Place the compass pointer at point A and adjust A B
your compass such that the pencil goes beyond
halfway line AB. Then draw two arcs one above - Here instead of bisecting lines, we bisect angles
line AB and the other below AB. - The point of intersection of the bisecting lines is
 Without adjusting your compass, place the the point where we place our compass pointer to
pointer at point B, adjust your compass such draw an inscribing circle
that the pencil goes beyond halfway AB and - Following the principles of bisection of angles
discussed earlier, bisect any two angles in the

234
above triangle. You can even bisect all the angles In such a figure, you may be asked to find the area
if you have enough time. between the two circles drawn.
- Label the point of intersection of the bisecting If asked,
line O. -First measure the radii of the circles
- Adjust your compass with your pointer at O such  Radius for small circle is 3 cm
that the pencil estimates any of the lines forming  Radius for big circle is 4.2 cm
the triangle and draw a complete circle. Area between the two circles
 Area of   Area of 
  
 big circle   small circle 
  R 2big   r 2 small

   4.22    32
  (8.64)

Taking  to be 3.142
= 3.142 × 8.64
= 27.147 cm2
2
Hence area between the two circles is 27.147 cm
The radius of the circle is 1.8 cm. Now we can handle questions which involve several
aspects of construction
Example
Using a pair of compasses and ruler only,
Example a) Construct triangle PQR such that QR =10.6cm
and angle PRQ = 750 and
Circumscribe and inscribe a square PQRS of side 6 PQR = 600
cm on the same diagram b) Construct a circumcircle of triangle PQR with O
as its center
Solution
c) Measure lengths PQ and PR and the radius of
- The same principle of bisection of lines and the circle.
angles still holds as for triangles. Find the area of the circle and the area of triangle
- Remember to first construct the square of side 6 PQR
cm
Solution
Please note that the question stretches the use of only
a pair of compasses and ruler. Only use a protractor
to measure the angle you have constructed
Sketch

235
1
Area of the triangle PQR = ab sin 
2
1
  10.6 14.7 sin 60
2
= 67.47 cm2
Example 2
Using a pair of compasses and a ruler only,
i) Construct triangle PQR, such that angle PQR =
600, QR = 9.0 cm, PR = 8.5 cm. measure length
QP
ii) Bisect the sides PQ and PR . Produce the line
bisectors to intersect at point M.
iii) Using M as the center, draw a circle to
circumscribe triangle PQR. Measure the radius
of the circle; hence calculate the area of the
- Recall how to construct angles 600 and 750 from
circle.
the previous examples
Solution
- Also recall how to circumscribe
c) PQ = 14.7 cm Sketch
PR = 13.0 cm
Radius = 7.5 cm.
Area of circle = πr2
22
  7.52
7
= 176.785 cm2
Accurate  25  3.142
 78.55 cm2
Example 3
Using a ruler, pencil and pair of compasses only
a) Construct a triangle ABC, where angle ABC =
1050, BC = 9.2 cm and AC = 12.0 cm. measure
and state length AB and angle BAC.
b) Draw a circle circumscribing triangle ABC, state
the radius of the circle.
Solution
Sketch

ii) Note that this part gives a clue on how to


circumscribe this triangle
iii) Radius = 5 cm
Area of the circle   r
2

   52
 25
Taking  to be 3.142
236
Accurate diagram

- Recall how to draw a perpendicular bisector to a line


from a point.

Accurate diagram
To construct an angle of 1050, we construct
(900 + 150), i.e. we construct an angle of 900 and then
we construct 150 to get 1050.

CD = 5cm
iii) Radius = 4.1cm
iv) Area of triangle ABC
 Area of   Area of 
  + 
 ADC   CDB 
1
Area of triangle ADC =  AD  CD
2
1
Area of triangle CDB =  DB  CD
2
AD = 5cm
Radius = 6.3 cm
DB = 3cm
Example 4 1 1
=  AD  CD   DB  CD
2 2
Using a ruler, pencil and pair of compasses only, 1 1
=  5 5   3 5
2 2
i) Construct triangle ABC such that AB = 8cm, 25 15
∠ABC = 60° and ∠BAC = 45°.    20 cm 2
2 2
ii) Construct a perpendicular from C onto AB to
meet it at D. Measure the length CD.  Area of triangle ABC = 20cm2
iii) Draw a circle circumscribing triangle ABC.
Measure its radius. Question 5
iv) Find the area of triangle ABC. Construct triangles PQR and PQS on the same
Solution diagram such that ∠RPQ = 60°, ∠PQR = 45°, PQ =
Sketch
8cm, ∠QPS = 45° and PQS = 60 . Circumscribe
0
C
triangle PQR and call the center of the circumcircle O.
join O to P and O to Q. Measure this OQ. This is the
radius of the circle. Shade the area between PQ and
the arc of the circumcircle on the side of triangle PQS.
45° 60°
A 8cm B

237
Draw a perpendicular bisector from O to meet the line But we don’t know θ, we use the cosine rule to get 
P. Find the area of the shaded part. i.e.
Solution a 2  b2  c 2  2bc cos A
This question is wordy, however a nice sketch and
being organized can make everything easy. 82  4.12  4.12  2  4.1 4.1cos 
82  4.12  4.12  2  4.12 cos 
Sketch
30.38  -33.62 cos 
30.38
cos  
33.62
  154.60
154.6 1
  3.142  4.12   8  h
360 2

Still we can get h through calculation

Accurate diagram

h 2  (4.1) 2  (4) 2
h  (4.1) 2  (4) 2
h  0.81
h  0.9cm

154.6 1
  3.142  4.12   8  0.9
360 2
Radius = 4.1cm  22.682  3.60
Note that radius  PQ = OQ = 4.1cm
= 19.08 cm2
Area of shaded part:
 area of shaded part = 19.08 cm2
Alternatively; we can get all the measurements by
directly measuring. However, some degree of error
has to be expected.
Height, h = 1.1cm
POQ  1500
Area of shaded part
Radius = 4.1cm
 area of   area of 
   Hence using our formula
 whole figure   OPQ   1
=  r 2  bh
 1 360 2
=  r 2  bh
360 2

238
150 1
  3.142  4.12   8 1.1
360 2
 17.6
Area of shaded part = 17.6 cm2

You note that the two answers vary by some error.


This is expected because there is a varying degree of
accuracy in construction.

239
BEARING (6) Never attempt any question on bearing
without a protractor, a ruler, squared paper
Most students always ask themselves about the and a pencil.
relevance of bearing in the day to day life not (7) Neatness may earn you a bonus mark.
knowing that their journeys to schools are all about (8) Know how to calculate speed, distance and
bearing. By the end of this topic, you will be able to time.
know why some students arrive late at school while The formula distance = Speed x Time
others arrive early at school. i.e. D = S x T has to be mastered.
(9) Last but not least, always pray for God’s
A student who knows bearing will always have the guidance as you interpret the many words that
best short cuts to school! some times constitute bearing numbers.
(10) Read the question at least two times before
As a rule, to be successful in any task, you must have starting to atempt any bearing number.
rules to guide you through any given task. The
following must be understood in order to be REMEMBER:
successful in any question concerning bearing;-  A sketch diagram is enough to summarise the
(1) All drawings in bearing must be drawn on a many words in any question in a very simple
graph paper. diagram.
(2) All angles in bearing are measured starting  A well drawn sketch diagram is the only aid
from the North in a clockwise direction. to getting all the marks in bearing questions.
(3) You must know the compass and all the  A sketch diagram is drawn using free hand.
different directions described on it. You don’t have to measure.
(4) A sketch diagram is a must as any answer  A sketch diagram may score you some marks
without a sketch diagram is invalid. even if time does not allow you to draw the
(5) Be in position to convert from one metric unit accurate diagram.
to another.
ANGLES IN BEARING
Angles in bearing are expressed using three digits for example; 0300, 0400, 0700, 2250, 1800. All angles are measured
starting from the North of the compass.
See the examples below:-
0300 0400 2250

300 400

2250

1800 0900 1500

1800 1500

Note the direction of the arrow, it is in the clockwise direction and that all angles are measured from the north
direction.
It may be confusing as to which place a protractor is to be placed on the compass.
Now we will concentrate on the location of different angles on the compass to measure a specific angle in bearing.

240
Consider a circle enclosing a compass. From the diagram below, you notice that four quadrants A, B, C and D have been created.
A quadrant means a quarter of a circle.
Assuming O to be the centre of the circle each quadrant forms an angle of 900 at O.

3600

D A
2700 O 900
C B

It follows that any angle


1800 0 between 00 and 900 lies in quadrant A.
0
Any angle between 90 and 180 lies in quadrant B.
Any angle between 1800 and 2700 lies in quadrant C.
Any angle between 2700 and 3600 lies in quadrant D.

Example
Measuring from the north in the clockwise direction, locate the following angles on the compass;
(i) 300 (ii) 450 (iii) 1200
(iv) 175 0 (v) 220 0 (vi) 2800
Solution:

450
300
1200

1750

2200

2800

Considering the diagram below. If you start your journey from P, you will go through the following bearings at
different points marked by the letters.

S 241
Q

R
At P, the bearing is 0000
At Q, the bearing is 0900
At R, the bearing is 1800
At S, the bearing is 2700
Remember the three digit rule in bearing.
Assuming that you are required to move along a football field mentioning the bearings indicated by different letters at
different points. Start
O
L K

300
60
0
400

35
0

N M

At O, the bearing is 0000


At K, the bearing is 0300
At M, the bearing is 900 + 400 = 1300
At N, the bearing is 900 + 900 + 350 = 2150
At L, the bearing is 90 + 90 + 90+ 60 = 3500
THE COMPASS AND BEARING
You can not talk about bearing without a compass. Pilots are able to move in air and sailors are able to move on water
because they use these two components.
You will encounter statements in questions on bearing like “he moved 300km South East” She move 40km to the
north” and other statements.
Many students tend to confuse where to put East and West on the compass. To cram this in a simple way, always
make sure that W (west) and E (East) form the word “WE”.

N
NW NE
E
242
450 450
450 450
W 450 450 E
450 450
This means that a person who moves in the different directions, moves through the following bearings illustrated in the
table below:
Direction Bearing
N 0000
NE 0450
E 450 + 450 = 0900
SE 450 + 450 + 450 = 1350
S 450 + 450 + 450 + 450 = 1800
SW 450+450+450+450+450=2250
W 450+450+450+450+450+450=2700
NW 450+450+450+450+450+450+450 =3150

You may also encounter statements like N200E, N400W, S400E and so many others. These may mean the following:-
N200 E: Means starting from the north, you move 200 in the Eastern direction
N400W: Means starting from the north, you move 400 in the Western direction.
S400E: Means starting from the South, you move 400 in the eastern direction. P

See the diagrams below:- K


a) N200E
L
300 L
700
200 200
The bearing here is 0200 U 0 M
10 300
150
300 200 Y
T N
b) N400 W
The bearing here is 3600 – 400 = 3200 R
S Q

400

243
c) S 400 E

The bearing here is 1800 - 400 = 1400

400

NOTE:
You may encounter questions like express S400E, N500E, N200W in three digit form. Such questions require you to
give the corresponding bearings.

Exercise
In the diagram below, give the bearings of the following points if OP is the starting point.

b) Using the form N  W, N  E, S  E and S  W where  is the angle, describe the above bearings for the different
points.

244
METRIC CONVERSIONS IN BEARING
It is not possible to have papers which can accommodate the so many kilometres always described in bearing
questions.
What is done is to give scales which can be used to convert the big units to smaller units like centimeters.
Example:
(1) Using a scale of 1cm to represent 50km, convert the following into centimeters.
a) 200km b) 2500km c) 300km
Solution:
a) If 50km are represented 1cm;
1
- 1km is represented by
50 cm
- 200km are represented by 1  200  4cm
50
b) If 50km are represented by 1cm
2500km are represented by 1  2500  50cm
50
c) 1km is represented by 1
cm
50
300km are represented by 1  300  6cm
50

2. If 100km are represented by 2cm, convert the following centimeters


i) 600km ii) 200km iii) 800km

Solution
(i) If 100km are represnted by 2cm
2 1
- 1km is represented by  cm
100 50
1
- 600km are represented by  600  12 cm
50
1
(ii) 1km is represented by cm
50
1
200km are represented by  200  4cm
50
1
(iii) 1km is represented by cm
50
1
800km are represented by  800  16 cm
50
Exercise
Using a scale of 1cm to represent 100m convert the following to cm.
(i) 200m (v) 500cm
(ii) 600m (vi) 6000m
(iii) 800m (vii) 300m
(iv) 90m (viii) 400m

245
Other Principles of angles Used in bearing
B D



I  J
F


G 
 H
E

A C

(a) If line AB is parallel to line CD, it follows that BEF = CFE = 


(b) If GH is parallel to IJ, it follows that
HEF = EFI = 
(c) If BA is perpendicular to GH, then
 +  =900

Some important Phases Used in bearing


(1) “FROM”
This phrase has to be used correctly because it rarely misses in bearing questions.

Examples:
What is the bearing of P from Q in the figures below?
a)

Q
0
30

Solution:
246
P from Q means that measurement of the angle is done at point Q starting from the north of Q up to the LINE joining P
and Q.

Q
300
300
P

The bearing is = 900 + 900 + 300


= 2100

b)

200
Q

Let the bearing by y


y + 20 = 90
y = 90 – 20
y = 700
Bearing of p from Q is 700
c)

1000
P

Q
1000

= 900 + 900 + 1000 = 2800

d)
247
P
200
0
20
Q
P from Q is 0200.

e) P

Q
You notice that Q and P share the same line so we don’t move through any angle from the north at Q. So the bearing
of P from Q is 0000.

Q
f)

Bearing of P from Q is 1800.


Let us recall how to calculate speed, distance and time.
Distance = Speed x time
Speed = Distance
Time
Time = Distance
Speed
Example I:
(1) Tom moves at 240kmh-1 from point A to point B at a bearing of 0200.
Calculate:
(i) Distance travelled if he used 2 hours to travel
(ii) Bearing of A from B

Solution:

B
200
200
248
A
(i) Distance = Speed x Time
= 240 x 2
= 480km

(ii) Bearing of A from B


Note that measurements are done at B from this question.
= 180 + 20 = 2000

(2) If Tom travelled 600km at the same speed, Calculate the time he spent travelling:
Time = Distance
Speed

= 600 = 2.5hours
240

Having handled the basics in bearing now we consider, the different applications of bearing.

Start the next section with the following at hand:-


 A ruler
 A pencil
 A protractor
 Concentration
 All the concepts discussed so far at hand.

Example II:
1. Sharon travels from home to a trading centre at a bearing of 030 0, a distance of 300km. From the trading centre,
she then travels 600km to the school which is at a bearing of 1300 from the trading centre. Assuming that she travels at
a constant speed of 20km / h and that 1cm represents 50km.
a) Draw a sketch to show the journey to school
b) Draw an accurate diagram for the journey
c) What is the bearing of:
i) Home from school
ii) School from home
d) Measure the distance between home and school
e) Calculate the total time spent on her journey
f) If she uses the shortest route to school, calculate the time she takes if she moves at the same speed.

Solution
a) Sketch

Trading centre

1300
300km
249
(6cm)
600km
300
(12cm)

Home
Converting km to cm
1cm represents 50km
⇒ 50km are represented by 1cm
1
⇒ 1km is represented by cm
50
1
1km is represented by cm
50
1
600km are represented by  600  12cm
50
b)

Trading centre

1030

6cm

(12cm)
300

Home
School

250
From the accurate diagram
c) (i) Bearing of home from school is 2830
ii) Bearing of school from home is 1030
d) Distance between home and school is 12.1cm
1cm represents 50km
12.1km represent 12.1 x 50 = 605km

e) Time = Distance
Speed
Since the speed is constant, we get the total distance and divide it by the speed.
300  600
Time =  45Hrs
20
f) From part d above, shortest route between home and school is 605km
from Time = distance
speed
= 605 = 30.25hours.
20
If she uses the shortcut (shortest route) to school, she uses 30.25 hours.

Bearing of home from school


= 900 + 900 + 1030 = 2830
Bearing of school from home
= 1030

Example III
Oketch has to travel from town A to town D but because of a very big island between towns A and D, he travels from
town A at a bearing of 1500 a distance of 200km to town B. From town B, he moves at a bearing of 090 0 a distance
of 300km to town C. From C he moves to town D which is 400km from C at a bearing of 020 0. Using a scale of 1cm
to represent 45km.
a) Draw a diagram to show his journey.
b) Measure
i) Distance of A from D
ii) Bearing of D from A
c) If he took a total time of five hours to travel and assuming that he travelled at a constant speed. Calculate his speed.
Solution:
a) Sketch

D
0
A 150

(8.9cm)
400km
200km
(4.4cm)
200
300km
(6.7cm) 251
B C
Converting km to cm
⇒ 45km are represented by 1cm
1
200km are represented by cm  200  4.4cm
45
1
⇒ 1km is represented by cm
45
1
300km are represented by cm  300  8.9cm
45

A 1500
(8.9cm)

(4.4Cm)
200

B (6.7km) C

b) i) Distance of A from D is 13cm


But 1cm represents 45km
⇒ 13cm represent 13 x 45km = 585km Bearing of A from D = 1800 + 690 = 2490
Bearing of D from A = 690

c) Speed = Total distance


Total time

200  300  400


 180km / hr
5

 Constant speed is 180km/h

252
Example IV
From the airstrip at Johannesburg, an aeroplane moves at a bearing of 090 0 to Botswana a distance of 200km. It then
moves at a bearing of 0200 a distance of 300km to town A. At town A, it rests for two hours and then moves at a
distance of 1700 to Kigali which is 500km from town A.
If 1cm represents 50km
a) Draw a diagram to show the journey of the aeroplane.
b) Distance between Jonannesburg and Kigali in centimeters
c) Bearing of Johansburg from Kigali
d) Time spent, if the plane travelled at a constant speed of 80km / h.

Solution
a) 1700

300km Town A
6cm
200
200km
4cm Botswana
Johane
urg
500km
(10cm)

Converting km to cm
50km are represented by 1cm
1
1km is represented by cm Kigali
50
1
200km are represented by  200  4cm
50
1
300km are represented by  300  6cm
50
1
500km are represented by  500  10cm
50

253
Town A

6cm

200
30
6
20 0k
Johaneurg 44cm c
Botswana (10cm)
0k m
c m
m
m

Kigali

Using the accurate diagram


b) Distance between Johansburg and Kigali is 8.6cm
c) Bearing of Johansburg from Kigali is 2980
d) Time = Distance
Speed

200  300  500


 12.5hrs
80

The plane takes 12.5 + 2hours for resting giving us 14.5hours for the whole journey.
Island C
Bearing of Johansburg from Kigali
= 1800 + 1180 = 2980

Example V
30
10.8cm
A cargo ship sails from island A to island B at a bearing of 250 0 and a distance of 250km
6 from island B. It moves
0
N30 W a distance of 550km to island C. If 1cm represents 60km. 0k
c
a) Draw a diagram to illustrate the ship’s journey. 9.2cm m
m
b) Distance between island C and A in centimeters.
c) If the shop took 2hours to travel from island A to B and 3 ½ hours to travel from B to C. Calculate 250
the0 respective
speeds. 300 4.2cm
d) Bearing of
Island A
i) Island C from A
ii) Island A from C Island B
Solution:
a)

Island C
Sketch

Island A
550km 254
(9.2cm)
2500
(4.2cm)
250km
300
Converting from km to cm
60km are represented by 1cm
1
500km are represented by  550  9.2cm
60
1
250km are represented by  250  4.2cm
60

From the accurate diagram


b) Distance between Island C and A is 10.8cm
c) Respective speeds
from Island A to Island B
Speed = Distance
Time
= 250
2
= 125km/h
From island B to Island C
Speed = Distance
Time
= 550

= 157.14km/h

d) i) Bearing of Island C from Island A is 3070

255
ii) Bearing of Island A from C is 1270

Bearing of Island C from Island A


= 1800 + 1270 = 3070
Bearing of Island A from Island C = 1270

Example VI
In the last year’s motorcycle sport competitions. Motorcycles started from town P and moved at a bearing of 0600 for
a distance of 120km to town Q. They then moved in a direction described as N 800E a distance of 150km to town R.
Then they moved at a bearing of 2300 for 300km to town S. If 1cm represents 20km
a) Draw a diagram to illustrate their journey
b) Bearing of P from S.
c) If a car was to take a short cut from Q to S, what distance did it go through.
d) What is the bearing of Q from S.
R
Sketch 800 150km
Q
(7.5cm) 2300

120km
(6cm)
600 (15cm)
300km
P

Solution:

Converting from km to cm
20km are represent by 1cm
⇒ km is represented by
1
cm
20
120km are represented by 6cm
150km are represented by 7.5cm

300km are represented by = 15cm

256
R

Q 800 7.5cm 2300


30
6cm
0k
600 m

P
15cm

Bearing of P from S = 1800 + 1690


= 3490
Bearing of Q from S = 270
c) Distance between Q and S is 9.3cm
But 1cm represents 20km
9.3cm represent 9.3 x 20 = 186km
d) Bearing of Q from S is 270

Example VII
Dorcus Inzikur started running from station P at a bearing of 3300 for a distance of 10km to station T. She then moved
in the Eastern direction at a speed of 5km / hr for 2 hours to junction M. She then turned through a bearing of 072 0 and
moved for 15km to station N. She then moved to station K which is 18km south of station P. If 1cm represents 2km.
a) Draw a diagram to show her journey
b) Distance and bearing of P from K
c) Distance and bearing of P from N
d) Total time spent if she was running at a speed of 2km / h

e) Distance and bearing of K from N

Sketch

257
N
15km

10km 720 (7.5cm)


T M
(5cm)

10km (5cm)

P
3300

Distance from T to M = S x T
18km (9cm)
= 5 x 2 = 10km

2km are represented by 1cm; 1km is represented by


1
cm
2
⇒ 10km is represented by 5cm
⇒ 15km is represented by 7.5cm
⇒ 18km is represented by 9cm

258
7.5cm N

(5cm) 720
T M

11.6cm

5cm 18.2cm

3300
P

9cm

b) Distance of P from K is 18km


Bearing of P from K is 0000
c) Distance of P from N is 11.6cm
But 1cm represents 2km
⇒ 11.6 represents 2 x 11.6 = 23.2km
Bearing of P from N is 2350

d) Time = Total distance


Speed
= 10 + 15 + distance KN
2
From the accurate diagram distance KN = 18.2cm
But 1cm represents 2km
18.2cm represents 18.2 x 2 = 36.4km
10  15  36.4
Time =  30.7hrs
2
259
e) Distance of K from N is 18.2cm OR
36.4km from part (d) above
Bearing of K from N is 2100

Bearing of P from K is 0000


Bearing of P from N = 1800 + 550 = 2350
Bearing of K from N = 1800 + 300 = 2100

Example VIII
Points A, B, C and D are oriented on a play ground such that point B is at a bearing of 0000 from point A, point D is
at a bearing of 3400 from point A, point C is at a bearing of 0700 from point B. If distance AD is 100m, AB is 40m
and BC is 30m and that 1cm represents 7m
a) Draw an accurate diagram to show the relationship between the points.
b) Bearing of D from B and C
Solution:
Sketch

C
(14.3cm)
700
30m
100m
B (4.3cm)

40m
(5.7cm)

3400
Conversion from m to cm
7m are represented by 1cm
1
1m is represented by cm
7
It follows that for
100m : 100  1  14.3cm
7
30m 1
: 30   4.3 cm
7
40m : 40  1  5.7 cm
7

260
D

11cm

700
4.3cm
14.3cm
B

5.7cm

A
3400

b) Distance between points D and C


from the graph paper it is 11.0cm
if 1cm represents 7km.
11.0cm represent 7 x 11 = 77km

c) Bearing of D from C is 3050


Bearing of D from B is 3290

Example IX

261
L
1500

8cm 10cm
600

K
300

7cm

400

N
The figure below shows the shape of Ankankunda farm formed by poles K, L, M and N

262
1500

8cm
600 P
K

12.5cm
M

400

If the owner wants to add a pole between L and M such that LP : PM = 1 : 3 where P is the pole point.
a) Draw an accurate diagram including point P to represent the farm.
b) Measure distance KP and PN
c) Bearing of (i) P from K
(ii) K from P
(iii) P from N
d) Distance KN and bearing of K from N.

Solution
From LP : PM = 1 : 3
1
LP =  LM
4
But LM = 10cm

263
1
LP =  10  2.5
4
PM = 10 – 2.5 = 7.5cm

b) Distance KP = 8.4cm
Distance PN = 11.9cm

c) Bearing of
i) P from K 0800
ii) K from P is 1800 + 800 = 2600
iii) P from N is 0060

d) Distance KN is 12.5cm
Bearing of K from N = 1800 + 1450=3250

264
RELATIONS AND MAPPING
A relation is a statement connecting domain and the
range. This statement is either in words or in terms of Example II
mathematical operation. It can also be defined as a set Consider a family which consists of brothers and
of ordered pairs. In everyday life. A relation can be sisters.
defined as a connection between events, items, Gladys, Harriet, John, Tom, Daniel and Sharon. Draw
numbers, things, people etc. a papygram to represent the above information with a
relation sister of
For example
Shamim is a sister to Sharifa, James is a brother of
John. Elizabeth is younger than Sophie.

The above relation can be expressed in form of ordered Shamim


Gladys
pair.
John
(Shamim, Sharifa)
(James, John) Tom
(Elizabeth, Sophie) Daniel

A relation may be expressed in three ways:-


(i) papygrams Sharon
(ii) In a mapping notation
(iii)Arrow diagram
Example III
PAPYGRAMS Draw a papygram for the set of values
Papygrams are used to represent relations whose {1, 2, 3, 4, 5, 9, 10} showing the relation is a factor.
domain and range belong to the same set. The
papygram contain a set of values forming a certain
relation in an enclosure, in most cases the enclosure is 2
a circle or any oval shape. 3
1
Example
Yawe 12 years Enock 6years
4 10
Immaculate 14 years Daniel 18 years
Represent this information on a papygram with a 5
relation is young than. 9
Solution:

Example IV
Yawe (12yrs) Enock
(6yrs) Grand father (Sempala)

Immaculate
14 yrs Daniel
18yrs Herbert
Rose

Draw a papygram showing the relations


(i)
Brenda is Daniel
a child to Joseph Alice

265
(ii) is a grandfather of Draw a papygram for the set of values
(iii)is a grandchild of {-2, -1, 0, 1, 2, 4 } representing the relation of a square
root of.
(i) Is a child to
Solution:
Sempala
-1 1
Rose -2
Herbert 0
Brenda Daniel
4

Joseph Alice 2

ARROW DIAGRAMS
Arrow diagrams show the relation between each first
(ii) Is a grand father of element and second element. It shows which items are
related.
Ssempala
For Example:
Herbert
A = {3, 4, 5} and
Rose B = {4, 12, 16, 20}
Draw an arrow diagram to show the relation is a
Brenda Daniel
Joseph Alice quarter of:

3 12

(iii) Is a grand child of


4 16
Ssempala

5 20

Rose Herbert

Domain
Brenda Daniel Range
Joseph Alice

Example III
Make a list of five girls in the S.2 class and also list the
Example V games they play. Draw arrows from the name in the

266
first list to the games in the second list to show which
games are played by each student. (b) Domain {1, 4, 5, 6, 7} x   x
(c) Domain {0, 1, 2, 4, 5} x  x2
Shamim Netball (d) Domain {15, 14, 13, 12, 11, 10}
x  2 x  1
Sharifa Football
(a) x 
 4 x  1
Carol Badminton 2 7
4 15
Dorah Basketball
6 23
Joan Volleyball 8 31
9 35
Domain Range 11 43
Example III 12 47
Express the relation (-3, -9) (-2, -6) (0, 0) (2, 6)
(i) as an arrow diagram x (Domain) Range
(ii) In a mapping notation
(b) x   x
-3 -9 1 -1

-2 -6 4 -4
5 -5
0 0
6 -6
2 6 7 -7

Domain Range x (Domain) Range

We note that the second number in the pair is three (c) x  x 2
times the first number thus the mapping relation is
0 0
x  3x where it is read as x is mapped with 3x 1 1
2 4
4 16
5 25

Example IV
Draw an arrow diagram representing the following x (Domain) Range
mapping below using the given domains.
(a) Domain {2, 4, 6, 8, 9, 11, 12}
(d) x  2 x  1
x 
 4 x  1 15 31

267
14 29 (Domain) Range
13 27 Example
Find the domain representing the ranges {15, 13,
12 25
14, 11, -12 and 17}
11 23
Solution:
10 21
x  2 x  1
x (Domain) Range
a 15
b 13
Express the following as an arrow diagram
c 14
(a) x  0.4 x where d 11
x  {0, 5 , 10, 15}
f -12
e 17
b  x  5 x  2 where
x  {1, 2 , 3, 4}

= 2a + 1 = 15
2a = 14
a =7
Solution: 2b + 1 = 13
x  0.4 x b
2c + 1 = 14
= 6

0 0 c = 6.5
2d + 1 = 11
5 2 d =5
2f + 1 = -12
10 4 f = -6.5
2e + 1 = 17
15 6 e = 16
e =8
7 15
(Domain) Range
6 13
x  5 x  2 6.5 14
1 3 5 11
2 8 -6.5 -12
3 13 8 17
4 18 Domain Range

268
Find the unknown values in the arrow diagrams for the The above mapping shows the relation is a square
mapping. root of

 2 x 1
x 
ONE TO MANY MAPPING
5 12 It’s the type of mapping where one element of the
domain is related to more than one element in the
a 16 range.
b 22 2 4
11 24
8
13 c 3
9
Solution:
2(a + 1) = 16 15
a +1 = 8
a= 7 4 20
2(b + 1) = 22 Domain Range
b+1 = 11
b = 10 MANY TO ONE MAPPING
This is a mapping where by more than one member of
2(13 + 1) = c the domain is related to one element of the range.
28 = c a
c = 28 b g
c
TYPES OF MAPPING
In the mapping notation, there is a general relation
between the 1st and 2nd set we have basically four types d
of mapping.
(i) one to one mapping
e h
(ii) one to many mapping
f
(iii)many to one mapping
(iv) many to many mapping
Domain range

ONE TO ONE MAPPING MANY TO MANY MAPPING


Is where each member of the domain is related to only This is a mapping where more than one member of the
one element of the range. domain is related to more than one member of the
range as illustrated below.
1 1 8 2
9 81
4 16
10 3
8 64
12 4
Domain Range
16 5
269
18 6

270
FUNCTIONS
As great mathematicians tried to formalize all of Domain range
mathematics using sets theory. Dirichlet defined a
function as a special case of relation whereby two sets
are related using a special rule or operation. The above relation is a function because different
According to Dirichlet a function f from set x to set y objects in the domain are mapped onto one image in
written. the range. This is known as many to one mapping.

f:x 
 y is a relation between two sets x Example III
and y which satisfies the following conditions.
(i) For each value in x there is only one value in y.
(ii) Many values in x can be related to one value in y. a x
Note:
If each member of x (domain) has one and only one y
corresponding member in y (range) then it is a
function. Therefore a function is a special type of z
relation that maps each member in domain to only one
member in the range such as one to one mapping. p
Example
b
Consider a relation below which involve pairing r
between members of one set with members of another
set.
a is related to x, y and z
1 3
b is related to y, p and r.
2 6
The above relation is not a function because each
3 9 object in the domain is mapped onto more than
one image in the range.
4 12
Example IV
5 15 a 3 x a
The above relation is a function because each object in
the domain is mapped onto only one image in the b 5 y e
range.
Example II
a x c 8 z f

A B
b y Relation A is not a function because b has not been
mapped to any value in the range and a is mapped to
more than one value. Relation B is a function because
c each member in the domain is mapped onto exactly
one member in the range.
d z Example I
4x  8
e Given that f(x) = find f(3)
x 2

271
Solution Example V
4x  8 Given that f(x) = (x – 4)2 . Find the value of x for
f(x) = which f(x) = 25.
x 2
Solution
438
f  3   4 f(x) = (x – 4)2
1 f(x) = 25
⇒ f(3) = 4 ⇒ (x – 4)2 = 25
x–4 = +5
Example II x–4 = 5
Given that f(x) = x2 + 3x + c. Find the value of c if
f(2) = 9 ⇒ x=9
Solution: x -4 = -5
f(x) = x2 + 2x + c
f(2) = 9 ⇒ x = -1
f(2) = 22 + 3 x 2 + c ⇒ x = -1 or x = 9
f(2) = 4 + 6 + c
f(2) = 10 + c Example VI
⇒ 10 + c = 9 2
c = 9 – 10 A function f(x) is defined by f(x)= 3
c = -1 x
Example III Find the value of x for which f(x) = 4
g(x) = ax2 + 5x – 3 if g(1) = 9. Find a
Solution Solution
g(x) = ax2
+ 5x – 3 f(x) = 2  3
g(1) = a(1) + 5(1) – 3 x
=a + 2 f(x) = 4
a+2 =9 2
a=7  3 4
x
2
Example IV:  43
Given that f(x) = ax2 + b, f(-2) = 3 and f(1) = -3. Find x
the values of a and b 2
 1
Solution: x
f(x) = ax2 + b 2  x
f(-2) = a(-2)2 + b
 x  2
= 4a + b
but f(-2) = 3
⇒ 4a + b = 3
Example VIII
f(x) = ax2 + b
f(1) = a(12) + b If f(x) = x2 - 3x. Find the value of x for which f(x)
but 4a + b = 3 ………………………(1) =4
Solution:
⇒ a + b = -3 ………………………(2) x 2  3x  4
eqn 1 – eqn 2
x 2  3x  4  0
⇒ 3a = 6
a = 2  4, 1
Substituting a = 2 In eqn 2 - 4
⇒ 2 + b = -3 ⇒ x2 – 4x + x – 4 = 0
b = -3 – 2 x (x – 4) + 1 (x – 4) = 0
b = -5 (x + 1) (x – 4) = 0
272
x(x – 4) = 0 or x – 4 = 0 m 1 3x

x = -1 or x = 4 1 5
 5m  1  3x
THE INVERSE OF A FUNCTION 3 5m  1
If a function f(x) has got its inverse, then the inverse x 
f(x) is denote d by f -1(x) 3 3
Example I 5m  1
x 
If g(x) = 2x – 5. Find g-1(x) and g-1(-3) 3
Solution 5 x  1
h 1
x  
Let g(x) = y 3
⇒ y = 2x – 5 54  1
h 1 4 
3
Make x the subject 54  1
y  5  2x 
3
y 5 5
x
2 5x  1 1
3  5  h 1 x   , h 4  5
g 1  3  3
2 Example IV
 1 If m(x)= 4x + 9
x + 4
Example II Find m-1(x) hence evaluate m-1 (3)
Solution
If f(x) = 3x . Find f -1(x) hence evaluate f -1(12)
4x  9
5 m x 
Solution: x4
3x 4x  9
Let y  y 
5 x4
 5 y  3x y  x  4  4x  9
5y xy  4 y  4 x  9
 x
3
xy  4 x  9  4 y
5y
 x  x  y  4  9  4 y
3
5x 9  4y
 f 1  x   x 
3 y4
5  12 9  4x
f 1 12   m 1  x  
3 x4
 20 9 43 3
m 1  3  
34 1
Example III
m 1  3  3
If h (x) = 3 x  1 . Find h-1(x) hence evaluate h-1(4) Example V
5 Find g(3) given that g-1(x) = x + 1
Solution x
3x Solution:
Let m 1
5 g-1(x) = x + 1
x
let y = x + 1
x
xy = x + 1

273
xy – x = 1
If h(x) = 3x  1 find the values of x for
x(y-1) = 1
x  3 x  40
2
1
x = which h(x) is meaningless.
y 1 Solution
⇒ g(x) = 1 h(x) = 3x + 1
x - 1 x2 + 3x - 40
for meaningless function the denominator = 0
g(3) = 1
3 -1
 x 2  3x  40  0
= 1
2 8, - 5
x  8 x  5 x  40  0
2

UNDEFINED FUNCTIONS / INFINITE


FUNCTION / MEANINGLESS FUNCTION x  x  8  5  x  8  0
A function is said to be undefined, infinite or  x  5  x  8  0
meaningless of the denominator if the function is equal
to zero. x5 x  8
Example I
x 1 COMPOSITE FUNCTIONS
Given that f(x) =  A composite function is a function of a function. It is
2x  4
a combination of two or more functions
find the value of x for which f(x) is not defined. Example I
Solution If f(x) = 2x + 3 and g(x) = 6x
If f(x) is not defined,the denominator = 0 (a) Find fg(x) and gf(x)
2x  4  0 (b) Find fg(3) and gf(3)
2x  4 Solution:
4 f(x) = 2x + 3, g(x) = 6x
x  fg(x) = f(g(x))
2 f(x) = 2x + 3
x 2 f(g(x))= 2(g(x)) + 3
Example II = 2(6x) + 3
Find the values of x for which = 12x + 3
g(x) = 5x + 6 is not defined. gf(x) = g[f(x)]
9 – x2 g(x) = 6x
Solution g[f(x) = 6[f(x)]
g(x) = 5x + 6 g[f(x) = 6[2x + 3 ]
9 - x2 = 12x + 18
g(x) is not defined fg(x) = 12x + 3
gf(x) = 12x + 18
 9  x2  0 fg(3) = 12 x 3 + 3
= 36 + 3
9  x2 = 39
x2  9 gf(x) = 12x + 18
gf(3) = 12 x 3 + 18
x2  9 = 36 + 18
x  3 = 54
Example II
x  3, x   3
If f(x) = 4x + 1 and g(x) = 3x – 2 . Find
Example III (i) fg(x)
(ii) fg(1)
274
(iii) gf(x) 1x
f  x  , g  x   5x
(iv) gf(-4) 1 x
Solution fg  x    fg  x  
f(x) = 4x + 1, g(x) = 3x – 2
1 x
fg(x) = f[g(x)] f  x 
f(x) = 4x + 1 1 x
f[g(x) = 4[g(x)] + 1 1  g  x
= 4[3x – 2] + 1 f  g  x   
1  g  x
= 12x – 8 + 1
= 12x – 7 1  5x

fg(1) = 12 x 1 – 7 1  5x
=5 1 52
fg  2  
gf(x) = g[f(x)] 15 2
g(x) = 3x – 2 11
gf(x) = 3[f(x)] – 2 
9
= 3(4x + 1) – 2 1x
= 12x + 3 – 2 g  x  5x , f  x  
1 x
= 12x + 1
gf  x   g  f  x  
gf(x) = 12x + 1 5 1  x 
= 12x – 4 + 1 
1 x
gf(-4) = -48 + 1
5  5x
= -47  gf  x  
1x
Example III 5  10
gf 1 
If f  x   1  x and g x   5x , Find: 1 2
1x 15

(i) fg(x) (ii) fg(2) 1
(ii) gf(x) (iv) gf(2)  15
Solution: gf 1  15

Example
If f(x) = 3x  2 and g x   4x
4 3
Find:
fg x  fg2 gf x  gf  2
Solution:

275
3x  2 4x 4x
f  x  g  x  g  x 
4 3 3
fg  x   f  g  x  
4
g  f  x     f  x  
f  x 
3x  2 3
4
4  3x  2 
3g  x   2 
fg  x  
4 3  4 
4  3x  2
3 x   2 
 
3  3
4 3x  2
4x  2 gf  x  
fg  x   3
4
2  2 x  1

4
4x  2
fg  x  
4
42 2
fg  2  
4
6

4
 1.5
3x  2
f  x 
4

STUDENTS’ EXERCISE
x2
f x  
1
1. If 7. Given that f(x) = 3 x  5 . Find the
2x  4 2
Find: value of x such that f(x) = 10
(a)f(4) (b) f(0) (c) f(-2)
8. Find the values of x for which the
2. f(x) = 4 x  3x  c if f  4   30
2 following functions below are equal to zero
Find c (a) g  x   x  9
2
(b) h x  
2x  4
x 4 2x  1
3. hx   2 x 2  bx  3 If h2  13
x 2  3x  2
f x  
Find b
(c)
4. g x   ax 2  5x  3, g 1  9 4x  1
Find a
(d)
1  x  x  3
2

x2 3x  1
5. Given that f(x) =  5 Find :
(e) 4  x  x  5
3 2
(i) f(2)
(ii) x for which f(x) = 17
4x  8

6. Given that f(x) = ax – 7 and f(8) = 17 9. Find the values of x for which the
Find the value of following functions are not defined.
8 x 9  2x
(i) a (ii) f(4)
(a) f x   (b)
9  x2 x 2  3x  2
276
(a) f(x) =2x and g(x) = x + 3
5 (b) f(x) = 3x and g(x) = x + 5
(c) p  x   2  6 (c) f(x) = 2x + 3 , g(x) = x + 5
x 9
(d) f(x) = x – 5 , g(x) = 5x - 3

(d) p x   8x  1 16. What is the composite?


2 x  1 x  1 f : x  5 x  2 followed by
(e) g  x   7x  8 g : x 
 2 x  4
 9  x  x
2 2
 36 
17. What is the composite of :
(f) f  x   4x  9 f : x 
 2 x  5 followed by
20 x  x  1
2
g : x 
 x  5

2 4  
18. If f x  2 x  5, g x  x   2
6
(g) h x    2 Find: (a) gf(x) (b) gf(2)
x  3 x 9 (c) fg(x) (d) fg(3)

10. If f x   4 x  9 . Find f -1
(x) hence x3 1  2x
evaluate f -1(13) 19. Given that f(x) = and g x  
2 5
Find the values of x such that
9x
11. If f  x   x 
1
 9, Find: f fg(x) = 9  24 x  8 x
2
5 10
hence evaluate f 0
1
20. If f(x) = 3 x  1 g(x) = 4x. Find
3x  1
12. sx   14  8x Find s 1  x  (i) fg(x) (ii) fg(2)
(iii) gf(x) (iv) gf(-2)
hence evaluate s 1 0 
21. Given the function
1  2x x 3
f x   g x  
13. Find the inverse of the following functions.

(a) f x  2 x (b)
5 2
f x   5x  7 Determine the values of x for which gf(x)
(c) hx   3x  6 (d) mx  7 x  9 =
1
10
8x 2  24x  9 
14.For each of the following function f(x) and
g(x). Find the composite functions fg(x) 22. If f(x) = x2 and g(x) = x – 3. Find
and gf(x) (i) fg(x) (ii) gf(x)
(a) f(x) = x + 1, g(x) = 3x + 2
1 23. Given that f(x) = x2 – 1 and
(b)f(x) = 2x + 4, g(x) = x  6 g(x) = 2x + 1. Find
2 (a) fg(x)
(c) f(x) = x2 + 1, g(x) = 3x – 4 (b) gf(x)
(c) fg(-1)
15. Domain x = {1, 2, 3, 4, 5}
Find the range of gf(x) If
277
24. Given that f(x) = 2x + 4 and fg0, gg 36, ff 36
g(x) = x + 5. Find fg(x) hence
evaluate fg(4).
ANSWERS
25.Given that f(x)= x2 +3 and g(x)= x – 1
Find the value of a for which 1.(a) 1.5 (b) – 0.5 (c) 0
fg(x) = gf(a) 2. -46
3. 4
26. Find the inverse function of 4. 7
f(x) = x3 + 2 19
5. (i) (ii) x6
3
4x  1 6. (i) a = 3 (ii) f(4) = 5
27. Given that f(x) = Find f -1(3)
2x  3
7. 5
8. (a)x = + 3 (b) x = 2
2 (c)x -2, x = -1 (d) x = -3, x = + 1
28. Given that f(x) = and
3 x (e) + 2, 5
g(x) = x + 1 9.(a) = x + 3 (b) x = -1, x = -2
(i) find g-1(x)
1
(ii) fg(x) (c) x = + 3 (d) x x  1
(iii) The value of x for which fg(x) is not 2
defined. (e) x   6 , x  3
1 1
4 x 1 x  x 
f 2
(f)
29. Given that f(x) = . Find 5 4
2x  3 (g) x3
30. If fx  x  3 g x  
x  2
x  9
x 
and 1
x5 x7 10. f  , 1
Find: 4
(a) f -1 (3) (c) g-1(0.5)
(b) fg (3) (d) gf(1) 11. 5x  45 ,  5
(e) fg-1(0) (f) gf-1(10) 5
14  x 7
31. If f(x) = 2x + 5, g(x) = x + 9 12. ,
8 4
Find f -1(x) and g -1(x)
x
Show that [gf(x)]-1 f-1 [g-1(x)] 13. (a) (b) x  7
2 5
g(x) = 3x – 8
32. If f(x) = 5x + 6
x6 x  9
Find f-1(x) g-1(x) fg-1(x) and (c) (d)
gf(x) 3 7
33. If f(x) = 3x, g(x) = 2x = 5 14. (a) 3x  3, 3x  5
Find ff(x), ff(2), gf(x),
gg(x), gf(0) fg(0) (b) x  10 , x  5
ggg(x), fff(x) gfg(1) (c) 9 x 2  24 x  17, 3x 2  1
1
34. If f  x   3x  12, g x   x  12
2 15. (a) {5, 7, 9, 11, 13,}
Find: gf x, fgx, gf 0 (b) {8, 11, 14, 17, 20}
278
(c) {10, 12, 14, 16, 18}
(d) {-2, -18, -13, -8, -3}
16. gf(x) = 10x + 8

17. gf(x) = 2x

18.(a) 4x2 + 20x + 19 (b) 75


(c) 2x2 – 7 (d) 11

19. (a) x = -3.5, x = 0.25


(b) x = 3

20. (i) fg(x) = 12 x  1


12 x  1
(ii)fg(2) = 23
25
12 x  4
(iii)
3 x 1

(iv) 8
5

21. x = -3.5, x = 0.25

22. (x – 3)2, x2 - 3

23. (a) 4 x 2  4 x
(b) 2x2 - 1
(c) 0

24. 2x + 14, 22,

25. a = 1

x  2
1
26. 3

27. 5

28. x  1, 2
, x 2
2 x

29. 1

30.
279
KINEMATICS -v2 + 2496 = 4v
Example I v2 + 4v – 2496 = 0
A mini-bus travels from Mijera to Kampala a NB: from ax2 + bx + c = 0;
distance of 156km at a certain average speed of b  b2  4ac
v km/hr. On the return journey, it increases the x
2a
average speed by 4 and takes 15 minutes less. 4  100
Find the average speed, v from Mijera to v
2
Kampala.
100  4
Solution v
2
Distance = 156 km
96
Speed = v km/hr 
2
Mijera 156 km Kampala = 48 km/hr

Distance Example II
Speed =
Time Musoke and Kizito stay in the same home.
Let the time taken by the minibus to travel When Kizito walks from home to school at a
from Mijera to Kampala be t. constant speed of 10.8km/hr, he arrives 20
156 minutes early. When Musoke walks at a constant
v speed of 7.2 km/hr, he arrives late by 30
t
vt = 156 ............................................... (i) minutes. Calculate how far the school is from
15 1 home.
 hrs Solution
60 4
t1   t  14  hrs
Let time taken by Kizito and Musoke, if they are
on time be t.
=  t  14  hrs The time taken by Kizito to reach school form
Distance home =  t  60
20
 hrs
Speed =
Time =  t  13  hrs
156
(v  4)  Distance = speed × time
t  14
D = 10.8 ×  t  13 
(v  4)(t  14 )  156
D = (10.8t – 3.6) ................................ (i)
v(t  14 )  4(t  14 )  156
vt  14 v  4t  44  156 Time taken by Musoke to reach school from
v home
vt   4t  1  156
4 30
t
4vt  v  16t  4  156  4 60
4  156  v  16t  156  4  4 1
t
2
v  16t  4
 1
 156  Distance = 7.2  t  
v  16  4 2 
 v 
= (7.2t + 3.6 ................................. (ii)
2496
v  4 Equating Eqn (i) and (ii);
v
280
10.8t – 3.6 = 7.2t + 3.6 Time at which they met:
10.8t – 7.2t = 3.6 + 3.6 9 : 45
7.2 3 : 30
t 12 : 45
3.6
They met at 12:45 pm (but they are still
t = 2 hrs continuing)
Distance = (7.2 × 2) + 3.6 Remaining distance to be covered= 273–210km
= 18 km from home to school. = 63 km
Distance 63
Time = 
Example III speed 60
Moses and James wish to travel from Kireka to
Kawempe which is 273 km apart. They travelled = 1.05 hrs  1 hr and 3 minutes
12 : 45
by vehicles. At 7:45 am, Moses started his
+ 1 : 03
journey at a steady speed of 42km/hr. One and 1 : 48
half hours later, James started his journey at a  James reached Kawempe at 1 : 48 pm
steady speed of 60km/hr.
(a) Find the time and the distance at which Time taken by Moses to reach Kawempe
James over-took Moses. 63
from starting point = = 1.5 hrs
(b) The times when James and Moses are in 42
Kampala = 1hr, 30 minutes
(c) The difference in their times of arrival 12 : 45
+ 1 : 30
Solution 2 : 15
7:45 Difference in their time of arrival
(D – 63)km
Meeting point
Sm = 42km/hr 2 : 15
Kireka 273 km
Kawempe –1 : 48
0 : 27
The difference in their time of arrival is 27
minutes.
Considering Moses; Example IV
D =S×t The distance between two towns A and B is 432
= 42 × 1 12 km. A lorry travelling at a speed of 72km/hr
= 63 km leaves town A at 6:45 am for town B. One and
D  63 half hours later, a mini-bus leaves town A at a
tm  steady speed of 108km/hr non-stop heading for
42
D town B. Calculate
ts  (a) the time and distance from town A
60
D  63 D when the mini-bus over-takes the lorry,
 (b) The time when the two vehicles arrived in
42 60
60(D – 63) = 42D town B.
60D - 3780 = 42D Solution
60D – 42D = 3780 Distance covered by the lorry in 1 12 hrs
18D = 3780 = speed × time
D = 210 km = 72 × 1 12
Time taken by James to meet Moses = 108 km
210
  3.5 hrs Time taken by the lorry before meeting
60

281
D  108 D  18
 ...................................... (i) Tt = ....................... (i)
72 24
Time taken by the mini-bus to meet lorry D
D Tb = .............................. (ii)
= ........................................... (ii) 30
108 Equating Eqn (i) and Eqn (ii)
Equating Eqn (i) and (ii);
D  18 D
D  108 D =
 24 30
72 108
108D – 11664 = 72D 30D – 540 = 24D
108D – 72D = 11664 30D – 24D = 540
36D = 11664 D = 90 km
11664 D
D  324km Time taken by Bob to meet Tom =
36 30
90 90
The two vehicles met at a distance of 324 km =  = 3 hrs
30 30
Distance
 time when they met =  Bob met Tom after 3 hrs
Speed
(b) Time taken by Tom from the meeting point
324
 = 3 hours to B
108
Distance 48
At which they met 8 : 15 =  = 2 hrs
+3 : 00 Speed 24
11 : 15 am Time taken by Bob to travel from the
∴ the two vehicles met at 11:15 am meeting point to B
Distance 48
=  = 1.6 hrs
Speed 30
Example IV
Two towns A and B are a distance of 138km
apart. Tom leaves town A for town B cycling at Difference in time of arrival
a steady speed of 24 km/hr. When he had = 2 – 1.6 hrs
travelled a distance of 18km from A, Bob sets = 0.4 hrs
off from the same spot Tom started cycling at a = 0.4 × 60 = 24 minutes
steady speed of 30 km/hr.
(a) Find when and where Bob overtook Tom Example V
(b) Bob maintains his speed even after Town A is 120 km from town B. A lorry took
overtaking Tom. Determine how long it off from town B for town A at 8:25 am at a
took him waiting for Tom to join him. steady speed of 40km/hr. A saloon car left town
(c) Given that Tom then increased his speed A for B at 8:55 and travelled at a steady speed of
such that they both arrive at town B at the 80km/hr.
same time, how much did Tom increase his (a) Calculate the distance from town A at
speed immediately after he was overtaken? which the two vehicles met.
Solution (b) The time at which the two vehicles met.
138 km (c) Just as they met, the lorry driver increased
D his speed by 10km/hr. Find the difference
18 km (D – 18) in their times of arrival at the destination.
(138 – 90) B
A = 48 km Solution:
SB =30km/hr
ST =24km/hr Meeting point

282
170 km 100
D (150 – D 20 km
= = 2 hrs
50
Lorry B = 120 minutes
A Time = 8:25am
8:55 Speed = 40km/hr Time taken by the saloon car from the meeting
Saloon =80km/hr point to town B
Difference in time of departure Speed = 80 km/hr
= (8:25 – 8:55) Distance = 120 – 100
= 30 minutes = 20 km
30 1 Distance
  hrs Time =
60 2 Speed
1
Distance of the lorry after 2 hrs: 70
=
Distance = Speed × time 80
1 = 0.875
  40 = 53 minutes
2
=20 km The difference in time of arrival
Time taken by the lorry from town B to the = (120 – 53) minutes
150  D = 67 minutes
meeting point = ...................(i) = 1 hr and 7 min
40
D
Time of the saloon car = ................ (ii) Hence the difference in their time of arrival is 1
80 hour and 7 minutes
Equating Eqn (i) and Eqn (ii);
150  D D Example VI
=
40 80 The distance from town A to town B is 360km.
12000 – 80D = 40D An express bus leaves town A at 6:30 and
12000 = 120D travels at a steady speed of 80km/hr towards
12000 town B. At the same time, a taxi leaves town B
D=
120 travelling non-stop towards town A at a steady
D = 100 km speed of 100km/hr.
Distance (a) Find the difference in the time of arrival
Time taken by the saloon car = of the bus and the taxi.
Speed
D 100 (b) Determine when and at what distance
  from town A the two vehicles met.
80 80
= 1.25 Solution
360 km
= 1hr : 15 minutes
(360 – D D
Thus time of meeting 8 : 55
+1 : 15 taxi B
A Speed = 100km/hr
10 : 10 am 6:30
Bus =80km/hr Meeting point
Hence they met at 10:10 am
(c) Time taken by the lorry from the meeting Time taken by the bus to meet the taxi =
point to town A 360  D
................. (i)
Speed of lorry = 10 + 40 80
= 50 km/hr Time taken by the taxi to meet the bus =
Distance = 100 km D
....................... (ii)
Distance 100
Time =
Speed Equating Eqn (i) and Eqn (ii);
360  D D
=
80 100
283
100(360 – D) = 80D Hence the distance from town A to the point
36000 – 100D = 80D where the two vehicles met is
36000 = 80D + 100D = (360 – D)
36000 = (360 – 200)km
D= = 200km
180 = 160 km

284
Example VIII
The distance between A and B is 432 km. A lorry travelling at a steady speed of 72km/hr leaves
A at 6:45am for town B. 1½ hours later, a mini-bus leaves from A at a steady non-stop speed of
108km/hr heading from town B.
(a) On the same axes, show the journeys of the two vehicles. (use scales of 2cm to represent
40m and 2cm to represent 1 hour)
(b) Use your graph to estimate the
(i) time and distance from A when the mini-bus overtakes the lorry
(ii) the times when the two vehicles arrive in town B
(iii) the difference in time of arrival of the two vehicles.
Solution
Considering the lorry;
Distance(km) 0 72 144 216 288 360 432
Time 6:45 7:45 8:45 9:45 10:45 11:45 12:45

Considering the mini-bus;


Distance (km) 0 108 216 324 432
Time 8:15 9:15 10:15 11:15 12:15

Distance – time graphs (students’ exercise)

(i) They met at a distance of 324 km


(ii) They met at 11:15 am
(iii) Difference in time of arrival = 30 minutes

Example IX
The distance from Gulu to Kampala is about 380km. Otada bus leaves Gulu at 7:30am and travels
non-stop to Kampala at 60km/hr. At 8:50am the head teacher of Gulu College leaves Kampala in
his car and travels towards Gulu at a steady speed of 120km/hr.
(a) Using the same axes, draw a distance-time graph showing the journeys of both vehicles,
hence find when and at what distance from Kampala they met.
(b) If the bus increases its speed by 10km/hr,
(i) calculate the time at which the bus arrived in Kampala
(ii) determine the difference in times of arrival of the vehicles (Use 2cm:50km and
2cm:1hr)
Solution
Otada bus
Time 7:30 8:30 9:30 10:30 11:30 12:30 1:30 2:30
Distance (km) 0 60 120 180 240 300 360 420

Head teacher’s car


Time 0 120 240 360 480
Distance(km) 8:50 9:50 10:50 11:50 12:50

Distance-time graph

285
Distance from Kampala = (380 – 180) km
= 200km
The Otada Coach and the headteacher’s car met at 10:30 am at a distance of 180km form
Kampala.

Example X
The distance from town A to town B is 360 km. An express bus leaves town A at 6:30 am and
travels at a steady speed of 80 km/hr towards town B. At the same time a taxi leaves town B
travelling non-stop towards town A at a steady speed of 100km/hr.
(a) Determine when and at what distance from A the two vehicles met.
(b) Find the difference in times of arrival of the two vehicles.
Solution
Express bus
Distance (km) 0 80 160 240 320 400
Time 6:30 7:30 8:30 9:30 10:30 11:30

Taxi
Distance (km) 0 100 200 300 400
Time 6:30 7:30 8:30 9:30 10:30
Distance-time graph

286
(i) Distance when they met = 160km from A
(ii) They met at 8:30 am
(b) Time of arrival:
Taxi: 10:06 am
Express bus: 11:00 am
Difference: 11 : 00
– 10 : 06
00 : 54

∴ The difference in the time of arrival is 54 minutes

Example XI
Kampala and Mbale are about 229 km apart. A mini-bus heading for Kampala leaves Mbale at
8:55 am with a steady speed of 56km/hr. At 9:40 am, a saloon car travelling 80km/hr leaves
Kampala and travels steadily towards Mbale. Using a scale of 2cm to represent 20 km and 2cm :
1 hr:
(a) Draw on the same axes the distance-time graphs showing the journeys of the mini-bus
and the car. Hence or otherwise determine when and at what distance from Kampala till
the meeting point.
(b) Given that the mini-bus increases its speed by 14km/hr, calculate the time when the mini-
bus arrives in Kampala and the difference in their times of arrival.

287
Solution
Mini-bus
Distance (km) 0 56 112 168 224 280
Time 8:55 9:55 10:55 11:55 12:55 1:55
Saloon car:
Distance (km) 0 80 160 240
Time 9:40 10:40 11:40 12:40

Distance-time graph

(a) (i) They met at 11:01 am


(ii) The distance is 119 km from Kampala to the meeting point
(b) They met at a distance of 110km from Kampala.
New Speed = 56 + 14
288
= 70 km/hr
Distance 110
Time = 
Speed 70
= 1.57 hrs
= 1hr and 34 minutes
Time of arrival of minibus:
11 : 01
1 : 34
12 : 35
∴ The mini-bus reached Kampala at 12:35 pm

Example XII
Arua town is 540km from Kampala city. A pajero left Kampala for Arua at 7:00 am travelling at
a steady speed of 100km/hr. After travelling for two hours, the pajero had a mechanical break-
down which took exactly 2 hours to repair after which it continued non-stop to Arua at a steady
speed of 110 km/hr. One hour after the pajero left Kampala, a Gaga bus coach left Arua for
Kampala travelling at a steady speed of 80 km/hr but had a stop of 30 minutes at Karuma which
is 240 km from Arua town. The Gaga bus coach then continued non-stop to Kampala at a steady
speed of 20 km/hr more than its original speed.
(a) Using a scale of 2cm to represent 1 hour on the horizontal axis and 2cm to
represent 50 km on the vertical axis, draw on the same axes the distance-time
graphs for the two vehicles.
(b) Use your graphs to determine the time at which the two vehicles:
(i) met
(ii) distance from Kampala at which the two vehicles met,
(iii) the time of arrival of the pajero in Arua town
(iv) time of arrival of Gaga bus coach in Kampala
(v) difference in time of arrival of the two vehicles in their respective stations.
Solution:
Pajero
Distance- (km) 0 100 200 200 310 420 530 640
Time 7:00 8:00 9:00 11:00 12:00 1:00 2:00 3:00
Gaga bus coach
Distance- (km) 0 80 160 240 240 340 440 540
Time 8:00 9:00 10:00 11:00 11:30 12:00 1:30 2:30

Distance-time graph (Students’ Exercise)

289
Vectors
A vector is a quantity with direction as well  2  4
a   ,b   
as magnitude. A vector quantity is  1   3 
represented by a line segment with an arrow  2   4 
indicating a particular direction. a b    
 1   3 
 2  4 
  
 1  3 
 6 
 
 2
 6 
 a b   
From the above diagram, a man is said to be  2
in motion moving from A to B in a straight
 2   4 
line. We represent that displacement as vector ab    
AB. When we describe quantities by giving  1   3 
both their direction and magnitude (size),  2
then we are using vectors. For example;  
London is 100000000000000 km south east  4 
of Kampala.  2
 ab   
Algebra of vectors  4 
A vector can be represented by a column
3a  2b
 x
vector   where x denote the horizontal  2   4 
 y  3   2  
 1   3 
movement and y denote vertical movement.
To add or subtract vectors, we add or subtract  6   8 
   
corresponding components.  3   6 
To multiply a vector by a scalar, multiply
 6  8 
each component by scalar.   
Example I  3  6 
 2  4  14 
Given that a    and b     
 1   3   3 
Find a  b 3(a  b)
ab   2   4  
 3      
3a  2b   1   3  
3 a  b  6 
 3 
4a 2
Solution;  12 
 
 6 
Example II
 4  6 
a    and b   
 2  12 
Find 1 a  2 b and 1 a  5 b
2 3 4 6

Solution;

290
1  4  2  6 
    iii. 2 AB  3CD
2  2  3  12 
 4   6 
2    4  2    3 
      2   12 
1   8
 8   18 
  2    
    4   36 
 7
 8  18 
1 5  
a b  4  36 
4 6
 26 
1   5  6 
4  
     40 
4  2  6  12 
 26 
 1   5   2 AB  3CD   
     40 
 1 2   10  1
iv. AB
 1  5   6  4
   
 0.5  10  10.5  
1  4 
 
Example III 4  2
 4  6   1
 
AB    and CD     12 
 2  12 
Magnitude of a Vector
Find
The magnitude of a column vector is its
1 length. Another word for magnitude is the
(i) AB
2 modulus which is used to refer to the positive
5 square root of a number.
(ii) CD
3  x
Let A be a point with column vector  
(iii) 2 AB  3CD  y
1
(iv) AB
4
Solution
1 1  4
i. AB   
2 2  2
 2
 
1
1  2
 AB   
2 1
5
ii. CD ,
3
Using Pythagoras’ theorem
5  6 
  OA2  OB 2  ( BA) 2
3  12 
OA2  x 2  y 2
 5 
 3  6  OA  x 2  y 2
 
 5  12   a  x2  y 2
 
 3  a is the magnitude or modulus of a
 10 
  For example;
 20 

291
 3 r  42  32
If AB   
 4  16  9
AB   3   4 
2 2
 25
 9  16  5 units
 25  3   1
4. p q    
 5 units 1  2 
4
Example IV  
 1
 3  1 4
If p    , q    and r   3  . Find p  q  42  (1) 2
1 2  
1. p p  q  16  1
2. q  17 units
3. r  3   1
5. pq    
4. pq 1  2 
 2
5. pq  
 3
6. p  3r
pq   2    3
2 2
7. qr
8. 2r  q  49
9. 3 p  2q  13 units
 3  4 
10. p  2q  3r 6. p  3r     3  
1  3
 3   12 
Solution;    
1  9 
 3
1. p   15 
 
1  10 
p  32  12 p  3r  15   10 
2 2

 9 1  225  100
 325 units
 10 units
 1  4 
7. q r    
 1  2   3
2. q 
 5 
2  
 1 
q   1   2 
2 2

qr   5   1


2 2

 1 4  25  1
 5 units  26 units
4  4   1
3. r    8. 2r  q  2     
3  3  2 

292
 8   1 0  3
p   ,r   
   
6  2   4   2 

9 1 1 0   3 
pr    
  2 2  4   2 
 4
0  3
2r  q  9 2  4 2    
 2   2 
 81  16  3
 
 97 units  4 
 3   1 1
pr
9. 3 p  2q  3    2   The length of
2
1  2  1
 pr   3   4 
2 2
 9   2 
    2
 3  4   9  16
 9  2   25
   5 units
 3 4 
 11  Geometry of Vectors
 
 1  Addition of Vectors
3 p  2q  11   1
2 2

 122 units

 3   1  4 
10. p  2q  3r     2    3 
1  2   3
 3   2  12 
    
1  4   9 
 3  2  12 
 
 1 4  9  From the diagram, we can see that the
displacement PO followed by the
 13 
  displacement OQ is the same as the
14  displacement PQ. This is an example of
vector addition and we can write;
p  2q  3r  132  142
PQ  PO  OQ .
 169  196
But PO is the same as OP but in the opposite
 365 units direction
PO  OP
Example III  PQ  OP  OQ
3 0
If the vectors r    and p    . Find PQ  OQ  OP
 2   4 
1
the length of pr Example I
2  13 
Solution Given that OP  p    and
17 
5
OQ  q    . Calculate the length of PQ
 2

293
Solution AB  AO  OB
 OA  OB
5  3
        OB
 1 7
 5   3
OB      
 1  7 
8
OB   
6

 OB  8   6 
2 2

 64  36
PQ  PO  OQ  100
 OP  OQ  10 units.

 13   5  Example III
     
17   2  Q and R are in the same plane with position

 13  5   2 13 
 vectors q    and r    . Find the
  4 0
 17  2 
length QR. S is the midpoint of QR. Find the
 8 
  coordinate of S.
 15  Solution
Length of PQ  PQ QR  QO  OR
 OQ  OR
  8   15
2 2
 OR  OQ
 64  225 13   2 
   
 289  0   4
 17 units  11 
 
 4 
 The length of PQ is 17 units. Length of QR  QR

 11   4 
2 2
Example II
 3 5  121  16
Given that OA    and AB   
7  1  137 units
where O is the origin. Find the position
vector of B. hence find OB .

Solution

294
Area of OPQ
Q(2, 4)
 Area of the   Area of the 
   
  right-angled    right- angled 
 triangle OPM   triangle PQM 
   
1 1
  4 4   2 4
2 2
 84
 4 units.

Example V
Find the values of x and y, if;
 1  2   1 
i. x    y     
3  1   8 
8
ii. x    y     
4 4
OS  OQ  QS  3 1  0 
1 Solution;
 OQ  QR
x 1
i.        
2 2y
1
 OQ   OR  OQ   3x   y   8 
2
 2 y  x   1 
 2  1  13   2     
          3x  y   8 
 4 2  0   4  2 y  x  1
 2  1  11 
    3 x  y  8
 4  2  4 
 x  2 y  1....................(1)
 7.5 
 
 2  3x  y  8.......................(2)
S  7.5, 2  ii.
 4 x   4 y   8 
     
Example IV  3x   y   0 
 4  2 4 x  4 y  8..........................(1)
Given that OP    , OQ    ; where OP 3x  y  0...............................(2)
 4 0
From equation (1)
and OQ are position vectors of points P and
Q respectively. Find the area of OPQ. x  y  2
Solution  x  2  y..........................(3)
Substituting
Substituting equation
equation(3)(3) in eqaution
in equation (2) (2)
3  2  y   y  0
6  3 y  y  0
2 y  6

(2, (4,
0) 0)

295
y  3 parallel and they have a common point then
Substituting y  3 in equation (3) the vectors lie on the same line.
 x  2  3 Consider three points A, B and C
x  2  3 AB
If   or AB   BC
x 1 BC
 x  1 and y  3 where  is a scalar then,
AB is parallel to BC and the points A, B and C
Example VI are collinear (lie in the same straight line).
 2  1  3  Example I
If u    , v    and w    , find The coordinates of P, Q and R are (2, 4), (18,
 4  3  4
2) and (50, -2) respectively. Find PQ and QR
the values of a and b such that au  bv  w and show that P, Q and R are collinear.
Solution:
Solution; PQ  OQ  OP
 2   1   3 
a   b      18   2 
   
 4   3   4   2   4
 2a   b   3   16 
     
 4a   3b   4   2 
 b  2a   3  QR  OR  OQ
  
 4a  3b   4   50   18 
   
b  2a  3.................(1)  2   2 
4a  3b  4..................(2)  32 
 
From equation (1)  4 
b  3  2a...................(3) 16 
 2 
 2 
Substituting equation (3) in equation (2)
 4a  3(3  2a)  4  QR  2 PQ
4a  9  6a  4  2 PQ  QR
2a  9  4 So points P, Q and R are collinear
2a  5
a  2.5 Example II
Substituting a  2.5 in equation (3) PQRS is a parallelogram with P = (1, 1), Q =
 b  3  2(2.5) (5, 3) and R = (7, 7). Find the;
b  3  5 i. Column vector of PS
ii. Coordinates of S
b2 Solution;
 a  2.5, b  2.

Parallel Vectors
If PQ   DR then PQ and DR are
parallel.

Collinear Points
Points are said to be collinear if all of them
lie in a straight line i.e. if two vectors are
296
PS  QR
PS  OR  OQ

 7  5
PS      
 7  3
 2
PS   
 4
 2
PS   
 4
PS  QR
 OS  OP  OR  OQ OM  OA  AM
OS  OR  OP  OQ 1
 a  AB
 7   1  5  2
OS         
 7   1  3  1
 a   AO  OB 
3 2
 
5 a 
1
 OA  OB 
2
 The coordinate of S is (3, 5)
 a  OB  OA
1
2
1
a b  a 
2
Proving Geometrical Results 1 1
 a b a
2 2
i. If A and B have position vectors a and
1 1
b respectively, show that AB  b  a  a b
2 2
Solution 1
 a  b
2
Example I
6 14 
If P    , Q    and R is a point on
8   16 
1
PQ such that PR  RQ . Find the
3
coordinates of R.
Solution;

AB  AO  OB
 OA  OB
 a  b
 AB  b  a
ii. If A and B have position vectors a and
b respectively then the position vector
of the midpoint M, of the line joining A
1
to B is OM  a  b
2
Solution;
297
OR  OP  PR Alternatively;
1
but PR  RQ OC  OD  DC
3
PR 1 1
  BC  BD
RQ 3 2
total ratio = 1+3 = 4. BD 2
 
1
= 1PQ
PR OP BC 1
OR PQ
4 4
1
OR  OP  PQ
4
6 1
OR      OQ  OP 
8  4
 6  1 14   6  
        
 8  4  16   8  
 6 1 8 
   
 8  4  24 
6  2 
   
 8   6 
8 
 C is a midpoint of BD
OR   
 2
OC  OD  DC
 The coordinate of R is (8, 2)
1
Example I  d  DB
2
In the figure below OB  b , OD  d and
1
1  d   DO  OB 
BC  BD . Find OC 2
2 1
 d   d  b 
2
1
 b  d   as before 
2
Example II
ABCD is a quadrilateral in which BC is
parallel to AD, AD  4d , AB  b ,
3
BC  AD
Solution 4
OC  OB  BC Find BC, ACand CD in terms of b and d
1
 b  BD
2
1
 b   BO  OD 
2
1
 b   OB  OD 
2
1
OC  b   b  d  A
2
2b  b  d Solution

2
bd

2
1
OC   b  d 
2

298
A
PR  PS  SR
but PS  3QR  3r
PR  3r  s...................(2)
Equating equation (1) and equation (2)
3  q  r  3r  s
BC  AD
4
q  s  3r  r
but AD  4d
3  q  s  2r (as required)
 BC   4d  Example IV
4
BC  3d The diagram below shows points a and b
AC  AB  BC with position vectors and respectively. The
3 point M divides OB such that OM: MB = 3:1
AC  b  AD C divides AB in ratio 3:1 and N is a midpoint
4
 b  3d of AD. Also 3AB  4CD
 AC  b  3d
CD  CA  AD
  AC  AD
   b  3d   4d
 b  3d  4d
CD  d  b
Example III Find OC, OD and BN. Show that MN is
PQRS is a quadrilateral in which PQ  q and parallel to OA
QR  r , RS  s and PS  3QR Solution
Show that q  s  2r

OM : MB  3 :1
AC : CB  3 :1
Solution
1
AN  ND  AD
2
3 AB  4CD

PR  PQ  QR
 q  r...................(1)
Alternatively

299
OC  OA  AC OD  OA  AD
 a  AC OD  a  AD
3 but AD  AC  CD
but AC  AB
4 3
 AB  CD
3 4
 OC  a  AB
4 3 AB  4CD
AB  AO  OB 3
 CD  AB
 OA  OB 4
 a  b 3
AD  AB  AB
3
ba 4 4
6
3 AD  AB
OC  a  b  a  4
4 3
AD  AB
2
4a  3b  3a
 3
AD   b  a 
4 2
 OD  a  AD
a  3b
 3
4 a b  a 
2
Alternatively 2a  3b  3a

2
OC  OB  BC 3b  a

1 2
 b  BA
1
4 OD   3b  a 
1 2
 b    AB  BN  BA  AN
4
1 1
 b    AB    AB  AD
4 2
1 3
1
 b   AB    b  a   . b  a 
4 2 2
3
1  b  a   b  a 
 b  b  a  4
4
4b  4a  3b  3a
4b  b  a 
 4
4 a b
3b  a BN 
 (as before) 4
4 1
BN   a  b 
4
MN  MB  BN
1 1
 OB   a  b 
4 4
1 1 1
 b a b
4 4 4
1
 a
4
OA  a
1
a
MN 4

OA a
MN 1

OA 4
4 MN  OA
Example VI
300
OC  OB  BC
1
 OB  BA
2
1
 OB    AB 
2
1
 OB   AB 
2
1
In the diagram above, OA  12a , OB  6b  6b   12a  6b 
2
D and C are midpoints of OA and AB  6b  6a  3b
2
respectively. OG  OC  6a  3b
3  3  2a  b 
Find in terms of a and b
2
i. OC OG  OC
3
ii. OG 2
iii. GD OG  3  2a  b  
3
Show that D, G and B are collinear.  2  2a  b 
GD  GO  OD
Solution
1
 OG  OA
2
1
 2  2a  b  12a 
2
 4a  2b  6a
 2a  2b
 2(a  b)
In order to show that points D, G and B are
collinear we need to express vectors DG and
OC  OA  AC GB in terms of a and b .
1
 OA  AB DG   GD
2
1  2 b  a 
 OA   AO  OB 
2 GB  GO  OB
1
 12a   12a  6b   OG  OB
2
 12a  6a  3b  2  2a  b   6b
 6a  3b
 4a  2b  6b
 3  2a  b 
 4b  4a
Alternatively
 4 b  a 
DG 2  b  a 

GB 4  b  a 
DG  b  a 

GB 2  b  a 
DG 1

GB 2
 2 DG  GB
 point D, G and B are
collinear (lie in the same
straight line)

301
2
Example VII  b  b  a 
3
3b  2b  2a

3
1
  2a  b 
3
To show that R, T and P lie in the same
straight line, we need to find RT and TP
A quadrilateral PQRS with ST  2TQ , RT  TR
2
PQ  a , SR  2a and SP  b but TR   2a  b 
3
Find these vectors in terms of a and b 2
i. SQ  RT    2a  b 
3
ii. QT 2
iii. TR   b  2a 
iv. PT 3
Show that R, T and P lie on the same straight TP   PT
1
but PT   2a  b 
line
Solution; 3
SQ  SP  PQ 1
TP    2a  b 
ba 3
QT  ? ST  2TQ 1
TP   b  2a 
ST 2 3

TQ 1
1
TQ  SQ 2
RT 3 
3 b  2a 
1 
TQ  SQ TP 1 b  2a
3  
3
1
QT   SQ RT  2   1 
3     
1 TP  3   3 
  b  a 
3 RT 2 3
TR  TS  SR
 
TP 3 1
2 RT
  SQ  2a  2,  RT  2TP
3 TP
2
   b  a   2a RT  2TP.
3 R, T and P line on the same straight line
2b  2a  6a
 Example VIII
3
4a  2b The diagram below shows a parallelogram
 OABC, OA  a and OC  c P divides AB in
3
2 the ratio of 1:3 and Q divides OP in ratio of
TR   2a  b 
3 4:1
PT  PS  ST

302
4a  16c

20
4
  a  4c 
20
1
  a  4c 
5
1
 QB   a  4c 
5
a. (i) find the vector QB in terms of a
AC  AO  OC
and c
 a  c
(ii) Show that the point Q lies on AC
b. If D lies on AB such that AD: DB =  ca
2:3. express OD and DC in term of a AQ  AO  OQ
and c 4  4a  c 
a   
Solution: 5 4 
OA  CB  a, OC  AB  c 4a  c
QB  QP  PB  a 
5
but the ratio of 5a  4a  c

OQ : QP  4 :1 5
1
QP  OP c a
5
AQ 
5
1 QC  QO  OC
 QB  OP  PB
5
4 a  c
but P divides AB in the ratio of 1:3  c
5
1 1
 AP  AB  c 4a  c  5c
4 4 
5
3 3
 PB  AB  c 4
4 4  c  a
5
1 3
 QB  OP  c 1
5 4
AQ c  a
OP  OA  AP  5
QC 4 c  a
1 5
a c AQ 1
4 
4a  c QC 4

4 4 AQ  QC
1  4a  c  3  Q lies on AC
 QB    c
5 4  4
4a  c 3
  c
20 4
4a  c  15c

20

OD  OA  AD

303
2 PM
 OA  AB
5 M is a midpoint of PQ
2 1
a c  PM  PQ
5 2
5a  2c

5
1

PM  q  p
2

1 OM  OP  PM
  5a  2c 
5 1
OM  p  q  p
2
 
2p q  p
DC  DB  BC 
2
3 pq
 AB  a OM 
5 2

 
3 1
 c  a  pq
5 2
3c  5a ON : NM  2 : 3
 2
5 ON  OM
3
1
  3c  5a 
5
2 1
 . pq
3 2
 
 
Example IX 1
 pq
M is a midpoint of PQ in triangle OPQ. If 3
OP  p and OQ  q , find in terms of PN  PO  ON
vectors p and q the vectors PQ, PM and  OP  ON
  p  ON
OM. N is a point on OM such that ON: NM =
2:1. Express ON and PN in terms of p and q . 2
ON  OM
Given that S is the midpoint of OQ, use 3
vector methods to show that N lies on PS and
hence determine the ratio of PS: SN
1
 pq
3
 
Solution; 1 1
PN   p  p  q
3 3
3 p  p  q
PN 
3
q 2p
PN 
3
1
PN   q  2 p 
3
PS  PO  OS
1
  p  OQ
2
PQ  PO  OQ 1
 OP  OQ PS   p  q
2
 pq 2 p  q
PS 
q p 2

304
OA  OP  PA
1
PS 
2

q 2p 
1
 OP  PQ
PN 3
1

q 2p  3
 1

PS 1 q  2 p
  OP   PO  OQ 
2 3

 
PN  1   1  1
      OP  q  p
PS  3   2  3

 
PN 1 2 1
   p q p
PS 3 1
3
PN 2
 3p  q  p
PS 3 
 PN : PS  2 : 3 3
Example X 2p  q
In the diagram below OP  p and OQ  q OA 
3
1  2p  q 
the points A and B are such that PA  PQ  OC  h  
3  3 
1
and OB  OP 2hp hq
3 OC   .....................(1)
3 3
OC  OQ  QC
 OQ  kQB
but QB  QO  OB
1
 q  p
3
3q  p

3
p  3q

i. Express OA, QB in terms of p and q 3
ii. If OC = h OA and QC = k QB,  p  3q 
OC  q  k  
express OC in two different ways and  3 
hence deduce: 3q  kp  3kq
(a) the values of h and k OC 
3
(b) OC in terms of p and q
 3  3k  q  kp
only OC 
3
(c) That Q, C and B are
kp 3  3k
collinear. OC   q ...............................  2 
Solution: 3 3
OP = p Equating equation (1) and
1 equation (2) respectively.
OQ  q, OB  OP
3 2hp hq kp  3  3k 
   q
1 2 3 3 3 3
 OB  p, BP  p
3 3 2h k
 
OC  hOA 3 3
2h  k ..........................(3)
h  3  3k 

3 3

305
h  3  3k .....................................  4  QC 2 21
 
Substituting eqn  3 in eqn  4  CB 7 1
QC 6
h  3  3(2h) 
CB 1
h  3  6h  QC  6CB
7h  3  Q, C and B are collinear
3 (lie in the same straight
h
7 line)
k  2h
6
k
7
3 2p  q 
OC   
7 3 
1

OC  2 p  q
7

QC  kQB
p  3q
QB 
3
 p  3q 
QC  k  
 3 
6  p  3q 
QC   
7 3 

QC 
2
7

p  3q 
CB  CO  OB
 OC  OB
1
  2p  q  p
7
 1
3

6 p  3q  7 p

21
p  3q

21
QC 
2
7

p  3q 
CB 
1
21

p  3q 
QC 7
2

p  3q 

CB 1
21

p  3q 
QC 2 1
 
CB 7 21
306
Example XI 
OX  l 3 p  2q 
OX  3lp  2lq........................(2)
In the diagram below showing a quadrilateral
Equating eqn 1 and eqn  2 
OSRQ, OS  q , OP  p and SX  kSP
 kp  1  k  q  3lp  2lq
k  3l
1  k  2l
1  3l  2l
1  3l  2l
5l  1
1
l
5

i. Express vectors SP and OX in terms k  3l


of p , q and k. 1
 k  3 
5
ii. If OQ  3 p , QR  2OS and
k 3
5
OX  lOR . Find the values of k and
l. hence the ratio of SX: XP. SX  kSP
Solution:

3
5
pq  
OQ  3 p, OS  q, OP  p
SX  kSP XP  XO  OP
SP  SO  OP  OX  OP
 OS  OP

 l 3 p  2q  p 
 q  p
 pq
1

  3 p  2q  p
5

3 p  2q  5 p


SX  k p  q  5
2 p  2q
OX  OS  SX 
5

 qk pq  XP 
2

pq 
5
 q  kp  kq
 1  k  q  kp
OX  kp  1  k  q..................(1)

OQ  3 p,
QR  2OS
 2q

OX  lOR
OR  OQ  QR
 3 p  2q

307
SX : XP AM  AO  OM


3
SX  p  q   OA  OM
5  a  xb
XP 
2

pq 
5 OS  OA  AS
SX 5
3

pq 
  OA 
1
 
XP 2 p  q AM
2
5
1
SX  3   2  a  a  xb 
     2
XP  5   5 
2a  a  xb
SX 3 5 
  2
XP 5 2 a  xb
SX 3 
 2
XP 2 1
SX : SP  3 : 2   a  xb 
2
Example OL  OA  AL
 OA  yAB
In the triangle OAB, OA  a OB  b A point
 a  y b  a 
L is on side of AB and M is on side of OB. OL
 a  yb  ya
and AM meet at S such that AS = SM and
 a  ya  yb
3
OS  OL . Given that OM = x OB and AL =  1  y  a  yb
4
y AB, express the vectors;

i. AM and OS in terms of a , b and x 3


OS  OL
ii. OL and OS in terms of a , b and y, 4
3
hence find the values of x and y.  1  y  a  yb 
Solution: 4

Consider OS in terms of a, b
and x
1 1
OS  a  xb......................(1)
2 2

Consider OS in terms of a, b
and y
OM  xOB,
3 3
 xb OS  1  y  a  yb.........(2)
4 4
AL  yAB
 y  AO  OB  Equating the corresponding components of
 y b  a  equation 1 and equation 2
AL  yb  ya

308
1 3 3y
  1  y  x
2 4 2
2  3 1  y  ......................(3) 31
  
1 3 23
x y
2 4 1
x
2x  3y 2
x 1 ,y 1
3y 2 3
x ......................(4)
2

From equation (3)


2  3 1  y 
2  3  3y
3y  1
y 1
3

Students’ Exercise

1. u   3, 3 , v   5, 5  , w   4, 6  and 6  9 
5. OA  a    , OB  b    ,
x   1, 1  4   6 
a) Find u  w and u  w 3 1
Find OC if OC  a  b
2 2
b) Find xv and xv
 2  2  6. A   3, 1 , B   1,3 , C   2, 4 
2. If a    and b    Find
3  4  Show that A, B and C are collinear.
i. ab
1
ii. a b Find D such that CD  BD
2
iii. 2a  3b
7. P  2, 2  , Q  6, 2  , R  2,1 and S are
iv. a  4b
vertices of a parallelogram.
 2  3
3. a    and b  , and i. Find the coordinates of S
3 8 ii. Find the coordinates of M the point of
a b  t a b intersection of the diagonals
iii. Show that M and the midpoints of PQ
Find t. and RS are collinear.
3 
6 9 
4. PQ    , UT    , Find PQ : UT
8. P   2, 4  , PQ    
8  12   1
Find the coordinates of Q

309
A   2, 2  is mapped into A1   5, 1 19. In a triangle ABC, X is the midpoint of AB
and Y is a midpoint of AC
under a translation. What is the image of
AB  a and AC  b
A1 under the same translation?

9. A  1, 2  mapped onto D   3, 2  by


translation r. what would be the image of A
under a translation 4r?
10. A   3,1 and B   6,5 
i. What is the column vector of AB?
ii. The images of A and B under a
reflection in the x-axis are A1 and B1 .
Find A1 and B1 .
12. If k   3, 7  and m  1, 7  i. Express BC in terms of a and b
i. Find the position vectors of k and m ii. Express XY in terms of a and b
ii. km and mk iii. Hence show that XY is parallel to BC and
13. If T   8, 0  , W   2, 0  and Z   5, 0  . half its length.
20.
Find the position vectors of WZ, ZT and WT
 4
14. ABCD is a parallelogram with AB   
1 
6
and AC    , A  3, 1 Find the
8 
coordinates of D
 2  x  5
15.         . Find the values of x and y.
 x   y   6
3 0  3 In the figure above AB  b , AC  c , BD  2DC
16. a    , b    , c   
 0  2 1  and AE  3ED
Find the values of p and q such that
pa  qb  c Find

i. AD
1 
17. AB    , A  2, 3 , C   1,15 . Find ii. ED
 4  iii. CE
the values of x and y if xOA  yOB  OC . 21. In the figure , ABCD is a parallelogram
DE  EC , AB  b and AD  d
1   3 
18. AB    and BC    . Calculate AB
 3 1 
and BC
Calculate AC

What do you conclude about vectors AB and


BC?
310
Express AE and EB in terms of b and d Express AB and CD in terms of a and b .
What are the geometrical relations between
22. OA  a and OB  b X is a point such that AB and CD?
3
OX  OA and Y is a point such that
2 24. The position vectors of A, B and P are a , b
OY  3OB and p respectively
i. If AP = PB, Find P in terms of a and b .
ii. If 4AP = PB, find P in terms of a and b .
iii. If AP = -5PB, find P in terms of a and b .
25. OABC is a parallelogram with OA  4a and
OB  4b . The diagonals intersect at D. P,
Q, R and S are midpoints of OD, AD, DB and
Express in terms of a and b DC respectively.

(i) OX (ii) OY (ii) YZ

The lines YX and BA are established to


meet at Z.

Explain why the position vector of any point


on YX extended can be written as
Show, using vector algebra that PQRS is a
3 
3b  k  a  3b  and that the position parallelogram.
2 
vector of any point on BA extended can be 26. OA  a , OB  b OC  c
written as b    a  b  N is the midpoint of OB and M is a midpoint
of AC
Hence or otherwise find the position vector
of Z.

23.

i. Express AB in terms of a and b .


ii. Express AC, AM and OM in terms of a
and c
iii. Express ON in terms of b
iv. NM in terms of, a b and c .
In the diagram above, OA  a , OB  b 27.
AC  2OA BD  2OB OAC and OBD are
straight lines.

311
X and Y are points on AC such that AX: XC =
1:2 and YC: AX = 3:1 express the following
vectors in terms of p and q
In the figure above OA  a and OB  b .
(i) AC (ii) AY (iii) OY (iv) OX (v)AB. 3OB  2BD , M is a point on AD such that
MD : AM  1: 2
Hence show that O, Y, B are collinear.
OC  3CE  3AM
28.
i. Express the vectors AD, BM and DC
in terms of a and b .
ii. Show that AD : OE  3:8

30.

In the figure above, AD is parallel to OC and


OA is parallel to BD, 3OC = 5OB.

E is the point where AC meets BD; AE: AC


= 3:2

Find in terms of vectors a and b the vectors In the figure above PQ  s , PR  r ,


AC, DC, ED, AE and OE 2QT  3TR and PU : UQ  2 : 3

Find the ratio BE: ED Find in terms of vectors r and s vectors


QR, QT and P
29.
Show that UT is parallel to PR

31. In a triangle OPQ, X is a point such that


2
OX  OP and Y is the midpoint PQ. The
3
point Z on OQ is such that OQ = QZ
Given that OP  p and OQ  q

312
a) Determine in terms of p and q the
vectors;
(i) OX (ii) OY (iii) OZ (iv) XY (v) YZ

b) hence of otherwise show that X, Y and Z


lie on a straight line
c) state the ratio of XY to YZ i. Find BE in terms of a and k
32. In the Triangle ABC, M and D are midpoints
ii. DE in terms of a , c and h
of AC and CN respectively. N is a point on AB
such that AN = 3NB iii. Find the values of h and k
35.
If AB  p and AC  q , express the
following vectors in terms of p and q

(i) AM (ii) AN (iii) ND

(b) Show that MD is parallel to AB and that


MD : AB  3:8
Given that AB  a and AD  b
33.
2
AE  AD , EC  2 AB .
3

Express the following vectors in terms of a


and b ;

(i) AE (ii) BD (iii) CE (iv) CD (v) BC

36. In the diagram below O is the origin ABC is a


straight line and M is the midpoint of OA
In the figure above OA  a , OB  b . F and OA  a OB  b and AC  3AB
G are points on AC such that OD: DA = FB :
BG = 1:2

i. Express AG and AC in terms of AB. Hence


find in terms of AB. Hence find in terms of
vectors a and b AB, AC, DG and OF
ii. Determine the ratio of DG: OC
34. The diagram shows a parallelogram OABC
with OA  a and OC  c . D is a point on AB Find in terms of a and b the vectors
such that AD : DB  2:1 . OD produced
(i) AM (ii) AB (iii) the position vector of C?
meets CB produced at E
DE = hOD and BE = kCB
1
(b) given that MN  MC , find
5

(i) ON in terms of a and b


313
(ii) The ratio of ON: NB

37.

i. Find the vectors OC, OD and BN in


terms of a and b
ii. Show that MN is parallel to OA.
In the diagram above PQ = 4PT, PV = 2PS, State the ratio of MN: OA
3RS = 2RT, PT  3a , PR  3b 40.

Express in terms of a and b

(i) RS (ii) PV (iii) RQ

(b) Find RV: RQ

38. PQRS is a parallelogram. The point T divides PQ in


the ratio of 1:3 and U, V and W are
midpoints of SR, PS and QR respectively.

PT  a And PV  b

Express each of the following in terms of a and b .

i. PQ
ii. SU
iii. PU
In the diagram above OA  a and OB  b .
1
M is the midpoint of OA and P lies on AB (b) Show that XR   5a  2b 
2
such that 3 AP  2 AB
41.
i. Express AB and AP in terms of a and b .
ii. Express MA and MP in terms of a and b .
iii. If X lies on OB produced such that OB = BX,
Express MX in terms of a and b .
iv. Show MPX is a straight line.
39. The diagram below shows the points A and
B with position vectors a and b
respectively. The point M divides OB such
that OM: NB = 3:1 C divides AB such that
3
AC  AB and N is a midpoint of AD. Also
4
3AB = 4CD
314
ABCD is ad isosceles triangle. L is the
midpoint of BC. M divides the line LA in
ratio 1:5 and N divides AC in ratio of 2:5

BC  p and BA  q

Express the following in terms of p and q

i. LA
ii. AN
iii. MN
42. In the figure AB  a , AC  c ,
3 AD  2 AC BE  EC and that
AF : FE  4:1

Express in terms of vectors a and c

i. BD
ii. AF
iii. BF
iv. The ratio of BD: FD
A

ANSWERS

7
1(a)   , 58
3

4 
(b)   , 2 13
 6 

2.(i) 1

(ii) 65

(iii) 2 106

(iv) 205

315
73
3.
13

4. 2:3

5. 13.5, 9 

6. AB=4BC

A, B, C are collinear

 3,5
7(i)  6, 1

8.  8, 4 

9.  3, 3

10.  
9
14 

11.   A1  3, 1 B1  6, 5 
3
 
4

 3  1 
12.   ,  
7 7

 2   2
KM=   MK=  
0  0

3 3 6


13. WZ=   ZT=   WT=  
0 0 0

14. D  1,8 

15. x =3, y =3

316
16. p =1

q =0.5

17. x = 7.6

y = -5.4

18. (i) 10

(ii) 10

20  2 5 | AB |  | BC |

19.

ca  BC
1
XY   c  a 
2
2 XY  BC

1
20.  b  2c 
3

1
 b  2c 
12
1
 b  2c 
4

1
21. AE =  b  2d 
2

1
EB =  b  2d 
2

b
22. OX  a
2

OY  3b

23. AB  b  a

317
CD  3  b  a 
CD  3 AB

1
24. p  a  b
2
1
p  4a  b 
5
1
p   5b  a 
5

25.

26. AB  b  a

AC  c  a
1
AM   c  a 
2
1
OM   a  c 
2
1
ON  b
2
1
NM   a  b  c 
2

27. (i) q  p

2
(ii) q  p
3

1
(iii)  2q  5 p 
3

1
28. AC   5b  3a 
2

1
DC  2b  3a 
3
3
ED  a
5
1
AE   5b  3a 
5
1
OE   2a  5b 
5
2
BE  A
5

318
1
29.  5b  2a 
2

1
 a  2b 
3

30. QR  r  s

3
QT  r  a
5
2 3
PT  s  r
5 5
5UT  3PR

2
31. p
3

1
OY   p  q
2
PZ  2q
1
XY   a  p 
6
1
Yt   q  p 
2
1: 3

1
32. AM  q
2

3
AN  p
4
1
ND   4q  3 p 
8
MD 83 p

AB p
MD 3

AB 8
 MD : AB  3 : 8

319
34. BE  ka

1
DE  h  3a  2c 
3
1 1
h , k
2 2

2
35. b
3

BD  b  a
CE   2a
1
CD   b  6a 
3
1
BC   3a  2b 
3

1
36. AM   a
2

AB  b  a
OC  3b  2a
3
ON  b
5
ON : NB
3 :2

37. RS  2  a  b 

PV  2  2a  b 
RQ  3  4a  b 
RV : RQ  1 : 3

320
2
AP  b  a 
3
1
MA  a
2
4b  a
MP 
6
38. AB  b  a 1
MX   4b  a 
2
MP 1

MX 3
3MP  MX
 MPX lies on the same strength line

39.

1
OC   a  3b 
4
1
CD   3b  a 
2
1
BN   a  b 
4
1
MN  OA
4
4 MN  OA
MN 1

OA 4
MN : OA  1: 4

321
40. PQ  4a

SY  2a
PY  2  b  a 

1
41. LA   2a  p 
2

2
AN   p  q
7
1
MN   46q  11 p 
84

42.

1
BD   2c  3a 
3
2
AF   a  c 
5
1
3F   2c  3a 
5
BD : FD  5 : 2

322
TRIGONOMETRY
It is a branch of mathematics that deals with the opposite a
sin   
relationship between sides and angles of a hypotenuse c
triangle. Trigonometry began to resemble its Adjacent b
present form from about 150 AD with the work cos  
hypotenuse c
of Aryabhata 1. The knowledge was transmitted
opposite a
to the Arabs and Europe, where a detailed tan   
adjacent b
knowledge account of trigonometric knowledge
appeared under the title De triangulis omni In order to remember the trigonometric ratios,
modis, written in 1464 by Regiomontanus. we remember SOHCAHTOA.
Originally, trigonometry may have been used to
measure the steepness of the faces of the S – sine, O – opposite, H – hypotenuse, A –
pyramids, but it was also used by astronomy. adjacent, C – cosine, T – tangent
Right Angled Triangles Example I
For right angled triangles, you will use either Solve the following triangles
Pythogoras theorem (if no angles are involved)
(i) 𝜃
or SOHCAHTOA (if there is an angle involved)
8 x
Pythogoras theorem
B
15
a c
ii.

C A
b
2 2 2
BC  CA  AB
a 2  b2  c2
(a and b are the shortest sides and c is the
hypotenuse)
B

c a
hypotenuse opposite
iii.
𝜃
A A
b
adjacent
The longest side AB is called the hypotenuse.
Side AC is said to be adjacent to angle  .
The hypotenuse is always opposite to the right
angle.
From the diagram above;

323
iv. 15
tan  
8
15
 tan  
8
15
  tan 1
8
  61.9 0

ii.
v.

Apply Pythogoras theorem on the above triangle


we have,
x 2  402  502
x 2  1600  2500
Solution x 2  2500  1600
i. x 2  900
x  30
 x  30 cm
40
cos  
50
4
cos  
5
4
  cos 1  
5
  36.90
Using Pythogoras theorem
iii.
82  152  x 2
64  225  x 2
x 2  289
x  17
x  17 cm

324
42  y 2  62
16  y 2  36 42  42  x 2
y 2  36  16 16  16  x 2
y  20
2
x 2  32
y  20  2 5 cm x  32
3 y x
2 2 2
4 2
9  20  x 2
x  2 8 cm
x 2  29 25°
x  29 3cm
x
 x  29 cm
iv.
3
sin 25 
x
3
x
sin 25
x  7.099 cm

Example II
N
6cm
5cm
𝛽 M
𝜃
L
In the figure above, angle MNL is 900, MN is 6
cm and NL = 5cm. calculate the value of
sin   cos  correct to two significant figures.
Solution
N
3  y 5
2 2 2
6cm
 9  y 2  25 5cm
𝛽
y 2  25  9 𝜃
M

y 2  16 L
Using Pythogoras’ theorem
y4

325
2
NL  NM  LM
2 2 Considering triangle ACD
y
52  62  LM
2
 cos30 
4
y  4cos30
2
25  36  LM
LM  61 cm  3
y  4  
6 6  2 
sin   cos  
61 61 y2 3
6 61 6 61
sin   . 
61 61 61
6 61 6 61
cos   . 
61 61 61
6 61 6 61
sin   cos   
61 61
61  6  6 

61
2 2 2
12 61  AD  DB  AB

2 3  2 3
61 2 2 2
 AB

4 3   4 3 
Example III 2 2 2
 AB
In the figure, AD is perpendicular to BC. AD =
DB, AC is 4cm and CAD is 300. Find AB 2
AB  24
 2 6 cm.

Example IV

Solution

In the figure, BN is perpendicular to AC. Find


the ratio of area of triangle ABN to the area of
ABC.
Solution
Let BN be y and AN be x

326
x 2  y 2  62 Solution
x 2  y 2  36
 y 2  36  x 2
y 2  16  x   142
2

  36  x 2   256  32 x  x 2  14 2
292  32 x  196
32 x  96 From triangle ACB;
x  3 cm AC
tan 420 
y 2  36  9 12
AC  12 tan 42 
 y 2  27
x
y  27 sin 350 
AC
 3 3 cm x
B sin 350 
12 tan 42
x  12 tan 42sin 35
x  6.197 cm
3 3 cm
Example V
A N C
Solve the following triangles
3cm 13cm
1
Area of ABC   16  3 3  24 3 cm2
2
1 9
Area of ABN   3  3 3  3 cm 2
2 2
Area of ABN 9 2 3 cm2

Area of ABC 24 3 cm2
Area of ABN 3

Area of ABC 16
⟹The area of ABN : Area of ABC = 3 : 16

Example V
Find the length of side marked x in the diagram
below correct to two significant figures
327
A

17
8

B D
(6 + x)
⟹82 + (6 + x)2 = 172
64  36  12 x  x 2  289
100  x 2  12 x  289
x
x 2  12 x  189  0
60
Comparing x2 + 12x – 189 = 0 with
y ax2 + bx + c;
z 55
a = 1, b = 12 and= -189
25 12  122  4(1)(189)
x
2(1)
x 𝜃 12  144  4 189 
x
Solutions 2
12  30
(i)
A x
2
17 x  9, x  21
y 10 ⟹x = 9, y=8

ii.
B 6 C x D

Considering triangle ABC

Considering triangle ABC


 112  y 2  612
121  y 2  3721
y2 + 62 = 100 y 2  3600
y2 = 100 – 62
y  60 cm
y2 = 64
y=8
Considering triangle ABD

328
z 2  562  y 2
z 2  3136  y 2
z 2  3136  652
z 2  1089
z  33 cm
z 2  x 2  552
1089  x 2  3025
Considering triangle BDC; x  44 cm
 x  11  60  65
2 2 2
 x  44, y  65 and z  33
x  22 x  121  3600  4225
2

x 2  22 x  3721  4225  0 Sign of cosine, sine and tangent in the


different quadrants
x 2  22 x  504  0
x 2  22 x  504  0
22   22   4  504 
2

x
2
22  50
x
2
x  14, x  36
But x ≠ -36
⇒ x = 14 and y = 60
B
x
D
60
From the diagram above;
y
In the first quadrant, the sine, cosine and tangent
A z 55 are all positive.
y x y
25 sin   cos   and tan  
r r x
C
x
In the second quadrant, only sine is positive;
cosine and tangent are both negative.
E
y x y
Applying Pythogoras theorem on triangle ABC sin   cos   and tan  
r r x
602  252  y 2
In the third quadrant, only tangent is positive.
3600  625  y 2
Both the cosine and sine are negative.
y 2  4225
y  65 y x y
tan   cos   and sin  
x r r

329
In the fourth quadrant, only the cosine is
positive but the sine and the tangent are
negative.
x y y
cos   , sin   , tan  
r r x
4
If sin A  , find the values of;
5
i) cos A
ii) tan A
iii) cos A + tan A
1 4
iv) sin A  For the 1st quadrant, if sin A 
tan A 5
Solution: 3
cos A 
4 Opposite 5
sin A  
5 Hypotenuse 4
For the 2nd quadrant, if sin A 
The sine is positive in the 1st and 2nd quadrant 5
3
cos A 
 5
In the 1st quadrant, if sin A  4 , tan A  4
5 3
4 4
In the 2nd quadrant, if sin A  , tan A 
5 3
4 3
In the first quadrant, sin A  , cos A  ,
5 5
4
tan A 
3
cos A  tan A :
3 4
 
5 3
9  20 29
 
15 15
29
 cos A  tan A 
15
4 3
In the second quadrant, sin A  , cos A  ,
x 4 5
2 2 2
5 5
x 2  16  25 4
tan A 
3
x2  9
x3

330
3 4
cos A  tan A  
5 3
9  20

15
29

15
4 4
sin A  , tan A 
5 3
For the 1st quadrant,
Using Pythogoras theorem
1 4 3
sin A    2 2 2
tan A 5 4 AB  BC  AC
16  15
 2
AB  82  17 2
20
31 2
 AB  64  289
20 2
1 31 AB  225
 sin A  
tan A 20 AB  15cm
1 31
For the 2nd quadrant; sin A   In the second quadrant;
tan A 20
15 15
sin   , tan  
Example II 17 8
8 In the third quadrant
Given that cos   for 00    2700 , find
17 15 15
sin   , tan  
without using tables or calculator the values of 17 8
sin  and tan 
Solution Example III
Note: the cosine is negative in the 2nd and 3rd 15
Given that tan   calculate without
quadrants. 8
using tables the values of 4cos  sin 

17
15
𝛼
8𝛼 8
15
17

Considering the 1st quadrant:

331
15
tan  
8
15
sin  
17
8
cos  
17
 8  15
4 cos   sin   4   
 17  17
32 15 17
  
17 17 17
 4cos   sin   1
Considering the 3rd quadrant
In the second quadrant
15 3 3 4
sin   tan x  ,sin x  and cos x 
17 4 5 5
15 4 3 1
tan  
8 cos x  sin x   
5 5 5
8
cos   In the fourth quadrant
17
 4 cos   sin 
3 4
 8  15 sin x  and cos x 
 4   5 5
 17  17 3 4 1
32 15 17 sin x  cos x   
   5 5 5
17 17 17
1
4 cos   sin   1  sin x  cos x 
5
Example IV
Example V
3
Given that tan x  and 00  x  3600 , Given that sin x  0.6 and x is obtuse without
4
using tables or calculator, find;
without using tables or calculator, find the
i) cos x
possible values of cos x  sin x ii) tan x
Solution
Note: the tangent is negative in the second and Solution
fourth quadrant Since x is obtuse, it implies that 90  x  180
6 6
sin x  0.6  
10 10

332
From Pythagoras’ theorem ; CN  3
Considering the triangle ACN
In the second quadrant
6 8 4
sin x  ,cos x  
10 10 5
6 3
tan x  
8 4
 cos x  0.8 tan x  0.75

Example VI 3 3
sin 60  cos 30 
3 2
Given that cos x  and x is a reflex angle, 2
5 1 1
sin 30  cos 60 
without using tables or calculator, find 2 2
(i) sin x (ii) tan x 1 tan 60  3
tan 30 
3
Solution
Note: x is reflex ⟹ (180° < x < 360°) If tan   1 ;

45°
1 2

45°
1
1 1
 cos 45  sin 45 
2 2
Example I
The length of the sides of an equilateral triangle
ABC is x units
3 3 Show that sin 60 
3
sin x  and tan x  i)
5 4 2
ii) Without using table or calculator, find
Trigonometrical Ratios of 300 and 600  sin 60 
2

Consider an equilateral triangle ABC of sides 2 the value of   tan 60 


 sin 30 
units
Solution:

333
A 2
 sin 60 
  tan 60 
30°30°  sin 30 
x 2
x  3 

2
1
 3
 2 

 
60° 2
60°
B x/2 N x/2 C 3 3
x
 
2
3  2 3
From Pythogoras theorem
2 2
 4  3  12
BN  NA  x 2 2
 sin 60 
  tan 60   12
2
x 2
   NA  x
2

2  sin 30 
x2 2
Example II
 NA  x 2 Find the exact value of the following
4
1 1
2 x2 cos 30  sin 45
NA  x 2  3 2
4
2 3x 2 tan 60  tan 45
NA  1  tan 60 tan 45
4
3x sin 2 30  cos 2 30
NA  Solution:
2
1 1
A cos 30  sin 45
30° 3 2
x 3 1
cos30  ,sin 45 
2 2
60° 1 1
B N
 cos30  sin 45
x/2 3 2
AN
sin 60  1  3 1  1 
   
3  2 
BN
2 2
 3x  1 1
   0
sin 60   2  2 2
 x  1 1
   cos30  sin 45  0
  3 2
3 tan 60  tan 45
sin 60   as required 
2 1  tan 60 tan 45
x
x 1 1
sin 30   2

x 2 x 2
3
x 3 1
tan 60  12    3
2 x 2 2
tan 60  3

334
3 1

1 3


 
3 1 3 1 
1 3  3  1

 3  1
2


1 3
1 2 3  3

2
42 3

2 Note: angles measured from the x-axis in an
 2  3 anti-clockwise sense are termed as positive
angles while those measured in a clockwise
tan 60  tan 45
  2  3 sense are negative angles.
1  tan 60 tan 45
cos120   cos 60  0.5
sin 2 30  cos 2 30
1 3 sin120  sin 60   3 
sin 30  , cos30   2
2 2
2 tan120   tan 60   3
1  3
2

sin 30  cos 30     
2 2
 Example IV
 2   2  Without using tables or calculator, find the value
1 3 of
 
4 4 sin 390°
4 cos 780°
 1
4 Solution
 sin 30  cos 30  1
2 2

Example III
1
Given that sin 60  3 and cos 60  , without 30°
2 2
using tables or calculator, find the value of
cos120
sin120
tan120 1
Solution sin 390  sin 30 
2

335
Trigonometric Equations
Example I
Find the values of  lying between 00 and 3600
such that;
a) cos  0.3256
b) sin   0.9388
Solution:
cos   0.3256
  cos 1  0.3256 
1   70.998
cos 780  cos 60 
2   71
cos is positive in the first and fourth
Example V quadrants
Without using tables or calculator, find the 
values of
i) sin 210
ii) cos 210
iii) sin150
iv) tan 210
Solution:

150°
30°   289
30°  If cos  0.3256
  71 and   289
sin   0.9388
The sine is negative in the 3rd and fourth
1 quadrants.
sin 210   sin 30  
2 Note: we first find the acute angle whose sine is
3 0.9388
cos 210   cos 30  
2 If sin   0.9388
sin150  sin 30 
1   sin 1  0.9388
2
1   69.9
tan 210  tan 30 
3 But sine is negative in the 3rd & 4th quadrants.

336
  249.9
  290.1  if sin  
3
,
Example II 2
Solve the following equations for the values of   60 and   120
 from 00 to 3600 inclusive. tan    3
a) sin  
3 tan    3
2
First find the acute angle whose tan is 3
b) tan    3
If tan   3
c) cos3  0.866

d) sin 2  60   0.9272   tan 1  3
e) 2sin 2   sin   1  0   60
Solution: The tan is negative in the 2nd and 4th quadrants.
3
sin  
2
 3
  sin 1  
 2 
  60
But the sine is positive in the first and the second
quadrant,

 if tan    3
  120 and   300
cos 3  0.866
3  cos 1  0.866 
3  30

337
But the cosine is positive in the first and fourth 2 x2  x  2 x 1  0
quadrant x  2 x  1  1 2 x  1  0
1
x  1 and x  
2
1
 sin   1 and sin   
2
if sin   1
  sin 1 1
  90
1
if sin   
2
 3  30 ,3  330
  10 ,   110 Note: first get the value of  whose sine is 1
2
 if cos 3  0.866 1
 sin  
  10 ,   110 2
sin  2  60   0.9272 1
  sin 1  
2
 2  60   sin  0.9272
1

  30
2  60  68
2  60  112
2  60  68
2  8
 4
2  60  112
2  52
  26
 if sin  2  60   0.9272   330 and   210
  4 and   26    90 , 210 and 330
2sin 2   sin   1  0 Example III
let x  sin  If cos x  0.634 for 90  x  270 , find the
2x2  x 1  0 two possible values of x
2x2 – x – 1 = 0
Solution
cos x  0.634
– 2, 1
Note: first get the value of the acute angle whose
cosine is 0.634

338
cos x  0.634 cos 2 x  0.4
x  cos 1
 0.634  2 x  cos 1  0.4 
x  50.7 2 x  66.4
x  33.2
cosine is negative in the 2nd and 3rd quadrants
2 x  293.6
x  146.8
 if cos 2 x  0.4
x  33.2 and x  146.8

sin 2 x  0.8
2 x  sin 1  0.8 
2 x  53.1
2 x  126.9
if cos x  0.634 x  26.55
x  129.3 x  63.45
x  230.7 1
tan x  0.5
3
Example IV 1
x  tan 1  0.5
Solve the following equations in range indicated 3
from 0  x  180 1
x  26.6
a) sin 2 x  0.6 3
b) cos 2x  0.4 x  79.8
c) sin 2x  0.8 1
x  206.6
1 3
d) tan x  0.5
3 x = 619.8°
e) tan  2 x  45   1 ⟹ x = 79.8°
tan  2 x  45   1
Solution:
sin 2x = 0.6 2 x  45  tan 1 1
2x = sin-1(0.6) 2 x  45  45
2x = 36.9°
2 x  90
2x = 143.1°
x = 71.55° x  45
x = 18.45° 2 x  45  225
⟹ sin 2x = 0.6 x  135
x = 18.45° and x = 71.55°

339
Trigonometric Graphs
For any angle  , a single value of sin  or
cos can be found. The same applies for tan 
except when   90  270 , for which the
values of tan𝜃 are not defined, thus sin  and
cos are functions which are defined for all
positive and negative values of  . tan  Is a For y  tan x
function which is defined for all positive and
x -360 -270 -180 -90 0 90 180 270 360
negative values of  except 90 , 270 ...... . To
tan x 0  0  0  0  0
draw the graphs of sin  , cos and tan  , we
construct a table of values giving ordered pairs
for these functions and hence plot the graph.

Example I
Draw the graphs of y  cos x , y  sin x and
y  tan x for 360  x  360
For y  sin x
x sin x
-360 0
-270 +1
-180 0 Example II
On the same axes, draw the graphs y  5sin 
-90 -1
0 0 and y  5sin 2 for 0    360 . What is the
90 +1 amplitude of each wave? What is the period?
180 0 Find the values of  where 5sin   5sin 2
270 -1
Solution
360 0
y = 5sin 𝜃
θ sin θ 5sin θ
0 0 0
30 0.5 2.5
60 0.87 4.4
90 1 5
120 0.87 4.4
150 0.5 2.5
180 0 0
210 -0.5 -2.5
For y = cos x 240 -0.875 -4.4
x -360 -270 -180 -90 0 90 180 270 360 270 -1 -5
cos x 1 0 -1 0 1 0 -1 0 1 300 -0.87 -4.4
330 -0.5 -2.5
360 0 0

340
y = 5sin 2𝜃 180 360 0 0
θ 2θ sin 2θ 5sin 2θ 210 420 0.87 4.4
0 0 0 0 240 480 0.87 4.4
30 60 0.87 4.4 270 540 0 0
60 120 0.87 4.4 300 600 -0.87 -4.4
90 180 0 0 330 660 -0.87 -4.4
120 240 -0.87 -4.4 360 720 0 0
150 300 -0.87 -4.4

The amplitude of both graphs is 5 units


(maximum displacement) Solution:
x 3x cos 3x
For the graph y  5sin  , the cycle repeats 0 0 1
itself after 1800 15 45 0.7
30 90 0
 The period of the curve y  5sin  is 1800 45 135 -0.7
For the graph y  5sin 2 , the cycle 60 180 -1
repeats itself after 900 75 225 -0.7
 The period of the curve y  5sin 2 is 900 90 270 0
105 315 0.7
Example III 120 360 1
Draw the graph of the curve y  cos 3 x for 135 405 0.7
0  x  150 . Using your graph, determine the 150 450 0
values of 
x 0  x  150 for  which For
4cos3x + 3 = 0
4cos3x  3  0
4cos 3x = -3
3
cos 3 x 
4

341
4cos 3x + 3 = 0 3
y
Cos 3x = y 4

From the graph, the values of  for which of 2 cm for 0.5 units draw a graph of
3 sin 2
cos 3 x  are   46.5 and   73.5
4 c) From your graph, find the values of 
for which sin 2  0.6
Example IV
Solution:
a) Draw a table showing the values of
 0 15 30 45 60 75 90
sin 2 for 0    90 using values of 2 0 30 60 90 120 150 180
 at intervals of 150 sin 2 0 0.5 0.9 1 0.9 0.5 0
b) Use the table in (a) above, a horizontal
scale of 4cm for 150 and a vertical scale

If sin 2  0.6

342
   18.5
  71.25
Example V:
Draw the graph of y  sin 3 for values of  from 00 to 3600. Hence solve the equation sin 3  0
Solution:
𝜃 3𝜃 sin3𝜃
0 0 0
30 90 1
60 180 0
90 270 -1
120 360 0
150 450 -1
180 540 0
210 630 1
240 720 0
270 810 1
300 900 0
330 990 -1
360 1080 0

For sin 3  0
  60 ,120 ,180 , 240 ,300 ,360

Example VI
Draw the graph of 3sin   cos for values of 0    360 . Hence solve the equation
3sin   cos  0.5

343
Solution:
 0 30 60 90 120 150 180 210 240 270 300 330 360
3sin  0 1.5 2.6 3 2.6 1.5 0 -1.5 -2.6 -3 -2.6 -1.5 0
- cos𝜃 -1 -0.87 -0.5 0 0.5 0.87 1 0.87 0.5 0 -0.5 -0.87 -1
y =3sin𝜃 - cos𝜃 -1 0.63 2.1 3 3.1 2.37 1 -0.63 -2.1 -3 -3.1 -2.4 -1

  28.5 ,  189 , for 3sin   cos  0.5 Example I


Calculate the area of the triangles below
Area of Any Triangle (i) C

Consider a triangle ABC in which A, B, C are 53°


angles of a triangle and a, b, c are lengths of
11cm 12cm
sides opposite these angles.

A B

(ii) C

6cm

Area of the triangle ABC 61°


A B
1 1 1 4cm
A  bc sin A  ac sin B  ab sin C
2 2 2 Solution:
OR A  s  s  a  s  b  s  c  (Heron’s
formula)
abc
s
2

344
A  s  s  a  s  b  s  c 
(i) C
53°
13  13  24
11cm 12cm
s
2
s  25
Area = 25  25  24  25  13 25  13
A B
1  25 112 12 
Area of the triangle ABC  ab sin C
2  60 sq.units
1
  11 12  sin 53 Example III
2
Calculate the areas of the following triangles.
 52.71 cm 2
(ii) C
i.

6cm

61°
A B
4cm
1
Area = bc sin A
2
1 ii.
Area of the triangle   6  4  sin 61
2
= 10.495cm2

Example II
Find the area of the triangle whose sides are 24
cm, 13 cm and 13 cm.

Solution:

Solution:
i) A + B + C = 180°
77  38.7  A  180
115.7  A  180
A  64.3
1
Area = bc sin A
2
1
  8.6  5.5  sin 64.3
2

345
= 21.3105 cm2 a b c
 The formula   is
1 sin A sin B sin C
ii) A   6  4  sin123
2 called the Sine formula for a triangle
2
= 10.064 cm
Example I
In a triangle ABC, angle A = 400, angle B = 800
Sine and Cosine Rule and b = 5cm. find the length of side a.
Solution:
Sine Rule C
The formula for the area of a triangle A
1 1 1
A  bc sin A  ac sin B  ab sin C 5cm a
2 2 2
1 1 1
bc sin A  ac sin B  ab sin C.......()
2 2 2 40° 80°
A B
Multiplying the above equation by 2
bc sin A  ac sin B  ab sin C a b c
From the sine rule  
 bc sin A  ac sin B and sin A sin B sin C
ac sin B  ab sin C  a   b 
  
 sin A   sin B 
From
a 5
bc sin A  ac sin B 
sin 40 sin 80
b sin A  a sin B.....................(1) 5sin 400
a
Dividing equation (1) by sin Asin B sin 800
b sin A a sin B a  3.2635 cm

sin A sin B sin A sin B
b a
  ....................(2) Example II
sin B sin A
In a triangle ABC, angle C = 1000, b = 4 cm and
From ac sin B  ab sin C
c = 5cm. find the size of angle B?
c sin B  b sin C......................(3)
Dividing equation (3) by sin B sin C Solution
c sin B b sin C

sin B sin C sin B sin C
c b
 .........................(4)
sin C sin B
Comparing equation (2) and (4)
c b a
  
sin C sin B sin A

From the sine rule

346
a b c (iv)
 
sin A sin B sin C 60°
b c
  4.1
sin B sin C
4 5

sin B sin100 25°
4sin100 x
sin B 
5 Solution
 4sin100 
B  sin 1   i.
 5 
B  51.98

Example III
Find x in the following triangles

i.
From the sine rule
a b c
 
sin A sin B sin C
a c
 
sin A sin C
2.3 1.2

sin 88 sin x
(ii) C
1.2sin 88
sin x 
100° 2.3
1.2sin 88
5.3m sin x 
2.3
 1.2sin 88 
40°
x  sin 1  
A
x
B  2.3 
x  31.4
(iii) (ii) C

140° 100°

x 3
5.3m

15° 40°
A B
x
From the sine rule

347
a b c From the sine rule
 
sin A sin B sin C b c

b c sin B sin C
 
sin B sin C 4.1 x
 
5.3 x sin 40 sin 60

sin 40 sin100 4.1sin 60
5.3sin100 x
x sin 40
sin 40 x  5.52 m
x  8.12 m Cosine Rule
iii. The sine formula is easy to remember and easy
to use but it cannot be applied if you know the
lengths of the two sides and one angle, and you
wish to find the length of the third side.

Proof of the cosine rule


Consider the triangle ABC
C

A  B  C  180
a
15  B  140  180 b h
155  B  1800
B  180  155
A x c-x B
B  250 D
a b c
 
sin A sin B Using Pythogoras theorem on triangle ACD

3

x x2  h2  b2
sin15 sin 25
3sin 25
 h 2  b 2  x 2 ............(1)
x
sin15
Applying Pythogoras theorem on triangle DBC
x  4.8986 m
h2   c  x   a 2
2

iv.
h2  a 2   c  x  .........(2)
2

Equating equation (1) and equation (2)


 b2  x 2  a 2   c  x 
2

b2  x 2  a 2  c 2  2cx  x 2 
b2  x 2  a 2  c 2  2cx  x 2
a 2  b2  c 2  2cx.................(3)
From triangle ADC

348
x
cos A 
b
x  b cos A
Substituting x  b cos A in equation (3)
a 2  b 2  c 2  2bc cos A
a 2  b 2  c 2  2bc cos A is called the Cosine
formula for a triangle
Note: The formula a 2  b 2  c 2  2bc cos A is
symmetrical in letters a, b, c and A, B and a 2  b 2  c 2  2bc cos A
C. there are two other formulae like it.  42  52  82  2  5  8  cos A
These two formulae;
16  25  64  80 cos A
b2  a 2  c2  2ac cos B
80 cos A  25  64  16
c2  a 2  b2  2ab cos C 80 cos A  73
Example I 73
In a triangle, a = 4 cm, b = 7 cm and Angle C =
cos A 
80
730. Find the length of side c
A  24.1
Solution: b  a  c 2  2ac cos B
2 2

C
52  42  82  2  4  8  cos B
73°
25  16  64  64 cos B
7cm 4cm 64 cos B  16  64  25
55
cos B 
A B 64
c
B  30.8
c 2  a 2  b 2  2ab cos C
c 2  7 2  42  2  7  4  cos 73 c 2  a 2  b 2  2ab cos C
c  6.97 cm 82  42  52  2  4  5  cos C
64  16  25  40 cos C
Example II
23
The three sides of a triangle have lengths 5 cm; 4 cos C 
cm and 8 cm. find the largest angle of the 40
triangle.  23 
C  cos 1  
Solution:  40 
C  125.1
The largest angle of the triangle is 125.1°

Example III

349
From a point P, town A is 20 km away on a When an observer looks upwards at some
bearing of 0400 and town B is 30 km away on objects above his/her eye level, his/her line of
the bearing 1260. Find; sight is elevated at an angle  above the
a) The distance of B from A. horizontal. The angle between the horizontal and
b) The bearing of A from B. his line of sight is called the angle of elevation.
θ
A

θ
20km tree

Joel B

40°
From the diagram above, Joel was looking
86°
P
straight ahead and then turned his head to look
30km
up to the top of a tree. The angle through which
B he turned her head is called the angle of
AB  20  30  2  20  30  cos86
2 2 2 elevation.  

AB2  400  900  1200cos86


Angle of Depression
AB2 = 1216.292232
If the object O observed is lower than the
AB = 34.88 km
observer’s eye, the line of sight is depressed at
a2 = p2 + b2 – 2pbcosA
an angle  below the horizontal. The angle
302 = 34.882 + 202 – 2(20)(34.88)cos A
between the line of sight and the horizontal is
900 = 1216.6144 + 400 – 1395.2cosA
called the angle of depression.
716
cos A 
1395.2
A  59.1
A

40° 19.1°

40°

86°
P

Brian stands on the top of a house wall which is


B
near a lake and then turned his head down to see
a boat. The angle through which he turned his
Depression and Elevation head is called the angle of depression.
Angle of Elevation Example I

350
A church tower is 100 m high. Find the angle of 42
elevation of its top from the point on the ground
tan 52 
x
160 m away on the level ground. x tan 52  42
Let the angle of elevation be  x
42
tan 52
x  32.814 m

 The flower bed is 32.814 m from the tower.

Example III
A girl is looking at the top of a tree. The girl is 2
m tall and the angle of elevation of the bird on
top of a tree from the girl is 300. How high is the
tree if the girl is 30 m away from the tree?

Solution:
Opposite
tan  
Hypotenuse
100
tan  
160 h
H
 100 
  tan 
1

30°
30m
 160  2m B

  32 h
tan 30 
30
Example II
From the top of the tower, the angle of h  30 tan 30
depression of a flower bed is 520. If the tower is h  17.3205 m
42 m high, find the distance from the tower to
the flower bed Let the height of the tree be H

Solution  H  2  17.3205
H  19.3205 m

 The height of the tree is 19.3205 m

Example IV
A boy is looking at the top of the building which
is on the ground that levels with him. He
measures the angle of elevation of the top of the
building 320. He walks 30 m towards the
building and finds that the angle of elevation of
Opposite the top of the building is now 450. The boy is 2.3
tan   m tall. How far was he from the building when
Hypotenuse
he started?
351
Solution: Solution:

H
tower

28° 35°
100m (x-100)
x

h H
tan 45  tan 28 
 x  30  x
h   x  30  tan 45 H  x tan 28............................(1)
h  x tan 45  30 tan 45.............(1) H
tan 35 
h x  100
tan 32 
x H   x  100  tan 35
h  x tan 32..............................(2) H  x tan 35  100 tan 35.........(2)
Equating equation (1) and equation (2)
Equating equation (1) and equation (2)
⟹ x tan 45° – 30 tan 45° = x tan 32°
x tan 28  x tan35 100tan35
x tan 45  x tan 32  30 tan 45
x tan 35  x tan 28  100 tan 35
30 tan 45
x x  tan 35  tan 28   100 tan 35
tan 45  tan 32
100 tan 35
x  79.972 m x
tan 35  tan 28
 The boy was 79.972 m from the building.
From equation (1)
h  x tan 32
30 tan 45 tan 32 H  x tan 28
h
tan 45  tan 32 H
100 tan 35
tan 28
h  49.972m tan 35  tan 28
H  220.956 m
Height of the building   2.3  h 
= 2.3 + 49.972 Example VI
A flag mast slants towards the west at an angle
= 2.3 + 49.972
of 130 to the vertical from the point M to the east
Example V and 20 m away from the foot F of the mast. The
At a certain point on the level ground, the angle angle o elevation of the top P of the mast is 350
of elevation of the top of the tower T is 280. At from another point N, west of the flag mast; the
another point 100 m away from the first point, angle of elevation of the top P is 220. If M, N
the angle of elevation is 350. Find the two and F are on the level ground,
expressions for the height of the tower and hence i) determine the vertical distance of the top
the height. P from the ground.

352
ii) determine the distance of the foot of the 20 tan 35
But x 
mast F from N. tan 77  tan 35
iii) determine the length of PF. x  3.8565 m
Solution: ON  x  41.3451  3.8565
 45.2016 m
13°
 The distance of the foot of the mast from N
is 45.2016.
H
H
sin 77 
PF
22° 35° H 16.7045
N O
77°
20m
M PF  
x F sin 77 sin 77
H PF  17.1439 m
tan 77 
x
Example VII
H  x tan 77...........................(1)
The angles of depression of the two boats from
H
tan 35  the top of the cliff are 390 and 600. The boats are
 x  20  in a straight line from the point of observation.
H   x  20  tan 35 How far are the two boats if the cliff is 400m
high?
H  x tan 35  20 tan 35
Solution:
H  x tan 35  20 tan 35...........(2)
P
39°
Equating equation (1) and equation (2) 60°
 x tan 77  x tan 35  20 tan 35
x tan 77  x tan 35  20 tan 35 400

x  tan 77  tan 35  20 tan 35


60° 39°
20 tan 35
x O A B
tan 77  tan 35 400
20 tan 35 tan 77 tan 39 
H OB
tan 77  tan 35 400
OB 
H  16.7045 m tan 39
 The vertical distance of the top P is OB  493.959 m
16.7045m. 400
tan60 
H OA
tan 22 
ON 400
OA 
ON 
H  tan 60 
tan 220
OA  230.940 m
16.7045
ON  OB  OA  493.959 m  230.940 m
tan 22
ON  41.3452 =263.019 m
ON  x  41.3452  x  The two boats are 263.019 m apart.

353
TRANSFORMATION
Transformation means a change. There are a number If the point P (2, -1) undergoes a translation
of ways in which a figure may be changed in shape,  11 
size or direction. A transformation is a one to one represented by the matrix   , find the image of P.
mapping which moves a plane, figure (object) from  4 
one position to another (image). This means that Solution
under transformations each object point has exactly  column vector   column vector 
one image. In some transformations the lengths      translation 
remain the same (isometric) while in others they
 of image   of object 
change. The object and the image may be directly  2   11 
 image =     
congruent.  1  4 
Types of transformations
 Translations  13 
= 
 Enlargement  5 
 Reflection  P ' (13, -5)
 Transformations  The image of P is (13, -5)
Translations Example II
Under a translation, every point or a plane figure
moves the same distance. For any translation, the  4
To what position does the translation   move a
following must always be true.  2
i. Each point of the object moves through the same point A (5, 4)?
distance Solution
ii. Movement must be in the same direction
 column vector   column vector 
iii. The image and the object are directly congruent      translation 
 of image   of object 
 5  4
column image of A =     
 4  2
9
= 
6
 The image of A (5, 4) is (9, 6)
Example III
 3
When the translation   was applied to a point A,
 2 
it mapped onto the point A'(1, -4), what are the
From the diagram above, the translation of triangle coordinates of A?
PQR will look as shown. When a triangle PQR Solution
undergoes a translation, the image of PQR becomes
 column vector   column vector 
P'Q'R'. In vectors PP' represents a vector translation      translation 
from object point P to image point P'. Similarly, RR'  of image   of object 
represents a vector translation from object point R to  1   column vector   3 
image point R'.   =  
 4   of object   2  
a
Note: When a translation is written in the form   it  column vector   1   3 
b       
 of object   4   2 
is referred to as a column vector.
For a translation, the following relation is used  2 
 
 column vector   column vector   2 
     translation  The coordinates of A are (-2, -2)
 of image   of object 
Example I Example IV

354
A (2, 4), B (6, 4), C (6, 7) and D (2, 7) are vertices of B B'
a rectangle ABCD. Displace it through a translation 1 6 7
     
1  2  4  6
  in a Cartesian plane. C C'
 2 1 6 7
a. Find the coordinates of the vertices of the image.      
b. Using the same axes plot the object and image  2 7 9
D D'
rectangles
 1   2  3
Solution      
a. A (2, 4) B (6, 4) C (6, 7) D (2, 7)  2 7 9
A A'  A'(3, 6) B'(7, 6) C'(7, 9) D'(3, 9)
 1  2  3
     
 2  4 6
 The image of (-3, -4) is (-4, 1)
 1   1  0 
     
 6   5  11
 The image of (1, 6) is (0, 11)
 0   1  1
     
 0  5   5 
 The image of (0, 0) is (-1, 5)

Enlargement
Enlargement is a transformation under which an
object is made bigger or smaller. However,
enlargement is not possible without the scale
factor (linear scale factor). The linear scale factor,
k, of the enlargement tells us how many times the
length of the image is bigger or smaller. Under
enlargement, each angle remains the same, that is,
the image is similar in shape to the object. If the
linear scale factor, k, is greater than 1, the image
is larger. If 0  k  1 , the image is smaller.
Example V If k is negative, the image is on the opposite side
A'(2, 9) is the image of A (3, 4). Find the images of the object. If k is positive, the image is on the
of (-3, -4), (1, 6), (0, 0) under the same same side as the object.
translation. image distance fromthe
Solution
centre of enlargement
 column vector   column vector  L.S.F (k)=
     translation  object distance from the
 of image   of object  centre of enlargement
 2   3   translation 
  =  
But since displacement is the distance moved in given

 9   4   matrix  direction,
image displacement from the centre
 translation   2   3   L.S.F (k)=
      object displacement from the centre
 matrix  9  4
 1
 
5 Example I
Under enlargement of scale factor 2, the image of (3,
 3   1  4 
     
4) is (5, 3). Find the coordinates of the centre of
 4   5   1  enlargement
Solution
355
Centre of enlargement is given by the formula 1
1 centre =  k  object - image 
centre =  k  object - image  k 1
k 1 x 1   2   6.5  
   2.5      
x 1   3   5   y  2.5  1   3   4.5  
   2    
 y  2 1   4   3   1   5   6.5  
=    
 6 5 1  1.5   7.5   4.5  
=1       = 
8  3 5 1  1.5 
3  3 
=
 (x, y) = (1, 5) 2
Alternatively
2  1.5 
image displacement from =  
3 3 
the centre
L.S.F =  1
object displacement from  
2 
the centre  The centre of enlargement is (-1, 2)
x Alternatively
Let the centre of enlargement be   image displacement from
 y
the centre
5  x  L.S.F =
   object displacement from
2    
3 y
the centre
3  x 
    6.5  x 
 4  y   
4.5  y 
  3   x    5  x   2.5  
2           2 x
 
  4   y    3  y  3 y 
3 x  5  x   2  x   6.5  x 
2   2.5   
4  y 3 y  3  y   4.5  y 
 6  2x   5  x   5  2.5 x   6.5  x 
  
    7.5  2.5 y   4.5  y 
8  2y  3  y 
5  2.5 x  6.5  x
Equating the corresponding components
 6  2x  5  x  1.5 x  1.5
x  65 x 1
x 1 7.5  2.5 y  4.5  y
8 2y  3 y 7.5  4.5  1.5 y
y  83 3  1.5 y
y5 y2
 The centre of enlargement is (1, 5)  The centre of enlargement is (-1, 2) as before

Example III
Example II The image of the point (4, -7) under enlargement of
Under enlargement of scale factor 2.5, the image of scale factor -2 is (1, 2). Determine the coordinates of
(2, 3) is (6.5, 4.5). Find the coordinates of the centre the centre of enlargement.
of enlargement Solution
1
Solution
centre =  k  object - image 
Centre of enlargement k 1

356
x 1   4  1   5 1  1   x  
   2          4      
 y  2  1   7   2    6  4  1   2   y  
1  8  1  5 1  4  x 
=     =  
3 14  2  6 5  8  y 

1  9   4  x 
=    5   5 
3 12    = 
 6   8  y 
3   
=   5 
 4   4 x
 The centre of enlargement is (3, -4) 5  5 
Alternatively   
6  8  y 
1  x  
 5 

 
2 y 4  x  25
 2  
4 x  x  21
 
 7  y  8  y  30
 4  x  1  x  y  22
-2     The image of the point (1, 2) is (21, 22)
 7  y   2  y 
Alternatively
 8  2 x   1  x 
   x5
 14  2 y   2  y   
y  6
-8  2 x  1  x 4  
1  5 
3x  9  
 2  6
x3 x5
14  2 y  2  y  
y  6
4  
3 y  12  4 
 
y  4  4 
 The centre of enlargement is (3, -4) as before  4   x  5 
-4     
 4   y  6 
Example IV 16   x  5 
  
Determine the image of each of the given point under 16   y  6 
the enlargement with stated centre and scale factor.
 x   21 
  
1. (1, 2) centre (5, 6) and scale factor -4
1  y   22 
2. (1, 1) centre (10, 13) and scale factor
3  The image of (1, 2) is (21, 22) as before
3. (3, 2) centre (0, 0) and scale factor 5
Solution
1 2. Object (1, 1)
1. centre =  k  object - image  Centre (10, 13)
k 1
1
x Scale factor k =
Let the image be   3
 y x
Let the image be  
 y

357
10  1  1  1  x    0 1  3  x 
          5     
13  1  1  3 1  y    0  5 1   2   y  
3  0  1 15  x 
1   =  
x  0  4 10  y 
10  3  3
  =    15  x 
13  2  1  y 
  0  4 
3    = 
 0   10  y 
10   1   
   x  4 
13  =  3  x  15, y  10
3 1  y  The image of (3, 2) is (15, 10)
 
2 3  Graphical method of finding the centre of
 20   1  enlargement and linear scale factor
 3  3x Example I
   Consider a triangle PQR and its image P'Q'R' whose
 26   1  y 
    coordinates are P (1, 1), Q (2, 4), R (3, 1), P'(2, 2),
 3  3  Q'(4, 8), R'(6, 2) respectively.
 x  7 We can plot the object and the image triangle on the
   same axes
 y 9
 The image of (1, 1) is (7, 9)
Alternatively
 x  10 
 
1  y  13 

3 1  10 
 
 2  13 
1  9   x  10 
  
3  12   y  13 
 3   x  10 
  
 4   y  13 
 x  7
  
 y 9
 The image of (1, 1) is (7, 9) as before

3. Object (3, 2)
Centre (0, 0)
Scale factor 5
Joining RR', PP' QQ' and extending these joining lines
until they intersect at X which is known as the centre
of enlargement X (0, 0)
 The centre of enlargement is (0, 0)

358
XR ' 8 Since the linear scale factor is negative, it implies that
 2
XR 4 the image is on the opposite side of the object.
6
XP ' 4
 2 M 5
XP 2
L
XQ ' 12 4
 2
XQ 6 3

XR ' XP ' XQ ' 2


  2k
XR XP XQ K 1 K'
 The linear scale factor is 2
-7 -6 -5 -4 -3 -2 -1 0 1 2 3 4
-1
Example II
D' A' -2
On the same axes plot the squares P(2, 2), Q (4, 2), R
(4, 4), S (2, 4) and P'(1, 1), Q' (2, 1), R'(2, 2), S'(1, 2). -3
Note: P'P, Q'Q, R'R and S'S meet at a common point -4
called the centre of enlargement. L' -5

-6
-7 M'

-8
⟹ K'(1, 1) L'(-1, -5) M'(3, -7)
Example IV
Under an enlargement, triangle ABC with vertices at
A(3, 1), B(9, 1) and C(9, 10) was mapped into triangle
A'B'C' with vertices at A'(7, 5), B'(5, 5) and C'(5, 2).
State the linear scale factor and coordinates of the
centre of enlargement.

11 C

10

9
8

7
XR' 4
LSF   6
XR 8
1 5 B' A'

2 4
XP' 2
LSF   3
XP 4
2 C'
1
 1 A B
2
0 1 2 3 4 5 6 7 8 9
10 '
XB 1

Example III XB 3
Triangle KLM has vertices at K (-5, 1), L (-4, 4) and 1
M (-6, 5). Find its image K'L'M' after an enlargement LSF 
3
with scale factor -2, centre (-3, 1).
Centre of enlargement X (6, 4)
Solution
See the graph below

359
Transformations A parallelogram whose vertices are Q(7, 3), R(-5, -1),
When we pre-multiply the matrix of transformation by S(7, 5), T(19, 9) is transformed using the matrix
the object we get the image.  1 2 
 transformation   object   image    . Find the coordinates of Q'R'S' and T'
     0.5 1.5 
 matrix   matrix   matrix  Solution
Example I  transformation   object   image 
A rectangle ABCD with vertices at A(0, 0), B(0, 2),    
C(3, 0) and D(3, 2) is transformed under the matrix  matrix   matrix   matrix 
 3 1  image   1 2  7  5 7 19 
   
  . Find the coordinates of the vertices of the  matrix   0.5 1.5  3 -1 5 9 
 1 1
image A'B'C'D'  1×7+-2×3 1×-5+-2×-1 1×7+-2×5 1×19+-2×9 
 
Solution  -0.5×7+1.5×3 -0.5×-5+1.5×-1 -0.5×7+1.5×5 -0.5×19+1.5×9 
 transformation   object   image 
     7-6 -5+2 7-10 19-18 
 matrix   matrix   matrix   
 -3.5+4.5 2.5+-1.5 -3.5+7.5 -9.5+13.5 
 3 1  0 0 3 
A B C D
3    image   1 -3 -3 1 
     
 1 1  0 2 0 2   matrix  1 1 4 4 
 3×0+1×0 3×0+1×2 3×3+1×0 3×3+1×2   Q'(1, 1), R'(-3, 1), S'(-3, 4) and T'(1, 4)
 
1×0+1×0 1×0+1×2 1×3+1×0 1×3+1×2 
 0 2 9 11
   image matrix
0 2 3 5 
 A'(0, 0), B'(2, 2), C'(9, 3), D'(11, 5)
Example II
The vertices of a triangle ABC where A (-3, -2), B (3,
-2) and C (0, 4) are mapped into its image by a matrix
 2 1
transformation   . Find the coordinates of
2 3 
A'B'C'
Solution
 transformation   object   image 
   
 matrix   matrix   matrix 
 2 1 -3 3 0   image 
   
 2 3  -2 -2 4   matrix 
 2  -3  -1 -2 2  3  -1 -2 2  0  -1 4 
 
 2  -3  3  -2 2  3  3  -2 2  0  3  4 
 -6  2 6  2 0  -4 
 
 -6  - 6 6  - 6 0  12 
 -4 8 - 4 
   image matrix
 -12 0 12 
A'(-4, -12), B'(8, 0), C'(-4, 12)
Example III

360
Example IV 3 y  2  10
1 y   3 y  12
Under a matrix transformation of   , the point
 x 4  y4
A(3, -2) is mapped into A'(1, 17). Find the values of x x  9  8
and y.  x  98
Solution x 1
 transformation   object   image   x = 1 and y = 4
   
 matrix   matrix   matrix 
Example VI
 1 y  3  1 
      Find the matrix of transformation for which
 x 4  2  17  A(3, 0)  A'(3, 6) and B (0, -4)  B'(-8, 4).
1 3  y  2  1  Solution
    transformation   object   image 
 x  3  4  2  17     
 3  2 y  1   matrix   matrix   matrix 
   T  O=I
 3 x  8   17 
a c 
3 2y 1 let T =  
b d 
 3 1  2 y
 a c   3 0   3 8 
2  2y    
 b d   0 4   6 4 
y 1
 a  3  c  o a  0  c  -4   3 8 
3 x  8  17   
 b  3  d  0 b  0  d  -4   6 4 
 3 x  17  8
 3a 4c   3 8 
3x  9   
x3  3b 4d   6 4 
 x = 3 and y = 1 3a  3
a 1
Example V 3b  6
2 y b2
A transformation described by   maps
 x 3 4c  8
M'(-1, 3) into its image m’ (10, 8). Find the values of c2
x and y.
4 d  4
Solution
d  1
 transformation   object   image 
    1 2 

 matrix   matrix   matrix  
 2 1
 2 y  1 10  Example VII
     
 x 3  3   8  If A (2, 0)  A'(2, 4) and B (3, 1)  B'(2, 7). Find the
 2  1  y  3  10  matrix of transformation under this transformation.
   Find the coordinates of the image point P(4, 4).
 x  1  3  3   8  Solution
 3 y  2  10   transformation   object   image 
      
 x  9  8   matrix   matrix   matrix 

361
T  O=I b + 3d = 2
a c  b=2
let T =   0 2
b d   
 a c   2 3  2 2  2 0
    0 2
 b d  0 1  4 7    is the matrix of the transformation
 a  2  c  0 a  3  c 1   2 2  2 0
   Transformation and area scale factor
 b  2  d  0 b  3  d  1  4 7 
The determinant of a matrix of transformation
 2a 3a  c   2 2   Area Scale Factor
  
 2b 3b  d   4 7  Area of the image
2a  2 ASF=
Area of the object
a 1
Area Scale Factor   Linear Scale Factor 
2

2b  4
Example I
b2 The square S is the image of the square R by an
3a  c  2 enlargement. If the area S = 17.52 and the area of R =
3(1)+c  2 4.38 cm2. Find the Area Scale Factor and the Linear
Scale Factor.
c  1
Solution
3b  d  7 Area of the image
6+d  7 A.S.F=
Area of the object
d 1 17.52
 1 1 A.S.F 
  4.38
2 1 
A.S.F  4
 1 1  4   0 
      Let the Linear Scale Factor be k
 2 1   4  12   Area Scale Factor k 2
 P '(0, 12)
4  k2
Example VIII
The points P (0, 3) Q (1, 3) and R (2, 4) are vertices of k  2
a triangle PQR. The images of PQR under the  k  2 or k  -2
a c  Example II
transformation T =   are P'(6, 0) Q'(6, 2) and A triangle ABC whose area is 14cm2 is mapped into
b d   5 4
A'(8, 4). its image by a transformation   . Find the area
Solution  1 0 
 transformation   object   image  of the triangle A'B'C' the image ABC.
    Solution
 matrix   matrix   matrix 
 Determinant of 
 a c  0 1 2   6 6 8   Area scale   
        matrix of 
 b d  3 3 4   0 2 4   factor   transformation 
 
 0  3c a  3c 2a  4c   6 6 8 
    Area scale   4 2
 0  3d b  3d 2b  4d   0 2 4     det  
3c = 6  factor  0 1
c=2  (4  1) - (0  2)
3d = 0
4
d=0
a + 3c = 6 Area of the image
ASF=
a+6=6 Area of the object
a=0

362
Area of A'B'C' Combinations of Transformations
4 Suppose that a square S undergoes a transformation T1
14
to produce S' followed by a transformation T 2 to
4 14  Area of A'B'C produce S'' then the single transformation that maps S
 The Area of A'B'C' is 56 cm2 to S'' is T2  T1.
Example II Inverse transformation
A rectangle whose area is 24cm2 is mapped into its A transformation maps an object into its image. Then
image with an area of 96 cm2 by a matrix of the inverse transformation has an opposite effect.
9 8 Suppose a triangle ABC undergoes a transformation
transformation   . Find the value of x.
 x 4 of T1 to produce A'B'C', the image of ABC followed
by a transformation T2 to produce A''B''C'', then the
Solution
Determinant of T single transformation that maps A''B''C'' to ABC is the
inverse of a transformation that maps ABC to A"B"C"
 9 4  8 x
T = T 2 × T1
 36  8 x  T 1 maps A"B"C" to ABC.
Area scale factor Example I
96
 4 ABCD is a rectangle in which A, B, C, D have
24 coordinates (1, 2), (6, 2), (6, 4), (1, 4) respectively.
 36  8 x  4 The rectangle ABCD is given a transformation
36  4  8 x  0 -1 
  to produce A'B'C'D' followed by a matrix
32  8 x  -1 0 
x4  0 1
transformation   to give the final image
 1 0 
Example III
A"B"C"D"
A trapezium ABCD whose area is ycm2 is mapped
Find
onto its image A'B'C'D' by a transformation
a. The image points
 4 3 i. A'B'C' and D' (ii) A"B"C" and D"
represented by   . If the area of A'B'C' is
1 2 b. Describe a single transformation that maps ABCD
200cm2, find the value of y. to A"B"C"D"
Solution c. Describe a single transformation that maps
A"B"C"D" back to ABCD
 4 3
Det   =A.S.F Solution
1 2  transformation   object   image 
 4  2   1 3  A.S.F a.    
 matrix   matrix   matrix 
8-3=A.S.F i.
5  A.S.F  0 -11 6 6 1   image 
Area of image    
5  -1 0  2 2 4 4   matrix 
Area of the object
 image   0 -11 6 6 1 
200    
5  matrix   -1 0  2 2 4 4 
y
5 y  200  0×1+-1×2 0×6+-1×2 0×6+-1×4 0×1+-1×4 
 
200  -1×1+0×2 -1×6+0×2 -1×6+0×4 -1×1+0×4 
y
5  0+-2 0+-2 0+-4 0+-4 
 
y  40  -1+0 -6+0 -6+0 -1+0 
 -2 -2 -4 -4 
 
 -1 -6 -6 -1 
A'(-2, -1), B'(-2, -6), C'(-4, -6), D'(-4, -1)

363
 0 1  -2 -2 -4 -4  Solution
ii.     transformation   object   image 
 -1 0  -1 -6 -6 -1     
 0×-2+1×-1 0×-2+1×-6 0×-4+1×-6 0×-4+1×-1  matrix   matrix   matrix 
   0.5 0  2 6 6 2 
 -1×-2+0×-1 -1×-2+0×-6 -1×-4+0×-6 -1×-4+0×-1   
 -1 -6 -6 -1  0 0.5  0 0 4 4 
   0.5×2+0×0 0.5×6+0×0 0.5×6+0×4 0.5×2+0×4 
 2 2 4 4  
 A"(-1, 2), B"(-6, 2), C"(-6, 4) and D"(-1, 4)  0×2+0.5×0 0×6+0.5×0 0×6+0.5×4 0  2  0.5  4 
b. T = T2 × T1 1  0 3  0 3  0 1  0 
 
 0 1   0 -1  0  0 0  0 0  2 0  2
  
 -1 0   -1 0   A' B ' C ' D ' 
 0×0+1×-1 0×-1+0×1    1 3 3 1
 0 0 2 2 
  
 -1×0+0×-1 -1×-1+0×0 
 A'(1, 0), B'(3, 0), C'(3, 0) and D'(1, 2)
 -1 0   0 -3 1 3 3 1 
  
 0 1  
 -3 0  0 0 2 2 
 -1 0   0×1+-3×0 0×3+-3×2 0×3+-3×2 0×1+-3×2 
 T=   maps ABCD to A"B"C"D" 
 0 1 
 -3×1+0×0 -3×3+0×2 -3×3+0×0 -3 1  0  2 
c. T = T2 × T1
 0 0 -6 -6 
 -1 0   
   -3 -9 -9 -3 
 0 1
 A"(0, -3), B"(0, 9), C"(-6, -9) and D"(-6, -3)
1 1 0
T 1   
1  0 -1 b. T = T2 × T1
 -1 0  0 3   0.5 0 
T 1      
0 1   3 0   0 0.5 
 -1 0  0  0.5+-3  0 0  0+-3  0.5 
  
 maps A"B"C"D" back to ABCD.  -3  0.5+0  0 -3  0+0  0.5 
0 1 
Example II  0 1.5 
 
The square ABCD with vertices A (2, 0), B (6, 0), C  1.5 0 
(6, 4) and D (2, 4) is given a transformation T1=
 0 1.5 
 0.5 0   maps ABCD to A’’B’’C’’D’’
  to produce A'B'C' and D’ followed by a  1.5 0 
0 0.5 
T = T2 × T1
0 -3 
1  0 1.5 
transformation T2=   to produce A"B"C" and T 1 
 -3 0  
0  2.25 1.5 0 
D" the image of A'B'C' and D'
 2 
a. Find the coordinates of  0
3 
i. A'B'C' and D' T 1   
ii. A"B"C" and D"  2 0 
 
b. Describe a single transformation that maps ABCD  3 
to A"B"C" and D"  2 
c. Describe a single transformation that maps  0 3  maps A"B"C"D" back to ABCD
A"B"C"D" back to ABCD  
 2 0 
 
 3 

364
Example III Example IV
The vertices of a triangle ABC are the points (4, 6), The triangle whose vertices are A (1, 0), B (2, -2) and
(6, 3) and (2, 3) respectively. The triangle ABC is C (3, 2) is mapped onto its image by a matrix
given a transformation represented by the matrix  2 0
 0 3   2 3 transformation Q  followed by
M=   followed by the matrix N=    0 2
 1 2  1 0 3 1
to give the final image. P 
i. Find the image points after the two 1 2
transformations Find:
ii. Describe the matrix transformation that is the coordinates of the final image
represented by the combined matrix a single matrix transformation which maps the
transformation final image back to the object
iii. Obtain a single matrix that would map the Solution
final image back ABC T = PQ
Solution 3 1 2 0
T   
 transformation   object   image  1 2 0 2
   
 matrix   matrix   matrix   3×2+1×0 3  0+1 2 
 
 0 -3  4 6 2   image   1×2+2×0 1 0+2  2 
   
 -1 2  6 3 3   matrix  6 2
 
 image   0  4+-3  6 0  6+-3  3 0  2+-3  3   2 4
  
 matrix   -1 4+2  6 -1 6+2  3 -1 2+2  3   6 2 1 2 3 
  
 image   18 -9 -9   2 4  0 -2 2 
  
 matrix   8 0 4   6 1  2  0 6  2  2  2 6  3  2  2 
 
 2 3  -18 -9 -9   2 1  4  0 2  2  4  2 2  3  4  2 
  
 1 0  8 0 4   6 10 22 
 
 2×-18+3×8 2×-9+3×0 2×-9+3×4   2 4 14 
 
1×-18+0×8 1×-9+0×0 1×-9+0×4  The coordinates of the final image are (6, 2),
(10, -4) and (22, 14)
 -12 -18 -6 
   -3 0 
 18 -9 -9  T=  
A'(-12, -18  0 -3 
), B'(-18, -9), C'(-6, -9) Det T=  3  3   0  0 
T = T2 × T1  90
 2 3   0 3 
   1  3 0 
 1 0   1 2  T 1   
9  0 3 
 2  0  3  -1 2  -3  3  2   1 
 
 1 0  0  -1 1 -3  0  2   3 0 
 
 3 0   0 1
 
   3
 0 3 
 1 
 3 0   3 0
maps the final image back to the initial
  maps the object to the final image  
 0 3  0
 - 
1
 3
object

Example IV

365
The vertices of a triangle ABC are given by a matrix 0 1  3 -5 -8 
  
1 2 3  0 1  1 3  -1 -2 -3 
  . The matrix R    maps ABC to
 0 1 1  -1 0  1 2 3
 
1 3  0 1 1
triangle A'B'C' and the matrix S    maps
 0 1  R 1 S 1 maps triangle A"B"C" back to ABC
triangle A'B'C' to A"B"C" Example
Show that R-1S-1 maps triangle A"B"C" unto triangle Given the object triangle O with vertices (-1, 4), (-2,
ABC. 3) and (-4, 4) and T1 is a transformation given by
Solution  0 1  1 0 
 transformation   object   image    . T2 is a transformation given by  .
     1 0   0 1
 matrix   matrix   matrix   1 0 
 0 1  1 2 3   image  T3 is transformation given by  .
     0 1
 -1 0  0 1 1  matrix  Calculate
 0+0 0+1 0+1  i. T1T2 (O)
 
 -1+0 -2+0 -3+0  ii. T2T1 (O)
 0 1 1 T12 (O)
 
iii.
 -1 -2 -3  iv. T22 (O)
 A’ (0, -1) B’ (1, -2) C’ (1, -3)
1 3  0 1 1 v. T1T2T3 (O)
   Solution
 0 1 -1 -2 -3 
i. T1T2 (O) means carry out a transformation
 0+-3 1+-6 1+-9 
  T2 on O and then carry out transformation
 0+-1 0+-2 0+-3 
T1 on T2(0).
 -3 -5 -8 
  T2 (O)  T2  object matrix
 -1 -2 -3 
 1 0   -1 - 2 - 4 
 A"(-3, -1) B"(-5, -2) and C"(-8, -3) T2 (O )    
 0 1  0 1  4 3 4 
R=    -1  0 - 2  0 - 4  0 
 1 0  T2 (O )   
 0  4 0  -3 0  -4 
 det R  0  1
 -1 - 2 - 4 
1 T2 (O )   
 -4 - 3 - 4 
 0 1  T1  T2 (O) 
R 1   
1 0 
0 1  -1 -2 -4 
 1 3   
S=    1 0   -4 -3 -4 
 0 1
 0+4 0+3 0+4 
 det S  1  
 1 3 
 1+0 2+0 4+0 
S 1    4 3 4 
0 1   
 0 1 1 3  1 2 4 
 R 1S 1    
 1 0  0 1  4 3 4 
 T1T2 (O) =  
 0 1  1 2 4 
 
 1 3 

366
ii. T2T1 (O) means carry out the transformation v. T1T2T3 (O) means carry out transformation
T1 on O and then carryout a transformation T3 on O, a transformation T2 on T3 (O) and
T2 on T1(O) finally a transformation T1 on T2T3(O) to get
 0 1 -1 -2 -4  T1T2T3(O).
T1 (O)     0 1 1 0 
 1 0  4 3 4  T1    , T2   
 1 0  0 1
 0+-4 0+-3 0+-4 
T1 (O)     1 0
1+0 2+0 4+0  T3   
0 1
 -4 -3 -4   1 0  -1 -2 -4 
T1 (O)    T3 (O)  
1 2 4   
0 1  4 3 4 
 1 0  -4 -3 -4  1+0 2+0 4+0 
T2  T1 (O)       
 0 11 2 4  0+4 0+3 0+4 
 -4+0 -3+0 -4+0  1 2 4 
T2  T1 (O)      
 0+-1 0+-2 0+-4   4 3 4
T2 T3 (O)
 -4 -3 -4 
T2  T1 (O)      1 0  1 2 4 
 -1 -2 -4    
 0 1  4 3 4
iii. T12 (O) means carry out the 1+0 2+0 4+0 
transformation T1 on O and then carry  
 0+-4 0+-3 0+-4 
out a transformation T1 on T1(O) 1 2 4 
 0 1 -1 -2 -4   
T1 (O)      -4 -3 -4 
 1 0  4 3 4  T1T2 T3 (O)
 0+-4 0+-3 0+-4   0 11 2 4
T1 (O)      
1+0 2+0 4+0   1 0  -4 -3 -4 
 -4 -3 -4   0+4 0+3 0+4 
T1 (O)     
1 2 4   -1+0 -2+0 -4+0 

T12 (O)  T1T1 (O) 4 3 4


 
 0 1 -4 -3 -4   -1 -2 -4 
  
 1 0 1 2 4 Example
 0+-1 0+-2 0+-4  The coordinates of the vertices of a triangle OAB are
  (0, 0), A (1, 1) and B (1, 0). T1 is a transformation
 4+0 3+0 4+0 
3
 -1 -2 -4  given by   . T2 is a transformation given by
 T (O)  
2

1
4 3 4   2
iv. T22 (O) =T2 T2(O)  2 0 
  . Find the coordinates of OA'B' under the
 -1 -2 -4   0 3 
T2 (O)    transformation T2T1 (O)
 -4 -3 -4 
Solution
 1 0  -1 -2 -4 
T2 2 (O)     T2T1 (O) means carry out the transformation T1 on O
 0 1 -4 -3 -4 
and then carry out a transformation T2 on T1(O)
 -1+0 -2+0 -4+0   translation 
T2 2 (O)       object= image
 0+4 0+3 0+4   matrix 
 -1 -2 -4      3
0 3
T2 2 (O)         
4 3 4  0  2  2

367
 1  3   4 
     
 1  2   3 
1   3   4 
    
0  2  2
T1(0) =  3 4 4
 
 2 3 2
Transformation  object = image
 2 0  3 4 4 
T2 T1 (O)=   
 0 3  2 3 2 
 -6+0 -8+0 -8+0 
 
 0+-6 0+-9 0+-6 
 -6 -8 -8 
 
 -6 -9 -6   i' j' 
 -6 -8 -8 
T2T1 (O)   T=  0 1 

 -6 -9 -6 
1 0 
 
Obtaining Different Matrices Corresponding to a  The matrix (T) corresponding to the
Given Transformation transformation of the reflection of the line y  x (x –
a. Reflection in the line x  y  0 (y   x)
0 1
y = 0) is  .
1 0
c. A reflection in the X-axis ( y  0 )

The matrix of transformation is obtained from the


coordinate i ' j '
 0 1
 T=  
 1 0   i' j' 
 The matrix corresponding to the transformation of T=  1 0 
 0 -1 
 0 1   
the reflection of the line x  y  0 is  .
 1 0 
d. A reflection in the Y-axis (line x  0 )
b. A reflection in the line y  x

368
 i' j' 
T=  0 1
1 0 
 
 i' j'   The matrix of transformation which
T=  1 0  corresponds to a positive quarter turn about the
 0 1   0 1
  origin is  
 The matrix of transformation which 1 0 
corresponds to the reflection in the Y-axis is
 1 0  g. Half turn about the origin
 
 0 1
e. Rotation of 3600 or 00 about O(0, 0)

 i' j' 
T=  1 0 
 0 -1
 
 The matrix of transformation which
 1 0 
 i' j'  corresponds to a half turn about O is  
T=  1 0   0 1 
0 1
  h. Negative quarter turn (-90° or +270°)
 The matrix of transformation which corresponds
1 0
to a rotation of 3600 about the origin is  
0 1
f. Positive quarter turn about the origin

369
3
iii. Positive turn about the origin (+270)
4
 cos 270  sin 270 
R  270  =  
 sin 270 cos 270 
 0 1
 
 1 0 
iv. Positive quarter turn about the origin
 cos   sin  
T  R   =  
 sin  cos  
 cos 90  sin 90 
R  +90  =  
 sin 90 cos 90 
 0 1 
 
1 0 
v. Negative quarter turn about the origin (-900)
 i' j'   cos 90  sin  90 
T=  0 1  = 
 -1 0   sin  90 cos  90 
 
 0 1
 
Rotations  1 0 
A rotation is a transformation which is as a result of Note: for enlargement at centre (0, 0) and
turning linear scale factor k, the matrix of
How to obtain different matrices corresponding to 2 0
a given rotation transformation is  
If an object is rotated through an angle  about the 0 2
origin (0, 0), the matrix of transformation Example I
corresponding to the rotation is Find the image of (4, 2) under a reflection in
 cos   sin   the Y-axis followed by a rotation of +270
T  R   =   about the origin
 sin  cos   Solution
i. Positive half turn about the origin
 cos   sin  
T  R   =  
 sin  cos  
 cos180  sin180 
R  +180  =  
 sin180 cos180 
 1 0 
 
 0 1
ii. Negative half turn about the origin (-180)
 cos   sin  
T  R   =  
 sin  cos  
 cos  180  sin  180 
R  -180  =  
 sin  180 cos  180   1 0 
T1   
 1 0   0 1
 
 0 1

370
 cos   sin   Solution
T2  R   =    0 11 2 
 sin  cos   PQ    
 1 1 0 -1
 cos 270  sin 270 
=   0+0 0+-1 
 
 sin 270 cos 270   -1+0 -2+-1
 0 1 0 1 
   
 1 0   1 3 
Combined matrix 0 1  3 
  
T  T2  T1  1 3  2 
 0 1  1 0  0 2 

   
 1 0  0 1   3  6 
 2
 0  0 0 1   
  3
 1 0 0  0   A’ (2, 3)
0 1 Alternatively
   1 2  3 
1 0   
 0 1 2 
0 1  4 
    3  4 
1 0  2   
0 2 
 0  2   1
   
40  2 
 2   0 1   1 
     2, 4    
4   1 1   2 
Alternatively 0  2
 
 1 0   4  1  2 
  
 0 1   2   2
   as before
 4  0  3
 
 o  2 
Example III
 4  The points A (1, 1), B (-2, 1), C (1, 4) and D (-2, 4)
 
 2  are vertices of a square ABCD. The images of A, B,
 0 1  4  C, and D under a reflection in the line x  y  0 are
   A', B', C', and D' respectively. The points A', B', C',
 1 0  2  and D' are further mapped onto points A", B", C", and
 0  2  D" respectively under an enlargement scale factor 2
 
40  and centre (0, 0).
a. Write down the matrices for the reflection and
 2  enlargement
 
b. Find the coordinates of the points:
4 
i. A', B', C' and D'
 ( 2, 4) ii. A", B", C" and D"
c. Determine the matrix of a single transformation
Example II which maps A"B"C"D" back to ABCD
0 1  1 2  Solution
Matrices P    and Q   0 -1  are
 -1 1  
used to map the point A (3, -2) onto A' under
the matrix Q followed by P

371
T=T2  T1
 2 00 1
  
0 21 0
 0+0 2+0 
 
 0+2 0+0 
 0 2
 
 2 0
 0 2
 T 
 2 0
1  Adjunct 
T 1   
det T  T 
1  0 2 
T 1   
0  4  2 0 
 i' j'   1
T=  0 1   0
1 0  2
  T 1   
1 0
 The matrix for the reflection in the line  
2 
0 1
x  y  0 is T1     1
1 0 0 2
  maps A"B"C"D" back to ABCD
For enlargement with scale factor k and centre (0,  0 
1
0) 
2 
k o
T  Example II
0 k 
A triangle ABC with vertices A (0, 0) B (1, 0) and
 2 0
 T2    is the matrix of enlargement
C (1, 1) underwent two transformations
 0 2 represented by T2 T1 . If T1 is a translation
with linear scale factor 2 and centre (0, 0)  2
 matrix of   object   image  represented by   , and T2 is a reflection in the
    3
 transformation   matix   matrix  x-axis. Find the coordinates of the final image.
 0 1 1 -2 1 -2  Solution
  
1 0 1 1 4 4  T2T1 (O) means carry out a transformation T1 on
 0 1 0 1 0  4 0  4  the object and the a transformation T2 on T1 (O)
 
1  0 - 2  0 1  0 - 2  0  to obtain the final image
1 1 4 4  T1 = translation
 
1 -2 1 -2   translation   object   image 
 A'(1, 1) B'(1, -2) C'(4, 1) and D'(4, -2)    
 matrix   matrix   matrix 
 2 0 1 1 4 4   2 0  2
       
 0 2 1 -2 1 -2  3  0 3
2 2 8 8   2  1   3 
      
 3   0   3
 2 -4 2 -4 
 2   1  3 
 A"(2, 2) B"(2, -4) C"(8, 2) and D"(8, -4)     
 3   1  4 

372
 i' j' 
 i' 
j' T2 =  0 1 
 -1 0 
T=  1 0   
 0 -1 
   matrix of   object   image 
   
 matrix of   object   image   transformation   matix   matrix 
     0 1   3 5 5
 transformation   matix   matrix 
  
1 0  2 3 3 1 0 0 0 3
    0+0 0+0 0+-3 
 0 1  3 3 4
 
 2+0 3+0 3+0   3+0 5+0 5+0 
   0 0 -3 
 0+-3 0+-3 0+-4 
 
 2 3 3 3 5 5 
   A'(0, 3) B'(0, 5) and C'(-3, 5)
 -3 3 -4   0 1   0 0 -3 
 A'(2, -3) B'(3, 3) C'(3, -4) are the coordinates   
 1 0   3 5 5
of the final image
 0+-3 0+-5 0+-5 
 
Example IV  0+0 0+0 3+0 
Triangle ABC has its vertices at A (3, 0) B (5, 0)  -3 -5 -5 
and C (5, 3). The triangle is given a positive  
0 0 3 
quarter turn about the origin (0, 0) to produce
 A"(-3, 0) B"(-5, 0) and C"(-5, 3)
A'B'C' the image of ABC followed by a reflection T = T2  T1
in the line x  y  0 to produce A"B"C"
 0 1 0 1
i. Determine the coordinates of A'B'C' and T  
 1 0  1 0 
A"B"C"
ii. Describe fully a single transformation which  0  1 0  0 
T 
maps A"B"C" to ABC  0  0 1 0 
Solution  1 0 
 
 cos   sin    0 1
R   =  
 sin  cos   det T =  11   0  0 
 cos 90  sin 90   1
R(+90) =  
 sin 90 cos 90 
 0 1
 
1 0 

373
det T =  11   0  0   cos   sin  
 1 R   =  
 sin  cos  
1  1 0 
T 1  
1  0 1 
  cos  90  sin  90 
R(-90) =  
1 0 

 sin  90 cos  90 

 0 1  0 1
1 0  T2   
  is a matrix of transformation which maps  1 0 
 0 1 For the enlargement of linear scale factor k centre (0,
A"B"C" to ABC 0)
Example V k 0
The points A (1, 1) B (1, 4) C (4, 4) and D (4, 1) are  
vertices of a square ABCD. The images of A, B, C and 0 k
D under a reflection in the line x  y  0 are A'B'C' and  3 0 
D'. Under a negative quarter turn about the origin, the T3   
images of A'B'C' and D' are transformed to A"B"C" and  0 3 
D" respectively. A"B"C" and D" are further mapped When we pre-multiply the matrix of transformation by
onto A'"B'"C"' and D"' respectively under an the object we get the image
enlargement of scale factor -3 and centre (0, 0).  0 11 1 4 4 
a. Write down the matrix for the   
 1 0 1 4 4 1 
i. Reflection
ii. Rotation  0+-1 0+-4 0+-4 0+-1 
 
iii. enlargement  -1+0 -1+0 -4+0 -4+0 
b. find the coordinates of
 -1 -4 -4 -1 
i. A'B'C' and D'  
ii. A"B"C" and D"  -1 -1 -4 -4 
iii. A"'B"'C"' and D"'  A'(-1, -1) B'(-4, -1) C'(-4, -4) and D'(-1, -4)
Solution  0 1  -1 -4 -4 -1 
  
 1 0  -1 -1 -4 -4 
 0+-1 0+-1 0+-4 0+-4 
 
1+0 4+0 4+0 1+0 
 -1 -1 -4 -4 
 
1 4 4 1 
 A"(-1, 1) B"(-1, 4) C"(-4, 4) D"(-4, 1)
 3 0  -1 -1 -4 -4 
  
 0 3 1 4 4 1 
 3+0 3+0 12+0 12+0 
 
 0+-3 0+-12 0+-12 0+-3 
 3 3 12 12 
 
 -3 -12 -12 -3 
 i' j'   A"'(3, -3) B"'(3, -12) C"'(12, -12) D"'(12, -3)
T2 =  0 1 
 -1 0 
  Example VI
The unit square formed by O(0, 0) A(1, 0) B(1, 1) and
C(0, 1) is reflected in the line y   x to give
OA'B'C'
a. (i) Obtain the matrix of transformation R for the
reflection
(ii) Use R to find the image points of OA’B’C’

374
b. If OA'B'C' is the enlarged by a linear scale factor -  2 0   0 0 -1 -1 
2 at the origin to give OA"B"C". Find   
 0 2   0 -1 -1 0 
i. The matrix for the enlargement
ii. The coordinates of the image OA"B"C"  0+0 0+0 2+0 2+0 
 
iii. The matrix which maps OA"B"C" back to  0+0 0+2 0+2 0+0 
OABC 0 0 2 2
Solution  
0 2 2 0
 O(0, 0), A"(0, 2), B"(2, 2) and D"(2, 0)
T  T2  T1
 2 0   0 1
  
 0 2   1 0 
0  0 2  0
 
0  2 0  0
0 2
 
2 0
Det T = 0 – 4
=-4
1  0 2
T 1   
4  2 0 
 1 
1
 0 2 
T   
 1 0 
 
 i' j'   2 
T2 =  0 1   1 
 -1 0    0 2  maps OA"B"C" back to OABC
   
1  1 0 
0  
 R   2 
 1 0 Reflection
 0 1 0 1 1 0 A reflection as a transformation is fully defined when
  
 1 0  0 0 1 1 we are given the object and the mirror line and we are
required to find the position of the image or when we
 0  0 0  0 0  -1 0  -1 
  are given the object and its image and we are required
 0  0 -1  0 -1  0 0  0  to find the position of the mirror line.
 0 0 -1 -1  Properties of reflection
  1. The object and the image are appositively
 0 -1 -1 0 
O(0, 0) A'(0, -1) B'(-1, -1) C'(-1, 0) congruent
Linear scale factor k, centre (0, 0) 2. The image is of the same size as the object
3. The line joining the object to its corresponding
k 0
  image meets the mirror line at 900.
0 k
 2 0  Finding the position of the image under a reflection
  in line L
 0 2  1. Draw the object and the mirror line as shown
below

375
2. Drop the perpendicular from each of the vertices
of the object into the mirror line. Extend this
line past the mirror line
2. Join any object vertex to its corresponding image
vertex, for example A to A’, B to B’ or C to C’

3. Locate the position of the image such that the


object distance from the mirror line is equal to
the image distance from the mirror line 3. Draw a perpendicular bisector of the line AA’,
BB’ or CC’. This perpendicular bisector to the
line AA’, BB’ or CC’

Example
In the x  y plane, draw the triangle ABC with
How to find the mirror line when the object and the coordinates A (2, 3), B(5, 1) and C (6, 5). Find the
image are given image of triangle ABC under a reflection in the
1. Draw the object and its image following lines;
i. x y 0
ii. x3
iii. 3x  2 y  6

376
iv. x y 3
Solutions
For x  y  0
0 1 3
0 -1 -3
For x  3
3 3 3
0 1 2
For 3 x  2 y  6
0 2
3 0

For x  y  3
0 3
3 0

A'(-6, 3)
B'(-9, 1)
C'(-10, 5)

A’ (-3, -2)
B’ (-1, -5)
C’ (-5, -6)

377
1990 PAPER ONE Answer x  3 y
7
5 3
SECTION A 10. Mr. Mugabi put Shs 2,400 in his savings account at
0.42  360 the Bank. The Bank’s simple interest rate was 5% per
1. Evaluate Answer (0.0108)
14000 annum. Find the number of years he should leave the
2. Solve the inequality 2x + 4 ≥ 5x – 5 money in the Bank in order to be able to receive a
Answer (x ≤ 3) total sum of Shs 2,700. Answer (2.5 years)

3. Without using tables, find (32)2/5 x (2)-4


11. Given that α*b = ab2 + b –a, evaluate 0.01 x 150.
Answer (1/4 ) Correct to 3 significant figures. Answer (375)
4. The points P(8,11) and Q(12,19) lie on a line which is
parallel to another line passing through 0(0,0). Find the 12 N
Equation of the line through 0(0, 0). Answer (y = 2x)
5. A chord of a circle of radius r centimeters is 10cm N A
long and subtends an angle of 1300 at the centre. Find
r correct to two decimal places. Answer (5.52cm)
6.
B C
In the diagram shown, angle ABC is 500. Determine the
bearing of B from A. Answer (2200)
13. Given that log10a = 1.621 and log10b = 1.152 ,
In parallelogram ABCD, AB = 2 cm, BC = 3cm and
evaluate log10a + log10b1/2. Answer(1.197)
AC = 4cm. Find the size of angle ABC. 14. Two men leaving a point B, walk in opposite
Answer 104.5 directions along a straight road with the same speed.
7. Given that a    , b    5  , and c =   1
3 Given that the first man takes four minutes to walk
  2  4 
  100 metres. Find the distance from B walked by the
      5 second man in 2.5 minutes? Answer (62.5)
Find the length of a + b + c. Answer 3 2
15 14 cm
1
8. In the figure below, OP = p, OQ = q, PY = PQ and
3
X is mid-point of OP. Find XY 40 cm

Y
7 cm

The diagram above represents a light circular pail with a


Answer 12 , 15 circular base of radius 7cm and a circular top of radius
53 53 14cm. The pail is 40cm high. Find the capacity of the
9. The transformation described by the matrix  3 x  pail. Answer (14.37334 litres)
 

(b)
 y 3
SECTION B
D C 16 (a)

A B 6cm
maps the point A(3, 5) onto the point A1 (6, 8), find
the values of x and y.

B C

378
In the diagram above ABC is an isosceles right–angled  3  1
triangle. The shaded area is bounded by the circular arcs. (b) Given thatM    ,find a matrix N such that
The outer arc is a semi-circle with AC as diameter and the 4 6 
inner arc is a quarter of a circle with centre B. Find the area
14 0 
of the shaded region. MN   
Answer (18cm2), (3.146cm2)  0 14
Hence or otherwise find the inverse matrix for M.
In the figure above AB and CD are targets to the circle at
 6 1 
points B and D respectively. ABCD is a parallelogram with
Answer M-1=  22 22 
AB = 4.5cm and BC = 6.3cm. Find (i) the radius of the  
circle  4 3 
 
(ii) The length of the chord DE.  22 22 

17. (a) Given the equation ax2 + bx + c = 0, (a ≠ 0) derive  6  1


21. (a) Given the vectors, PQ   , QR    and the

 8  5
 b  b 2  4ac
the formula ; x  point Q(2,3) find:
2a
for finding the roots of the above equation. Use the formula (i) PR
to solve the equation 3x2 + 14x – 24 = 0 (ii) The coordinates of the point P.
 7 
 
Answer (x = -6), (x = 4/3)
(b) To print wedding cards at the diamond printery one has Answer   , 4, 11 x  2
to pay a deposit of Shs 50 and an amount which is  3 
directly proportional to the number of cards to be (b) The positions of the three points in a plane are L(-3, 4),
printed. The table below gives the total cost c, required M(8, -5) and N(x, 3). Given that OL is parallel to MN,
to print d cards. where O is the origin, find the value of x.
d 1 3 6 8 22. Draw on the same coordinate axes the graphs of
c 100 200 350 450 y = 2x2 – 3x ……………………… (i)
Find (i) c in terms of d. y = 3(2x – 3)…………………….. (ii)
(ii) The total cost of printing 248 cards. (a) Using your graph, find the points of intersection of
Answer (12450) (i) and (ii).
18. A port B is 2.5 km East of Port C. A navigator observes (b) Find a quadratic Equation whose roots are the x-
that the bearing of C from his ship is 310 0 and that of B is coordinates of the points you stated in (a)
0
018 . By an accurate scale drawing or otherwise; find the Answer
position and the bearing of the ship from B. given that the
-1
1.5, 0  and 3, 9  , 2 x2  9 x  9  0
ship begins to sail at a speed of 10 kmh on the bearing of
2410. Find by drawing or otherwise the bearing and the
position of the ship from C after 48 minutes.
Answer (7.5km), (222.40)
19. In a class of 53 students 30 study chemistry, 20 study
physics, and 15 study mathematics. 6 study both
chemistry and Physics, four study both mathematics and
Chemistry, 5 study both Physics and Mathematics. All
students study at least one of the subjects.

(a) Find the number of students who study all the three
subjects?
(b) A student is selected at random from the class, find the
probability that:
(i) He studies physics only.
(ii) He studies physics but not mathematics.
Answer M 
20. (a) Using matrix methods find the values of x and y
which satisfy the equations.
2x – y = 1
3x + 2y = 12
Answer (x = 2, y = 3)

379
1990 PAPER TWO 1 7
11. Given that a   , b    Find the number x and y
 6 9
SECTION A
 15 
1. Given the sequence 1 , 4 , 9 , 16 such that xa  yb   
4 9 16 25  24 
49 Answer (x = 1, y = 2)
Write down the seventh term Answer
64
12. E D
2. Factorize 25 – (x + 2xy + y )
2 2
(3 marks)
F
Answer ( 5 + x + y) (5 – x – y)
3. Given that cos   5 and that  lies between 00 x
13
A B C
and 1800. Find without using tables the values of
(i) sin  The figure above shows a rectangle ACDE in which
(ii) tan  AB : BC  5 : 3 . Given that AC=12cm, find x
Answer  12 , 12  Answer(4.5)
 
 15 5  13. Find the matrix A such that
4. Find, without using tables, the square root of 2500 x 1 0  3 1
AP   , Where P    (3 marks)
  1 3
1764. (2 marks)
0 1
Answer (2100)
5. A map is drawn to a scale of 1:250,000. Find actual  3 /10  1/10 
Answer  
distance in km, of a piece of a road represented by
1/10 3 /10 
3.6cm on the map. Answer : 9km
14. Given that x  y  10 and a  b  3, evaluate
6. The functions f and g are defined as follows;
10 3
f ( x) 
1
, g ( x)  x 2  1 . Find the value of x such  (2 marks)
2x  6 yx ba
that fg (x) is meaningless. Answer (0)
Answer (x = + 2) 15. Express 2.3 as a rational number (3 marks)
7. Solve the simultaneous Equations Answer (7/3)
-x + 2y =10
y–4=x (3 marks)
Answer (x = 2, y = 6) SECTION B
3 12  1 5 6  311
8. The following are the percentage marks obtained by ten 16. (a) Evaluate
pupils in a mathematics test. 1 34  7 23  3 56
12, 3, 39, 61,40,10,28,40,15,52. (b) In the triangle XYZ, YZ  5.5cm, XZ  3.7cm and
Find the probability of a pupil selected at random from this XY  7.8 . Find
group having obtained a mark below the mean mark. (3 (i) sinXYZ
marks) (ii) The radius of the circumcircle of the triangle XYZ.
Answer (0.5) Answer (4/5, 0.432, 4.287)
9. The position vectors of the vertices of a triangleABCare
17(a) A shirt and a pair of trousers were each sold at 6000.
 2  5
OA   , OB    and OC    . Find the area of the
2 The shirt was sold at a profit of 25% and the pair of
 
0  
0  4 trousers was sold at a loss of 20%. Find the
triangle.(2 marks) percentage loss on both articles.
Answer (6 units) (b) A total of 1200 exercise books is to be shared by four
10. Petrol costs Shs 0.85 per litre in U. S. A. Find the price
classes, 4A, 4B, 4C and 4D. Senior 4A is given 1
of petrol in pound sterling (₤) if $1= ₤ 0.48. 3
Answer (£0.408) 2 is to go to 4B.The
of books, of the remainder
5

380
other two classes share the remainder with 4C 60 – 64 5
getting 60 more books than 4D. Find the fraction of 65 – 69 11
the total number of books obtained by 4D. 70 – 74 21
Answer (5%, 7/40)
75 – 79 20
18. Use graph paper for this question. 80 – 84 17
Scale: 1cm to 1 unit on the x-axis. 85 – 89 10
1cm to 0.5 units on the y-axis. 90 – 94 6
(i) Plot the triangle PQR: P(1,2), Q(0,0), R(2,0). 95 – 99 4
(ii) Write down the coordinates of PQR as a 2 by 3 matrix 100 –104 2
A.
105 – 109 1
(iii) Multiplying A on the left, by T1 the transformation
Find;
1 2
matrix.   to give the image of triangle PQR (i) The mean weight.
0 1 (ii) The median weight.
under T1. (iii) The model weight
(iv) Plot P'Q'R' Answer (77.8, 77, 74.045
(v) Find the coordinates of P"Q"R", the image of P'Q'R'
21. (a) The daily cost per child in a Kampala family is
 2 0
under T1 whose matrix is   partly constant and partly inversely proportional to the
 0 2 number of children in the family. Given that the cost per
(vi) Plot P" Q" R" child for a family of 10 is Shs 350 and for a family of 20 is
(vii) Write down the matrix of a single transformation Shs 300, find the cost per child for a family of
which would map PQR onto P"Q"R". (i) 50 children
1 0 2  5 0 2 (ii) n children
 ,   , (b)
Answer  2 0 0 2 0 0
y 5 P(x, y)
P II 10, 4  Q II  0, 0  R II  4, 0  4
2 4  3
  2 Q(1, 2)
0 2 
1
19. Use graph paper. Draw x and y axes for: 1 4 5 x
O 2 3
x – axis from -3 to +5
y – axis from -4 to +6. In the figure above the point P moves in the plane in such
On both axes use 1cm: 1unit. away that its distance from (0, 0) is Equal to its distance
(a) Draw and label the graphs of the following lines: from Q. find the locus of P in terms of x and y.
(i) x + y = 3
(ii) y = x – 4 1000
(iii) y + 3x = 0 Answer : 270, C = 250 +
n
(b) By shading the unwanted regions, show clearly the
22 V
region R which satisfies the inequalities
x+y≤3
y≥x–4
y ≥ –3x C
Given that P(x, y) = 5x + 4y, find the two positive values of A
P(x, y) in R for which x =1 and y is an integer. Answer (1,
1) (1, 2) O

20. The table below shows the frequency distribution of B


weights in kg of luggage for 100 passengers boarding the The figure above shows a regular tetrahedron VABC with
Uganda air lines plane traveling from Dubai to Entebbe.
VA = 6cm.
Frequency Calculate
Weight (kg)
(i) The height of V above the base ABC.
50 – 54 1 (ii) The angle between the edge VA and the base
55 – 59 2 ABC.

381
(iii) The volume of the tetrahedron.
Answer (4.918, 550 , 25.558cm3)
1991 PAPER ONE
SECTION A
1. Without using tables or calculator, evaluate
3 log102 + log 1020 – log101.6. Answer (2)
2. Given f(x) = (x – 3) , find the values of x such that f(x) =
2

16. Answer (-1 and 7)

3. Factorize x3 – xy2 Answer x(x+y) (x-y)


4. Express the recurring decimal number 0.36 36 as a
fraction. Answer  4 
 
 11 
5. y
4
y=x+1

x+y=4
1
-1 4 x
Copy the above diagram and show the region satisfying the
inequalities. y  x  1, x  1  4 and y  0
Solution

6. Given that operation  is defined by L  M = the


smaller of the two numbers L and M, find -3  (4  3)
Answer (-3)
7. A man bought a shirt at 20% discount. If he paid Shs
2,000, find the original price of the shirt.
Answer (2500)
8. Find all the integers x that satisfy the inequality
7 x 2  63 Answer x={-2, -1, 0, 1, 2}

382
9. Given that tan   5 , calculate without using tables  4 1  x   4 
12 (b) If       .
or calculator, the value of cos   sin   x  1 y   8 
Determine the values of x and y
(3 marks)
Answer 1 144  36 
7   , x   6, x  2, y  28, y   4
Answer   360  246 64 
 13  17. (a) By plotting suitable graphs on the same axes, find the
solution of the equations.
-3x + 2y = -16
10 x+y=7
(b) Plot the graph of x2 − 5x − 24 for -5 ≤ x ≤ 10 use your graph
D
O to find the roots of the equation x2 – 5x – 24 = 0
Answer (x=6, y = 1, x = -3, x = 8)

A C 18. Using a ruler, pencil and a pair of compasses only,


135 (i) Construct a triangle PQR with angles RPQ = 60 0,
B 5O PQR = 450 and PQ = 8.4 cm, measure the length of
PR and QR.
In the diagram above O is the centre of the circle. Given
(ii) Construct the line ST 12.6cm long bisecting and
that angle ABC=1350 find the angle ADC and the reflex perpendicular to QR and meeting PQ at T. What is the
angle AOC. Answer (270) size of angle STQ?
(iii) Join S to R and Q. Draw the circle circumscribing the
11. The mean of three numbers is 3 and the sum of the triangle QRS. From your diagram determine the
smallest and the middle numbers is 5. Find the largest radius of the circle.
number. Answer (4) Answer (r = 5.1cm)
19. Copy and complete the table below showing the number
of senior four candidates of a certain school who passed
12. The scale of a map is 1cm: 20km. Two towns on the a zonal mock examination in mathematics.
map are 5cm apart. A man driving a car covers this distance Marks x f fx
in 80 minutes. Find his speed in kmh-1. 35-39 37 60 2220
Answer (75kmh-1) 40-44 ___ 72 ____
45-49 47 ___ 3760
E2 50-54 52 ___ 2600
13. Given that E = 1.42 ×102, D =144×106 and V 
D 55-59 ___ 48 ___
.Find v in the scientific form . Answer (1.4 x 10-2) 60-64 ___ 35 ___
65-69 ___ ___ 2010
14. The bearing of P from C is 060 0. What is the bearing of 70-74 ___ 25 ___
C from P? Answer (240) 75-79 ___ ___ 924
80-84 ___ 5 ___
 2
15. A spider made the following four moves OA =   , 85-89 ___ 2 ___
 3
90-94 ___ 1 ___
 3  4 2
AB =   , BC =   and CD =   . What single ∑f=420 ∑f x =____
1  0  8 
vector is equivalent to these four movements? (a) State:
Answer  5 (i) the class width.
  (ii) the modal class
 4 
(b) if a distinction was awarded for a score of 70 or more
marks, determine the percentage number of candidates who
SECTION B passed with distinctions.
(c ) Calculate the mean mark.
1 2 9 9
16. (a) Given the matrices A   , B  Answer (5, 10.7, 52.19)
4 6 1 1 20. Three points P, Q and R in a plane have position vectors
5 0  4 1 12
and C    Find (ABC)-1 p=   , q    and r    respectively .
 1 4   6  2 0
a) Find:

383
1991 PAPER TWO
(i) The lengths of PQ, QR and PR.
(ii) The size of the angle QPR
(iii) The area of triangle PQR
b) Given that S is the midpoint of QR, find:
SECTION A
(i) The coordinates if S, 3 x  9 x 1
(ii) The equation of the line through S having the same 1. Simplify x 1
Answer (35)
27
gradient as PQ
Answer (5 units, 11.18 units, 10 units) 2. Without using tables or calculators, evaluate 5.2 x (3.75 2
– 1.252) (3 marks)
21. A plane flew west from Entebbe (E) at the speed of Answer (65)
200kmh-1 for 1 12 hours to reach Kasese (K). At Kasese it
altered its course and flew North-East to Moroto (M) at 3. Given that f(x) = x2 + 1 and g(x) =x − 1, find the value of
150kmh-1. The total time when the plane was in air was 5 a for which fg(a)  gf (a) (3 marls)
hours. Answer (a = 1)
4. Given that the operation * is defined by the relation a*b
(i) By using a scale drawing determine the distance and
bearing of Entebbe from Moroto. (Use the scale 1cm to = a + b + ab, evaluate 2 * (1 * 3). Answer (23)
50km). N
(ii) On its way to Moroto the plane passed over Soroti 5.
which is North of Entebbe. Estimate the distance between S
Soroti and Moroto.
1000 8cm
(iii) If the plane flew back to Entebbe via Soroti at the
speed of 200kmh-1, determine the time it took to fly from
R T
Moroto to Entebbe. In the diagram above determine the length of the
Answer (90km, 2hrs)
perpendicular NT from T to RS. (Correct your answer to 2
22. Two transformations T1 and T2 are represented by matrices s.f)
3 1 3 1 Answer (7.878cm)
  and   respectively. A point P(a, b) in 6.
 2 4  4 2
Q
the plane under T1 followed by T2 is mapped into the point
P(a + 2, 12b + 48). Find the values of a and b and the
coordinates of the image P1.
Answer PI(5, 0) O P

In the diagram above, the length of the minor arc PQ is


equal to diameter of the circle. Determine the obtuse angle
POQ where O is the centre of the circle. (4 marks)
Answer (114.60)
7. Given that x : y = 6:4 and that x + y = 30, determine the
value of y. Answer (12)
 5   2
8. It is given that OP    and OQ    . Find the
 3  5
magnitude of QP. Answer (7.29 units)

9. Given that  2 4  a    24  , find the values of a and b .


 1 0  b  6

Answer (a = 6, b = 3)

10. Given that R  4m xn k  express x in terms of


3

R, m, n and k. (2 marks)

384
Answer  x  k R 
3 2 In the figure above find all the inequalities satisfying the un
2 shaded region OPQ. Determine the maximum value of
 16nm  2x  3y
over the region OPQ. Answer (16)
11. A number is selected at random from the set B = {3, 6,
9, 12, 15, 18, 21}. Find the probability that the number
is even. 17. In a senior four class of 30 students, 18 play football
(F), 15 play volleyball (V) and 13 play hockey (H). The
Answer  3 
  number of students who play all the three games equals the
7
number of those students who do not play any of these
12.
games. Ten students play both F and V, and 3 play only H
A
and V.
Determine;
y 540
D (i) The number of students who play all the three
x games.
C B (ii) The number of those who play only one game.
In the diagram above AB is the diameter of the circle and (iii) The probability that a student selected at random
DA and DC are tangents to the circle at A and C from the class plays two or more of these games
respectively. Given that angle CAB = 540, find the values 8
Answer (5, 9, )
of x and y. (3 marks) 15
Answer (x = 36, y = 108) 18. Plot the points A (-2, 1), B (-1, 2), C(2, 2) and D (0, -1)
13 The size of an interior angle of a regular polygon is one on a graph paper. The quadrilateral ABCD is enlarged to
and a half times the exterior angle. Find the number of another one whose points are P (1, 3), Q (3, 5), R (9, 5)
sides of the polygon. Answer (5) and S(5, -1) respectively.
14. When the rays of the sun make an angle of 30 0 with a (i) Determine the coordinates of the centre T and the
horizontal ground the length of the shadow of a pole on the scale factor of the enlargement.
ground is 10cm. find the height of the pole Answer (5.77m) (ii) Determine the area of the quadrilateral PQRS.
Answer (-5, -1), (2) , (26squnits)
15. Find the median of the following numbers 27, 28, 04,
19, 11, 32, 10, 46, 03, 14. 19. (a) The height of a right cone is 12cm and the angle at
the vertex of the cone is 300. Find the surface area of the
cone. Take  =3.14
SECTION B O
(b)

L P Q

0
0 70
3cm 15
10cm

M X N
The diagram above shows a triangular prism LMNOPQ of
length 10cm and edge LM=3cm. LX is perpendicular to
MN such that MLX  15 and MLN  85 . Find the
0 0

volume of the prism.


Answer (125.58cm2), (12.69cm3)

16

385
20. P S 1992 PAPER ONE
4cm
0. SECTION A
A D
a
Q R 1. Express 1  2  0.56 in the form , where a and b are
9cm 6 9 0.64 b
0. integers. Answer  1 
12cm  
B C  8 
0.
2. Factorise ax  a completely
4

The above diagram is a cuboid with the dimensions as


Answer a(x2 + 1) (x + 1) (x – 1)
shown. 3. A man borrowed Shs 200,000 from the bank at a simple
Calculate interest rate of 2.5% per annum. He paid back the money
(i) the length QC
in 24 Equal monthly installments over a period of two
(ii) The length PC
years. How much money did he pay every month?
(iii) PCQ and
Answer (8750)
(iv) The angle between the planes PQC and PQRS.
Answer (12.65cm, 15.52cm, 35.40, 18.40) 1
4. Given that log 125 x  log 125 5 x  ,
3
21. Asabat, Bitumoko and Cholimar form a trade Find the values of x. Answer (-1, 1)
partnership. Asabat contributes Shs 750,000, Bitumoko Shs 5 B
500,000 and Cholimar Shs 900,000. Twenty percent (20%) D
of the annual gross profits are to remain as development
capital and a monthly taxation of Shs 10,000 is to be paid C
A
by each share holder. E
The net profit is shared in the ratio of the initial In the figure above AD: AB = 4 : 5 and DE is parallel to
contribution of the shareholders. BC. Find the ratio of the areas of the triangle ADE and the
If at the end of the first year the partnership recorded gross quadrilateral BCED. Answer (16 : 9)
profits amounting to Shs 3,160,000, how much did each
6. Solve the inequalty  x  16  3x Answer (x < - 4)
member get as his net profit?
Answer (756279, 504186, 907535) 7. In a group of 10 people, 7 people speak English, 4 speak
French and 2 speak neither of the two languages. How
22. At Jenga-mwili supermarket, Ali bought 5 trays of eggs many people speak both languages?
and 7 kg of Irish potatoes at Shs. 11,800. Moses bought 6 Answer ( 3)
trays of eggs and 8kg of Irish potatoes at Shs. 14,000. If 6
8.  a 3  b 3 , find the values of a and b.
Shs t and Shs p are the prices of a tray of eggs and a kg of 3 2 2 3
potatoes respectively, Answer (a = 3, b = 2)
(i) Write two equations to describe the purchase of the 3
two men. 9. Given the function f ( x)  , find the value of x
(ii) By combining the two equations to a matrix form 4x  3
determine the cost of purchasing each item. for which f(x) = 3. Answer (x = 1)
(iii) How much would Dulu pay for 2 trays of eggs and 2
x y , evaluate 3 23
1 2
kilograms of Irish potatoes? 10. Given that xy 
Answer (1800,400,4400) 3
Answer (12)
11. Given that 1 3 4 y   7 7 ,
 x 2
 
Find the values of x and y.
Answer (x = 1, y = 1)
12. The numbers 3, 4, 5 are arranged in a random order so
as to form a three-digit number. No digit is repeated in a
number formed.

386
(i) Write down the possibility space for the numbers (ii) If the above data is representative of the type of
formed. patients that visit the doctor. Find the probability that
(ii) Determine the probability that the number formed is the weight of the first patient in the next week
not odd. 21
Answer  1 
belonged to the model class. Answer (74.5, )
  100
 3
13 18. A transformation represented by the matrix
 4 6
  maps the vertices A, B, C of a triangle
 1 2
onto the points A1 (6, 2)
B1 (16, 7) and C1 (22, 9) respectively.
Find
(i) The coordinates of A, B and C.
The diagram above shows a circle with centre O. Given that (i) The determinant of the matrix.
POR  140 O and POQ  120 O . Determine the (ii) The areas of ABC and its image
angle of triangle OQR. A1 B1 C1.

 QOR  100 , ORQ  OQR  40


0
Answer (0, 1) (-5, 6), (-5, 7) 2.5, 5
Answer 19. Mukasa is to travel from station A to station B,
14. Find the image of the point (2, 1) under the reflection 400km a part, on the bearing of 0650. On his start of
in the line y  x the journey, he makes a mistake and sets off on a
Answer PI (1, 2) bearing of 0560 and moves for 300km. Using scale
15. Given that cos   0.599 and 0    90 , find in
o o
drawing
degrees, the value of Ө. Answer (53.20) (i) Determine how far he then is from station B.
(ii) If he is to move to station B from where he is, on
what bearing should he set off?
SECTION B (iii) If his speed is 80kmh-1, determine the time
16. Copy and complete the table for the relation wasted due to the mistake made at the start of the
y   x  2  draw and state the line of symmetry for
2 journey?
Answer (11.5km, 0900, 11.25minutes)
your graph
20 (a)
(i)
x -7 -6 -5 -4 -3
y 4
-2 -1 0 1 2 3
4 25
Solve the equation (x + 2)2 = 3
Answer (-0.2, -3.8, x = -2)
In the figure above BN is perpendicular to AC. Find
17. The following is a frequency table for the weights, the ratio of the area of triangle ABN to that of ABC.
in kg, of adult patients who visited a certain doctor (b)
in a certain week.
Weight (kg) Frequency
50-54 3
55-59 5
60-64 8
65-69 11 Using the information given in the diagram above,
70-74 21 (i) Find in terms of b the area of CDEK.
75-79 19 (ii) Show that area of RBKT is equal to 6b – 6.
80-84 18 Given that the area of RBKT is three times the area of
85-89 11 ARTE, find the value of b and hence the dimensions of
90-94 4 CDEK. Answer (3 : 16) (DC = 8, BC = 4cm)
(i) Calculate the mean weight of the patients.

387
21.
V
1992 PAPER TWO
SECTION A
1. Simplify 20.1− .623 ÷0.23 Answer (0)
  3
S 2. If the position vector of point A is   and BA   5 
 2   2
, find the position vector of B(2 marks)
P R  8 
Answer  
 0
Q
The figure above shows aright pyramid standing on a 3. In a group of 22 tourists visiting Uganda it was found
horizontal rectangular base PQRS. Given that PQ = 6cm, that 12 had been to Karuma falls and 11 to Entebbe zoo.
QR = 8cm and V is 12cm vertically above the horizontal Find the minimum possible number of tourists who had
base PQRS. Find: visited both the falls and the zoo. (2 marks)
Answer (1)
(i) The length of VQ.
(ii) The angle between VQ and the horizontal base.
1 1
(iii) The angle between the planes VPQ and VSR. 4. Given that h( x)  and k ( x)   9 , evaluate
Answer (VQ = 13, 67.38, 400) 2x  9
2
x
kh (-1) Answer (2)
22. A school lorry and a school bus are to be used to
transport students to a certain function. The capacities of 5.
the lorry and the bus are 50 and 70 students respectively.
The number of students to attend the function should not
exceed 350. Each trip made by the lorry or the bus cost Shs
3,000/=. The money available for the transportation is Shs
18,000/=. The number of trips made by the lorry should not
exceed that made by the bus. If x and y are the number of
trips to be made by the lorry and the bus respectively.
(i) Write down five inequalities representing this In the figure above AD is perpendicular to BC.
information. AD  DB. AC = 4cm and CAD =300. Find AB
(ii) Plot these inequalities on the same axes.
(iii) By shading the unwanted region show the region Answer (4.90)
satisfying all the inequalities.
(iv) If all the available money for transport is to be used, 6. In a certain class there are 72 boys. If the ratio of the
list all the possible number of trips that each vehicle number of girls to the total number of pupils in the class
will make. (Assume that for each trip a vehicle makes is 3:7, find the number of girls in the class.
it must be full). Answer (54)
(v) Find the greatest number of students that can be
transported. 7. Given that 81x =( 1 3 )x-5 find the value of x
Answer (360) Answer (1)
8. Given that tan x = 3 4 and 00 ≤ x ≤ 3600. Without using
tables or calculator, find the possible values of cos x + sin x
Answer  1 , 1 
 
5 5 
9. Express 4.454 as a national number. Answer  49 
 
 11 
10. An insect moves along straight lines from point A(2, 0)
to point B(0, 3) and finally to point C(5, 4). How far away
is the insect from its starting point?
Answer (5 units)
11. The scale of a map is 1:25000. The distance between
two schools on the map is 8cm. find the actual distance, in
km, between the two schools.

388
Answer (2km) (ii) Hence write down det(A2)
12. Form a quadratic equation whose solution set is (-2, 3) (iii) A geometrical figure of area 18cm2 is
transformed by the combined matrix AB. Find
Answer (x2 − x-6 = 0) the area of the transformed figure.
Answer (a = 0.6, b = -2.6, 100, 90cm2)
13. Factorize 20x2y2+xy - 1. Answer (5xy – 1) (4xy + 1)
14. If the determinant of the matrix  3a a  8 
  6 a  2 18.
 
C D
is zero. What are the values of a? Answer (a = 4, a = -4)
15. The total weight of a train with n coaches is T. The F
weight of the engine alone is E and the average weight E
of the coaches is A. Write down an equation connecting
T, E, n and A. Answer (T = nA + E) B
b
~

SECTION B O a
A
16. (a) A farmer bought a machine at sh. 2,200,000/-. ~

If the machine depreciates at the rate of 15% per


annum, find the value of the machine after two In the diagram above OA = a and OB = b; 2OD =
years. 5OB and AC = 3AB, E and F are midpoints of OD
(b) In a certain country the income tax is levied as and AC respectively. Find in terms of a and b , the
follows; vectors ED, OF, and CD
A person’s monthly gross income has certain
allowances deducted from it before it is subjected 5 3b  a 4a  b 
Answer  b, , 
to taxation. (this includes family relief and 4 2 2 
insurance value). 19. A circle passes through the points A(-3.5, 1)
The allowances are as follows B(0.5, 5) and C(4, 3).
Married man sh 1,800 (i) Using 2cm to represent one unit, plot the points
Unmarried man sh 1,200 A, B and C on a graph paper.
Each child below eleven years sh 500 (ii) By construction find the centre and radius of the
Each child above eleven but circle.
Below eighteen years sh 700 (iii) Calculate the area of part of the sector
Insurance premium sh 1,200 (segment) cut off by the line segment AB.
Peter earns sh 64,000. He is married with 3 children of Answer (centre (0.5, 1) radius = 8cm, 18.24cm2)
ages between eleven and eighteen years and 2 children 20. Of the 35 candidates in senior four, 13 registered
below eleven years. for Biology (B), 20 registered for History (H) and 17
Given that he is insured and has claimed transport registered for Fine Art (A).
allowance of sh 1,700, calculate the income tax he If 9 registered for both Biology and Fine Art and
pays under the income tax rates below; n(B∩H) = 3, n(B∩H∩A) = 2 and n(H∩A∩B1) = 8,
Taxable income rate (%) represent these information on a venn diagram.
0-10,000 10 From the diagram
10,001- 20,000 25 (a) Find
20,001- 30,000 30 (i) the number of candidates who registered for History
30,001- 40,000 45 only.
40,001 and above 50 Ans. (1589500, 19100) (ii) the number of candidates who registered for at least
two of the three subjects.
17. (a) The solution of the simultaneous equations (b) Which of the subjects had to be taken with at least one
2ax + by = -4 and bx +3ay = 1 where a and b are other subjects?
constants is x =1 and y = 2. (c) How many candidates did not take any of the three
Find the value of a and b. subjects?
Answer (9, 18, 5)
3 1  5 7
(b) Given that A    and B   2 
 2 4  1 3
(i) Show that det(AB)=det(A) det(B)

389
1993 PAPER ONE
21. When Mukasa was 5km from home and walking at
2 12 kmh-1 on his way to visit his aunt 15km from his home,
his brother Musoke decided to run after him at 4mkh-1. SECTION A
(i) When and where did Musoke catch up with Mukasa? 1. Given that f ( x)  px  3 and f (5)  33 , find the
(ii) If Musoke continued to run at the same speed, how value of (i) p, (ii) f(-2). (4 marks)
long did he have to wait at his aunt’e home before Answer (6, -9)
Mukasa joined him? (Express your answers in 2. A rectangular piece of land is 12m by 15m. Find the area
minutes)
of the land in cm2 on a map whose scale is 1:500.
Answer  40 km , 15minutes  Answer (7.2cm2)
 
 3 
22. From a point P on the top of a cliff 150m high , two 3. Given that 4
px 2  a  L express x in terms of a, L and
ships A and B are observed on the bearings of 240 0 and p. hence determine the values of x for which a = 4, p = 4/3
1500 respectively.  4 
and L = 2. Answer  L  a ,  3,  3 
 p 
If B is 720m from A on the bearing of 120 0, calculate the  
angles of depression of A and B from P. 4. In a class, 15 pupils play cricket, 11 play hockey, 6 play
Answer (22.60) both games and every one plays at least one of the games.
Find (i)The number of pupils in the class.
(ii) The probability that a pupil picked at random
plays only one game. Answer (0.7)

The diagram above shows a circle inscribed in a square


ABCD of side 2x and a square EFGH inscribed in the
circle. Find the expression for the area of the shaded
 8x2 
region. Answer  
 7 
6. Astick 10cm long on a 0.57m high plat form rests against
a vertical wall making an angle of 400 with a horizontal
ground. Find the height of the top of the stick above the
ground. Answer(7m)
7. The lengths of the sides of a right angled triangle are a,
2a - 1, and 2a + 1. Find the value of a and hence the
sides of the triangle. (a > 0) Answer (a = 8)
8. The image of (0, 2) under an enlargement scale factor 3
is (4, 6). Determine the centre of enlargement.
Answer (-2, 0)
 3 x
9. Given that matrix A   
 y 4
det(A) = 3x +12, determine the value of x and y.
10. Determine the equation of the line passing through the
points (2, 1) and (3, 3). Answer(x = 0, y = -3)

390
SECTION B (iii) The ratio of the male population employed to the
female population employed.
11. Triangle ABC has its vertices at A(2,0), B(4 , 0) and (iv) The total number of people employed in the country.
C(4 ,3). The triangle is given a positive quarter turn about Answer (6300000), (5775000), (81 : 55), (476000)
(0, 0) to produce AIBICI the image of ABC; followed by a S
15
reflection in the line x + y = 0 to produce A11B11CI1, the
image of AIBICI .
O
(i) Determine the co-ordinates of A1B1C1 and A11B11C11. P 50O 6cm
(ii) Describe fully a single transformation which maps ABC
onto A11B11C11.
T
Answer AI(0, 2) BI(0, 4) CI(-3, 4)
AII(-2, 0) BII(-4, 0) CII(-4, 3) (a) In the above diagram PT and PS are tangents to the
1 0  circle with centre O. if
  OT = 6cm and angle POT=500. Calculate the area of the
 0  1 shaded region. (Take  =3.14).
12. (a) Find the inverse of A   4  1
(b) Two Equal circles of radius 5cm intersect at right
2 3  angles.
 
(i) Find the distance between the two centers of the circles.
(b) Tom bought 2 eggs and 3 tomatoes at a total cost of (ii) Calculate the area of the common region of the circles.
Shs 370. The cost of 4 tomatoes is Shs 90 more than Answer (11.504, 14.25)
that of one egg. 16.
(i) Write down this information as a pair of simultaneous
equations. y
(ii) Find the cost of one egg.
(iii) Calculate the cost of one tomato.
(iv) Determine the number of eggs and tomatoes Shs
1470 fetched if twice as many tomatoes as eggs were
Number of students

obtained.

Answer
1  3 1
  , 110, 50, 7, 14
14  2 4 
13.
20
P 3a
~
T Q
S
3b V
~

R
29.5 39.5 49.5 59.5 69.5 79.5 x
In diagram above PQ = 4PT; 2PS = PV; 3RS = 2RT; PT =
3 a and PR = 3 b Study the bar graph given above showing the ranges of
~ ~
(a) Express in terms of a and b. marks obtained by students in a certain math test
(i) RS (i) Determine the number of students who sat the
(ii) PV test.
(iii) RQ 10(ii) Write down class groups and their frequencies.
(b) Find the ratio of RV to RQ (iii) State the modal and median class.
Answer (2a – 2b) (4a + 2b), (12a – 3b) (1 : 3) (iv) Use your results in (ii) above to calculate the
14. In a country with a population of 14,000,000 people, mean mark obtained in the test.
55% are females, 45% of the male population are Answer (40, 40 – 49, 40 – 49, 50.25)
employed and 25% of the females are employed. 17. Draw the graph of the curve y  cos 3x
Find:
for 0o  x  150 o . Using your graph determine the
(i) The male population in the country.
(ii) The female population unemployed. values of x(00≤ x ≤1500) for which 4cos3 x  3  0
Answer (45, 75)

391
1993 PAPER TWO
SECTION A
1. Determine the values of k for which the expression
k2  k 6
is zero . Answer (3, -2)
k2
2. A fair die is tossed once. Find the probability that
(i) The die shows a number greater than two.
(ii) An odd or even number less than four shows up.
Answer  2 , 1 
 
 3 3
3. Two right angled triangles ABC and PQR are similar.
Given that angle ABC=angle PQR=900
.
AB =10cm, BC = 6.5cm and QR = 52cm, The graph above shows motion of a lorry from Tako’s
Find PQ . Answer (80) home to the city, O to C.
(i) How far did the lorry travel in the first two hours?
4. Factorise completely x - x2 + y + y2 (4 marks) (ii)For how long did the lorry have a stop over?
Answer (y + x) (y – x + 1) (iii) Determine the average speed of the lorry for the
5. journey from home to the city.
 100 
Answer  60km , 1hr , km / hr 
 3 

3
10. Without using a calculator, evaluate 0.002406 to 3
decimal places.
Answer (0.134)
The figure above shows a circle centre O circumscribing a
triangle ABC. SECTION B
Determine (i) The reflex angle AOB. 11. Plot on the same axes the graphs of xy = 24, for 1≤ x
(ii) An expression of y in terms of x (4 marks) ≤24 and x + y = 12
Answer (y = 90 – x) Using your graph
6. Solve the simulatenous equations (i) Find the coordinates of the points of intersection of
3x + 4y = 20 the line and the curve.
x +2y = 0 Answer (x = 20, y = -10) (ii) Estimate the area enclosed between the line and the
curve.
7. The images of I(1,0) and J(0,1) under a transformation Answer (2.4, 9) (9.6, 2.4) (7sq.units)
represented by a 2x2 matrix are I1(2,0) and J1(0,3) 12. A plane flies 540 km from station A to station B on
respectively. Determine the coordinates of K1, the bearing 0600 . From B it travels 465 km to station C on
image of K(1,1) under the same matrix transformation. bearing of 150, From C it heads for station D 360km away
Answer (2, 3) on bearing of 2650.
8. The coordinates of the points P and Q are (2, 3) and (4, - (i) Draw to scale a diagram showing the route of the
plane (use the scale 1cm:5km)
1) respectively. Calculate the length of PQ.
(ii) (ii)From your diagram determine the distance and
Answer (4.47 units) bearing of station A from station D.
9 (iii) Determine how long it would take a plane traveling at
a speed of 400kmh-1 to travel direct from station A to
station C.
Answer (375km, 2960, 710km)

392
13. The following results were obtained in an experiment to 16. Using compases and ruler only, construct a triangle
measure the lengths of leaves from the stalk to the a pex ABC in which AC = 8cm, AB = 6cm and angle BAC =
(to the nearest tenth of a cm) 300. On the same side AC construct a triangle such that
6.6 8.5 7.4 10.8 11.2 9.1 8.7 9.9 AS = SC and BS = 8cm. A point K is on the same side
12.4 10.0 6.5 7.3 12.8 8.2 6.4 8.9 of BS as C. Its distance from BS is the same as the
8.9 8.1 8.3 9.0 7.6 7.1 8.8 10.4 distance of C from BS.
11.7 9.2 10.2 9.8 9.5 12.3 6.2 8.8 Construct the locus of K.
7.0 7.9 9.3 6.9 7.7 6.2 8.6 7.4 Given that angle BKS = 900, find by construction two
possible positions K1 and K2 of K. Measure K 1K 2 .
(i) Starting with 6.0-6.9 as the first class and using classes Answer (4.8cm)
of Equal length draw up the freqnency table for the data in
(i) above. 17. A manufacturer makes two types of hoes A and B. The
(ii) Give the modal class for the grouped data in (i) above. following conditions apply to daily production.
(iii) Calculate the mean length of the leaves (i) Each type of A costs sh 3000/- and each type of B
Answer (8.0 – 8.9, 8.775) costs sh. 5000/- and the manufacturer has a
maximum of sh. 450,000/= available.
(ii) Due to labour shortage the production of type A plus
14. A line whose end points are P(1, 3) and Q(3, 4)
four times that of B should not exceed 160.
undergoes a rotational transformation. The images of the (iii) A study of the market recommended that the number
endpoints of line PQ are P1 and Q1 respectively. of type B produced should not exceed twice the
(i) Draw the line and its images in the same set of axes number of type A produced.
using a scale of 2cm to 1 unit. (a) Given that x hoes of type A and y hoes of type B
(ii) Find the centre and angle of the rotation. are made, write down three inequalities a part from
(iii) Determine the image of PQ when its image P 1 Q1 x ≥0, y ≥ 0, satisfying the above conditions.
further undergoes a rotation of 120 0. State the size of (b) Show graphically the region containing the points
angle formed between PQ and the image of P1 Q1. satisfying the above conditons.
15. (c) Taking x + 2y as a suitable expression for the
manufacturer profit, find the number of each type
of hoe that should be made to obtain the greatest
profit.
Answer (maximum profit = 147)
(135 hoes of type A and 6 hoes of type B)

In the diagram above, OP = p , OQ = q and OT = t . R


~ ~ ~

and T are mid-points of OQ and PR respectively.


(i) Express t in terms of p and q .
~ ~ ~

(ii) Given that OP is parallel to QS such that


OP  2QS , find OS in terms of p and q
~ ~
(iii) Taking O as the origin and P(0, 8) and Q(6, 4)
OS and PS .
determine the lengths of
 1 1 
Answer  t  p  q  , 10 units, 6 units.
 2 4 

393
Find the price of the car after (i) one year (ii) two
1994 PAPER ONE years. Answer (4930000), (295800)
9. Given that log 10 7  0.845 and log 10 2  0.301 , use
SECTION A  49 
this information to find log 10   Hence determine the
1. (i) Express 108 as a product of its prime factors.  64 
8
49
4 2  2 3 16 1 / 2  33  value of giving your answer to 3 significant figures.
(ii) Simplify Answer  , 1 64
83  2  Answer (-0.116)
ky 2 10. A and B are two matrices such that
2. x ,y and z are connected by the relation x  , where
z 1 3    1 2
A  ,B   
k is a constant. Given that x = 6 when y = 1000 and z = 9, 4 11  1 3
find the value of z when x =2 and y = 5. (Give your answer Find: (i) matrix P = (AB)
in standard form) Answer (6.75 x 10-4) (ii) P-1
3. Answer 1  41  11
 
5  7 2

O SECTION B
11(a)The table below shows a sample of ages (to the
110O
A D nearest tenth of a year) of patients randomly selected from a
group of patients who sought medical treatment at a certain
B clinic during a certain week
C 0.6 2.0 18.0 3.4 19.0
In the figure above, O is the centre of the circle and ABC is 15.0 16.3 14.0 7.0 12.2
a straight line BD = CD and angle AOD=1100. Find the size 18.9 5.9 1.5 12.0 9.0
of angle (i) DBC (ii) BDC. Answer (550, 700) 5.0 12.8 17.0 7.7 0.2
8.0 14.0 5.4 15.8 17.8
4. Betty and Alex are to share 144 oranges, with Betty 5.5 11.4 6.0 6.9 16.0
getting twice as many oranges as Alex. Find how many 10.0 0.8 13.6 11.0 3.9
oranges each will get. Answer (48, 96) 13.0 9.0 6.6 10.9 4.0
5. Two areas on a map represented by two rectangles (i) Form a frequency distribution table for the ages having
equal class intervals of 5 years and starting with 0.0 −
OABC and OPQR are similar with length AB
4.9 class. State the modal class of the distribution
corresponding to PQ = 42cm, and OC to OR; and width (ii) Draw a histogram (bar chart) to show the data
BC = 5cm corresponding to QR = 20cm and AO b) The mean of four numbers is 25, their median is 23. if the
corresponding to PO. largest number exceeds the smallest by 30, and the
(i) Determine the length of AB. largest number is one and a half times the second largest
(ii) If the width QR = 20cm on the map measured on actual number, determine the four numbers
ground is 50km, state the scale of the map. Answer (modal class = 5.0 – 5.9) (12, 18, 28, 42)
Answer (10.5cm, 1 : 250000) 12. (a) Show by shading the unwanted regions the region
6. Given that f ( x)  x  2 x 2 and gf ( x)  3  x determine satisfying the inequalities y  2 x  1 and y  3
an expression for gf (x). Hence evaluate gf(-2). (b) Find the equation of the line through the points A(2, 7)
Answer (13) and B(5, 13). The points A and B are reflected in the line
7. y is known to be inversely proportional to the square of x. x = y.
(i) Determine the coordinates of A1 and B1 the images of A
when x = 2, y = 2. Find the value of x when y = 32.
and B respectively.
Answer (x = + 1 ) (ii) Find the equation of the line through A1 and B1.
2 Answer (y = 2x + 3) AI(7, 2) (13, 5)
8. In a showroom, the price of a car is given as Shs (2y = x – 3)
5,800,000. During sale, a discount of 15% is allowed. 13. In a certain game a die and a coin are each thrown and
(a) How much does a customer pay for a car? tossed once respectively. One side of the coin is labeled
(b) After the car has been bought, in the first year its value T(tail) and the other H(head). The number which appears
deprecates by 25% and by 20% during the second year. on the upper face of the die is the players’ score. In

394
addition, if a tail appears the player receives a score of 4, In the above figure angle MNK = 900, NM=6cm and NL =
and a score of 6 when a head appears. The score obtained 5cm. Calculate the value of sin  o  cos  o , correct to
by tossing a coin is then divided by the score obtained by
two significant figures.
throwing a die. If this quotient is a prime number a player
(b)
takes the first prize. A player takes the second prize if his
quotient is a recurring decimal and a third prize if the
quotient is a triangular number.
Copy and complete the table below giving the possibility
space of the game
Score on die 1 1 6 6
Score on coin 4 6
Quotient 4 6
The figure above shows a container EFGH (part of a
Find the probability that a player wins cylindrical can) used by shopkeepers for scooping out sugar
(i) The first prize from a sack. Calculate the
(ii) The second prize (i) Maximum volume of sugar the container can scoop
(iii) The third prize 22
(iv) None of the prizes given that there are only three (volume of a cylinder is  r2h,  = )
7
prizes. (ii) Ratio of the volume of the cut-off piece of the
1 1 1 1 cylindrical container to that of the container EFGH.
Answer  , , ,  Answer (1.5, 891cm3, 1 : 7)
 4 6 3 3
16. (a) Mr. Kapere deposited Shs 2.421 million on his
14. The points A(1, 5): B(4, 2):C(11, 5) and D(4, 8) are
savings account at the bank at a compound interest rate
vertices of a quadrilateral ABCD.
of 8.5% per annum. Determine the number of years his
(i) Find the lengths of the sides of the quadrilateral. Hence
money will take to accumulate to Shs 2.85 million.
state the name of the quadrilateral.
(ii) Given that AC meets BD at a point I, find the
coordinates of I and show that the points A, I and C are (b) The following is an advertisement of a canon
collinear. photocopier.
(iii) Find the area of the quadrilateral
GET YOURSELF A PHOTOCOPIER CHEAPLY
WHILE STOCK LASTS:
y
TERMS: CASH AT USH 960,000
8 D(4, 8)
OR HIRE PURCHASE: DEPOSIT 15% OF MARKED
I(4, 5) PRICE AND PAY EITHER USH. 75,000 WEEKLY
5 A(1, 5) C(11, 5)
FOR 12 WEEKS OR USH. 245,000 MONTHLY FOR
2 4 MONTHS.
B(4, 2)

0 Calculate:
4 11 x
(i) The saving a customer would make by buying the
photocopier on cash terms rather than weekly hire
Answer purchase.
 AD  AB  3 2 , DC  BC  58  (ii) The percentage profit made on the monthly hire
  purchase if the wholesale cost of a photocopier is
 The quadrateral is a Kite 30sq.units  17.5% below the cash prize.
Answer (1.997years, 84000, 41.92%)
15(a) N

5cm 6cm 17. Using a ruler and a pair of compasses only,


(i) Construct a triangle ABC where AB = 3cm and AC =
𝜃° 5cm and angle ABC = 900
L 𝛽° (ii) Bisect the triangle ABC and let the point at which the
M
angle bisector cuts line AC be the centre of

395
enlargement of triangle ABC. Using the centre, enlarge
ABC by a linear scale factor of -2 to form A1B1C1.
1994 PAPER TWO
(iii) Determine the area of the figure ABCA1B1C1
Answer (30cm2)
SECTION A
1. Convert
(i) 0006 hours to the 12 hours time
(ii) 250 US dollars ($) to pounds sterling (£). If 1 US $ =
Ush 980 and 1(£) = U.sh 1750 Answer (12:06am) (£140)
 3  5
2. Given that OP    , and PQ    Where O is the
7 1
origin, find
(i) The position vector of Q.
8 
(ii) OQ Answer   , 10 units
6
3. Given that f (x) = ax2 + bx, f (1) = 5 and f(2) = 14, find
the values of a and b. Answer (2, 3)
4.

120°

A 69.8 B
mm

In the figure above, AB is a chord of the circle whose


centre is O. Angle AOB is 1200 and AB = 69.8mm.
Calculate the radius of the circle. (Give your answer to 3 sf)
(4 marks)
Answer (40.30mm)
5. In a certain game Bob scored the following points 3, 12, 2,
8, 0, 3, 5 and 7. Determine the median and mean of the
points Bob scored in the game. (4 marks)
Answer (4, 5)
 2  2 3 2 
6. Given matrices P , Q  and
0 1   4  1
 3  4
R  . Determine
1 2 
(i) P.Q + R (ii) The determinant of (P.Q + R)
3 2
Answer  ,  3
3 1 

396
( A) two
7. nGiven ( B)Aand
sets
12, n 13,B A  that
n(such B)  20 n( )  24 Answer (10, 112.890, 3.32cm, 5.47cm, 33.560
and ,
find, 13. The unit square OIKJ where O(0,0), I(1,0), K(1,1) and
J(0,1) is reflected in the line y = -x to give image OI1K1J1.
(i) n( A  B )
1
(i) Obtain the matrix of transformation R for this
(ii) n( A  B' ), where ε is the universal set and B1 reflection.
represents the complement of B. Answer (7, 11) (ii) Use R to find the image points of OI'K'J'.
If OI'K'J' is then enlarged by a linear scale factor of -2 at
the origin to give OI"K"J", find
8. Find the solution set of the equation (iii) The matrix E for the Enlargement.
2x2 + x – 10 = 0 Answer (-2.5, 2) (iv) The coordinates of the image OI"K"J"
9. An observer at a point A sees an object on a bearing of (v) The area of OI"K"J"
1000. Another observer at point B sees the same object on (vi) The matrix which maps OI"K"J" back to OIKJ.
bearing of 1500. Given that the distances of the object  0 1 
Answer R   
 1 0 
from A and from B are equal. Determine the bearing of A
from B. Answer (215) O1(0, 0), II(0, -1), KI(-1, -1), JI(-1, 0)
 2 0 
10. Without using tables or calculator, evaluate;  
0  2
3.83  5.96 O’’(0, 0), I’’(0, 2), K’’(2, 2) J’’(2, 0)
correct to 3 significant figures
1.952 4 square units
Answer (6.00) 0 1
2
1 
2 0
SECTION B 14.
11. On the same axes draw the graphs of
y = x3 − 2 and y = 3x + 2: for -3 ≤ x ≤ 3
From your graph, estimate
(i) The value for x3 − 2 = 0
(ii) The solution of the equation x3 − 2 = 3x + 2
Answer (x = 1.4 + 0.1) (x = 2.2 + 0.1)
12.

In the diagram above AD is parallel to OC and OA parallel


to BD. 30C=50B. E is the point where AC meets BD .
AE: EC = 3 :2
Find (i) in terms of the vectors a and b, the vectors AC,
DC, ED, AE and OE.
(ii) The ratio BE : ED.
In the figure above, a pyramid whose base ABCD is a Answer  5b  3a , 2b  3a , 3 a , 1  5b  3a  2a  5b 
 
rectangle of sides 8cm by 6cm has slanting edges AE =  3 3 5 5 5 
DE = BE = CE = 6cm.F is the point of intersection of 15. (a) The speed at which water comes out of a pipe is
inversely proportional to the cross-sectional area of the
the diagonals of the rectangle. G is a point on EF such
pipe. Given that water comes out of a pipe of cross-
2 section 5cm2 at the speed of 1ms-1, determine the
that FG = FE .Find
3 difference in the cross-section of two pipes from which
(i) angle AEC water comes out at speed of 0.8ms-1 and 1.2ms-1.
(ii) The lengths EF and AG
(iii) The angle which each of slanting planes makes with b) A circle of radius 2cm and centre O, has points A, B and
the base. C along its circumference. A and C are joined to form

397
chord AC such that it subtends an angle of 800 at point B, 1995 PAPER ONE
calculate the perimeter of the region enclosed between the
SECTION A
chord AC and the major arc ABC.
Answer (2.08cm2) , (6.98cm)
16. The distance from Lira to Kampala is 380km. A bus 1. Without using tables or calculator evaluate
2 / 5 2
leaves lira at 0730 hours and travels non stop to  8   5 
Kampala at 60kmh-1. At 0850 hours a Pajero car leaves   . 1 / 2  Answer (2)
Kampala and travels towards Lira at a steady speed of  125  8 
120kmh-1. On the same axes draws distance–time 2. The functions f(x), and g(x) are defined as f(x) = x + 2,
graphs showing the journey of both vehicles. Hence or
x2
otherwise determine when and at what distance from and g ( x)  . Find the values of x for which fg(x) = 3
Lira they meet. If the bus then increases its speed by 4
10km-1, Answer (x = + 2)
(i) Calculate the time at which the bus arrived in 3. Juma takes 10 days to dig a certain piece of land. John
Kampala. takes 15 days to dig the same piece of land. Assuming
(ii) Determine the difference in the times of arrival of that both work at the same rate, determine the number
two vehicles (Use scales of 2cm to represent 50km
of days they will both take do dig the same piece of land
and 2cm to represent 1 hour).
Answer (10:30hrs) at a distance 180km from Lira, if they work together. Answer (6)
13:22hrs, 4. Given that log10 x 2.304 and log 10 y  2.872 , find the
1:22hrs
value of x 1 / 4 y 1 / 2 . Correct to 2 decimal places.
17. The table below shows the tax structure on taxable
income of citizens in the working class of a certain country. Answer(13.80)
15
Income (sh) per annum Tax rate 5. Given that tan   , calculate without using tables or,
(%) 8
(i) 1st sh 80,000 7.5 calculate the value of 4 cos   sin  . Answer (1, -1)
(ii) Next sh 80,000 (160,001-240,000 12.5
(iii) Next sh 80,000(160,001-240,000 20.0 6. Find the equation of the line which is the perpendicular
(iv) 240,001- 320,000 30.0 bisector of the line passing through the points A(3, 4)
(v) 320,001- 400,000 36.5 and B(1, 10). Answer(3y = x + 19)
(vi) 400,001- 480,000 45.0 7. If matrices M and N are such that
(vii) 480,001- and above 52.6  2 1 12 0 
M    and MN    . Find matrix N.
 2 3  0 12 
A man’s gross annual income is sh 964,000. The
 9  3
allowances including insurance accrued to him were; Answer N 
(i) Housing sh 14,500 per month.  6 6 
(ii) Marriage: one tenth of his gross annual income
8. The mean weight of a class of 30 boys is x kg. When two
(iii) Medical sh 50,700 p.a.
(iv) Transport sh 10,000 per month boys with a total weight of 150kg are absent, the mean
(v) He has to pay an insurance premium of sh 68,900 per weight of those present is 2kg less than the mean weight
annum. of the whole class. Find the value of x. Answer (47)
(vi) Family allowances for only four children at the
9. Given 5  20  a 5 Determine the value of a.
following rates sh 3,400 for each child above the age of
5
18, 4,200 for each child above 10 but below 18 years
and sh 5,400 for each child below 9 years. Given that Answer 3 5
he has a family of five children with three of them
below the age of 8, one 16 and the elder child 20 years,
determine; SECTION B
(a) His taxable income.  3  1.5 
(b) The income tax he pays annually as a percentage 10. If vectors a  
  2  ~  3 
, b 
of his gross annual income.
~
   
Answer (435600), (10.5) 1
Find the length of a 3b
2~ ~ Answer (10)

398
A B
11.
14. On the same axes draw the graphs of the lines
2m y –2x = 1, and y + 3x = 6 for -3  x  3. Use your
C M D graphs to solve the equations.
E F 2x – 1 = 0
3x – 6 = 0
G H Answer (x = -0.5, y = 0), (x =2, y= 0)
15. Aggrey and Bob are to travel from town A to town B
In a seminar, the high table CD which is 3m long is used by ridding on a bicycle and motorcycle respectively. When
guest speakers. The table is placed in front of and parallel Aggrey is 21km away from town A and ridding at a steady
to wall AB. Some chairs are arranged behind the high table speed of 18.5kmh-1, Bob sets off for town B on his motor
with the front legs of each chair occupying 0.5m along CD. cycle at a steady speed of 36kmh-1. Bob is expected to ride
Participants are seated on chairs arranged in circular form 1
placed in front of the high table along the arcs EF, GF, and for 3 hours to reach town B.
IJ of circles whose centers are the point O along AB as 4
shown in the diagram above. The chairs are also arranged (a) Calculate:
such that each occupies 0.5 m of the length along the arcs. (i) The distance between A and town B.
Given that the perpendicular line from O bisects CD at (ii) When and where Bob will catch up with Aggrey.
point M and CE = EG = GI = 2M (iii) How long Bob will take waiting for Aggrey to join
(i) Find the angle COD him in town B.
(ii) The maximum number of guest speakers that can (b) Represent Aggrey’s and Bob’s journeys on the same
get seated at the seminar. distance time graph.
(iii) The maximum number of participants that can get Answer (117km) ; Bob will catch up with Aggrey after
seated in chairs arranged along the arcs. (correct 1.2hours and he would have travelled 43.2km;
your answers (number of people to the nearest 1.94hrs
whole number)
Answer (73.4, 6, 48) 16. A flag mast slants towards the west at an angle of 13 0 to
the vertical. From a point M to the east and 20 metres away
12. A circle passes through the points from the foot F of the mast, the angle of elevation of the top
P(0.5, -3), Q(2, 4.7) and R(4.5, 1). P of the mast is 350. From another point N to the west of
the mast the angle of elevation of the top P is 220. If M, F
(i) Plot and join the points P, Q and R on a graph paper. and N are on level ground, determine to 4 significant
(Use 2 cm to represent one unit on either side). figures.
(ii) By construction determine the centre and radius of (i) The vertical distance of the top P from the ground.
the circle. (ii) The distance of the foot of mast F from N.
(iii) Calculate the area of the minor segment cut off by (iii) The length PF.
the chord PQ. Answer: 16.72, 41.38, 17.16
Answer centre = (0.5, 1) radius=4cm, 78cm 2
17. A triangle ABC where A, B and C are the points (2, 3) (6, 3)
13. The monthly salaries of 300 employees working in a and (4, 6) respectively is given a transformation represented
certain bottling company are as follows;  0  3
by the matrix M  
  1 2  followed by the matrix
 
Number of
 2 3
Salary range employees N    to give the final image A1, B1 and C1
26001 – 36000 56  1 0 
36001 – 46000 74
(i) Find the image points A1, B1 and C1
46001 – 56000 82 (ii) Describe the single matrix transformation that is
56001 – 66000 38 represented by the combined matrix
transformation M followed by N.
66001 – 76000 25
(iii) Obtain a single matrix that would map A1, B1 and
76001 – 86000 15 C1 back onto ABC.
86001 – 96000 10 A (-6, -9), BI(-18, -9) and CI(-12, -18)
I

(i) Represent this data on a histogram.  3 0    13 0 


(ii) Calculate the mean monthly salary.  ,  1
(iii) Estimate the median salary. 0 3 0  3 
Answer (50567.2, 48439.5)

399
1995 PAPAR TWO
SECTION A
2 x  8 x 1
1. Simplify Answer (2)
16 x 1
Q In the figure above OA = a ; OB = b 3AD=AB
2 ~ ~

 2a  b 
Find OD in terms of a and b . Answer  
~ ~
 3 
132°
O 9. Determine the solution set of the inequality
P 120° x2 2 3
< Answer x 2
4x  2x 2
3 4
R 10. A three digit number is formed using each of the digits
2, 4 and 6 only once. List the possible numbers that are
The diagram above shows a circle centre 0 circumscribing a
formed. Calculate the probability that the number
triangle PQR. Given that angle POQ=1320 and angle
POR=1200, find the angles of triangle PQR. 1
formed is greater than 430. Answer
Answer (54, 600, 66) 2
3. Given that f(x) = 1 SECTION B
1 x 11. The figure below shows the marks in percentage
(i) find f(2) obtained by candidates in an English test.
(ii) State the value of x for which f(x) is not defined. 43 70 50 35 64 62 50 53
Answer (-1, 1) 46 62 65 83 59 54 58 64
4. A butcher sells 5kg of meat at sh 7,000. If the cost of 52 54 32 59 48 54 35 48
meat is increased by 25%, determine how much 40 58 64 40 71 74 55 70
kilograms of meat can be bought with the same amount 72 48 75 45 55 40 57 55
of money after the increase. Answer (4kg) (i) Starting with 30 as the lower class limit of the first
y class and using equal class intervals of 5 marks, form
5 y=x+5
A a frequency distribution table for this data.
B
(ii) Plot a cumulative frequency curve for the data. Use
your data to estimate the median mark.
(iii) Calculate the mean mark using an assumed mean of
57%.
Answer (52.5, 55.75)
O x 12. (a) A line passes through the points (a, 0) and (o,
C 2 units
In the diagram above the Equation of the line AB is y – 5 = b). Find the Equation of the line.
x and C is 2 units from 0. Find the area of OABC. (b) Given that a line, L passing through the point (0, 2) is
Answer (8sq.units) perpendicular to the line 2y = 5x + 3, find the point of
intersection of the line L with the line 2x = 3y - 5.
6. Solve the Equation 106 x  (20  0.5) x
2

Answer (-3. 2) Answer (ay = ab – bx) (5y + 2x = 10) 5 , 6


 4.5 1   2  1 29 29
7. Given the matrix A    and B   , 13.
 0 7  3 1 
find matrix M such that 3M – 2I = 2A – B , where I is an

Answer 
3 3
identity matrix of order 2. 
 1 5 
8.

400
The diagram above shows a solid object with a regular (iii) The area of minor sector BOC and hence the area of
pentagonal base ABCDE of side 20cm and centre 0. The the shaded region.
Answer (120, 6.93cm, 50.3cm, 32.5cm 2)
vertex V is vertically above 0 and VO = 30cm.
17. A soccer club wishes to intensively train its top and
second division players by residential training in
(i) Find angle BCO.
preparation for soccer league tournaments. The cost of
(ii) Calculate the length OC.
maintaining a player is sh 60,000 and sh 45,000 per top and
(iii) Obtain the length of VC and the angle at which it is per second division player respectively. The club has a
slanting to the horizontal (give your answer correct to a maximum of sh 1,800,000 for the residential training. One
decimal of a cm) and a third times the number of top players must not exceed
0 0 the number of second division players. Given that the club
Answer 54 , 17cm, 60.5
can only train up to 35 players who must be selected from
14. Using a ruler and a pair of compass only, construct a
the two divisions of players.
triangle ABC such that AC =9.6 cm, BC = 4.8cm and the (i) Write down the set of inequalities representing the above
angles, BAC=300 and ABC=900. D is a point on BC information.
(ii) Using a scale of 2cm to represent 10 units on each axis,
produced 2.7cm away from AB . Construct angle BDE = draw on the same axes graphs for these inequalities.
450 with DE =10.1cm. join the points A to D and B to E, (iii) Shade out the unwanted regions and find the maximum
construct a circle circumscribing triangle ACD such that it number of players from each division the club can train.
also passes through the point E. Answer (4x + 3y < 120, x + y < 35, 3x < 3y, x > 0,
Measure: y > 0)
(i) Lengths AB and BE. Maximum no. of players = 35
(ii) Angle ADC.
Answer (8.2cm, 8.6cm) ABC  70 ; radius = 5cm
15. Sarah bought a four-inch mattress (a mattress whose
thickness is 4 inches). She then went to John, a tailor, and
bought a cloth which can exactly fit the mattress as a cover.
John sold her 4.6m2 of cloth while according to John’s
experience the 4.6m2 of cloth was exactly enough to cover
all the sides of the mattress.
(a) Given that 1 inch is approximately 2.5cm; the width of
the mattress is w cm and length 2w cm, find w.
(b) If she paid the mattress and its cover cloth Uganda
shillings 52,500 and 36,500 respectively, calculate in
pounds sterling, ₤, given that 1 United States dollar $
is equivalent to U Shs 950 and 1₤ = 1.8$, the
(i) Price of the mattress.
(ii) Total cost of the mattress and its cover.
Answer (100, £30.7, £21.35, £52.05)
16.
B
12cm
A
12cm O
C 12cm
C
C
In the figure above, AB and ACCare targets to the circle at
points B and C respectively. O is the centre of circle. Given
that AB  BC  AC = 12cm. Determine
(i) The obtuse angle BOC.
(ii) The radius of the circle.

401
1996 PAPER ONE  0  4 8
O   , p   , and q    . Find the coordinates of
SECTION A  
0  
7 9
R, the mid-point of PQ and the distance of R from O.
1. Without using tables or calculator, evaluate
Answer (6, 8), (10 units)
32 .135 2  17 .865 2
Answer (1000)
0.7135 10. Two mirrors M1 and M2 are placed 12cm a part parallel to
2. A car’s petrol tank is three quarters full at the beginning each other. An object O is placed 4cm away from M2. If
of a journey. If the car uses two-thirds of a tank-full of M1 (0) denotes the image, O1 of O after a reflection of O
petrol for the journey, what fraction of a tank full in M1, determine the distance from O of M2 [(M1 (O)], the
remains at the end of the journey? image, O11, of O1 after a reflection of O1 in M2.
On returning the car refuels such that at the end of the
return journey the car is one-fifth tank full. What Answer (24cm)
fraction of a tank full of petrol was bought?
Answer  1  ,  9  SECTION B
   
2  20  11. In a certain trading centre there are 56 shops, 28 of
3. Without using tables or calculator, simplify which sell soft drinks, 24 sell food stuffs and 32 sell
1 a textiles. 10 shops sell both food stuffs and soft drinks, 6
log 10 16  2 log 10    log 10 a 2 Answer (2) sell both food stuffs and textiles and 4 shops sell all the
2 5
three categories of commodities; both food stuffs and
4. Given that 124n= 52ten, determine the value of the natural
textiles and 4 shops sell all the 3 categories of commodities.
number n. Answer (6)
(a) (i) Represent this information on a Venn diagram.
1 (ii) Determine the number of shops which sell both
5. In a class of boys and girls, the average age is 15
2 soft drinks and textiles only.
years. The class has 12 boys whose average age is (b) If a customer is to choose a shop at random, what is
3 the probability that the shop he goes to sells
16 years. Find the size of the class if the average age
4 (i) At least two of these categories of commodities.
of girls is 15 years. (ii) Only one of the three categories of commodities.
Answer (42) Answer (12, 3 , 4 )
7 7
6. y
12. Using a ruler, pencil and a pair of compasses only,
A y=4 B
(i) Construct a triangle ABC, where
y=x
y = 12x
BC = 7.2 cm, AC = 8.4 cm and angle
ABC = 750. Measure AB and angle ACB.
(ii) Draw a circle circumscribing triangle ABC. State
O C x
the radius of the circle.
In the diagram above OABC is a trapezium formed above
Answer (AB = 6.5cm, ACB  48 , radius =
0
the x-axis by the intersections of lines y = x, y = 4 and y
=12 − x. 4.3cm)
Answer A(4, 4) B (8, 4) C(12, 0) 13. (a) Two saloon cars A and B are hired to carry people
going to attend a wedding ceremony. With six trips
7. Find the solution set of the equation x  32  4 2 each, the two cars can carry 60 people. A total of 62
Answer (x = -1, 7) people can be transported if A makes seven trips and
8. Two right cones are similar with linear scale factor 2. If
B makes five trips. Determine the number of people
the larger cone has radius of 14cm and height of 27cm,
each car can carry per trip.
calculate the volume of the smaller cone. (Take   22 , (b) Otim and Mukasa stay in the same home. When
7
volume of cone = 1 × height × base area Answer (693) Otim walks from home to school at a constant speed
3 of 5.4 kmh−1, he arrives 10 minutes early. When
Mukasa walks at a constant speed of 3.6kmh−1, he
9. Three points O, P and Q in the same plane have position arrives late by 15 minutes. Calculate how far the
vectors school is from their home.
Answer (A carries 4 people and B carries 6 people)

402
4.5km 1
14. A food Aid Agency carried out a survey to ascertain the 060oto air strip D at 240 kmh-1 for 1 hours. The
2
average monthly expenditure on food by a family in a
certain urban centre. The expenses on food were found to total time of flight between the four air strips was 4 1
2
be in two parts; a constant expenditure and another part
hours.
varying as the square of the number of children in the
(i) By scale drawing, determine the distance and
family. bearing of A from D. (use a scale of 1cm to
A family of 3 children needed Shs19,000/= while that of 5 50km).
children needed Shs31,000/=. (ii) Determine the total distance of flight from A to D
(a) Write down an expression for the total expenditure and hence the average speed for the journey.
on food, E, Spent per month by a family with n (iii) If the plane flew directly back to A at a speed of
children.
200kmh-1, determine how long it took to fly back to
(b) What is the monthly food expenditure for
(i) A childless family? A.
(ii) A family with 4 children? Answer (575km, 21.3cm, 236.67km/hr, 2.875hrs)
(c ) How many children are in a family which needs an
average food expenditure of Shs39,250/- per
month?
Answer (E = 12250 + 750n2) (12250, 24250, 6)
15. Draw on the same co-ordinate axes, graphs of
y  x2 x  3 and y  2 x  1 for  3  x  4
(i) Using your graph, determine the points of
intersection y  x2 x  3 and y  2 x  1
Use your graph to find the roots of 2 x  3 x  0
2
(ii)
Answer (x = 0, x = 1.5)
16.
Class Frequency X Class
Scores Mark Frequency Mark
40-49 _ _ 450
50-58 _ 16 _
59-67 63 _ 1575
68-76 _ _ 864
77-85 _ 13 _
86-94 90 4 360
______ _________
The table above shows the number of students who passed
an end of year English promotional examination in terms of
mark scores.
(a) Study the table and the information available to
complete the missing details.
(b) (i) State the class interval of the scores.
(ii) Calculate the average score of the marks.
(c) If all the above students were promoted and represented
4/5 of the class, find the number of students in the class
who sat the examination. Answer (9, 64.575, 100)

17. A plane flew due west from air strip A at a speed of


280 kmh-1 for ¾ hours before reaching air strip B. It
then altered its course and flew North-west to airport
C at 220kmh-1. From there, it flew on a bearing of

403
1996 PAPER TWO 7. When a cyclist has traveled a distance of 105km in
1 23 hours, he cycles at an average speed of 54kmh-1 for
SECTION A further 2 13 hours. Calculate the average speed for the;
1. Given that a*b = a2 − b2, find the value of x in
(i) First 1 23 hours
x 3  74
Answer (x = + 6) (ii) Whole journey
Answer (63km/hr), (57.75km/hr)
2. A
8. Using matrix methods, determine the values of x and
540 y which satisfy the equations.
6.5cm 2y – 4x + 2 = 0
4.2cm 3x – 2y = 5 (4 marks)
Answer (x = -3, y = -7)
B C
D   2  1    3
9. If U   , V    and W =   , Find the
In triangle ABC above, AD = 4.2cm is the altitude,
 4 
 3  4 
angle BAC = 540 and
value of a and b such that
AB = 6.5cm. Find:
a ( u ) + b( v ) = w
(i) The size of angle CBA ~ ~ ~

(ii) Length AC Answer (a = 2.5, b = 2)


Answer (40.30, 4.21cm) 10. Express 0.891as a rational number in its simplest
3. Given that y varies as the cube of x and y = 8 when x = 4,  99 
find the value of x when y = 1. terms. Answer  
Answer (2)  111 
4. Solve the equation 3 x  7 x  2  0
2 SECTION B
Answer (x = 2, x  3 )
1

P
5. If x : y : z  2 : 3 : 5 , determine the value of z when:
(i) x  y  z  200
9m
(i) y  84 S 6m T
Answer (z = 100, z = 140)
6. A 16cm B
Q U R
11.
The diagram above shows part of a rafter of a building. QR
h 17cm 17cm is parallel to ST. PQR is an isosceles triangle with Q and R
as base angles. U is the mid-point of QR.
PS : SQ  3 : 2, ST = 6cm and PR = 9m.
C
The diagram above shows a right circular cone ABC of
Calculate the;
vertical height h cm and slant side AC = BC =17cm
(i) Lengths of QR, TR and PU
and base diameter AB = 16cm.
(ii) Size of angle PQU.
Find:
(iii) Area of DPQR.
(i) h
Answer (QR = 10, TR = 3.6, PU = 7.48m, 37.4m 2)
(ii) The capacity of the cone.
(Use  =3.142, volume of cone= 1 h × (base 12. (a) If x3 = 3.375, use tables to find the value of x;
3 correct to three significant figures.
area) (b) Given that log102 = 0.3010 and
Answer (1.0048ltrs) log103 = 0.4771, find without using tables or calculator, the
value of:
(i) log1072

404
(ii) x if log10x = 1.6020 (ii) For how long the particle will be less than 2m away
Answer (1.8572, 40) from P.
(iii) The distance the particle travels between t = 2.5 and t
13. A builder makes concrete blocks by mixing cement and = 4.5 seconds.
(iv) The speed of the particle between t = 0.5 and t = 2.5
sand in the ratio 1 : 20 . From every 3 buckets of mixture he
seconds.
makes 2 blocks, one bucket of cement costs sh3,000/= and Answer (0.8, 5.3, <2m, 1.3m, 2.2m/s)
one bucket of sand sh 25/-. Find the cost of cement and
16. A
sand required to make 6,300 blocks.

(b) A grocery sells two kinds of meat products A and a


~
M
B. Athieno bought 4 kg of A and 6kg of B paying a B
O b D
total of sh 5280/=, Namusisi bought 5kg of A and 3kg ~

of B at a total cost of sh 4,440/-.


C
(i) Write down two equations to describe
Athieno’s and Namusisi’s purchase. E
(ii) By combining the two Equations in matrix In the figure above OA = a and OB = b , 3OB = 2BD
form, determine the cost of 1kg of each meat ~ ~
product. , M is a point on AD such that MD : AM = 1 : 2,
(iii) How much would Mugisha pay for 6kg of A OC = 3CE = 3AM
and 5kg of B. (i) Express the vector AD, BM and DC in terms of
Answer (600, 480), 600)
vectors a and b .
~ ~
14. In a certain school there are 87 students in S.3. Of (ii) Show that AD: OE = 3 : 8
Answer  1  5b  2a  , 1  a  2b  , 1  5b  4a  
these 43 play hockey, 42 play football and 47 play
volleyball, 15 play hockey and volleyball, 17 play
2 3 2 
volleyball and football and 21 play hockey and
17. A factory makes two kinds of bottle tops “Coca-cola”
football. Each student plays at least one of the three
tops and “Pepsi tops”. The same equipment can be used to
games while x students play all the three games.
make either. In making Coca-cola tops one man can
(i) Represent this information in a Venn diagram, supervise 10 machines and this batch will give a profit of
showing clearly the number of students in each pounds sterling (₤) 50 per week. Pepsi tops yield a profit of
region. ₤250 a week, using 25 machines and 8 men. There are 200
(ii) Write down an equation in x and hence find x. machines and 40 men available. By taking x batches of
(iii) If a student is chosen at random from the class, coca-cola tops and y batches of Pepsi tops, write down
what is the probability that he plays exactly two inequalities for;
games?
(i) The number of machines used.
(iv) Find the number of students who play at least two
(ii) The number of men employed.
of these games.
(iii) An expression for profit P.
1
Answer (x = 8, , 37)
3 Use these inequalities to draw a suitable graph showing the
region which satisfies them. From your graph, determine
15. (a) Copy and complete the table below of values of the numbers of Coca-cola and Pepsi tops which should be
4 made to obtain the maximum profit. Hence find the
S t 3 where S is the distance covered (in m)
t maximum profit.
by a particle from a fixed point P after t seconds. Answer (2x + 5y < 40, x + 8y < 40, p = 50x + 250y)

S 5.5 1.17 1.1 1.33 3.67


t 0.5 1.0 1.5 2.0 2.5 3.0 4.0 5.0 6.0
(b) Draw a graph of S against t for t = 0.5 to t = 6.0
inclusive. (Use 2cm to represent one unit of t and 4cm to
represent one unit of S)
From your graph estimate:
(i) The times when the particle will be 3cm from P.

405
1997 PAPER ONE SECTION B
11. The vertices of triangle ABC; A(1, 0); B(1, 2); C(5, 2)
SECTION A are mapped onto the triangle A1 B1 C1 by the
1. A teacher awards two marks for each correct answer plus 1 0
transformation whose matrix M   
three marks as an incentive for test attendance. If Q is the  2 4
number of questions attempted and M the total marks a (a) Find the
candidate gained in the test. (i) Coordinates of the vertices of the image triangle.
(i) Write down a relation between Q and M. (ii) Ratio of the area of triangle ABC to the area of
(ii) What is the maximum mark score for a test with triangle A1 B1 C1.
only nine questions?
(b) Plot on the same axes triangle ABC and its image.
Answer (m = 2Q + 3, 21)
(c) Determine the matrix of the transformation which
x 3 x  2 x 1 28
2. Solve for x in    . Answer maps A1B1C1 back to ABC.
5 3 2 3 19 Answer A1(1, 2), B1(1, 0) and C1(5, 18);
3. Otti takes 6 days to plough a certain piece of land.  1 0
Mpoza takes 12 days to plough the same piece of land. 1 : 4;  1 1 
Assuming that both work at the same rate, how long will  2 4 

the two men take to plough the piece of land if they 12. The table below shows the marks scored in an English
work together? Answer (4) test marked out of 100 by students of S.2 in a certain
school.
4. Given that f(x) = px – 4and f(3) = 14, find: Score f(x)
(i) The value of P. 0-19 2
(ii) f (1) Answer (6, 2) 20-39 6
5. Factorise a  2ab  5a  2b  4 40-49 12
2

Answer (a – 1) (a – 4 – 2b) 60-79 9


80-89 1
 2  - 5  6 
6. Given the vectors p   , q    and r   ,
 
3  3   2 (i) Represent this information on a bar-chart.
(ii) Use the frequency table above to estimate the
Find the length of (p + q + r ). Answer (5 units)
median and mean mark score.
Answer (50.17, 51.17 )
7. Given that log 10 x  1.699 and log 10 y  1.913 ,
13. Fifty-two students of S.3 in a certain school were
evaluate without using tables or calculator interviewed to find out how many of them had ever visited
log 10 x  log 10 y1/ 3 . Answer (1.670) the towns of Arua, Kasese or Mbale. Only 4 had visited
1 3 . Answer 4 3  2 2 neither towns. It was found out that an equal number of
8. Simplify 
3 2 3 2 students had visited Arua and Kasese of whom 12 had
9. The numbers 0, 1 and 2 are arranged in a random order visited both Arua and Kasese. 24 students altogether had
so as to form two digit and three-digit numbers. No digit visited Mbale of whom 11 had visited both Mbale and Arua
is repeated in a number formed. and 13 had visited Mbale and Kasese. Eight had visited all
(i) Write down the possible numbers that can be the towns,
formed. (a) Present this information on a Venn diagram.
(ii) Find the probability that a two-digit number formed (b) How many students in the class had
has the same numerical value as the three-digit (i) visited Kasese
number formed (ii) Not visited Arua?
1 (c) Given that three students are taken at random from the
Answer   class, what is the probability that they belong to the set of
6 students who only visited two towns?
10. Given that H denotes half turn about the origin and X
3 1
denotes reflection in the X-axis, find a single Answer (26, 26, , )
transformation to XH on the points of the unit square. 13 4
Answer 
 1 0 14. Using matrix methods find the values of x and y which
 satisfy the equations.
 0 1 2x − 3y = 12

406
x + 2y + 1 = 0
1997 PAPER TWO
 3  2
(b) given that matrix A    , find a matrix B
 4 5  SECTION A
7 0 1. A musical tape costs pounds sterling (₤) 8.95. Given that
such that AB    the exchange rates are US dollar $1.56= ₤1.00 and Ug
0 7
sh. 1045 = 1$, find the equivalent cost of the musical
Hence otherwise find the inverse matrix of A.
tape in
 5 2   75 2
7 (i) U.S dollar.
Answer (x = 3, y = -2),  , 2 3
 4 3  7 7
(ii) Ug. Shillings Answer ($13.962, Sh. 14590.3)
2. Determine the area of figure enclosed by the x-axis, the
15 E
8cm
line 2x + y = 8 and the reflection of this line in the y-axis.
Answer (32 sq.units)
D C 3. The scale of a map is 1:200,000. Two trading centres on
the map are 4.5cm a part. Determine in km the actual
5cm distance between the two trading centres. Answer (9km)
F
600 1 a 
A 5cm B 4. Under the matrix transformation   , the point P
b  4
The figure shows a pyramid whose base ABCD is a
rhombus of sides 5cm and whose acute angle is 60 0. AE = (3, -2) is mapped on the point P1(-1, 17). Find the values
DE = CE = BE = 8cm. F is the point of intersection of the of a and b. Answer (a = 2, b = 3)
diagonals of the rhombus. Find 5. Four athletes ran a 100m races. Their times taken to
(i) Length EF (ii) Angle AEB (iii) The angle each of the complete the race in second(s) were 16.5, 13.6, 10.8 and
slanting planes makes with the base. 12.4.
Answer (45.25, 6.73, 36.4, 69.6) (i) Write down the median, in (s) of the four times.
(ii) Calculate the time taken by a fifth athlete, if the mean of
16. (a) The length of the sides of an equilateral triangle all the five times was 13.4 seconds. Answer (13, 13.7)
ABC is x units. 6.
3 y
(i) Show with the help of the triangle that sin60 0 =
2
(ii) Without using tables or calculator, find the value of 12
2 A1
 sin 60 0
0
 
 sin 30 0  tan 60 
11
 
10 C1
(b) Draw the graph of y = cos3x for 0  x  150
0
B1
Answer (12) 9

17 8
0.8m
7 C B
2.0m 6
1 2 3 4 5 6 7 8 9 10 x
5 A
1.0m
The figure above shows an open metallic water tank made In the4 graph A'B'C' is the image of ABC under an
of material which is 2.0 cm (0.020m) thick. The metal used enlargement with scale factor f. Determine the
for making he tank costs Shs 100 for every 50cm3 (5 x 10-5 (i) Value
3 of f.
m3). (ii) Co-ordinates of the centre of enlargement.
(i) Find the cost of making this tank. Answer 2 -1(4, 5)
(ii) If the tank is to be filled with water at a fee of 5 7.
cents (Shs 0.05) per litre, determine (in Shs) the
cost of filling the tank. (1m3 = 106 litres). 1
Answer : (264.704, 73.4)
0

-1
407
-2
h metres high, is 2m higher than building A and MO =
24m.
Calculate:
(i) The height of building A.
(ii) How far building B is from point O.

(b) A ship is observed moving away from the top of a cliff


which is 80m high. Within a time span of 10s the angle of
depression decreases from 300 to 200. Determine the
distance covered within this time range. Hence find the
speed of the ship in meters per second (ms-1)
The pie chart shows the percentage of people living in three Answer (18.75, 15.08, 8.124m/s)
L.C.1 zones I, II and III of a certain village. The sector 12 (a) A dispenser produces medicine in two different sized
cylindrical plastic cans. The cans are similar. The area
representing people living in zone II is 18% while the one
of the bottom of the small can is 12cm2 and that of the
representing those living in zone I is 42%. If the number of bottom of the large can is 48cm2, given that the small
people living in zone III is 240, find the can holds 100ml of the medicine, calculate;
(i) Population of the village.
(ii) Number of people who live in zone I. (i) How much medicine is contained in the large can.
Answer (600, 252) (ii) The height of the large can.
8. A blouse and skirt were each sold at sh. 12,420. On the (b) The dispenser uses exactly half of the medicine of the
small can and then mixes it thoroughly with 37.5 ml of
blouse a profit of 15% was realized while on the skirt a loss
distilled water to obtain a mixture for treatment of
of 12.5% was recorded. Calculate the percentage loss on patients. Calculate the height (in cm) of the mixture in
both articles. Answer (0.62%) the small can. Answer (800cm3, 16.67, 17.29)
9. A
13.
F

a B
O ~

a
D b G
C A ~
b O C
B In the figure above OA = b ; F and G are points on AC
In the figure above, O is the centre of the circle and b = ~
1180, find the value of a. Answer (1240) such that AF: AB=3:4 and AG:AC=2:3, respectively. D is a
10. Mukasa, Kaija and Obita are to share sh. 90,000 in the point on OA such that OD: DA = FB : BG =1:2.
ratio 3:4:5, respectively. Calculate the amount each will
(i) Express AG and AC in terms of AB. Hence find in terms
receive. Answer (22500, 30000, 37500)
of vectors a and b the vectors; AB; AC; DG and OF.
~ ~
(ii) Determine the ratio DG: OC
SECTION B Answer
11 (a) 3 9 9 9b  5a a  3b 
 AB, AB, b  a,  b  a  , , , 2 : 3
2 4 4 6 4 
14. (a) The line y = 2x meets the line x = 3 at the point A.
(i) Give the coordinates of A.
(ii) If the x-axis is the axis of symmetry of triangle OAB,
find the coordinates of B.
(iii) Find the equation of OB.
(iv) Write down the inequalities for a point (x, y) inside
the triangle OAB.
(b) Calculate the area of triangle PQR where P is (3, 3), Q
From the point O on level ground MON, between two is (4, -1) and R(-8, -4).
buildings, A and B, the angles of elevation of the tops of Answer (3, 6) (3, -6) (y = -2x), (y < 2x),
buildings A and B are 380 and 540 respectively. Building B,

408
(y > -2x), (x < 3)(25.5sq.units) (c) Distance covered by taxi A by the way of estimating
15. 2m the area under curve described by the motion of taxi
A for a period of 85.
A D Answer (3.6seconds, 7m/s, 6m/s, 64m)

2xm
B C

2xm
The diagram shows four rectangles A, B, C and D which
together form a square of side 2xm. Given that, one side of
D is 2m, and that the area of A+D is 4m2, express the other
side of D in terms of x.
Given also that area of A + B + C is 11m2.
Show that 4x3 = 11x + 4.

16. The position vector of a point (x, y) is written as the


 x
column vector   . A geometrical transformation is
 y
represented by the equation.
 x1   a b  x 
 1     
 y   c d  y 
a) Write down the image of (1, 0) and (0,1) under this
transformation.
b) Show that the image of (1, 0) under a rotation of +30 0
about the origin is  3 , 1  and find the image of
 
 2 2
(0,1) under the same rotation.
c) Using your answers to part (a) and (b) above or
otherwise, write down the matrix M of this rotation.
d) Calculate M2 and write down the coordinates of the
image of (1, 0) and (0,1) under the transformation of
M2.
Explain your answer geometrically.
Answer (a, c), (b , d),
 23  12   12  3 

1
, 
3  3
2
1
,
  1
2 , 2
3
,  2
3
, 1
2 
2 2   2 2 
m2 represents a rotation of +60 about the origin
17. When traffic lights flash green, two taxis A and B move
off from rest in the same direction and on a straight road.
The speed of taxi A increases at a uniform rate of 2ms-2
while taxi B moves as shown in the table below.

Time (S) 0 1 2 3 4 5 6 7
Velocity (ms-1) 0 0.5 1.5 4 10 15 18 19.5
Using suitable scales, draw on the same axes the velocity-
time graphs find the,
(a) Time the two taxis have Equal speeds and state the
magnitude of that speed.
(b) Difference in the speeds of the two taxis after a
period of 6 seconds.

409
1998 PAPER ONE that of cubes with at least a blue face and Y the cubes with
at least a yellow face.
SECTION A a) Represent all the above information in a Venn
1. (a) Without using tables or calculator evaluate diagram. Show any remaining information
14 × 398 – 198 × 14 b) Find the number of cubes with
(b) Factorise completely 2a2-32 i. All the three different colour faces
Answer (2800) 2(a+ 4)(a – 4) ii. At least one of each of the three colour faces.
c) If a cube is picked at random what is the probability
 12
2. Given that tan  = and  lies between 00 and that it is grey or blue `only?
5 Answer (4, 52, 6 )
1800. Find, without using tables or calculator the values of 13

sin  and cos  . Answer  12 , 5 


12. Sixty 13–year old senior one students from a certain
  school were tested to find their resting Pulse-rates and the
 13 13 
3. Kato bought a car and sold it to Tom at a loss of 25%. If following figures were obtained for a number of beats per
his selling price was sh. 3.6 million, find the cost price of minute.
the car. Answer (4.8m)
72 70 66 74 81 70 74 53 57 62
4. The image of the point (4, -7) under an enlargement of 58 92 74 67 62 91 73 68 65 80
scale factor -2 is (1, 2). Determine the coordinates of the 78 67 75 80 84 61 72 72 69 70
centre of enlargement. Answer (3, -4) 76 74 65 84 79 80 76 72 68 63
82 79 71 86 77 64 72 56 70 67
5. Given that f(x) = x2 and g(x) = (x – 1), determine fg(x).
Hence evaluate fg(-1). Answer (x2 – 2x + 1, 4) 76 56 86 63 73 70 75 73 81 64
a) By arranging the data in classes of 50 - 54, 55 - 59,
6. A family spends its income on the following items in a etc make frequency a table.
month. b) Draw a bar Chart displaying the given data.
Item Food Rent Transport Others c) Using your grouped data, calculate the mean and
Amount (sh) 35,000 12,000 10,000 15,000 median pulse-rate
Show the family’s expenditure in a pie-chart. Answer (72.17, 72)
13. A plane flies from air strip K due north for 350km to air
7. Use the matrix method to solve the pair of simultaneous strip R. it then flies on a bearing of 295 0 for 250km to
equations. airstrip N. From there it flies on a bearing of 090 0 for
2x − 3y = 7 500km to another airstrip M.
14 + 4y = 5x Answer (x = 2, y = -1) a) Draw a sketch diagram to show the route of the plane.
Hence draw accurate diagram using a scale of 1cm to
8. Soma college school is located on a stretch of land of represent 50km.
area 22.5km2. On a certain map its area is 3.6cm2. b) From your diagram, find the distance and bearing of
Determine the scale of the map. Answer (1 : 250000) airstrip K from M.
9. Two dice are thrown once. Find the probability that both c) If the plane flies back to airstrip K by the direct route,
1 and it travels at an average speed of 250kmh-1, find
dice show even number. Answer   the time (in hours) taken for the whole journey.
4 Answer (210, 535, 5.54)
10. Given that curve y = 2x2 + 3x and y = 5x + 4, determine
14.
the coordinates of the points of intersection of the curve and
the line. Answer  2, 14  ,  1,  1

SECTION B
11. Faces of 52 small wooden cubes are to be painted either
grey, blue or yellow. Of the cubes, 15 have all their faces
painted grey, and 9 all faces blue. There are 6 cubes with
grey and blue faces, 10 with yellow and blue and 7 cubes
with grey and yellow faces. The cubes whose faces are all The diagram shows a piece of wood of uniform cross-
yellow are four more than those whose faces are all blue. section PQRS in which OPQR is a rectangle and ORS is a
Given that G is the set of cubes with at least a grey face, B

410
quadrant of a circle, centre, O. the other rectangles are a) Write down five inequalities representing the above
PQYK and PXZS. information.
Given that PQ =15cm, PO = 10cm and QY=40cm, calculate b) Plot on the same axes the above inequalities.
c) By shading the unwanted region, show the region
(giving your answers correct to 3 s.f), the
satisfying all the inequalities.
(a) Area of the cross section PQRS d) List the possible number of trips each vehicle will
(b) Volume of the wood. make, given that all the money for transport is to be
(c) Total surface area of the piece of wood (Take  = used.
3.142) e) What is the greatest number of supporters that was
Answer (327cm3, 13080cm3, 2334cm2) transported?
15 (a) Copy and complete the following table of values for Answer (2x + 3y < 12, 4x + 5y < 40, x – y < 2,
the curve y=(x-1) (x-3) between x = -1 and x = 5. x > 0, y > 0
X -1 0 1 2 3 4 5
x-1 0 2
x-3 0
y = (x-1) (x-3) 0 0
(b) Using the values in (a) Draw the graph of y= (x-1) (x-3)
for -1≤x≤5
(c) Use your graph to solve
(i) x2 - 4x + 3 = 0
(ii) x2 - 4x + 1 = 0
(d) From your graph find the
(i) Range of values for which (x-1) (x-3) <0
(ii) Minimum value of the function.
Answer (x =1 , x = 3), (x = 0.3), (x = 3.7)
(1 < x < 3), (2, -1)
16. Towns A and B are a distance of 138km from each
other. Dick leaves town A for town B cycling at a
steady speed of 24kmh-1. When he has traveled a
distance of 18km from A, Bob sets off from the same
spot Dick started, cycling steadily at 30kmh-1.
a) Find when and where Bob over took Dick.
b) If Bob maintained his speed even after over taking dick
determine how long it took him waiting for Dick to
join him.
c) Given that Dick then increased his speed such that they
both arrived in town B at the same time; by how much
did Dick increase his speed immediately after he was
overtaken?
Answer (90km, after an interval of 3hrs from the time
Bob started his journey, 24minutes, 12km/hr)
17. Supporters of a certain soccer team wish to accompany
their team for a soccer match. They are to travel by a taxi
and a mini-bus. The capacity of the taxi is 18 people while
that of the mini-bus is 27 people. The number of supporters
to go will not exceed 108. Each trip the taxi and mini-bus
make, costs sh.24,000 and sh. 30,000 respectively. The
money contribution for transportation of the supporters is
sh. 240,000. The number of trips made by the taxi should
not exceed those made by the mini-bus by more than 2. If x
and y are the number of trips made by the taxi and mini-bus
respectively.

411
1998 PAPER TWO SECTION B
SECTION A 11. By using a ruler and a pair of compasses only, construct
1. Without using a calculator or tables, simplify a quadrilateral ABCD in which AB = AC  AD = 8cm.
4 13  2 14 Angle ABC=600 and BAD=1200.
22 
1
4 16 (c) Identify the type of quadrilateral. Bisect angle ACB.
2. By expressing each of the numbers in the form Let the bisector meet AB at E and DA produced at F.
a × 10n, where n is an integer, evaluate 0.24 Construct a circle circumscribing triangle AEF.
0.0006 (d) Measure the distance from the centre of circle O to the

 
9 vertex B. What is the radius of the circle?
3. Express in the form a b  c , where a, b Answer (Rhombus, OB=7cm, radius = 4cm)
5 2 12. Mbarara is about 260 km away from Kampala. An express
and c are integers. bus leaves Kampala for Mbarara at 6.45 a.m traveling at a
steady speed of 52kmh-1. A commuter taxi leaves Kampala
4. Find the unknown values in the arrow diagram for the
mapping x  2(x + 1) given below. 1 13 hours later and travels non-stop at a speed of 84kmh-1.
5 12 Draw on the same axes distance-time graphs showing the
journey of the two vehicles.
p 16 [Use scales of 2cm to represent 30km and 2cm to
represent 1 hour]. Hence or otherwise determine the
q 22 time and distance from Kampala when the commuter
Answer (q = 10, p = 7, r = 28) overtakes the bus. If the bus then increases its speed by
5. TM is a11
tangent drawn24from a point T to a circle, centre 20kmh-1
O, and radius 8 cm. if OM = 17cm, calculate the length of Calculate the
TM. 13 r (a) Time when the bus arrives in Mbarara.
(b) Differences in the times of arrival of the two vehicles.
Answer (15cm)
Answer (10:18am at a distance 183km, 11:22am,
 1 2  2  1
6. Given that A   , B  , find AB – 11minutes)
 2 1  -1 2  V
13
 4 0
BA. Answer  
0 4 13cm

7. If A and B are two sets of objects and n(A) = 10, n(B) =


A O 5cm B
7, n( A  B)  13 . n( A  B)1  2 , find
The figure above shows a right circular cone AVB. The
(a) n( ) , where  , is the universal set
radius of the base is 5cm and the slanting edge 13cm.
(b) n( A  B) Answer (15, 4)
a. Calculate angle AVB.
8. M is the mid-point of the line PQ , where P is (2, 6) and b. Find the:
Q(-8, 2). Calculate the distance of M from the point R(1, i. Volume of the cone.
0). Answer (5.66 units) ii. Total surface area of the cone [take  =3.142].
Answer (45.240) (314.2cm3) (282.78cm2)
9. Without using tables or calculator, evaluate log1040 + 14. The points A(-2, 1), B(-2, 4), C(1, 4) and D(1,1) are
log1050 − log1020. Answer (2) vertices of a square ABCD. The images of A, B, C and
D under a reflection in the line x – y = 0 are A', B' and
10. OAB is a triangle with position vectors OA = a OB = D'. The points A', B', C' and D' are then mapped onto the
~
points A", B", C" and D" respectively under an
b . Express in terms of a and b the position vector of enlargement with scale factor 2 and centre of
~ ~ ~
enlargement the origin O(0, 0).
OC, where C divides AB in the ratio 1:2
a) (a)Write down the matrices of the reflection and
 2a  b  enlargement.
Answer  
 3  b) Find the coordinates of the points
i. A', B', C' and D'

412
ii. A", B", C" and D"
c) Determine the matrix of a simple transformation (a) The taxable income and the income tax she pays under
that would map ABCD onto A", B", C", D" the income tax rates below.
Answer AI(1, -2), BI(4, -2), CI(4, 1), D1(1, 1)
Taxable income (sh) Tax rate %
AII(2, -4), BI(8, -4), CII(8, 2), DII(2, 2) 0-15,000 8.50
0 2 15,001-84,000 16.50
 
2 0 84,001-170,000 24.00
15. Three points A, B and C lie on the same level ground. 170,001-285,000 30.00
A vertical pole NP stands in between points A and B 285,001-435,000 37.50
Above 435,000 48.50
such that AN =21m. Angle BAC=720 and AC =21m. (b) Determine the percentage of her gross monthly income
The angles of elevation of the top of the pole, P from A paid in tax.
and B are 180 and 570 respectively. Answer (23.84%)

Calculate the
(a) Height of the pole NP.
(b) Length AB.
(c) Angle of elevation of P from C.
Answer (6.82m, 25.43m), (15.40)
16. H G
E F
12cm

D
C
A 16cm B
The figure ABCDEFGH is a rectangular box with square
ends BCGF and ADHE of side 12cm. AB = 16cm.
Calculate the
(a) Lengths BD and BH.
(b) Angle between lines BH and plane ABCD.
(c) Angle between planes CFH and BCGF.
Answer (20, 3.96, 620)
17. A certain country’s income tax structure is such that a
person’s gross monthly income has certain allowances
deducted from it before it is subjected to taxation. The
allowances spelt out are as follows;
Marriage allowances one-twentieth of the gross monthly
income.
Family relief and Insurance sh. 120,000 per annum.
Water and electricity sh. 12,500 per month.
Housing sh. 35,000 per month.
Medical (self and family) sh. 240,000 per annum.
Transport sh. 800 per day.
Family allowance for four children only at the following
rate; sh. 5,800 for each child above the age of 16, sh. 7,200
for a child above 10 years but below 16 years and sh. 9,000
for a child below 10 years. Joy has a family of four children
with two of them below the age of 9, the elder child is 20
and the other 14 years. Given that she earns sh. 680,000,
calculate

413
1999 PAPER ONE ^
(a) a quadrilateral PQRS such that Q R S  45 , QR
0

SECTION A =4.5cm, RS = 6.0cm, SP = 7.5cm and PQ


1. Without using a calculator or tables, simplify =10.5cm
3
(b) Point T on QR produced such that PT  ST join
40.04 
1
2
 8(4 1 )(16 ) 4 Answer (4)
the point P, S and T. Measure length PT and
2. By writing each of the number in the form a  10 where
n
angle PTS
n is even, find the square root of 0.25 × 0.64.
Answer (0.4) (c) A circle passing through the points P, T and R.
Measure the radius of the circle.
3. Show that 18  50  72  2 2 Answer (9.3cm, 480, 6.8cm)
4. Given that f(x) = ax2+4x and f(3) = 21, find the value of 12. Kampala and Mbale are about 229km a part. A mini-
a. Hence find f(-1) Answer (a = 1, -3) bus heading for Kampala leaves Mbale at 8.55a.m with a
steady speed of 56kmh-1. At 9.40a.m, a saloon car traveling
A 12cm at 80kmh-1 leaves Kampala and travels steadily towards
5 B
300 Mbale. Using scales of 2cm to represent 20km and 2cm to
represent 1 hour, draw on the same axes distance- time
graphs showing the journeys of the mini-bus and the car.
O
Hence or otherwise determine when and at what distance
from Kampala the two vehicles will meet given that the
In the figure above, AB=12cm is a tangent to a circle, at A. mini-bus then increases its speed by 14kmh-1, calculate the
(i)Time when the mini-bus arrives in Kampala
Angle OBA=300. Find the
(ii) Difference in the time of arrival of the two vehicles.
(i) Length of OB. Answer (11:03am at a distance of 110km from Kampala)
(ii) Radius of the circle. Answer (13.86cm, 6.93) (12 : 37pm), (12 : 33pm) (4minutes)
 0 1 1 2  13
6. Matrices P    and Q    are used to
  1 1  0  1
map the point A(3, -2) onto A1. What are the co-
ordinates of A1 under the matrix transformation Q
followed by P. Answer (2, 3)
7. P and Q are two sets of objects such that n( )  12 ,
n( P  Q )  5 , n(Q)=8 and n( P  Q)1 = 3 Find
(a) n( P  Q )
(b) n(P) Answer (9, 6)

8. R is a point which is 13 units from the origin O. If its x- ABCDE is a right solid cone. CE = 10cm,
coordinate is 12, find the possible values of the y-
 = 300,
CD : DE = 2 : 3. The cone BCD was cut off.
coordinate. Answer (5, -5)
Calculate the:
9. Evaluate log a5 b2, without using tables or calculator, if (i) Total surface area of the remaining portion ABDE.
loga = 0.234, log b = 1.185 Answer (3.54, 17) (ii) Volume of the cone BCD.
12   4 Answer (198cm2, 14.516cm3)
10. Given that OA= a    OB= b   
~ 6 ~ 1 14. The points P(0, 2), Q(1, 4) and R(2, 2) are vertices of a
Calculate the length of AB. Answer (17) triangle PQR. The images of P, Q and R under a reflection
in the line x-y = 0 are P', Q' and R' respectively. The points
P', Q' and R' are then mapped onto the points P", Q" and R"
SECTION B respectively, under an enlargement with scale factor -2 and
11. Using a ruler and pair of compasses only construct. centre of enlargement O(0, 0).

(a) Write down the matrix for the


(i) Reflection
(ii) Enlargement

414
b) Determine the coordinates of the points Family allowances for four children only. For children in
(i) P', Q' and R' the age bracket 0 to 10 years sh. 12,500/- per child;
(ii) P", Q" and R" between 10 and 18 years sh. 8,250/- per child; and over 18
c) Find the matrix of a single transformation which would years sh.5,000 per child.
map triangle PQR onto P" Q" R" (a)Calculate the man’s taxable income and the income tax
0 1  2 0  he pays, given that he has three children, two of
  ,  whom are aged between 0 and 10, and the other
1 0  0  2 child 13 years.
P'(2, 0), Q'(4,1) and R'(2, 2) (b)What percentage of his gross income goes to tax?
P"(-4, 0), Q"(8, -2) and R"(-4, -4) Answer (166096.25, 22.15%)
 0 2 
 
 2 0 

15. The points P, Q and R are on level ground, a vertical


flag pole ST stands between P and Q such that Q is 12m
away from S, the base of the pole. The angles of elevation
of T from P and Q are 610 and 150 respectively. If angle
PQR = 22.60 and QR = 13cm, calculate the
(a) Height of the flag pole ST.
(b) Length PQ
(c) Angle of elevation of T from R.
Answer (3.215, 13.782, 32.6)

16.

The figure ABCDEFGH is a rectangular box with square


ends BCFG and ADEH of side 9cm, AB=12cm.
Calculate:
(a) Length AC and AG,
(b) The angle between the line AG and plane ABCD.
(c) The angle between the planes DEG and ADEH.
Answer (15cm, 30.96, 62)
17. The table shows the tax structure on taxable income of a
certain working class of people.

Income (sh) per month Tax rate (%)


0-30,000 10.0%
30,001-90,000 16.5%
90,001-190,000 23.5%
190,001-340,000 32.0%
340,001-500,000 40.0%
Above 500,000 49.5%
An employee earns sh. 750,000. His allowances include;
Marriage allowances one-fifteenth of his gross monthly
income
Water and electricity sh.15,000 per month
Relief and insurance sh. 180,000 per annum
Housing allowances sh. 40,000 per month
Medical sh. 300,000 per annum
Transport allowance sh. 36,000 per month

415
1999 PAPER TWO 10. Find the coordinates of points of intersection of the
curve y = x2 - 3 and the line y = 5x – 9. Answer (2, 1), (3, 6)
SECTION A
1. (a) Without using tables or calculator evaluate 0.25 ×
2195 – 1795 × 0.25 (2 marks)
SECTION B
(b) Factorize completely 27-3x2
11. Faces of 36 small wooden cubes are to be painted either
Answer (100, 3(3 + x) (3 - x)
green or black or white. Of these, 10 cubes have all their
2. Given that sin  = 0.500, find the two possible values of
faces painted green and 6 have all their faces painted black.
 , what would be the two values of  if sin  = -0.500 .
There are 5 cubes with green and white, 8 white and black
Answer (2100, 3300)
and 4 green and black. Cubes with all faces painted white
3. The price of a car in a show room is sh. 6.4million. A
are three more than those with all faces black.
7.5% cash discount is allowed when a customer pays
Given that G represents the set of cubes with at least a
cash for the car, while on hire purchase basis a
green face, W represents that of cubes with at least a white
customer, pays sh. 1.65 million per installment for four
and B represents the cubes with at least a black face.
months. Determine how much a customer saves by
paying cash for the car than purchasing it by hire
(a) Represent the above information in a Venn diagram,
purchase. Answer (0.68m)
showing the remaining information.
4. Under an enlargement of scale factor 3, the image of (1,
(b) Find the number of cubes with
3) is (4, 5). Find the coordinates of the centre of
(i)All the three different colour faces.
enlargement. Answer (-0.5, 2)
(ii)At least one of each of the three colour faces.
5. If g ( x)  2 x and f ( x)  x  3 ,find gf (x) . Hence (c) If the cube is picked at random, what is the probability
evaluate gf ( 2) Answer (2x + 6, 10) that it is black or white only?
Answer (3, 36, 5 )
6. The number of visitors accommodated per night by a 12
certain hotel was recorded over a period of a month. 12. The table below shows ages of 120 students entering
The figures are given in the table below. senior one.
Age: 12.5-12.9 13.0-13.4 13.5-13.9 14.0-14.4 14.5-
No. of visitors 0-<10 10-<20 20-<30 30-<40 40-<50
years 14.9
No. of nights 2 8 12 5 3 No. of 8 35 52 17 8
students
Using a scale of 1cm to represent 2 nights and 1cm to (a) State the (i) class with
represent 10 visitors, display the given data on a bar- (ii) Modal class
graph. Use your bar-graph to estimate the modal (b) Determine the mean and median age of the students.
number of visitors accommodated by the hotel. Ans. Answer (0.5, 13.5 – 13.9, 13.625, 13.61)
(24) 13. Three points A, B and C on the same horizontal level
are such that B is 150km from A on a bearing of 0600. The
7. Use the matrix method to solve the pair of simultaneous
bearing of C from A is 1250. The bearing of C from B is
equations.
1600.
x + 4y = 4
(a) By scale drawing, using 1cm to represent 25km. Find
9y – 5x = 9 Answer (x = 0, y = 1)
the distance of C from
8. A piece of land measures 33.6m by 16.5m. Find the area
(i) A
of the land, in cm2, on a map whose scale is 1:120.
(ii) B.
Answer (385cm2)
(b) An aeroplane flies from A on a bearing of 340 0 at
9. In a certain game a die is thrown once. When a 1 or 6
300kmh-1. After 40 minutes of flying, the pilot changes
appears that player wins and when a 3 or 4 appears the
course at point D and flies directly to C at the same speed.
player loses. Determine the probability that a player
Include in your diagram in (a) above the route of the
neither wins nor loses.
plane. Hence find the
1 (i) Time (in hours) the plane takes to travel from A to C.
Answer  
 3 (ii) Bearing of D from C.
Answer (252.5km, 232km, 2.1hrs, 3220)

416
centre, there by arriving 0.6 hours later than if he had
maintained the 7kmh-1 speed.
i. Calculate by how much he reduced his speed.
ii. For how long was he in the trading centre before
14. Three solids, a sphere, a right cone and a right cylinder
Otim joined him?
are of equal surface area and the radii of their circular Answer (Mukasa overtook Otim after 3hrs at a distance of
sections are also equal. Given that the volume of the sphere 21km, 2.1km/hr, 27minutes)
is 288  cm3, find the 17. A wildlife club in a certain school wishes to go for an
(i) Radius of the sphere. excursion to a national park. The club has hired a mini-bus
(ii) Height of the cylinder.
and a bus to take the students. Each trip for the bus is sh.
(iii) Length of the slant slide of the cone.
Hence calculate the volume of the: 50,000 and that of a mini-bus sh. 30,000. The bus has a
(iv) Cylinder capacity of 54 students and the mini-bus 18 students. The
(v) Cone. maximum number of students allowed to go for the
4 2 excursion is 216. The number of trips the bus makes do not
[Take  =3.142, volume of sphere =  r and volume have to exceed those made by the mini-bus. The club has
3 mobilized as much as sh. 30,000 for transportation of the
1 2
of cone =  r h. students. If x and y represent the number of trips made by
3 the bus and mini-bus respectively.
Surface area: sphere = 4  r2; Cone (curved surface area) =
 rl; where l is length of slant side]. (iii) Write down five inequalities representing the above
Answer (6, 6, 18, 678.672cm 3, 639.84) information.
(iv) Plot these inequalities on the same axes
(v) By shading the unwanted region, show the region
15. (a) Copy and complete the following table of values for satisfying all the above inequalities.
the curve y = x2 - 2x - 6 and (vi) List the possible number of trips each vehicle can
y = 4x-5 for values of x between x = -4 and x = 7. make.
(vii) State the greatest number of students who went for
x -4 -3 -2 -1 0 1 2 3 4 5 6 7 the excursion.
Answer (3x + y < 12) (5x + 3y < 30) (x < y) (x > 0)
x2 16 9 16 25 36 49
(y > 0) ; (1, 1), (2, 1), (2, 2), (3, 1), (3, 2),
-2x 8 - 4 2 - -2 - - - - -12 -14
(3, 3), (216)
x2-2x-6 18 - 2 - - - - -3 - - 18 29
4x - - -8 -4 0 - 8 - - - 24 28
y = 4x-5 -21 - -13 - -5 - - - - - 19 23

(b) On the same axes plot the graph of the curve


y =x2-2x-6 and the line y = 4x - 5 for -4≤ x ≤7
(c) Using your graph estimate the
(v) Coordinates of the points of intersection of the
curve and the line.
(vi) Roots of the equation y= x2 –2x – 6= 0
Answer (-0.1, -5.75, 6.1, 19.5) (x = -1.6, x = 3.7)

16. Otim and Mukasa wish to travel to the next trading


centre which is 30.8km away. They will travel by their
bicycles. When Otim had covered 9km, traveling steadily at
4kmh-1, Mukasa started riding at a steady speed of 7kmh -1
from where Otim started. Both Mukasa and Otim
maintained their cycling speeds until Mukasa overtook
Otim.
a. Find the time and distance at which Mukasa overtook
Otim.
b. Given that Mukasa then reduced his speed and
maintained the new speed till he arrived at the trading

417
2000 PAPER ONE SECTION B
11. Using a ruler, pencil and a pair of compasses only,
SECTION A Construct a triangle PQR, where angle QPR = 1350, PQ =
1. Without using tables or calculator evaluate 8.4cm and QR = 12.5 cm. state the length of PR.
30 .25 2  30 .15  30 .25 S and T are points such that TS bisects QR, where T
Answer (1210) is on QR and, S on the same side as PQ. Draw circle
0.0025
2. Given that x2 - y2 = 135, and x - y = 9. Find the values of x to circumscribe the points P, Q, R and S. Measure
and y. Answer (x = 12, y = 2) and state the
(i) Length ST
3. Given the sets: (ii) Radius of the circle.
A= {all natural numbers less than 30} Answer (PR = 5cm, ST = 2.5cm, radius = 9cm)
B= {all prime numbers between 10 and 30}
Find (i) n{AB1} 12. (a) Express 1.24 as a fraction in its simple form.
(ii) n{A1∩B}, where A1 and B1 are complements of (b) If S  kd(l  d ) express l in terms of d, k and s.
sets A and B, respectively. Answer (23, 0)
4. Solve the simultaneous equations 3 2
(c) Express in the form p  q r ,
4y - 3x = 2
3 2
2y + 1 = 2x Answer (x = 4, y = 3.5)
where p, q and r are constant.
Answer
5. A straight line passes through the origin and the point (1,
-1). Find the equation of the line  41 s 2  kd 2 
 , L  , p  5, q   2, r  6 
Answer (y + x = 0)  33 kd 
6. Shs. 6,000 is to be shared among David, Daniel and
13. The diagram below shows the allocation of the
Diana. Daniel is to get one and half times as much as
members of the board of governors of a school to different
David, while Diana is to get three and half times as
committees?
much as David. Determine the ratio in which the money
is to be shared. Answer (2 : 3 : 7)
n(production)=8
n(academic)=8
7. Given the points L(3, 4) and M(7, 7), Find the: 2
(i) Vector LM 2 x
(ii) Lengh of LM
 4
Answer  , 5 2 z
3 y
8. By removing the brackets factorize completely y(ay – x)
+ x(y – ax) 1 -
(iv) Determine the values of x, y and z.
Answer a(y + x) (y – x) (v) What is the total number of members on the board
9. In the figure below PQ is parallel to RS. Angle of governors?
SVW=550. UV is perpendicular to TW. Determine the (vi) What is the probability that a member chosen at
values of the angles labeled a and b. random from the members of Board of governors
belong to.
T U n(finance)=7
(iv) Both finance and production committee.
P Q
a b (v) Only one committee?

Answer (x = 2, y = 2, z = 2, 15, 4 1)
,
R S 15 3
V 55O
W 14. The data below represent the times in seconds of an
Answer (125, 145) oscillation of a given pendulum as recorded by
10. Find the value of x, correct to 2 decimal places, given different students.
that 3x = 5
Answer (1.465) 10.3 9.7 10.2 9.8 10.1
9.9 10.1 9.9 10.1 10.2

418
10.3 10.0 10.2 10.1 9.8 B
9.9 10.1 10.0 10.1 9.9
10.1 10.1 10.1 10.1 9.9
C D
9.8 9.8 10.0 9.9 10.2
(a) The frequency table below was drawn out to
represent the above data.
Copy and complete the table
Time(s) x Freq.(f) Cum. Freq x(f)
9.7 __ 1.0 __
9.8 4.0 5.0 __ A
9.9 __ __ __
10.0 3.0 __ __ Given that AB  CD = 40cm, calculate the:
10.1 __ __ __ a)
10.2 __ __ __ (i)Height of the vertex of the pyramid from the
10.3 __ __ __ square base.

 f  ___  fx =_
(ii) Angle between a triangular face and the base of
the pyramid
(b) Use the table to (iii) Volume of the pyramid.
(i) State the modal time of oscillation. b) If the pyramid is cut horizontally at a vertical height of
(ii) Calculate the mean and median times of oscillation. 2.6cm from the square base, and the upper part of the
Answer (10.1, 10.01, 9.8) pyramid containing the vertex is thrown away, what
15. The table below shows the speed (vms-1) of a car volume remains?
after time (t s) Answer (12.65cm, 64.620, 607.2cm3, 297.95cm3)
vms-1 0 15 28 42 48 35 0 17. (a) Customs duty and purchase tax are levied on certain
time(ts) imported goods as follows
(i) Plot a speed – time graph showing the motion of Customs duty = 35% of the value of the good.
the car. Purchase tax =15% of (value + duty)
(ii) Use the graph to estimate the: Find the total amount levied on an electric kettle valued
(c) Speed of the car at t = 6s. Shs. 40,000. Hence calculate the percentage rate for the two
(d) Times when the speed of the car is 32.5 ms-1. taxes combined.
(e) Distance traveled between time interval
t = 10s to t = 14s.
(b) A man borrowed Shs. 39.6 million from a housing
(iii) Describe the motion of the car between
finance company to build a commercial house at a
(iv) t = 10s and t = 14s
(v) t = 14s and t = 20s compound interest rate of 10.5% per annum. He has to
(d) By drawing a tangent to the curve at t = 17s, repay the loan and interest within two years in 8 equal
estimate the rate of speed at that instant. instalments.
Answer (35m/s, 5.5seconds, 17seconds, 192m, Calculate
between t = 10s and t = 14s the is travelling with a (i)Total amount of money the man paid the
uniform speed of 48m/s, between t = 14s and 4 = 20s company.
the car is retarding to rest at an acceleration of - (ii)Interest he paid on the loan.
7m/s2) (ii) Amount he paid per installment.
16. The diagram below shows a square of side 12 cm Answer (22100, 55.25%, 48.352590,
and four congruent isosceles triangles, representing the 8752590, 6044073.75)
net of a pyramid on a square base.

419
2000 PAPER TWO Answer (-6, 2)

SECTION A SECTION B
a 2  b 2  2ab 11. Draw the graph of the curve x  2 x  1 for
2
1. Given that a  b 
ab  3  x  3 . Use your graph to find the solutions of the
i) 4 * 3 following equations.
(i) x  2 x  1  0
2
ii)8 * (4 * 3) Answer (1, 7)
(ii) x  x  6  0
2. In the diagram below angle ABC=500. What is the 2

bearing of B from A. (04 marks) Answer (1, -2, 3)


N

A
12. The image of the vertices P(2,3); Q(2, 2) and R(4,2) of
a triangle PQR under a rotational transformation are P 1(-
N 1, 2), Q1(0,2) and R1(0,4) respectively. The image of
PQR, P'Q'R' then undergoes a further rotation of 52 0 to
give the image P"Q"R".
B 500 C
(i) Represent triangle PQR and its images on the same
coordinate axes. (Use a scale of 2cm to 1 unit).
(ii) Determine the centre and angle of rotation of PQR.
Answer (220) (iii) Find the coordinates of the final image P"Q"R".
1 x 5 State the angle formed between PQR and P"Q"R".
3. Solve the inequalities   . Answer (x < 18) Answer (1, 1, +90, PII(-1, 0), QII(-0.4, 0.8)
2 6 2
RII(-1.9, 2), (380)
4. In a triangle ABC, angle BAC =1500, AB = 5cm and 13. In triangle ABC L, M and N are the mid-points of
AC = 4cm, calculate the area of the triangle ABC. BC,CA and AB respectively. AM= m; AN = n and
Answer (5cm2) 3AG = 2AL.
8
5. Given that cos  for 00≤  0≤1800, (a) Express in terms of vectors m and n. the vector
17 (i) AB (ii) AC
Find without using tables or calculator, values of sin  (iii) BC (iv) BG
and tan  . Answer  15 , 15  (v) GM
 
 17 8  b) Show that B, G and M lie on a straight line and
6. Given that f ( x)  x 2  3 and g ( x)  x  1 , find the 3BG  2BM
value of a for which fg(a)  gf (a) Answer (1) 2 1
Answer (2n, 2m , 2m – 2n,  m  2n  ,  m  2n  )
1 3 3
7. Without using tables or calculator evaluate (0.008) 3

Answer (0.2) 14. The cumulative frequency graph below shows the
8. A pole is fixed on horizontal ground. The angle of marks scored by 150 students in an end of term
depression of the foot of the pole from the top of a cliff examination marked out of 80 marks. Study the graph
57.7m high is 300. Find how far away the foot of the and use it to:
cliff is from the pole. Answer (99.94m)
9. Two coins have each one side labeled H and the other T.
they are together tossed once.
Write down the possibility space. Find the probability
that the labels on the two coins are different.
Answer (0.5)
10. Given the matrix M   3a a  6  , find the values of

  6 a  2
a for which the determinant of M is zero.

420
16. On a shore running from east to west are two ports P
160 and Q which are 18km apart. A town R on an island on the
Number of
same level as P and Q is on a bearing of 230 0 from P and
candidates
1400 from Q respectively. A pilot flying a plane above port
140
P observes town R at an angle of depression of 60.
Calculate the
120 (i)Distances PR and QR
(ii)Vertical height of the plane above P.
100 (iii)Angle of elevation of the plane from port Q.
Answer (13.79, 11.57, 1.45, 4.60)

80 17. The table below shows the tax structure on taxable


income of employees of a certain industry.

60
Income (sh) per month Rate (%)
0 20 30 40 50 60 70 80 Marks 18,001-36,000 8.75
a) Estimate40the; 36,001-54,000 12.50
(i) Median mark 54,001-72,000 18.00
(ii) Number of students who obtained distinction if 60 72,001-108,000 24.50
20
marks and above are distinction scores. 108,001-180,000 30.00
(iii) Number of students who scored 30 or less marks. Above 180,000 40.50
(iv) Pass mark if 92 students passed the examination. An employee earning a gross income of sh. 425,000 a
b) Construct a table of the frequency distribution of the month is allowed the following.
students’ performance. Hence calculate the mean mark.
Answer (26, 12 students, 90 students, Allowance Amount (sh)
pass mark = 24) Transport and lunch 45,000 per month
15. The diagram below shows a hollow right cylinder Housing 80,000 per month
PQRSTU of negligible thickness, Part of which has been Water and electricity 2,100 per month
cut off as shown below.
Annual medical 900,000 per annum
If the radius of the circular end is 50cm, RS =150cm and Marriage 1 th
of gross monthly
20
PQ  TU =20cm, income
[NB. A month is taken to be 30 days and a year 360 days].
U The employee is allowed a family allowance for any three
of his children according to age distribution.
C Age Shs
S 0-12 6,000
13-18 4,500
C
19-21 2,500
Q 20cm P 150cm
Given that this employee has a family of five children with
the older child aged 22, the other 15 years and the rest aged
C C 50cmC C
between 2 and 12 years, calculate the employee’s.
Find: R C (i) Total monthly allowance
(i)The area of the cross-section PQR (ii) Taxable income
C
(ii)How much water (in litres) would fill this container (iii) Income tax
Determine the percentage of the employee’s income that
[use  =3.14].
goes to tax. Answer (241350, 38963.25, 9.16)
Answer (7836.73cm2, 1175.509 litres)

421
2001 PAPER ONE 12. (a) Find the point of intersection of the lines
y  2 x  3 and y = -x − 3. Calculate the area of triangle
SECTION A
2
1. Given that Q = I Rt , make I the subject. enclosed between the two lines and the x-axis.
Hence evaluate I for Q = 1000, t = 20, and R =2 (b) Find the Equation of the line which is a perpendicular
bisector of the line passing through points A(5, 4) and
 Q 
Answer  I  , 5,  5  B(3, 8).
 Rt  Answer (0, -3, 6.75sq.units) (y = 0.5x + 4)
13. Using a ruler, pencil and pair of compasses only,
2. In the figure below, AC  AD  BD , angle
DAC = 480. Find the size of angle x. Answer (33) construct triangle ABC such that AB = 8cm, angle ABC =
1 x  3   23  600 and BAC = 450.
3. Given that       , Find the values of x and y. Construct the perpendicular from C onto AB to meet it
1 1  y   7 
at D. Measure the length CD.
Answer (x = 5, and y = 4)
Draw a circle circumscribing triangle ABC. Measure its
4. A floor measuring 2.5m is to be covered by square tiles radius.
measuring 25cm each. Find the number of tiles that will Find the area of triangle ABC. Answer (CD = 5cm, 20cm2)
be needed to cover the floor. Answer (80 tiles)
5. A chord of length 6cm is 4cm from the centre of a circle. 14. The following table shows the marks obtained in a
Determine the circumference of the circle. mathematics test by S.5 students in a certain school.
Answer (31.43cm)
50 53 31 56 38
6. Without using tables or calculator, solve for x in the
33 39 51 38 41
1
equation 32 x   4 x 3  2 24 . Answer (3) 69 57 63 50 54
8
7. Find the discount on a bicycle priced Shs 64,000 but sold 40 41 45 48 64
off at a discount of 7 1 2 %. How much was paid for it? 59 61 55 36 52
Using class internal of 5 marks, make a frequency
Answer (59200)
distribution table, starting with the 30-34 class.
 4 1
8. In a parallelogram OBCA, OA    and OB    , Use your table in (i) above to estimate the mean mark. State
1  3 the modal class.
where O is the origin. Draw a histogram for the data. Use it to estimate the modal
Find (i) vector BC
mark. (12 marks)
(ii) The co-ordinates of C. Answer  4   5, 4 
  Answer (49, 50 - 54, 53 )
1  15. In a certain school there are 50 students who play three
9. Given that log10x = 2.852, and log10y = 2.581 . Use games, namely Chess, Tennis and Volleyball, 24 play
1
x 2 Chess, 26 play Tennis and 29 play Volleyball, 9 play
tables to evaluate
y both chess and volleyball, while 13 play both tennis
Connect to 3 significant figures. Answer (10162.5) and volleyball, 11 play both Chess and Tennis. Each of
these students play at least one of the three games.
10. A triangle PQR whose area is 12cm2 is mapped onto its (a) Represent the above information on a Venn diagram.
image A1B1C1 by a transformation represented by the
(b) Find;
2 1
matrix   . Find the area of A1B1C1 (4 marks) (i) How many students play all three games?
 3 4 (ii) The number of students who play only one game.
Answer (60) (iii) The probability that a student selected at random
SECTION B plays only tennis.
(iv) The probability that a student selected at random
x3 1  2x
11. (a)Given that functions f ( x)  , g ( x)  , plays only two of the games.
2 5
Answer (4),  3 , 21 
determine the value of x for which fg(x) + gf(x) = 0.  
 25 50 
(b) Express x2– x– 3
4 in the form (x + p)2 +q. Hence 16. A transformation represented by the matrix  6  4 
2 1
solve the equation x2– x– 3
4 = 0 Answer (3) maps the vertices of triangle KLM onto its image vertices
K1(8, 3), L1(32, 11) and M1 (2,2) respectively. The image

422
of triangle KLM further under goes transformation
2001 PAPER TWO
 2 0
represented by matrix   SECTION A
 0 2 1. Find the H.C.F of 18, 42 and 48. Answer (6)
Find (i) The coordinates of vertices K, L and M. 2. Without using tables or calculator evaluate
(ii) The coordinates of K", L" and M" 2
1 1
 8 3
(iii) A single matrix of the transformation which 16 2  64 3    Answer (4.41)
would map triangle K" L" M" back onto triangle KLM.  125 
Answer K(2, 1), L(6, 1) and M(3, 4) 3. A television set costs British pound sterling (₤) 220.
KII(16, 6), LII(64, 22), MII(4, 2) Given the exchange rates:
One United States dollar (1 US$) = ₤0.75, and
  14 1 1 U$ = USh.1, 800, determine the cost of the set in
 1 3 Uganda shillings. Answer (528000)
 2 2 4. Using a number line, find the integral values of x which
satisfy the sets
17. A whole saler wishes to transport 870 crates of soda {3x > 2x + 5} ∩ { 3x < 32 – x}
from a factory. He has a lorry which can carry 150 crates at Answer (6, 7)
a time and a pick-up truck which can carry 60 crates at a 5. In the table below, y is known to be inversely
time. The cost of each journey for the lorry is Shs. 25, 000 proportional to x.
and for the pick-up Shs. 20,000. The pick-up makes more y P 45 12
x 5 8 q
journey than the lorry because it travels faster. The amount
Find the values of p and q (4 marks)
of money available for transporting the soda is Shs. Answer (p = 72, q = 30)
220,000. 6. Olga bought a motor cycle and sold it to Okello at a loss
of 25%. If he sold it at Shs. 1, 200,000, find how much
(i)Write down five inequalities, representing the above money Olga paid for it.(4 marks)
information. Answer(1600000)
x 1 x 1
(ii)Plot a graph for the inequalities, shading out the 7. Solve the Equation 
2x  5 3
unwanted regions.
Answer (x = 2, x = -2)
(iii)How many journeys should the lorry and the pick-up 8. Express 0. 321as a fraction. Answer  107 
 
make so as to keep the transport cost as low as  333 
possible. State how much money the wholesaler saves 9. A cylindrical tank of diameter 1.4m and length 2m has a
by making these journeys. capacity of 3.08m3. Find the radius and height of a similar
tank of capacity 83.16m3.(4 marks)
Answer 2x + 5y > 29, 4x + 5y < 44, x > y, x > 0, y > 0 Answer (2.1m, 6m)
10. The table below shows the age of pupils in a certain
5 trips of pick up and 4 trips of a lorry should be
class.
made, 20000
Age (years) 11 12 8
No. of pupils a 10 a
If the mean age of the pupils is 10, find the value of a
Answer (a = 20)

SECTION B
11 (a) The length of a rectangular floor is 8 meters more
than its width. If the area of the floor is 65m2, find the
dimensions and perimeter of the floor. (6 marks)
(b) In the figure below ABCD is a rectangle. AB =10cm,
AD  AX = 6cm and xy is an arc of a circle, centre
D.

423
A X B
15. In a certain country, income tax is computed after
deducting the following allowances.
D Y C Type of allowances Amount UShs
Calculate the area of the shaded region. (Take  =3.14) Marriage 10,000
(6 marks) Single 4,000
Answer (13, 5, 36m, 46.29) Each child above 10 but below 20
12. Given the Equation of a curve y  2 x 2  5 x  3 years. 3,000
(i) Copy and complete the table below. Each child under 10 years 2,000
- Omoja is married with 3 children, two below 10 years of
x 4.0 -3.5 -3.0 -2.5 -2.0 -1.5 -1.0 -0.5 0.0 0.5 1.0 age and the other child 12 years old. Mbili is single but has
2x 2
8 0.5 two depedents aged 11 and 15 years. Each month Omoja
5x -10 2.5 and Mbili earn gross incomes of Shs. 130,000 and 120,0000
respectively. The income tax is calculated as follows.
-3 -3 -3 -3 -3 -3 -3 -3 -3 -3 -3 -3
y -5 Ush % age tax
st
1 :01-10,000 20%
(i) On the same axes and using the same scales plot the
Next:10,001-50,000 15%
graphs of y  x  1 and y  2 x 2  5 x  3 Rest:50,001 and above 10%
(ii) Using your graph solve the Equation x  2 x  2
2
(a) Calculate the
Answer (x = -2.75, 0.75) (i) Taxable income for Omoja and Mbili.
(ii) Income tax for Omoja and Mbili.
13. In the figure below OL = 4.5cm, PM =3cm NM =
b) Express the total income tax for each man as percentage
4cm and LN = 7.5cm. of their respective taxable incomes.
L Answer (113000, 110000, 12.65%, 12.73%)
16. A helicopter flies from Moroto due South for 300km. It
M then flies on a bearing of 2550 for 350km. From there is
flies on a bearing of 0200 for 400km.
O N (i) Draw an accurate diagram showing the journey of the
P
Find: helicopter using a scale of 1cm to represent 50km.
(i) lengths ON and OP (ii) From your diagram, find the distance and bearing of
(ii) The radius of the circle Moroto from the final position of helicopter.
(iii) Area of OLMP (iii) Given that the helicopter flies at a steady speed of
-1
Answer (6cm, 1cm, 2.3cm, 7.5cm ) 2 200kmh , find how long the whole journey took.
Answer(200km, 0860, 5.25hrs)
14. In the figure below, vector PQ = s, PR  r ,
~ ~ 17. The diagram below shows a right pyramid with a
2QT  3TR and PU : UQ  2 : 3 rectangular base ABCD AB =16cm, BC =12cm and
R each slant edge of length 26cm.
V
26cm
r T
~

s D C
P ~ Q
U s
~
O 12cm
(a) Find in terms of vectors r and s , vectors
~ ~ A B
16cm
(i) QR (ii) QT (iii) PT
Calculate the
(b) Show that UT is parallel to PR
(i) Height of OV above the base.
 3 2s  3r 
Answer  r  s, r  s, 
(ii) Angle between line VB and the base.
 5 5  (iii) Angle between the planes BCV and ABCD.
Answers (24, 67.4, 71.6)

424
2002 PAPER ONE SECTION B
SECTION A 11. Using a ruler, pencil and pair of compasses only,
(a) Construct a triangle ABC, where angle ABC=1050, BC
1. Without using tables or calculator evaluate
= 9.2cm and AC = 12cm. Measure and state length AB
(a) 3.1422 – 3.042 × 3.142 (2 marks)
and angle BAC.
(b) 1.21 × 10-2 × 40
(b) Draw a circle circumscribing triangle ABC. State the
2.2×11 (2 marks)
radius of the circle.
Answer (0.3142, 0.02)
2. Given that a2 – b2 = 63 and (a + b) = 21 find the values Answer AB = (5.9cm),  BAC=470, radius = 6.2cm)
12. The Venn diagram below shows representation of
of a and b .
members of a community council to three different
Answer (a = 12, b = 9)
committees of Finance (F), Production (P) and Security
3. Given that p= {the square of all prime numbers} and Q =
(S)
{the first ten square numbers}.
Write down the members of (P∩Q) (4 marks) n (P)=10 n(F)=10
Answer {4, 9, 25, 49}
2 a 3
4. Solve the pair of simultaneous Equations
2x + 3y = 8
2y – x = 3 Answer (x = 1, y = 2)
5. A straight line of gradient -1, passes through the point (3, c 3 b
-2)
(a) Determine the Equation of the line. 1
(b) Through which point does the line cut the y-axis?
Answer (y = -x + 1) (0, 1) Determine the values of a, b and c .
6. Three boys John, Michael and Tom share Shs 4,000. Find the:
Given that Tom gets six times as much as Michael and (i) total number of members that make up the
John gets half of what Tom gets, find how much money community council.
(ii) number of members who belong to more than one
each boy gets. Answer (400, 2400, 1200)
committee. n(S) =7
Given that a member is chosen at random from the council
7. Two points P(5, 2) and Q(2, 4) are in a plane. Find the members, what is the probability that the member belongs
(a) Coordinates of M, the mid-point of PQ to
(b) OM , where O is the origin (2 marks) Only one committee
Answer M(3.5, 3), (4.6units) Not more than two committees?
8. Factorise completely 16 x 3 y  2 y (4 marks) Answers (a = 3, b = 1, c = 2), (18, 9, 1 , 5 )
3 6
Answer 2y(4x2-2x + 1)(2x + 1) 13. (a) Express 1.252525 as a fraction.
9. In the diagram below, C and O are centers of two (b) Given that 2g – e = 3g(g – e), express g in terms of e in
intersecting circles. Angle CQS= 370 its simplest form.
4 3
T (c) Express in the form a  b c , where a , b
x 4 3
S R and c are constants.
C
370 O Answers
Q y  124   3e  2  9e2  4 
P   , g    ,  a  7, b  4, c  3
 99  6 
N  
Find the values of x and y Answer (53, 74) 14. The distance d(m) traveled by a car after time t(s) is
10. Without using tables or calculator, evaluate given by the following table.
d(m) 0.0 10.0 22.0
4 log 10 2  log 10 48  log 10 30 Answer (1) 29.0 31.5 36.0
t(s) 0 2 54 7 8 10

425
(a) Plot a distance – time graph to show the motion of the
car. (Use scales: 2.0cm to 1.0 sec and 2.0cm to 5.0m on
2002 PAPER TWO
the horizontal and vertical axes respectively) SECTION A
(b) Use your graph to estimate r 2  s2
(i) The distance the car had traveled after t = 4s. 1. Given that r * s  , find
10 s
(ii) At what time the car had covered 34.0m.
(a) 4*–8
(iii) The average speed of the car between
(b) 7*(4* −8) Answer (-1, -5)
t = 3s and t = 7s.
2. Express the bearing South South-East (SSE) in degrees.
Answer (18m, 9.1s, 3.75m/s)
Answer (157.5)
15. The marks obtained by a class of 40 pupils in an
1x x
English test are given below. 3. Solve the inequality  5  1  (4 marks)
50 71 40 48 61 70 30 62 4 2
44 63 60 51 55 25 32 65 Answer (x < 16)
54 62 65 50 45 40 25 45  3  2
4. Given that OA    , OB    , where OA and OB
48 45 30 38 30 28 24 48  5  0
30 48 28 35 50 48 50 60 are position vectors of A and B respectively, find the
area of the triangle OAB. Answer (3sq.units)
(a) Using a class internals of 5 marks, construct a frequency
5
table starting with the 20-24 class group. 5. Given that tan  = , without using tables or
(b)Represent this information on a histogram. Use the 12
histogram to estimate the model mark. calculator , find the value of cos  -sin  .
(c)Estimate the mean using a working mean of 47.
7 7 
Answer (48.5, 47.125) Answer  , 
16. The base of a right pyramid is a rectangle 10.0cm by  13 13 
3x  5 find the value of x such
8.0cm and the slant edges are each 13cm. 1
6. Given that f ( x) 
Calculate the; 2
(a)Total surface area of the pyramid. that f(x) = 10. Answer (5)
(b)Angle between two opposite slanting sides of the 4
7. Without using tables or calculator find 40.0081
pyramid whose base length is 10.0cm.
Answers (298.96cm2) (38.90) 8. K, L and M are three points on a circle of radius 8cm
17 (a) Customs duty and purchase tax levied on certain such that KLM is an equilateral triangle. What is the
imported commodities are calculated as follows; shortest distance of any side of the triangle from the
Customs duty: 30% of the value of the commodity. centre of the circle? Answer(4cm)
Purchase tax: 20% of (value + duty) 9. Two dice are thrown up at once. What is the probability
Calculate the total amount of money levied on a that the sum of the scores on the dice is less than 8?
Answer  7 
commodity valued at Shs 18,600. Hence determine the
percentage rate for the two taxes combined.  
 12 
(b) A company borrowed 14.85 million shillings to boost its
10. Given that matrices A  
1 3  1 2
business. The bank rate is 12% compound interest per , B   
annum. The company had to repay the loan and interest  4 11   1 3
within two years. It is to repay these bank dues in six Find (a) P = A.B
Equal installments. Calculate the  415  115 
Answer 
2 
–1
(i) Total amount the company paid to the bank. (b) P
7
(ii) Interest the company paid to the bank. 5 5 

(iii) Amount of money the company paid per installment. SECTION B


Answers (10416, 56%, 18.62784m, 3.77784m, 3.1046m) 11(a) Copy and complete the table below
x -4 -3 -2 -1 0 1 2 3 4
x2-2
-x2+6
b) Plot on the same axes the graphs of y = x2 –2 and

426
y = 6 – x2, for -4 ≤ x ≤ 4 Answer(37, 72 students, 20, 20 students, 36, 36.3)
Using your graphs solve the equation x2-2 = 6-x2 15. Three towns A, B and C lie on the same level ground.
Answer (2, -2) Town B is 15km away from Town C. the bearings of
2 towns B and C from A are 0600 and 1500 respectively.
12. In a triangle OPQ, x is a point such that OX  OP The bearing of C from B is 2000. To a pilot flying an
3 aircraft above A, the angle of depression of C is 7.5 0.
and Y the mid point of PQ . The point Z on OQ is such Calculate the:
that OQ  QZ . Given that OP= p and OQ = q . a) distances AB and AC .
~ ~ b) vertical height of the aircraft above A.
a) Determine in terms of p and q , the vector (i) OX (ii) c) Angle of elevation of the aircraft from B.
~ ~ Answer (11.49, 9.64, 1.26km, 6.260)
OY (iii) OZ (iv) XY (vi)YZ 16. A rectangular swimming pool is constructed such that
b) Hence or otherwise show that x, y and z lie on a straight when the pool is completely full, the shallow end is 1
metre deep and the deeper end is 4 metres deep. The
line. State the ratio of the lengths XY and YZ .
pool is 25 metres long from the shallow end to the deep
Answer  2 p  ,  p  q  ,  2q  ,  3q  p  ,  3q  p  end and 20 metres wide.
3   2   6   2  (a)Calculate the:
13. The end points of line AB whose coordinates are A (3, -1) (i)Inclination of the floor of the swimming pool to the
and B (4, -3) undergo a rotational transformation to give the horizontal.
image line A'B' with A'(1,3) and B'(3,4) respectively. (ii)Volume of the water (in m3) that can fill the pool.
a. Plot the line AB and its image on the same set of axes (b)Starting with the pool empty, a tap which delivers water
using a scale of 2cm to 1 unit. at a rate of 400 litres per minute is used to fill the pool.
b. Determine the centre and angle of rotation of the line. How long (to the nearest hour) will the pool take to fill?
c. Find the new image A"B" of AB when its image A'B' Answer (6.840, 1250m3, 52hrs)
further undergoes a rotation of 1280. State the size of
the angle formed between AB and A"B". 17. In a certain school a teacher’s salary includes the
Answer (Centre 0, 0), (+90), (1450) following tax-free allowances.
14. The cumulative frequency graph below shows marks Type of allowances Amount
scored by 180 students in an end of year examination Legally married teacher Shs. 10,000/-
marked out of 80 marks. Study the graph and use it to. Each child under 10 years Shs. 2,500/-
180 Each child above 10 years Shs. 2,000/-
PTA Shs. 50,000/-
Head of department/ subject Shs. 10,000/-
160
Class teacher Shs. 5,000/-
House master/mistress Shs. 5,000/-
140
Unmarried teacher Shs. 6,000/-
Number of candidates

Mr. Mugisha and Ofuti are senior teachers in this school.


Mr. Mugisha is married with two children under 10 years
120 and one child above 10 years. He is also a class teacher and
head of mathematics department. Mr. Ofuti is single but has
two children under 10 years and is also a house master and
100 a class teacher. Their gross incomes at the end of the month
are each subjected to a “PAYEE” (Pay As You Earn) which
has the following rates.
80 For the fist Shs. 10,000 taxable income the tax is 20%
while the rest is taxed at 15%. At the end of the month Mr.
Mugisha’s gross income was Shs 150,000 and Ofuti’s gross
60 income Shs 130,000
10 20 30 40 50 60 70 Marks
Calculate the:
a) Estimate the
a) Taxable income for each teacher.
(i) Median
40 mark.
b) Tax paid by each teacher.
(ii) Number of students who scored 30 or less marks.
c) Tax paid as a percentage of the gross income for each
(iii) Number of students who obtained distinction if 61 or
teacher.
more
20 marks were distinction scores.
Answer
(iv) Pass mark of the examination if 94 students passed.
(68000, 59000, 10700, 9350, 7.13%, 7.19%)
b) Construct a table of the frequency distribution of the
students’ performance. Hence calculate the mean mark

427
B F 4cm C
2003 PAPER ONE 3cm
( x  2)cm G E
SECTION A
2  33
2 A (2x+3) cm D
1. Simplify Answer (6)
2 4  36  18 Given that EC = 3cm and FC = 4cm, AB = (x +2) cm and
63  28 AD = (2x + 3)cm.
2. Simplify as far as possible (a) Find:
175  63
(i) The area of triangle ECF.
Answer (2.5)
(ii) An expression for the area of shaded region
3. The diagonals of a rhombus are 20cm and 48cm
ABFGED in terms of x.
respectively. Determine the length of the side of the
(b) If the shaded area is 43cm2, show that 2x2 + 7x – 49 = 0.
rhombus. Answer (26cm)
Hence find the length of AD
4. Factorize 8a  18 b completely (4 marks)
2 2
Answer (6m2, 2x2 + 7x – 6), (x = 3.5, AD = 10cm)
Answer = 2(2a + 3b)(2a – 3b) 12. Using a pair of compasses and a ruler only,
780 0.25
5. Use logarithm tables to evaluate correct to 2 Construct triangle PQR, such that angle PQR = 600, QR =
1.09
decimal places. Answer(179) 9.0cm, PR = 8.5cm. Measure the length QP ,
 2 0  4 8 Bisect the sides PQ and PR . Produce the line bisectors to
A , B   
6. If  0 2  2 6  ,Find det(A.B) intersect at point M.
7. Find the equation of the line passing through the points Using M as the centre, draw a circle to circumscribe
(2, -1) and (5, 8) Answer (32) triangle PQR. Measure the radius of the circle. Hence
8. Given that f(x) = ax – 7and f(8) = 17, find value of calculate the area of the circle. (Correct to 2 significant
(i) a figure) (12 marks)
(ii) f(4) Answer (y = 3x – 7) Answer (radius = 4.8cm, A = 72.5cm)
9. The pie-chart below shows the fruits popularly sold in a 13. A retail trader ordered for shirts from a Kampala
daily super market in Kampala. wholesale shop as follows;
Size
pineapples olour Small Medium Large Extra large
1200 Apples Blue 0 40 20 0
540 Green 30 0 25 0
960
Yellow 0 20 0 10
Bananas
Mangoes
Given below is the cost for each size of shirt.
If 420 apples were sold on a given day, Determine,
(i) The total number of fruits that were sold that day. Size
(ii) How many mangoes were sold that day. Small Median Large Extra large
Answer (2800, 700) Cost (UShs) 3000 3600 4200 4800
10. There are enough chicken feeds to feed 360 chicken for
21 days. Find how many more chicken would be (a) Write down a
needed for the same feeds to last 15 days. (i) 4 × 3 matrix for the order of the shirts made.
Answer (144) (ii) 4 × 1 cost matrix.
(b) Given that the trader had to pay a tax of 17% of the cost

SECTION B
of shirts purchased, find his expenditure on the order.
(12 marks)
11. The figure below shows a rectangular piece of paper
ABCD which has been folded along EF such that C maps
onto G.

428
0 30 0   3000  (iii) Differences in the times of arrival of the two
    vehicles
 40 0 20 
,  3600   635310  Answer (324km, 11:15am, 12:45pm, (12 : 15, 30minutes)
 20 25 0   4200  17. A farmer plans to plant an 18hectare field with carrots
    and potatoes. The farmer’s estimates for the project are
0 0 10   4800  shown in the table below.
14. The table below shows the marks scored by 90 students Carrots Potatoes
in a test marked out of 50 marks. Harvesting cost
Marks Frequency(f) Shs 95, 000 Shs 60,000
per hectare
15-19 1 Number of
20-24 13 12 days 4days
working hours
25-29 29 Expected profit
30-34 25 Shs 228,000 Shs 157,000
per hectare
35-39 19 The farmer has only Shs 1,140,000 to invest in the project.
40-44 3 The total number of working days is 120.
By letting x represent the number of hectares to be planted,
Represent the above on a histogram. Use your histogram to with carrots and y the number of hectares to be planted with
estimate the mode. potatoes,
Calculate the mean mark of the test using a working mean (a) Write down inequalities for;
of 27. Answer (28.5, 30.167) (i) Cost of the project
15. A school has a teaching staff of 22 teachers. 8 of them (ii) Working days
teach Mathematics, 7 teach Physics and 4 teach Chemistry. (iii) Number of hectares used in the project
(iv) The possibility that the field will at least be used for
Three teach both Mathematics and Physics and one teaches
planting either carrots or potatoes
Mathematics and Chemistry. No teacher teaches all the (b) Write down an expression for the profit, P, in terms of x
three subjects. The number of teachers who teach Physics and y
and chemistry is Equal to that of those who teach chemistry (c) (i) on the same axes plot graphs of the inequalities in (a)
but not Physics. and (b) above, shading out the unwanted regions
(ii) Use your graph to determine how the farmer should use
(a) Represent the above information on a Venn diagram the field to maximize profit. Hence find the farmers
(b) Find the number of teachers who teach maximum profit.
(i) Mathematics only. Answer (19x + 12y < 228, 3x + y < 20, x + y < 18, x > 0, y
(ii) Physics only. > 0, p = 228000x + 157000y, the farmer should plant
(iii) None of the three subjects. should plant 2 hectares of carrots and 16 hectares of
(c) Find the probability that a teacher picked at random potatoes.
teaches only one or none of these subjects.
8
Answer (4, 2, 9, )
11
16. The distance between two towns A and B is 432 km. A
lorry traveling at a steady speed of 72kmh-1 leaves town
A at 6:45 a.m. for town B. one and a half hours later, a
minibus leaves town A at a steady nonstop speed of
108kmhr-1 heading for town B.
(a) On the same axes, show the journeys of the two vehicles
(use scales of 2cm to represent 40km and 2cm to
represent 1hour)
(b) Use your graph to estimate the:
(i) Time and distance from town A when the mini-bus
overtakes the lorry.
(ii)Times when the two vehicles arrive in town A

429
(i) Find the sum of the areas of triangles BCP and CDQ
2003 PAPER TWO in terms of a.
(ii) Given that the area of triangle PQC is 40.5cm2, find
SECTION A the value of a
(iii) Express the area of triangle PCQ as a ratio of the area
of the rectangle ABCD.
1. Without using tables or calculator evaluate Answer (4a2 – 1.5a)(a = 6, 9 : 32)
1 1 12. On the same coordinate axes, draw the curve y = 4x2 for
1 2
 1 3
    -2 ≤ x ≤ 2 and the line y=1, show by shading the unwanted
 16   64  region, the region represented by
Answer (16) (i) y > 1
2. Without using tables or calculator, evaluate (ii) y < 4 - x2
7.46 2  2.54 2 Answer (49.2) Hence state the integral coordinates of the points which lie
in the region {y >1  y < 4-x2}
Answer (-1, 2) (0, 2) (0, 3) (1, 2)
3. Given that a*b= a 2  (ab) , find (1*4)*3
13. In a sports field, four points A, B, C and D are such that
Answer (12) B is due south of A and due west of D, AB = 10.8m, BD =
4. Use logarithm tables to find the square root of 0.0576. 18.8m, CD = 16.6m.  BAD = 600,  CDB = 400 and
Answer 0.24  BCD=800. A vertical pole created at D has a height of
4.8m.
5. Given that y is directly proportional to x3 and that y is
250 when x = 10, find the equation connecting x and y. (a)(i) Draw a sketch of the relative positions of the points
hence find the value of y when x = 4. on the sports field.
Answer (4y = x3, 16) (ii) Using a scale of 1cm to represent 2m, draw an accurate
6. Given that A = x: -2 ≤ x ≤ 1 and B = {x: 0 < x ≤ 5} diagram to show the relative positions of the points and the
represent A  B on a number line. State A  B. pole.
Answer {1} (b) Find the:
7. A student wrote (p + q)2 as p2 + q2. Find the percentage (i) Distances BC and AD
error the student made in evaluating (p + q)2 when p = 7 (ii) Bearing of B from C.
and q = 3. (4 marks) (iii) Angle of elevation of the top of the pole from B.
Answer (42%) (c) If an athlete runs from point A through points B, C and
8. A lake of area 120km2 is represented by the area of D and back to A in 16 seconds, find the athlete’s average
4.8cm2 on a map. Find the length (in km) of a horizontal speed.
road measuring 6cm on the map. Answer (30km) Answer (12.2m) (21.6m) (3300), (14.50) (61.20) (3.825m/s)
9. Mary is five years younger than John and Peter is twice 14. In a triangle ABC, M and D are midpoints of AC and
as old as Mary. The sum of their ages is 49. Find Peter’s CN respectively. N is appoint on AB such that AN = 3NB
age. Answer (22yrs) If AB = p and AC = q, express the following vectors in
10. A box contains green, blue and red balls. The terms of vectors p and q
1 (i) AM
probability of picking a green ball from the box is (ii) AN
5
(ii) ND
1
and that of a blue ball . What is the probability of (b) Show that MD is parallel to AB and that MD: AB = 3 :
2 8
 3 Answer  q,  3 p  4q  
1 3 1
picking a red ball from the box? Answer   p,
 10  2 4 8 
SECTION B 15. Town B can be reached using two different routes.
11. In the diagram below ABCD is a rectangle in which BC Using a shorter route, it takes the driver 2hrs 26min. the
= 4acm and CD = a cm. P and Q are points on AB and AD driver covered the first xkm at an average speed of 54km-1
respectively. Such that AP = AQ = 3cm. and then covered the remaining ykm at an average speed of
37.5km-1. using the second route, which is 5km longer than
A 3cm Q the shorter route, it takes the driver 2hrs 12min at an
D average speed of 60km-1
3cm (i) Show that the time taken using the shorter route is given
P a by the equation 25 x  36 y  3285
(ii) Form an equation in terms of x and y that represents the
time taken using the second route.
B 4a C

430
(iii) Find how long it would take a driver traveling at a
steady speed of 65kmh-1 to move from town A to town
B by the shorter route. 2004 PAPER ONE
Answer (x = 117, y = 10, 1hr and 57minutes)
16. The diagram below shows a cube ABCDEFGH of sides
SECTION A
8cm and EM = MF. A tetrahedron AMHE is cut off the 33  92  125 1 / 3
cube. 1. Simplify Answer (3 x 5)
93
H G
2
M 2. If =a b , find the values of a and b.
E F 3 2
Answer (a = -2), b = (6)
D C
3. ABCD is a quadrilateral in which angles ABC and CDA
A are 900 each. If AB = 6cm. AC = 10cm and CD= 5cm.
8cm B
Find
Find: (a) Length BC
(i)Area of triangle HAM (b) Angle ACD Answer (8 and 600)
(ii)Angle between HAM and the plane AEHD
(iii)Volume of the remaining part of the cube after the 4. Factorize x 3  9xy 2 completely (4 marks)
tetrahedron has been cut off Answer x(x + 3y) (x – 3y)
Answer (39.19, 35.26, 469.33cm3)  2 2 1 1 
17. The monthly income tax system of a country is given as 5. Given that A    and B    Find:
below.  1 3 1 2 
Basic pay (U Shs.) Tax (%) (a) The matrix P such that AB = P
1st 0-150,000 Free (b) P-1
Next:15,1000-250,000 10.0 4 6  74 3
2 
Answer   ,  
1 
Next:251,000-350,000 12.5
Next:351,000-450,000 16.0 4 7  1
Next:451,000-550,000 22.5 6. Use the fact that log 10 2  0.301 and x = 4, to find the
Next:551,000-650,000 30.5
value of log10x2. Answer (1.204)
An allowance in excess of Shs. 80,000 is subjected to a tax x2
7. Given that f ( x )   5 , find the value of x for which
of 25% of the monthly allowance. Two employees A and B 3
are such that A earns a basic monthly pay of Shs. 355,000 f(x( = 17 Answer (-6, 6)
and a top up allowance of Shs. 185,000 per month while B 8. Find the Equation of the line passing through the points
earns only a basic ,monthly pay of Shs 540,000. (-1, 3) and (4, 2) (04 marks)
(i) Who of the two employees pay more monthly? Income
x 14
tax than the other and by how much? Answer y 
(ii) Express employee A’s Income tax as a percentage of his 5 5
monthly earnings. 9. A food store has enough food to feed 200 students for 15
Answer (B pays more income tax than A by 8200, 9.18. days. For how long will the food last if 50 students join
the group? Answer (12 days)
10. The pie-chart below represents the number of students
who attend various courses in a commercial college.

TOURISM
500 MARKETING
400
CATERING
1500

If the number of students studying Accountancy is 120.


(a) Determine the student population of the college.
(b) Find the number of students who study marketing.
Answer = (288, 96)
ACCOUNTANCY

431
SECTION B (b) If three points are awarded for a win, two points for a
draw and no point for a loss, use matrix multiplication
11. (a) The diagram below shows a rectangle ABCD of to determine the winner of the tournament.
length 44cm and width 15cm. (c) Given that Shs. 475,000 is to be shared by the three
A B teams according to the ratio of their points scored in the
15cm tournament. Find how much money each team will get.
D C 1 1 3   2 2 1  3 3 4 
44cm Answers  3 0 2  ,  4 1 0  ,  7 1 2 
If it was curved in such a way that AD and BC come      
2 2 1 3 1 1  5 3 2
together to form a hollow cylindrical figure, find the      
volume of the cylindrical figure formed. Team Y was the winner with 23 points
(b) A rectangular piece of cardboard measuring 27cm long X gets 375,000/=
and 15cm wide rests against a vertical wall as shown Y gets 575,000/=
in the diagram below. Z gets 525,000/=
C 14. The distance from town A to town B is 360 km. An
express bus leaves town A at 6:30a.m and travels at a
steady speed of 80kmh-1 towards town B. At the same
B time, a taxi Omnibus leaves town B traveling non-stop
towards town A at a steady speed of 100kmh-1. On the
same axes draw a distance time graph for the journeys of
the two vehicles. Use a scale of 2cm to represent 1 hour
27cm and 2cm to represent 50kmh-1.
D From the graph;
15cm
Y (i) Find the difference in the time of arrival of the bus and
250 X the taxi.
A (ii) Determine when and at what distance from town A the
If angle DAY = 250, find the length of C above the ground two vehicles will meet.
Answer (11:00am, 10:06am, 54 minutes, 8 : 30am)
Answer (2310cm3, 30.81cm)
(160km)
12. Using a pair of compasses and ruler only
15. A packet has 60 different vitamin tablets. Each tablet
Construct triangle ABC such that
contains at least one of the vitamins A, B and C. Twelve of
BC =10.6cm and angles ACB = 750 and ABC = 600,
the tablets contain only vitamin A. Seven contain vitamin B
Construct a circumcircle of triangle ABC with O as its
only and eleven contain only vitamin C. Six contain all the
centre.
three vitamins.
(c) Measure lengths AB and AC and the radius of the circle.
Given that n(A'∩B∩C) = n(B'∩A∩C) = n(C'∩A∩B), find
Answer (AB = 11.4, AC = 13cm, radius = 7.5cm)
the:
(i) Number of tablets that contain vitamin A.
13. Three secondary school football teams X, Y and Z
(ii) Probability that a tablet picked at random from the
qualified for a football tournament which was played in packet contain vitamin C.
two rounds with other teams. In the first round: (iii) Probability that a tablet picked at random from the
Team X won one game, drew one and lost three games. packet contains both vitamins A and B.
Team Y won three and lost two games. Answer (34, 11 , 7 )
Team Z won two, drew and lost one game in the second 20 30
round. 16. The table below shows the weight (in kg) of 40 students
Team X won two, drew two and lost one game. of a class and their corresponding cumulative frequencies.
Team Y won four and drew one game while team Z won Weight(kg) cumulative Freq.
three games, drew one and lost one. 30-34 2
(a) Write down 35-39 7
(i) a 3×3 matrix to show the performance of the three 40-44 12
teams in each of the two rounds. 45-49 21
(ii) A matrix which shows the overall performance of the 50-54 28
teams in the two rounds. (4 marks)
55-59 34

432
60-64 38
65-69 40
2004 PAPER TWO
(a) Draw a cumulative frequency curve, use your graph to SECTION A
estimate the .
1. Express 784 as a product of prime factors. Hence find
Median weight of the students. the square root of 784. Answer (28)
25th and 75th percentile weights.
2. If the exchange rate of a Kenya shilling to Uganda
(b) Calculate the mean weight of the students. shilling is 1K.sh = 24 Ush, and an American dollar to
Answer (49, 42.5, 49.25) Uganda shilling is $1 = Ush 1,950. How many American
17. A private car park is designed in such a way that it can dollars would one get in exchange for Ksh 9,750?
accommodate x pick-ups and y min-buses at any given Answer ($120)
time. Each pick-up is allowed 15m2 of space and each min-
bus 25m2 of space. There is only 400m2 of space available 3. In the diagram below, BC is a tangent to the circle with
for parking. Not more than 35 vehicles are allowed in the centre O and angle BAO=300.(4 marks)
park at a time. Both types of vehicles are allowed in the
park. But at most 10 min-buses are allowed at a time. x0 B
(a) (i) Write down all the inequalities to represent the above
information.
A 300 y0
(ii) On the same axes plot graphs to represent the O C
inequalities in (i) above, shading out the unwanted
region.
(b) If the parking charges for a pick-up is Shs 500 and that
Find the size of the angles x and y.
for a min-bus is Shs 800 per day, find how many
vehicles of each type should be parked in order to Answer (x = 60, y = 30)
obtain maximum income. Hence find the maximum 4. Given that the representative fraction of a map is
parking income per day. (3 marks) 1
, find the length of a horizontal road on the
250 ,000
Answer (3x + 5y < 80, x + y < 35, y < 10, x > 0) map whose length on the ground is 66.25km long.
(25 pick-ups and 1 minibus should be parked)
(13300) Answer (26.5)

 2 b
5. The transformation described by the matrix  

 c 3
maps the point P(-1, 3) onto its image p1(10,8).Find the
values of b and c Answer (b = 4, c = 1)

6. In the figure below, DE = 8.0cm, BC =12.0cm and


BD = 5.0cm.
A

D 8.0cm E
5cm
B 12.0cm C
V
Given that DE is parallel to BC , find length AD .
Answer (10cm)
7. Solve the equation 3x2 +10x=8 Answer (x = -4, x  2 )
3
8. Given that 133n = 43ten, find the value of n.

433
Answer (n = 5) 25% less his original speed and arrives in trading centre Q
9. A fair die with faces marked 1,2,3 …., 6 and a fair coin fifteen minutes earlier than cyclist C2.
with one side showing a court of arms (C) and the other a) Determine the speeds of cyclists C1 and C2.
side a fish (F) are tossed together once. Construct a b) If cyclist C2 started from trading centre Q at the same
possibility space showing all the possible out comes. Find time as cyclist C1 started from trading centre P, both of
the probability that a six and a fifth will show up. them traveling non-stop on the way, find how far from Q
the two cyclists would meet. After how long would they
1
Answer   meet?
 12  Answer (10.58km/hr, 20.58km/hr, 15.85km, 36 minutes)
10. The angle of depression of the sun’s rays to a man’s 14. (a) Plot the graph of y = 3x2 + 2x − 16 for values x: − 3
head is 140. If the man whose height is 1.7m, is standing ≤x≤3
up right on horizontal ground. Find the length of his b) Use your graph to solve the equation
shadow, correct to 2 significant figures. 3x2 + 2x – 8 = 0
Answer (6.82) 3x2 + 2x – 16 = -8
Answer (x = -2, x = 1.4)
SECTION B 15. The bearing of tower A from point O is 060 0 and that of
11 (a) At the beginning of the year, a customer deposited sh tower B from O, 2000 OA =24km and OB =33km. tower
1,900,000 in a bank which offers a compound interest rate C is exactly half way between towers A and B.
of 2.75% per four months. Find how much interest he a) Using a scale of 1cm to represent 5km, draw an accurate
earned at the end of the year. diagram showing the positions of the towers.
(b) A cooking oil factory offers a trade discount of 2% to its b) Use your diagram to find:
customer. It also offers a 1% cash discount to any customer
i. Distances AB and OC .
who pays cash for the oil bought. If the factory price for a ii. The bearing of tower B from tower A.
20 litre jerrican of cooking oil is sh 30,000, find the amount iii. The bearing of tower C from O.
of money a customer saves by paying for 100 jerricans of c) Find:
the oil. Answer (161100, 289400) i. The average speed of a cyclist who takes 2 14 hours
to travel directly from A to B.
12. The coordinates of the vertices of a triangle OAB are ii. How long it takes another cyclist to travel from A
O(0, 0), A(1, 0) and B(1, 1) and B via O at a steady speed of 4.5kmh-1 faster than
a) Find the co-ordinates of the image formed when: that of the cyclist in (c) (i) above.
Answer (53.5km, 11km, 2130, 1500, 23.78km/hr, 2hrs)
 3
(i) Triangle OAB undergoes a translation of   to form
 2 16. The diagram below shows a quadrilateral OSRQ. OS =
O1A1B1. q , OP = p and SX = k(SP)
 2 0 
~ ~

(ii) OAB is transformed by the matrix   to form q


 0  3
~
O S
p
O11A11 B11. ~
P X
b) (i) plot triangle OAB and its images on the same graph
(ii) Use your graph to find the area of O11A11 B11
Answer OI(3, 2), AI(4, 2) BI(4, 3) Q R
OII(-6, -6), AII(-8, -6) BII(-8, -9) (i) Express vectors SP and OX in terms of p , q and k.
3 sq.units ~ ~

13. Two cyclists C1 and C2 start at the same time from (ii) If OQ = 3 p and QR = 2OS, and OX=lOR, find the
trading centre P travelling to trading centre Q which are 

24km a part. Cyclist C1 starts at a steady speed of 10kmh-1 values of k and l. hence find the ratio SX: XP.
greater than that of cyclist C2 who also travels at a steady 
Answers  p  q, kp  1  k  q ,
1 3 
, , 3: 2 
speed. When C1 has covered half the distance, he delays for  5 5 
three quarters of an hour, after which he travels at a speed

434
17. In the diagram below VABCD is a pyramid with a
rectangular base ABCD and V, the vertex, O is the centre
2005 PAPER ONE
of the base ABCD. SECTON A
AB = 8m, BC = 6m, VC  VB  VA  VD =13m. M is 1. Write down the next term of each of the given sequences.
(i) 2, 3, 1, 4, 0,…
a point on VO such that 3 MV  OV . M is also the
(ii) 1, 4, 20, 120,….
centre of base EFGH of a small pyramid VEFGH similar
Answer (5, 840)
to VABCD which is to be cut off from the original
2. Without using tables or calculator find the value of
pyramid VABCD.
i. Cos7800
V ii. Sin3900
Answer (0.5) (0.5)
13m
30  35
3. Without using tables or calculator simplify
H G
6 7
E M F
Answer ( 2 5 )
4. At lunch time a certain hotel received 80 customers. Of
these 45 had a posho (P) meal and 50 had matooke (M).
(i) Represent this information in a Venn diagram.
D (ii) Find the number of people who had a meal of both P
C
and M.
Answer (15)
6m 5. If the point (2, -1) undergoes a translation represented by
O
 11 
the matrix   , find the image of P. (4 Marks)
  4
A B
8m Answer (13, -5)
Find the (i) Dimensions of the base EFGH. 6. Calculate the simple interest on Shs 96,000 for 10
(ii) Height of pyramid VABCD. months at rate of 8 13 % per annum. Answer (6666.67)
(iii) Volume of the remaining part of pyramid VABCD
7. Using mathematical tables evaluate
when VEFGH is cut off.
Answers (2.67, 2, 12, 184.89m3) (0.48) 53 correct to 2 dp. Answer (0.64)
8. A stretch of land on a map of scale 1:15,000 has an area
of 300cm2. Determine the actual area of the land in km2.
Answer (6.75km2)
9. A floor measuring 6m × 4m is to be covered with square
tiles measuring 50cm each. Find the cost of covering the
floor, if the price of a dozen of tiles is Shs. 15,000.
Answer (120000)
10. Show that the points (3x, -2y), (2x, y) and (0, 7y) lie on
a straight line.
SECTION B
11. (a) Express x2 + x – 12 in the form (x + a)2 + b. Hence
solve the Equation x2 + x – 12 = 0
x3
b) Given that functions f(x) = 2 , and
1 2 x
g(x) = 5 , determine the values x for which

9  24 x  8 x 2
fg ( x) 
10

435
 1 49 1 7  (a) Using class width of 3 marks and starting with the class
a  , b   , x  , x   of 48 – 50, make a frequency distribution table.
Answer  2 4 4 2 b) Use your table to
12. (a) Use matrix method to solve the following pair of (i) draw a histogram.
simultaneous equations. (ii) determine the median and mean mark.
x y 3 Answer (55.5, 55.8)
3x  2 y  1 16. (a) Okello bought 3 pens and 2 rulers from a bookshop
at shs. 3,150. Mukasa bought 2 pens and 3 rulers from
b) A transformation maps (1, 2) onto (-1, 4), and (2, 3) onto
the same bookshop at sh. 2,850.
(-1, 7). Find the matrix of this transformation. Hence
(i) Find the cost of each pen and a ruler.
determine the image of (3, 0) under the transformation.
(ii) If Mugisha spends sh. 6,000 to buy n pens and n rulers.
1  1  Find the value of n.
Answer (x = 1, y = 2)   (3, 6)
2 1 (b) A pick up van can be bought by cash at Shs. 8,750,000 or
13. Using a ruler, pencil and pair of compasses only, can be bought on hire purchase by paying 25% deposit of
cash price and 12 monthly installment of sh.600,000 per
Construct a triangle ABC such that AB =8.7cm, AC = month.
10.6cm and angle BAC=600. Calculate:
Inscribe a circle on the triangle ABC, (i) Cost of pick up by hire purchase.
Construct a perpendicular from B onto AC to meet it at (ii) Extra money paid for the pick-up by hire purchase than
point D. by cash.
Answer (450, 750, 5, 93, 87, 500, 63, 7500)
Measure length BC and the radius of the cycle. 17. A transport company has 8 lorries of 8-tonnes carrying
Measure BD and calculate the area of triangle ABC. capacity each, and 5 lorries of 10-tonnes capacity each.
Answer BC = 9.8cm, BD = 7.5cm, radius = 2.7cm There are 12 drivers available. The company was
Area =39.93cm contracted to transport 480 tonnes of cement from the
14. The figure below shows a hollow pipe of external diameter factory to a town on a given day. The 8-tonne lorries can
16mm, internal diameter 10mm and length 50cm. make 6 journeys in a day and the 10 tonne lorries 4
journeys a day. The costs of using an 8-tonne lorry and a
10-tonne lorry are sh. 40,000 and sh. 60,000 respectively.
(i) Write down the inequalities to represent the above
information
(ii) Plot a graph for the inequalities, shading out the
unwanted regions
(iii) From your graph, find the number of 10-tonne and 8-
tonne lorries the company used, keeping its costs as
D d 50c minimal as possible
Answer (x < 8, y < 5, x + y < 2, 6x + 5y > 60) (6, 5)
m (7, 4) (8, 3)

(i) Calculate the surface area (in cm2) of the pipe correct to
2 dp. (use  =3.142)
(ii) What would be the surface area of a similar pipe of
length 150cm, external diameter 48mm and internal
diameter 30mm. Answer (253.81cm2, 2284.30cm2)
15. The table below shows the marks obtained in a
chemistry test by 54 students in a certain school.
54 49 60 58 54
60 51 57 56 54
53 59 56 52 55
57 62 54 54 56
48 51 52 55 58
65 55 54 57 61

436
2005 PAPER TWO 9. A fair coin with one side showing court of arms (A) and
other side showing a cow (C) is tossed twice. Find the
SECTION A probability that at least a cow (C) will show up in the
1. Find the highest common factor (HCF) of 18, 45 and 42. two tosses. Answer(0.75)
Answer(3) 10. The angle of elevation of the top of a flag pole to a
2. When thirty times a number is increased by 32, the result policeman of height 1.7m is 200. If the policeman is
is equal to twice the square of the number. Find the standing at a distance of 16m from the pole on level
number. ground, find the approximate height of the flag pole,
Answer (-1, 16) correct to 2 significant figures. Answer (7.52)
3. If the exchange rate for a French Franc to a pound sterling
is £1 = 9.00 Francs and £1 =$1.53 (American dollars), find
how many American dollars one would get in exchange
SECTION B
for 1,000 Francs. 11. Mr. Lwanga and Mr. Okot were each given Uganda
shillings 980,000 at the beginning of 1999. Mr. Lwanga
Answer ($170) exchanged his money to United States dollars and then
4. In the diagram below, O is the centre of the circle. AB banked it on his foreign currency account at a
compound interest rate of 2% per annum, while Mr.
and CB are tangents to the circle. Angle ABC=540. Okot banked his money without exchanging it, at a
(04 marks) compound interest rate of 12% per annum. The
C exchange rates in 1999 and 2000 were Ug. Sh 1,250 and
Ug. Sh1,500 to a dollar respectively. If Mr. Okot
withdrew sh. 120,000 at the end of 2000.
D O 540 B
(i) Calculate the amount of money (in Ug.sh) each man
had in the bank at the end of 2000.
(ii) Who had more money and by how much?
A Answer (Lwanga had 1223510.4sh, Okot had 1109312,
Find angle ADC Answer (630) Lwanga had more money in the bank by 114198.4)
12. Two cyclists C1 and C2 begin travelling at the same
1
5. The representative fraction of a map is . Find time from town A to town B, 18 km a part. C 1 travels at a
250,000 steady speed of 15 kmh-1 faster than that of cyclist C2 who
the area of a lake (in km2) which is represented on the map also travels at a steady speed. When C1 has covered half the
by an area of 4.6cm2. Answer (28.75km2) distance, he delays for half an hour, after which he travels
at a speed 20% less his original speed. He arrives in town B
15 minutes earlier than cyclist C2.
6. If 135n = 75ten, find the value of n (i) Determine the speeds of the two cyclists C1 and C2.
Answer (7) (ii) If cyclist C2 started from town B while C1 at the same
7. Use matrix method to solve the pair of simultaneous time started from town A and all the two travel non-
equations. stop, determine the distance from town A where the
2x – y = 8 two cyclists will meet. After how long will they
meet?
4x – 3y = 14 Answer (x= 5, y = 2)
Answer (12km/hr, 27km/hr, 12.46km, 28 minutes)
8. In the figure ABC, AB = 6cm, = 2cm, CD = 13. Using suitable scales, plot on the same axes the graphs
of y = 2x2 and y = 5 x  5 for -2≤ x ≤ 3. Use your graphs
4cm and CE = 5 cm. 2
C to estimate the solutions of the equations.
5cm 4cm (i) 4x2 − 5x − 10 = 0
(ii) 6x2 + 10x −30 = 0n Correct to 2dp
E D
2cm Answer (x = -1.05, 2.4, x = 1.5)
14. Town B is 100km away from town A on bearing of 135 0.
Town D is on a bearing of 0900 from town B, 124km a part.
B Town C 160km away from town D is on bearing 030 0 from
A 6cm D.
Using a scale of 1cm to represent 20km, make an accurate
If is parallel to AB , find length AE drawing to show the relative positions and distances of
Answer (10) towns A, B, C and D.

437
Determine the: Answer
a. Shortest distance and bearing of town C from A.
 a  xb  
1 1  3 3   1 1
a xb  ,  a  yb  ya  ,  1  y  a  yb    ,  
b. Distance and bearing of town B from town C. 2 2  4 4   3  2
Answer (290km, 2360, 250km) 17. The figure below shows part of a solid right circular
15. a) (i) Find the images of the points A(1, 4), B(1,1) and cone whose original height was 20cm before part of its top
C(2, 1) of a triangle ABC under a transformation L
was cut off. The radius of the base is 12cm and that of the
whose matrix is  1 0  top is 8cm. A circular hole of radius 8cm was drilled
 0 1
through the centre of the solid as shown.
1 1 1
(ii) Plot triangle ABC and its image A B C on the same
graph. Describe the matrix transformation L. hence deduce 8cm
the matrix transformation which would map triangle
A1 B 1C 1 onto triangle ABC.
1 1 1 11 11 11
b) Triangle A B C is mapped onto triangle A B C
by matrix transformation M =  1 0 
 0 1
(i) Find the coordinates of A11 B 11C 11 .
11 11 11
(ii) Plot A B C on the same graph in (a) (ii) above.
Use your graph to describe a single transformation that
11 11 11
will map triangle ABC onto triangle A B C . Hence 12cm
find the single matrix transformation which maps Calculate the volume of the remaining solid [Use  =
11 11 11
triangle ABC onto A B C . 3.142]. Answer (782cm3)
(ii) The single transformation that maps ABC onto
A11 B 11C 11 is half turn about the origin (0, 0). Single
equivalent matrix.
M . L
 1 0   1 0    1 0 
     
 0  1 0 1   0  1
Answer AI(-1, 4), BI(-1, 1), CI(-2, 1)
AII(-1, 4), BII(-1, -1), CII(-2, 1)
 1 0 
 
 0  1
16. In a triangle OAB, OA = a , OB = b . A point L is on
~ ~
the side AB and M on the side OB. OL and AM meet
3
at S. AS = SM and OS = OL.
4
Given that OM = xOB and AL = yAB. Express the
vectors,
(i) AM and OS in terms of a, b and x.
(ii) OL and OS in terms of a , b and y. hence find x
~ ~
and y.
B

a M L
S
~
~
O A
b
~

438
2006 PAPER ONE (i) Construct a grouped frequency table having class
intervals of 10 marks, beginning with the 15-24 class
SECTION A group.
3 1
(ii) Use your grouped frequency table to calculate the
1. Simplify
12 2  168 Answer (8)
mean mark of the mock examination.
27  18 (b) Represent the above mock results on a histogram
1 1
6 2
and use it to estimate the mode.
2. Solve for x in the equation (x + 2)(x – 4)<x2 – 6 Answer (48.5, 58.8)
Answer (x > -1)
12. (a) (i) Plot on a graph the triangle ABC whose
1 3 vertices are (1, 1), (3, 2) and (2, 4) respectively.
3. Express in the form a+ b 3 . Hence evaluate
2 3 (ii) On the same graph, enlarge the triangle ABC
1 3 using (-1, -1) as the centre of enlargement and a scale
correct to 3 significant figures if 3 = 1.732. factor of 2 to obtain its image A1 B1 C1 .
2 3 (iii) State the coordinates of A1 B1 C1 the image of
Answer  3  1, 0.732  triangle ABC.
(b) Using your graph, find the area of the triangle
4. A trader made a 35% profit after selling a goat at sh ABC. Hence, determine the area of the triangle
45,900. How much profit did the trader get?
A1 B1 C1 . (12 marks)
Answer (11900)
Answer A1(3, 3), B1(7, 5) C1(5, 9), (2.5sq.units)
5. Simplify Log75+2Log2-log3 Answer (2)
(5sq.units)
6. Find the values of a and b such that 13. Using a pair of compasses and ruler only;
 3 b   7a   43  15 122  a) (i) Construct triangle ABC, such that
       , 
 4 a  2   30 Answer  13 13  AB  10.0cm , BC  9.2cm ABC  105 0 .
(i) Measure length AC
7
7. A line of gradient passing through the point Q (3, 4), (b)(i) construct an inscribed circle of triangle ABC
9 with center O
cuts the y-axis at a point P. Find the coordinates of P. (ii) Measure the radius of the circle (12 marks)
 5 (AC = 15cm Radius = 2.5cm)
Answer  0,  14. A poultry farm has three units; A, B and C. unit A
 3 produces 30 trays of eggs and 20 broilers every
8. The height of a small box is 2cm and its volume 10cm3. month. Unit B produces 40 trays of eggs and 15
If the height of a similar box is 6cm, what is its volume? broilers and unit C, 35 trays of eggs and 10 broilers
Answer(270cm3) during the same period. If a tray of eggs costs Shs
9. The points: A, B, and C and D are on the circumference 3,000 and a broiler Shs 4,000.
of a circle of centre O and  ADC =300. Find the
a) (i) Represent the above information in matrix form
values of the marked angles a and b. (4 marks)
of order 3 × 2 for the eggs and broilers.
Answer (1500) (ii) Form a 2 × 1 cost matrix produced on the farm for
15  x 2 the eggs and broilers.
10. Solve the equation x = (4 marks)
2 (iii) Find the sales of the farm if all eggs and broilers
Answer (-5, 3)
were sold.
b) If the farm charges a 17% VAT, find the total income
from the sales of the farm every month.
SECTION B  eggs

broilers 

a) (i) 
11. Shown below are marks obtained by 50 candidates A 30 20 
in a certain S.4 mathematics mock examination. B 40 15 
 
25 30 29 60 72 59 40 40 62 70 C  35 10 
40 39 62 65 40 59 39 43 80 21
58 29 19 25 30 32 56 59 40 55
69 90 81 50 31 45 60 20 51 49
31 30 56 58 50 50 50 60 40 70

439
Answer B 21m C
 30 20 
   3000 
 40 15 ,  ,  495, 000  ,  579,150  10m
6m
 35 10   4000 
 
A B
15. (a) An FM radio commercial section charges fees
for any radio announcements as follows; the first ten
words Shs 5,000 and any additional word Shs 100 The figure ABCD shows a plot of land in form of a
each. Find the total charge for the announcement trapezium. Lengths BC  6m, CD  21m and
below DA  10m.
“Mr. John Musoke, the chairman organizing (a) Find the
committee of the wedding preparatory meetings of
(i) Length AB of the plot
Mr. James Lima and Miss Vanessa Tukko
(ii) Area of the plot. (6 marks)
announces the cancellation of the wedding meetings
which were scheduled to begin on Wednesday, 11th (b) The diagram below shows road AO intersecting
August, 2004 at Kalori gardens, national theatre, road OB at 900 at point O. the two roads are also
Kampala. Any inconveniences caused are highly connected to A and B by an arc-like shaped road
regretted. A new date and venue for the meetings measuring a quarter of a circle 70m in radius.
will be announced later”.
A
b) Mr. Ronald Anguyo bought a car at Shs 4,500,000.
The car depreciates at a rate of 12% per annum. After
2 years, Ronald decided to sell the car to his friend at 70cm 6
m
25% less of the value of the car then. Find the price at
which his friend bought the car. O B
Answer (55100, 2613600) Find the distance saved by a motorist who goes
16. At a graduation party, the guests are to be served through the arc-shaped road instead of going through
with beer and soda. At least twice as many crates of AO and OB.
beer as crates of soda are needed. A crate of beer Answer (50m, 150m2, 30m)
contains 25 bottles and a crate of soda contains 24
bottles. More than 200 bottles of beer and soda are
needed. A maximum of Shs 500,000 may be spent on
beer and soda. Assume a crate of beer costs Shs
40,000 and that of the soda costs Shs 15,000.
a) (i) Form inequalities to represent the above
information.
(ii) Represent the above inequalities on the same
axes.
(iii) By shading the unwanted region, represent the
region satisfying the inequalities in (a) (i)
above.
b) From your graph, find the number of crates of beer
& soda that should be bought if the cost is to be as
low as possible. Find the amount that was paid for
these crates of beer and soda.
Answer (y > 2x) , (24x + 25y > 200) (3x + 8y <100),
(x > 0), (y > 0), 6 crates of beer, 2 crates of soda)
Amount paid = 270000)
17.

440
2006 PAPER TWO 100 metres away from the first point, the angle of
elevation is 350. Find the two expressions for the height of
SECTION A the tower, hence find the height of the tower and give your
answer to the nearest metre.
1. Simplify (i) 1 14  2 12  1 34
b) If cosx = -0.634 for 900 < x < 2700, find the two possible
(ii) 2 12  3 13  1 65 Answer (2, 5) values of x. (5 marks)
Answer (221, 129.350, 230.650)
x2 4
2. Solve for x in  Answer (2) 13. A helicopter left Kampala at 06:00 hours and flew on a
2 x bearing of 0900, at a velocity of 300 kmh-1. It landed at
3. An examination is marked out of 130 marks. If Rita Nairobi Airport at 08 30 hours. At exactly 09 00 hours,
obtained 60% in the examination, how many marks did it left Nairobi Airport and flew on a bearing of 340 0 at
she get out of 130? Answer (78) the same original velocity. It then landed at Kitgum
4. Given that p{x, y}:2x − 3y≤6} and Q={x, y}:x + y < 0} Airstrip at 12 00 hours. Using graphical construction
Show by shading the unwanted region, the region and a scale of 1cm: 100km, find the
representing P  Q . (4 marks) a) Distance of Kitgum from Kampala.
22.60 b) Bearing of Kampala from Kitgum.
5. Use logarithm tables to evaluate correct
47.80  0.329 Answer (950km, 2080)
to 2 decimal places. Answer (1.44) 14. Two cars A and B start off from rest at the same time
6. Evaluate 5600  80,000, leaving your answer in the form moving in the same direction on a straight road. The speeds
a × 10n where 1 ≤ a < 10 and n is an integer of the two cars in ms-1 are shown in the table below.
Answer (7.0 × 102) t(s) 0 2 4 6 8 10 12
7. In a home work marked out of 20, a group of pupils Speed of car
0 4.5 9.0 13.5 18.0 22.5 27.0
obtained the following marks:- 15, 20, 18, 17, 8, 18, 16, 20, A(ms-1)
18, 17, 12 and 19. Find the mode and median marks. Speed of car
0 2.0 5.0 10.5 23.0 27.0 28.5
Answer (17.5) B(ms-1)
8. Under an enlargement of scale factor 3, the image of the
Using a suitable scale, draw on the same axes the velocity-
point P(0, 3) is P1(4, 5). Find the coordinates of the
time graphs of cars A and B. From your graph find the
centre of enlargement. Answer (-2, 2)

a) Time taken when the two cars have equal speed and the
9. Express 0.3 8 as a fraction in its simplest form. magnitude of that speed.
Answer  7  b) Different in speed after a period of 9 seconds.
  c) Distance covered by car A by way of estimating the
 18 
10. A fair die is tossed only once and the number which area under the curve described by car A for the 12
appears on its top face noted. What is the probability of seconds.
a top face showing? Answer (The two vehicles have equal speeds at 6.8secs
(i) A number greater than 4? and the magnitude of the speed is 15m/s)(5.5m/s)(162m 2)
(ii) An odd number or prime number? 15. OAB is a triangle, OA= a , and OB = b . Points C and
~ ~
Answer  1 , 2 
  E are points on lines OA and AB such that they divide the
3 3
lines OA and AB in the ratios 1:2 and 3:1 respectively.
SECTION B Point D lies on OE such that OD  2DE .
11. Draw graphs y = 2x2 + 3x – 3 and y – 7x + 3 = 0 for -3
≤ x ≤ 3 using a scale of 1cm: 2 units for the vertical axis A
a
and 1cm: 0.5 units for the horizontal axis. Using your graph ~

find the C
(a) Point of intersection of the line and the curve.
E
(b) Gradient of the curve between the points of intersection O D
of the line and the curve. Answer (0, -3), (3, 11), (7) b
~
12(a) At a certain point on the level ground the angle of B
elevation of the top of a tower T is 280. At another point

441
a) Find the vectors AB and CB in terms of vectors a and
~

b.
~

b) Show that the points B, D and C lie on a straight line.


Answer
 a  3b   3b  a   1 
b  a  ,  ,   ,  CD  CD 
 4   3   2 
16. A man earns a gross annual income of Shs 10,500,000.
He is entitled to the following monthly allowances.
Shs 15,000 for each child aged 12
Children and below.
Shs 12,000 for each child between The figure above (in thick, heavy lines) shows a lampshed
age 15 and 18 inclusive. ABCD bounded by circles of radii 15cm and 25cm. The
slanting side AB is 30cm. If the lampshed was cut from an
Lunch Shs 60,000
original figure OABCD, of a complete cone, calculate the;
Transport Shs 110,000
a) (i) Slanting length of the cone OAB.
Medical 1th (ii)The angle formed by producing CD and BA to O.
of gross monthly income.
10 b) (i) Vertical height of the lampshed.
(ii) Volume of the lampshed. (12 marks)
1th
Marriage of gross monthly income. Answer (75cm, 38.940, 28.28cm, 36263.86cm)
25

1th
Housing of gross annual income.
100
The man is married with five children of whom two are
aged 12 and below, the other two aged 21 and 24 and the
other aged 17. The following tax structure is applicable on
the taxable income in excess of Shs 30,000.
Taxable income (Shs) Rate (% ages)
00001–30,000 Free
30,001–130,000 8.0
130,001–260,000 14.5
260,001–380,000 23.0
380,001– 490,000 28.5
490,001– 590,000 35.0
590,001 and above 42.5
(NB: A month has 30 days and a year 360 days)
Calculate:
a) The man’s (i) Total monthly allowance
(ii) Monthly taxable income
(iii) Monthly income tax
b) The percentage of his gross annual income that goes to
tax. Answer (439500, 435500) (70267.5), (8.03%)
17.

442
2007 PAPER ONE
SECTION A If AD=12, find the area of the shaded region.
  Answer (21cm2)
1. Express2. 3 6 as improper fraction in its simplest form.
 26  SECTION B
 
Answer  11 
1 x
11 (a) Given that   1, solve for x.
3x  4 x  1
2. If a  14, b  8 and a  a  1  b, find the value of (b) Solve the simultaneous equations
c 2c
 41  x2  4y2  4
 
C. Answer  16  y  x 1
3. A line is given by the equation 45  15x  3 y  0
Answer
Find the co-ordinates of its x – intercept.
 8 3
 x  2.5, x  0 when y   1, x  when y  
Answer (3)
4. Given that f ( x )  2 x  4 and g ( x )  x  5 , find  5 5
fg (x). Hence evaluate fg (4).
12. Using a pencil, a ruler and a pair of compasses only,
Answer (2x + 14, 22) construct a triangle ABC in which AB = 9.2cm,
2
Expand the expression: a1 
ax  angle CAB= 450 and angle ABC=750
5.  (a) Measure the length of BC
 2
(b) Draw a circumscribing circle through the points A, B
 a x  3 2
and C.
a  a x 
2
 (c) Measure the radius of the circle.
Answer  4 
Answer (BC = 7.5cm, radius = 5.4cm)
6. A butcher sells 5Kg of meat at she 10,000. If the cost 13.(a) In the figure below, vectors OA = a and OC =
of meat is increased by 20%, find the weight of meat 1
which can be bought at shs3,600. c, CD  OA and AB  3OC.
3
Answer (1.5kg)
7. The data given below represents the ages in years of 30 B
Senior four students of a certain school:
Ages 15-17 18-20 21-23 24-26
Number of
7 11 9 3
students
Use the table above to draw a histogram and state the C D
modal class. Answer (18 – 20) c
8. Triangle ABC with vertices A(0, 0), B(1, 0) and C(1,1)
underwent two transformations represented by T 2T1. If
A
 2 (i) O By expressing
a vectors in terms of a and c, Find
T1 is a translation represented by   and T2 is a
OD, AB and OB.
3
reflection in the x-axis, find the co-ordinates of the
(ii) Show that points O, D and B are collinear.
final image of the triangle.
(b) Points A and B have co-ordinates (0,-1) and (-6, &)
Answer A11 (2, -3), B11(3, -3), C11(3, 4) respectively.
9. Given A  1 2  and B    1  2  , evaluate Find:
3 4   0 1
    (i) AB
Answer 
0
 A  B2 0 (ii) the magnitude of AB
 Answer
15 25 
 a  3c 
10. Study the diagram below  OD  3 , AB  3c and OB  a  3c 
C  
B   6  
 3OD  OB ,  8  , 10 
   
14. In a certain school, a simple of 100 students was
6 cm 8 cm
picked randomly. In this sample, it was found out that

A
D
E 443
78 students play Netball (N) 82, play Volleyball (V),
53 play tennis (T) and 2 do not play any of the three 2007 PAPER 2
games. All those that play tennis also play Volleyball.
48 play all the three games.
SECTION A
 , evaluate 1 * 2 Answer  
(a) Represent the given information on a Venn diagram. a b 25
1. If a * b 
(b) How many students play both Netball and Volleyball b a 2 3  12 
but not Tennis?
2. Make C the subject from the expression:

 b c 
(c) If a student is picked at random from the sample, what 2
is the probability that the student plays two games a b 2 2

only? Answer ( 14, 19, 0.19)


15 (a) Draw a table showing the values of Sin Answer  c  a  2ab 
2

2for0    90 , using values of  at


0 0

3. The point R(10,7) is reflected in the line y = x to given


intervals of 150
(b) Use the table in (a) above, a horizontal scale of 2cm point S. Given that M is the mid-point of RS. Find the
for 150 and a vertical scale of 2cm for 0.5units to co-ordinates of M.
draw a graph of sin 2  . Answer (8.5, 8.5)
(c) From the graph, find the values of Ө for which sin 2 4. Find the area of a triangle whose sides are 13cm, 24cm
Ө = 0.6 Answer (18.4, 71.6) and 13cm. Answer (60cm2)
16. A manager of an industry earns a gross salary of 5. Given the sets:
Shs2,000,000 per month, which includes an A = {All natural numbers less than 30}
allowance of Shs500,000 tax free. The rest of her
B = {All prime numbers between 10 and 30}
income is subjected to an income tax which is
calculated as follows: Find (a) n(A∩B')
7.5% on the first Shs 800,000 (b) n(A'∩B)
12.5% on the next Shs 500,000 Where B1 stands for the complement of the set B.
20% on the next Shs 100,000 Answer (24, 0)
30% on the next Shs 60,000
6. If 
2 4  3 1  8 6 
35% on the remainder.
 3   k  ,
(a) Find her taxable income.  3  0 n    3  1
(b) Calculate her monthly income tax. find the values of k and n. Answer 2,  2
(c) Express her monthly income tax as a percentage of
her monthly gross salary. 7. Use the prime factor method to find the cube root of
Answer (1, 500,000, 174,500, 8.725) 3375. Answer (15)
17. In the figure below, ABCDH is a right pyramid on a 8. In a Revenue Authority Department, the tax earned
square base ABCD of side 5m. Each of the slanting income is calculated as follows:
edges is 5m. The first shs120,000 is tax free and the remaining
A income is taxed at 25%. Find the tax payable on an
earned income of:
(a) Shs100,000
5 cm (b) Shs440,000 Answer (0, 80,000)
9. Given that V is inversely proportional to t2 and V =
5 cm
25 when t = 2 find V when t = 5. Answer (4)
D 10. The figure below shows a net of a cone which can be
folded to form a right circular cone.
C

A 5 cm

5 cm
Calculate the: 2100
B to two decimal places.
(a) height of the pyramid, correct
12 cm

(b) angle between the plane HBC and the base.


(c) Volume of the pyramid, correct to one decimal place.
Answer (3.55m, 54.80, 29.58m3)

444
Calculate the radius of the cone formed. 15. (a) Musa is a businessman who deals in an agricultural
Answer (7) produce business. Visited four markets in a certain
SECTION B week:
In market A he bought 3 bags of beans, 5 bags of
11. (a) Given that 212n = 25nine, find the base that n maize, 10 bags of potatoes and 3 bags of millet,
represents. In Market B, he bought 1 bag of beans, 4 bags of
(b) A positive integer r is such that Pr2 = 168, where p is potatoes and 2 bags of millet,
such that 3  p  5 . Find the integral values of r. In market C he bought 5 bags of beans, 1 bag of
Answer (3) maize,
12.(a) Find the length marked x in the diagram x in the In market D he bought 4 bags of beans, 3 bags of
diagram below correct to two significant figures. maize, 6 bags of potatoes and 1 bag of millet. He
A bought eash bag of beans at Shs 45,000, a bag of
x maize at Shs30,000, a bag of potatoes at shs 15,000
and a bag of millet at Shs 50,000. He later sold all
D the produce he had bought at shs 50,000 per bag of
35o beans, Shs35,000 per bag of maize, shs18,000 per
420 bag of potatoes and shs55,000 per bag of millet.
B
C 12 cm (a) Form a 4 × 4 matrix to show the produce Musa
(b) A dog tied a silk rope 4.5m long is tethered to a tree bought from the four markets.
stamp 2.5m from a straight path. For what distance (b)(i) Form a cost matrix for the price of the produce,
along the path is one in danger of being bitten by the (ii) By matrix multiplication, find the amount of money
dog? Answer (10.8, 6.19, 7.48) spent on the produce in each market.
(c) Find also the amount of money he got from the sale
13. By shading the unwanted regions, show the region of the produce.
which satisfies the inequalities. (d) Find Musa’s profit.
x+y3  3 5 10 2   45, 000   585, 000 
y>x–4      
1 0 4 2  ,  30, 000  ,  205, 000  ,
y + 7x  - 4 5 1 0 0  15, 000   255, 000 
Find the area of the wanted region.      
4 3 6 1   50, 000   410, 000 
Answer (17.5) Answer

14. The table below show the weight in Kilograms of 28 1, 655, 000/   ,  200, 000/  
children sampled in a Primary School: 16. Town A is 170Km from town B. A Tata lorry left
Weight(kg) Number of children town B for town A at 8:25am and travelled at a
15-19 2 steady speed of 40kmh-1. A saloon car left town A
20-24 4 for town B at 8:55am. And travelled at a steady
25-29 7 speed of 80kmh-1.
30-34 3 (a) Calculate the:
35-39 5 (i) distance from town A to the point at which the two
40-44 6 vehicles met.
45-49 1 (ii) time at which the two vehicles met.
(b) Just as they met, the Tata lorry driver increased the
(a) State the modal class
speed by 10kmh-1. Find the difference in their times
(b) Calculate the cumulative frequency and
of arrival at their destinations.
(i) hence, estimate the median weight correct to one
decimal place, Answer (100km, 10:00am, 1hr and 7.5mins)
(ii) Calculate the mean weight of the children
17. The figure QRSTUV below, is a plan of Mr. Rukidi’s
(iii) find the probability that a child selected at
farm. The area marked A is in form of an equilateral
random from the school weighs 40kg and above.
triangle, area B is
Answer (31.2, 31.8, 7, 0.25)

445
rectangular and C is a semi-circle.
2008 PAPER ONE
RQ  14m and RS  100m
Q SECTION A
1. Find the Lowest Common Multiple (LCM) and the
Highest Common Factor (HCF) of 54 and 84.
Answer (756,6)
R V 2. The pie-chart represents yields of beans from three
fields A, B and C.

100 m B 120
0
150
0

S U If the total yield of beans was 300 sacks, calculate the


number of sacks got from field C. Answer (75)
T 3. Express 2 log3 18  log3 31  log3 6 2  1 as a
Find the:
single logarithm log3 Q . Answer (log3 9)
(a) length QT which divides the farm into two equal
3 1
parts, 4. Given that P =   , find a matrix P -1 such that
(b) area of the farm  1 3 
(c) length of barbed wire required to fence Rukidi’s PP -1 = I where I is the identity matrix of order 2.
farm. Answer (119.12m, 1561.84m 2, 250m)
 0.3  0.1 
Answer  
 0.1 0.3 
5. Study the graph below:
y
6

4
-6 -4 -2 0 2 4 6 8
x

Find the inequality representing the shaded region.

Answer (y < 4x – 4)

-2
446
 16  4
3
 27  SECTION B
6. Evaluate     11. (a) The points (-1, q) and (r, 2) lie on the line y = 2 – x.
 81  Answer  8  Find the values of q and r.
(b) In the figure below, P is 4 units from O. the equation of
7. Solve for w:
1
w  6  1 2w  5  1 1  w the line MN is 4y + x = 12.
5 15 3
Answer (-3) y
4y + x = 12
8. Given that f(x) = 2x – 5, find N
(a) f(-2) M
(b) f -1(x)
x5
Answer (-9, )
2
9. In the triangle BCD, AD=15cm, BD = 25cm, AB = AC O 4 units x
P
and AB is perpendicular to CD.
B Find the area of OPMN.
Answer (3, 0) 10sq.units
12 (a) Adikini bought a television set for which the cash
price was shs:599,000. she bought the television set on
a hire purchase scheme and had to pay an extra
25cm shs:71,000. if she made eight equal monthly
installments, how much did she pay per month?
(04 marks)
(b) Mukasa wants to buy a house which is priced at shs:
56,000,000. a deposit of 25% of the value of the house
is required. A bank will lend him the rest of the money
at a compound interest of 15% per annum and payable
C D after two years.
A 15cm Calculate the:
Find the length of CB correct to one decimal place. (i) deposit Mukasa must make.
(ii) Amount of money Mukasa will have to pay the bank

Answer 20 2cm  after two years.
(iii) Total money which Mukasa will spend to buy the
10. In the diagram below O is the centre of the circle and house.
angle BOD = 1640 Answer (83750, 5545000, 69545000)
13. A club held swimming tests in Crawl (C),
A Backstroke (B) and Driving (D) for 72 members. Those
who passed Crawl were 49, 30 passed Backstroke and 30
O passed Driving. 5 passed Crawl and Backstroke but not
Diving. 4 passed Backstroke and Diving but not Crawl. 6
1640 passed Crawl and Diving but not Backstroke. 14 passed
B D all the three tests.
(a) draw a Venn diagram to represent the given
information.
(b) use the venn diagram to find the members who:
C (i) Passed the Crawl test only.
(ii) Did not pass any test.
Find (c) If a member is picked at random, what is the
(a) angle BAD probability that the member passed two tests only?
(b) angle BCD Answer (820, 980) Answer (24, 6, 15/72)
14. Given that the points A has co – ordinates (-8,6).
12
Vectors AB =   and M is the mid – point of AB.
4 
(a) Find the
(i) column vectors AM.
(ii) co – ordinates of M

447
(iii) magnitude of OM.
6
2008 PAPER TWO
Answer   ,  2, 8 , 68units SECTION A:
 2 1  1 1
15. (a) a unit square whose vertices are O(0,0), i(1,0), j(0,1) 1  8  2   11 
and k(1,1) is transformed by rotating through a positive Simplify: 2  3 2  Answer  
quarter trun about the origin. Find the matrix for this 1  1 2  21 
1 of 1  1 
transformation. 5  4 3
0 1 2. Factorise completely:
  and
0 
(b) given T = 2p2q3 – pq3 + pq – 2p2q
1 Answer: pq(q + 1) (q – 1) (2p-1)
 1

0 
 , find the: 
3  10 13  6  105 
2

1
M= 3. Simplify . Give your answer in
0 80
(i) image of the points A(0,3) and B(5,3) under the standard form. Answer (1.35 x 10-3)
transformation TM. (04 marks) 1 
(ii) matrix of transformation which will map the images of 4. Given the vectors QR = , ST =   and SR =
A and B back to their original positions.  6
0 1  0 1   4 7
Answer   (3, 0) (3, -5),     , find the vector QT. Answer  
1 0 1 0   3 7
16.(a) Copy and complete the table below for the
4x  2y
5. If x  2 y  2 , express p in terms of x and y.
p
equation y = 2x2 – 3x – 7
x -
1½ -1 -½ 0 ½ 2
2x2 2 0 Answer (x – y = p)
-3x 3 0 6. Given that D = {All odd numbers less than 20} and M=
-7 -7 -7 {All multiples of three less than 20}, find n(D∩M).
y -2 -7 Answer (3)
3
7. Find the equation of the line of gradient and passing
1 1½ 2 2½ 3 5
through the point (3, 4). Ans. (5y + 3x = 29)
8. A farm is on a piece of land whose area is 5.6km 2. What
would be the area of this farm in cm2 on a map whose scale
is 1: 40,000? Answer (35cm2)
9. A forex Bureau buys one US dollar at Ug.shs 1,900 and
(b) Plot the points (x, y) obtained from the completed table sells one pound sterling at Ug.shs3, 450. Atim wants to
on a graph paper using 2 cm to represent 1 unit on the x exchange 3,000 US dollars to pound sterlings. How many
– axis and 1 cm to represent 1 unit on the y – axis. pound sterlings will she get?
Hence draw a graph for y = 2x2 – 3x – 7. Answer (1652.174£)
(c) Use your graph to solve the equation 10. Two points A (5, 1) and B(6, 0) are given a
2x2 – 3x – 8 = 0
3 1
Answer (x = -1.4, 2.9) transformation defined by the matrix   . Find
17. (a) The dimensions of a rectangle are 60 cm by 45 cm. 1 0
if the length and width are each reduced by 10%, the Co-ordinates of their images.
calculate the percentage decrease in area. Answers AI(16, 5) BI(18, 6)
(b) A container has a volume of 6400 cm3 and a surface
area of 8000 cm2. find the surface area of a similar
container which has a volume of 2700 cm3. SECTION B
Answer (19%, 4500cm2) 11.(a) A man gave half of his welfare allowance to his wife,
1 to each of his two sons and the rest to his daughter.
5
Find
(i) the fraction given to the daughter.
(ii) his welfare allowance if each son was given shs:
16,000.

448
(b) The difference between the values of y when x = 10 is Find the
16. Given that y is inversely proportional to the square of x, (a) length of:
find the equation relating x and y. (i) the arc XRY. [take π = 3.14]
(ii) MQ
1 (iii) MX (09 marks)
Answer   , 80,000
 10  (b) Perimeter of the given figure
12. The table below shows the weights in kilogrammes of
thirty pupils. 16. A school constructed an office block which required 34
48 44 32 52 54 44 tonnes of sand. The school hired a lorry and a tipper truck
53 38 37 35 53 46 with capacities of 7 tonnes and 5 tonnes respectively to
59 51 32 37 49 42 transport the sand. The cost per trip either by lorry or by
48 59 52 40 54 46 tipper truck was shs: 30,000. The money available for
45 62 35 54 48 35 transportation was shs: 180,000. The trips made by the
(a) Construct a frequency table with a class width of 5 lorry did not exceed those made by the tipper truck.
starting from the class of 30 – 34. (b) Use table in (a) to:
(i) Estimate the mean weight of the pupils. (ii) Draw a (a) If x and y represent the number of trips made by the
histogram and use it to estimate the modal weight of the lorry and the tipper truck respectively.
pupils. (i) Write down five inequalities to represent the given
Answer information. (04 marks)
13. Four students; Kale, Linda, Musa and Naana went to a (ii) Plot these inequalities on the same axes, shading the
stationary shop. unwanted regions. (05 marks)
Kale bought 4pens, 6 counter books and 1 graph book. (b)(i) From your graph in (a)(ii)above, list all the possible
Linda bought 10pens and 5 counter books. numbers of trips, that each vehicle can make so as to
Musa bought 3 pens and 3 graph books. maximise the total tonnage of sand transported.
Naana bought 5 pens, 2 counter books and 8 graph (ii) Find the number of trips by each vehicle that made the
books. greatest total tonnage. (03 marks)
The costs of a pen, a counter book and a graph were shs: 17. The figure below shows a cuboid ABCDEFGH in
400, shs: 1,200 and shs: 1,000 respectively. which BC = 8cm, BF= 6cm and CD = 5 cm. K is the
(a)(i) Write a 4  3 matrix for the items bought by the four mid-point of AB.
students.
(ii) Write a 3  1 matrix for the costs of each item. E H
(b) Use the matrices in (a) to calculate the amount of money F G
spent by the each student.
(c) If each student was to buy 4 pens, 10 counter books and
6 graph books, how much money would be spent by all Find the:
the four students? (a) (i) length AG. D
K
(ii) Angle which AG makes with the planes ABCD.
14.(a) The lines ax + 2y = 3 and (08 marks)
ax – by = 5 intersect at (1, 2). Find a and b. (b)Bangle between planes KGHC and FGHE.
Answer (11.81cm, 32.50, 36.90)
4 1  x   4 
(b) If       , determine the values of
x 1 y   8 
x and y. (07 marks)
15. In the figure below, QXRYP is a semi-circle with centre
O and radius 10cm. MN is parallel to the diameter QP.
Angle XOQ = 400.
R
M X Y N

400
Q O P
10cm

449
2009 PAPER ONE SECTION B
SECTION A 11 A group of 55 students were asked if they liked the food;
1. Use factors to evaluate Matooke (M), Posho (P) or Rice (R). 19 liked Matooke,
617 × 793 + 786 × 793 + 597 × 793 24 liked Posho and 25 liked Rice. 3 liked Matooke and
Answer (158600) Rice only. None of the students liked Matooke and
2. Determine the solution of the inequality Posho only. 4 students disliked all the foods.
x2 − x − 6 < 0 a) Represent the given information on a Venn diagram.
Answer (-2 < x < 3) b) Find the umber of students who liked:
 p (i) all the three types of food
3. Given that log p = 2.476 and log q = 1.811 , find log  2 
 
q (ii) Matooke only
(iii) Posho only
Answer (1.854)
(iv) Rice only
4 p 1 3 p 1 5  2 p
4. Solve for p in the equation   c) Find the probability that a student selected at random
3 2 4 from the group liked only one of the foods
Answer (3.25) 8
x 1 Answers (6, 10, 16, 14, )
5. Find g(3) given that g  x  
1
11
x Answer (0.5)
12. Triangle ABC with vertices A(2, 1), B(4, 4) and C(2, 4)
2 16
(y ) 1 is reflected in the line is reflected in the line is
6. Evaluate when x = 16 and y = 8 Answer  
6
1
(9 x) 2 reflected in the line y = 0 to get triangle A'B'C'.
 2 3 Triangle A'B'C' is then given a negative quarter turn
7. Given the matrix A    , find a matrix B such that about the origin to get triangle A"B"C"
5 7
a) i) Draw the three triangle on the same graph paper
1 0  1  3  ii) Write down the coordinates of A'B'C' and A"B"C"
A+B=   Answer  
 0 1   5  6  b) Use your graph to describe fully the transformation which
8. The table below shows the number of goals scored by a maps A"B"C" back onto ABC
team in a series of football matches. Answers AI(2, -1), BI(4, -4) and CI(2, -4);
Number of goals 1 2 3 4 5 AII(-1, -2), BII(-4, -4) and CII(-4, -2)
Number of matches 3 4 1 x 2
If the mean number of goals is 3, find x  0  1
 
Answer (6)  1 0 
9. Find the length of BC in the diagram below 13. a) Given the points P(2, 4) and Q(−4, 8), find the
A
(i) Coordinates of the midpoint of the line segment PQ
100 550
12cm 3
(ii) Equation of the line with gradient passing through
2
B C D the midpoint of PQ
Answer (8.6)
10. In the figure below, O is the centre of the circle and b) Find the coordinates of the point of intersection of the
angle AOC = 140° line y − 5x = 2 and the curve y = 2x2 + 5
B Answers(-1, 6) (2y = 3x + 15) (1, 7) and (1.5, 9.5)
14. A motor company had an advertisement as shown
O below:

A 1400 C

D
Find: (i) angle ABC
(ii) angle ADC
Answer (70, 1100)

450
b) its length and breadth
EASY TERMS ON SALOON CARS c) its perimeter
Answer (1.25) (4cm, 2.5cm, 13cm)
MADE IN JAPAN

CASH: SHS. 8.5 MILLION

CASH DISCOUNT: 8% OF THE CASH VALUE

HIRE PURCHASE: DEPOSIT 60% OF THE


VALUE AND PAY 7 MILLION PER
MONTH
a) Calculate FOR 3 MONTHS
the saving Chris would make if he bought
the vehicle by paying cash rather than by hire purchase.
b) Chris bought the vehicle by hire purchase and then
sold it at 35 million after one year. Find the percentage
loss he made Answer (5.48, 30.14)

15. a) Copy and complete the table below


x -2.0 -1.5 -1.0 -0.5 0 0.5 1 1.5 2 2.5 3
2x2 -8 -2 -0.5 0 -2 -4.5 -8 -18
3x -6 -3 -1.5 0 3 4.5 6 9
6 6 6 6 6 6 6 6 6 6 6 6
y -8 1 4 6 7 6 4 -3
b)i) Use the completed table in (a) above to draw a graph of y
= 6 + 3x − 2x2 for values of x for −1 ≤ x ≤ 3.
Use 2 cm to represent one unit on the x-axis and 1 cm
to represent one unit on the y-axes
ii) On the same axes, draw a line whose equation is y = 2x
c) Use the graph in (b) above to solve the equation 6 + x
− 2x2 = 0
Answer (-1.5, x = 2)

8 4
16. Given the vectors a = , b =   and c =   , find:
 11  5 
(i) a + 2 b + c.
(ii) the length of a + 2 b + c.
6  12 
(b) the position vectors of D and E are   and  
 4  11
respectively. M is on DE such that DM : DE = 2:3.
Find:
(i) DE
(ii) DM,
(iii) The position vectors of M
18  6  4  10 
Answers   , 30 units,  ,  ,  
 24   15   10   6 

17. A rectangle of length (4x − 1) cm and breadth 2x cm has


an area of 10cm2.
Find:
a) the value of x

451
long will it take the two men to plough the garden
2009 PAPER TWO if they worked together?
SECTION A Answer (24, 6), (12 days)

3 16  1 32 12.The diagram below shows a circle with centre O and


1. Simplify Answer (7.25)
3  12
2 5 radius 10.5 cm. Two tangents PR and PS are drawn
from a point outside the circle .Angle POR = 600 and
2. Find the equation of a line which passes through the
PO = intersects the circle at Q
point (0,5) and is parallel to the line 3x – y = 7
Answer (y = 3x + 5) R
3. Given A  2 C
B 10.5cm
(i) express B in terms of A,  , and C cm 600 Q P
(ii) find the value of B if C= 240,  = 3.14 and A = 12.56 O 
 4c 2 
Answer  2
, 60 
 A  S
4. If set F = {All factors of 12} and set T = {All triangle
numbers less than 20}, F = {1, 2, 3, 4, 6, 12}} Calculate the:
Find the members of F  T. Hence find n(F  T). (a) lengths of the tangents (04 marks)
Answer {1, 3, 6}, 3 (b) area bounded by the tangents and the arc SQR.
 22 
5. Factorise completely 2ab − 3 + 2a − 3b  Use    (08 marks)
Answer (b + 1) (2a – 3)  7 
6. A translation T maps A (−2, 3) onto A1 (−6, 10). Find the Answer (18.17cm), (18.17cm), (66.2cm)
image B1 of B(5,4 ) under the translation T 13. Agroup of students obtained the following marks in a
Answer (1, 11) Maths test.
1 28 35 94 78 70 56 57
7. Express with a rational denominator 58 60 76 77 62 84 66
5 2
67 68 69 70 51 64 73
 5 2 74 75 61 62 54 80 83
Answer  
 3 88 90 41 47 64 70 75
 
8. The price of an article is shs 24,000. If a discount of 12% (a) (i) Form a grouped frequency table for the data
is given, calculate the selling price of the article starting from the Class 20−29.
(ii) Represent the marks obtained in the Maths test on a
Answer (shs. 21,120)
bar chart Answer (66.8)
 2  8 14. (a) Solve the following simultaneous equations
9. Given that OA =   , OB =   and M is a point on
2  4 x − 2y = 12
AB such that AM : MB  1:1 find; x = 12 + 2y
(b) In a certain supermarket, a school bag costs b
10   5  shillings and a pair of shoes costs S shillings. Kato
(i) AB (ii) AM Answer  ,  
 2  1  bought 3 school bags and 2 pairs of shoes at shs
10. If an area of 4cm2 on a map represents an area of 103,00 and Atim bought 5 school bags and 1 pair of
576km2 on land, find the Representative Fraction shoes at shs 132,000.
(R.F) of the map? Answer (1 : 1200000) Find the cost of:
(i) a school bag
SECTION B (ii) a pair of shoes (07 marks)
11. (a) The ages of Lacor and Nankya are in the ratio 4 : Answer (x = 2, y = -5)(23,000) (17,000)
1.After 6 years, the ratio of their ages will be5 : 2. 15. A bicycle factory assmbles two types of bicycles;
Roadmaster and Hero on different assembly lines. An
Find their present ages. (06 marks)
assembly line for Roadmaster occupies an area of
(b) Ouma takes 20 days to plough a garden. Mukasa 60m2 and that of Hero occupies an area of 30m2 of the
takes 30 days to plough the same garden. How floor space. The floor space available for all the

452
assembly lines is 420m2. The assembly line for
Roadmaster needs 10 men to operate it and that of 2010 PAPER ONE
Hero needs 16men to operate it. The assembly lines SECTION A
need a maximum of 120 men to operate them 1. If 125n = 82ten, find n.
(a) if x and y represents the number of assembly lines for Answer (8)
Roadmaster and Hero respectively 2. In a group of 29 girls, 22 liked Rice (R) and 18 liked
(i) form four inequalites to represent the given Matooke (M). All the students liked at least one of the
information foods. How many liked both?
(ii) draw graphs on the same axes to represent the Answer (11)
inequalities in (i) a bove. Shade the unwanted regions 3. Solve the inequality: 10 – 3x < 4(x – 1).
(04 marks) Answer (x > 2)
(b) The assembly line for Roadmaster produces 30 bicycles 4. Without using mathematical tables or a calculator,
per day and that of Hero produces 20 bicycles per day. 21.35  41.35  21.352
Find the: evaluate:
0.02
(i) number of assemly lines for each type of bicycle
Answer (21350)
that should be operated so as to produce the highest
5. Two quantities y and x are related by the equation y = a +
total number of bicycles per day
bx. When y = 4, x = 2 and when y = 6,
(ii) highest total number of bicycles that can be
x = 4. Find the values of a and b.
produced per day.
Answer (a=2, b = 1)
Answer (5x + 8y < 60, 2x + y < 14, x > 0, y > 0)
3
( 5 roadmasters and 4 heros), (230) 6. Given that sin   and α is obtuse, without using
16. (a) Find the values of x and y given that: 5
mathematical tables or calculator, find the values of cosα
 4 3 and tanα.
1 3 2   x 2    39 25  4 3 
10 y  Answer 
4 
,
   5
(b) Given that matrix P = 
2 2  , find a matrix 7. A shopkeeper bought an item at Shs5,500 and sold it at
  30% more than the buying price. Find the shopkeeper’s:
 3 4 
(a) selling price,
Q such that PQ =  2 0  .Hence find the inverse (b) profit.
 
 0 2 Answer (7150, 1650)
of matrix P.  5 6  , find P2.
8. Given the matrix P   
4 2  2 1  2 2 
Answer (x = 5, y = 8),  ,  
 3 2  1.5 1  13  18 
17. The figure below shows a prism ABCD with an Answer  
isosceles right- angled triangle as the cross- section  6  8
and a horizontal rectangular base ABCD. 0.0875  0.0243
9. Use tables or logarithms to evaluate:
0.003142
E
Answer (0.6767)
10. Solve the following pairs of simultaneous equations
F
D 5x – 9y = 1
4y – 2 = x
C
Answer (x = 2, y = 1)
10cm
A SECTION B
8cmm B
11. The following table shows marks obtained by 40 pupils
mm in a Mathematics test.
Calculate the
(a) Lengths of 11 17 35 34 42 45 28 46
(i) AF 16 21 14 36 41 31 49 37
(ii) BE
(b) angle between BE and the base ABCD 20 33 37 38 18 38 39 27
(c) Volume of the Prism 26 28 40 33 43 32 29 47
Answer  0
32, 29.5 , 160cm 3
 29 32 41 24 44 35 36 23
(a) Draw a frequency distribution table for marks
starting with a class of 10 – 14.

453
(b) State the handkerchiefs sold, and 12% for any handkerchief sold in
i. class interval, excess of 20.
ii. modal class. (a) Express the number of handkerchiefs sold in the 3 rd
(c) Calculate the:
week as a percentage of the number sold in the first
i. mean mark,
ii. median mark. week.
Answer (5, 35-39), (32.25), (33.667) (b) Calculate the commission he received in the third
12.(a)(i) Determine the range corresponding to the domain week.
{-3, -2, 0, 1, 3, 4} for the mapping x ⟶x2 + 1 (c) If in the fourth week the hawker received a
(ii) Represent the mapping in (i) on an arrow diagram. commission of 2,000/-, calculate the number of
(b) Given the functions h(x) = x + 2, g(x) = x2 and f(x) handkerchiefs he sold in that week.
= -x; find the values of x for which g[h(x)] = f(x) 17. In the diagram below, OA = a, OB = b, OB  BC = 1 :
Answer -1, -4 3, 3OF = 2OA and E divided AC in the ratio 3 : 2.
13. A triangle with vertices A(2, 4), B(6, 4) and C(1, 6), A
undergoes two successive transformations P1 and P2. The F
 0 1
transformation P1 is represented by the matrix   E
1 0  a

and P2 by matrix 
0.5 0  O C
. b B
 0 0.5 
Express the following vectors in terms of a and b:
(a) Find the coordinates of the vertices of:
(a) BC.
(i) triangle A'B'C' the image of ABC under P1,
(b) CA.
(ii) triangle A''B''C'' the image of A'B'C' under P 2
(c) BE.
(b) Show on the same axes the three triangles ABC, A'B'C'
(d) FE.
and A''B''C''
(c) Use your graph in (b) above to describe fully the
transformations represented by
(i) P1 (ii) P2
Answer : P1 is matrix of positive quarter turn about the
origin.
P2 is a matrix of enlargement with linear scale
factor ½ about origin
14. (a) A student had his photograph of dimensions 30cm by
20cm framed with uniform border. If the area of the
border is 216cm2, how wide is the border?
(b) A cone has a radius of 7cm and vertical height of
30cm. Find:
(i) it’s volume,
(ii) the volume of another similar bigger cone which has
22
linear scale factor 2. [use π = ]
7
15. (a) Find the equation of the line passing through a point (2,
0) and perpendicular to the line joining the points ( –10, 3)
and (6, –9)
(b). A triangle PQR has vertices with coordinates P(3, –
1), Q(7, 6) and R(0, 2). Find the equation of its line of
symmetry.
16. A hawker sells handkerchiefs at Shs500 each. He sold 50
handkerchiefs in the first week. In the second week, he
sold 20% more than in the first week. In the third week
he sold 10% more than in the second week. Each week he
receives a commission of 8% on the price of the first 20

454
2010 PAPER TWO (c) Calculate the
(i) total time taken for the speed-boat to move from M
SECTION A to P.
1. Express 0.341666... in the form p/q, where q ≠ 0. (ii) speed-boat’s average speed for the whole journey.
3 1
2. Solve for x in 32 5  x 2  2 . 12. The Venn diagram below shows the members of a
3. Given two points P(4, 5) and Q(-2, 9), find the equation District Council who sit on three different committees of
of the line through P and Q. works (W), Production (P) and Finance (F).
4. Simplify 20  45  125 . Give your answer in the n(W) = 12 n(P) = 13
form a b where a and b are constants. z
3 3
5. A rectangle 6cm long and 5cm wide is enlarged so that
its area becomes 270cm2. Find the linear scale factor of 2
the enlargement. (04 marks) y x
6. In the figure below, O is the centre of the circle, angle 4
1
JKQ = 40° and KOQ is a straight line
J n(F) = 12
(a) Determine the values of x, y and z.
Q (b) Find the total number of members who
i. make up the District Council
p ii. belong to more than one committee
n (c) Given that a member is selected at random from
40° O
the District Council, find the probability that the
K member belongs to only two committees.

2 4 4( x  4)
Find the angles marked n and p. (04 marks) 13. (a) Express   in the form
 2  7 ( x  4) ( x  3) ( x2  x  12)
7. Given that a =   , b =   and m = a + 2b, find the

  9  2 a
(05 marks)
magnitude of m. (04 marks) ( x  b)
(b) A mini-bus travels from Migyera to Kampala, a
 2 
8. If n  x  2  , express m in terms of n and x. distance of 156km, at a certain average speed of
 4m  1  Vkm/hr. On the return journey, it increases the average
3 speed by 4km/hr and takes 15 minutes less. Find the
9. A function f ( x)  . Find the values of x for which average speed V from Migyera to Kampala.
1  x2
14. The table below shows the time (t) in seconds and
f(x) = 4. (04 marks)
velocity (V) in m/s of an object.
10. Three girls; Auma, Asiimwe and Nakato shared Shs t(s) 0 1 2 3 4 5 6
10,500. Nakato got twice as much as Asiimwe and V(m/s) 0.0 1.0 1.7 2.0 1.7 1.0 0.0
Asiimwe got twice as much as Auma. Find how much (a) Using a scale of 2 cm to represent one second on the
money Asiimwe got. horizontal axis and 4 cm to represent 0.5 m/s on the
vertical axis, plot the values of t and V and join the
SECTION B points with a smooth curve.
(b) Use your graph in (a), to find the
11. A speed-boat sets off from an island M on a bearing of i. times at which the speed of the object is 0.8m/s,
080° to an island X at an average speed of 150 kmh–1. ii. acceleration of the object when the time is 2
Island X is 450 km from island M. At X, it alters its seconds.
course to a bearing of 200° and maintains the average (c) If the total distance covered by the object was 7.5m,
speed of 150 kmh–1 for 3 hours until it reaches island Y. what was its average speed?
It then moves to island P which is west of island M at an 15. (a) Without using mathematical tables or calculator,
average speed of 200 kmh–1. Island P is 400 km from find the value of
island M. 2log1050 + log1080 – log102.
(a) Using a scale of 1 cm to represent 50km, construct a (b) (i)Find the prime factors of 150.
scale drawing to show the route of the speed-boat. (06 (ii) Use your result in (i), find log10150, given that
marks) log105 = 0.6990, log103 = 0.4771 and log105 =
(b) Use the scale drawing in (a) above to find the distance 0.3010.
PY.

455
16. (a) Solve the following simultaneous equations using
the matrix method 2011 PAPER ONE
5x + 2y = 5, SECTION A
3x – 0.2y = 10. 3x
2 1 1. Make p the subject of the formula S  1
(b) Given that P   1 5 
 , Q   2 3 and
2y  p .
 3 2   
Answer  p  2 y 
3x 
R= 
 4 3  find:
  S  1 
 1 2 
n 1 n  3 n  2
(i) QR – P 2. Solve for n in the inequality   .
2 4 3
(ii) the determinant of QR – P
17. Mr. Oketcho’s monthly salary is shs900,000 which Answer (n > 7)
includes the following allowances: 3. In the figure below, O is the centre of the circle. Angle
Water and electricity 20,000 ABO = 43° and angle OAC = 62°.
A
Relief and insurance 30,000
62°
Housing allowance 50,000 C
Medical allowance 25,000 B 43°
Transport allowance 28,000 O
Marriage allowance 20,000 x
Family allowance
(for only 4 children):
– From 0 to 9 years 20,000 per child
Find the value of x.
– Between 9 and 16 years 15,000 per child Answer (x = 210°)
– Over 16 years 10,000 per child 4. Point A(0, 3) is reflected in the line y + x = 0. Find the
Mr. Oketcho has five children; two of whom are aged coordinates of the image A'
between 0 and 9 years, one aged 14 years and the other two
Answer A'(-3, 0)
are over 16 years. The income tax structure is shown in the
table below: 5. A box contains 5 black balls and 3 red balls. Two balls
Taxable income per are randomly picked one after the other without
Tax rate % replacement. Find the probability that both balls are red.
month in shillings.
01 – 50,000 10.0  3 
Answer  
50,001 – 110,000 20.0  28 
110,001 – 200,000 24.5 6. Given that a*b = ab2, find the value of
200,001 – 350,000 35.0 (i) 2 * 5
(ii) a if a*3 = 63.
350,001 – 600,000 40.0
Answer (i) = 50 (ii) 7
Above 60,000 49.0
7. Solve the equation 2x2 + x – 1 = 0.
(a) Calculate Mr. Oketcho’s
1
i. taxable income Answer : x = -1 and x 
ii. income tax. 2
8. Find the inverse of matrix B = 
(b) Express the income tax as a percentage of his 5 4
.
monthly gross salary. 3 2
 1 2 
Answer   
1.5 2.5 
9. In the figure below, AD is perpendicular to BC, AD =
DB, AC = 4 cm and angle CAD = 30ctorisation method.

456
C (ii) Measure the radius of the circle
Answer (11.8, SQ=12.8)

D 13. The histogram below shows the marks scored by 40


4cm O students in a test.
Answer
(a) Marks No of students
30°
10 – 14 1
A B 15 – 19 4
Find the length of AB. 20 – 24 5
Answer 2 6  25 – 29
30 – 34
8
12
10. Use the grouped frequency distribution table below to 35 – 39 9
answer the questions that follow.
40 – 44 2
Class Frequency Cumulative
45 – 49 1
frequency
(b) 29.5
30 – 39 19 –
14. (a)Factorise completely the following expressions
40 – 49 21 – (i) a2 + b2 – 4 + 2ab,
50 – 59 19 – (ii) a2 – 5a – 36 + am + 4m
60 – 69 12 – (b) Given that x + y = 10 and xy = 5, find the values of:
70 – 79 08 –
1 1 1
80 – 89 01 – (i)  (ii) y
(a) Complete the cumulative frequency column x y x
(b) Determine the median class. Answer
Answer (a)(i) (a + b + 2)(a + b – 2)
Class Frequency Cumulative (ii) (a + 4)(a – 9 + m)
frequency (b)(i) = 2 (ii) = 5
30 – 39 19 19 0 4 
40 – 49 21 40 15.(a) The matrix   is pre-multiplied by the
 3 1 
50 – 59 19 59
60 – 69 12 71  x  8 
70 – 79 08 79 column matrix   to give  x  . Find the
80 – 89 01 80  y  
values of x and y.
Median class = 40 – 49.  2 3
(b) Given that matrix P =   and
1 2
SECTION B  2 3 
11. (a) Given that a – b2 = 16 and a + b = 8, determine the
2 Q  , find
values of a and b..  1 2 
(b) Two taxis, a Nissan and a Toyota transported (i) PQ
students from Jinja to Kampala. When the Nissan had (ii) a 2 × 2 matrix R such that QR = P
made 3 journeys, the Toyota had made 4, and they had Answer: x = -1 and y = -2
transported 116 students altogether. When the .Nissan 1 0  7 12 
had made 2 journeys and the Toyota 4, they had (b)(i)    (ii) R =  
0 1 4 7 
transported 110 students. If each journey made was at
full capacity, find the capacity of each taxi. 16. A rectangle ABCD has vertices A(1, 0), B(3, 0), C(3,
Answer (a) a = 5 and b = 3 1) and D(1, 1). Rectangle ABCD is mapped onto
(b).capacity of the Nissan is 20 students and that of the rectangle A'B'C'D' by the transformation matrix
Toyota is 14 students. 2 0
12. Using a pencil, a ruler and a pair of compasses only,  .
0 2
(a) (i) Construct a triangle PQR in which PQ = 6.5cm, (a) Find the coordinates of A'B'C'D'.
angle PQR = 30° and angle QPR = 120°. S is a (b) Rectangle A'B'C'D' is mapped onto A''B''C''D'' with
point on the opposite side of RQ as P such that vertices at A''(2, 0), B''(6, 0), C''(10, 2) and D''(6, 2).
angle RQS = 90° and RS = 12cm. Find the matrix of transformation.
(ii) Measure the length of RQ and SQ. (c) Find a single transformation matrix which maps
(b) (i) Draw a circle that passes through the points P, Q rectangle ABCD onto A''B''C''D''.
and S.

457
Answer
A'(2, 0), B'(6, 0), C'(6, 2) and D'(2, 2) 2011 PAPER TWO
1 2 SECTION A
(b) Matrix of transformation =   1. Convert 5.272727… to a fraction.
0 1
(c) The single transformation matrix that maps ABCD Answer:  5 3 
 11 
to A''B''C''D'' is equal to the product of the two
2. Given that F = {all factors of 24} and G = {all factors of
2 4  30}, find n(F'∩G) where F' is the complement of F.
matrices. Hence we have:  
 0 2 Answer ( 4)
17. A company wishes to transport at least 480 3. Find the equation of a line whose gradient is  1 and
mattresses from its stores to one of its sales point. It has 2
two types of trucks, A and B. Truck A can carry 40 passes through the point (-4, 5).
mattresses at a cost of shs30,000 per trip. Truck B can Answer y   12 x  3
carry 60 mattresses at a cost of Shs45,000 per trip. 4. A sales woman earns a basic salary of shs120,000 and a
There is Shs600,000 available for transport. The number
commission of 8% of the month’s total sales. If the
of trips made by A should not exceed 12. Those made by
month’s total sales were shs1,350,000, find her income
B should not exceed twice the number of trips made by
for that month.
A.
Answer ( Shs 228,000)
(a) If x and y are the trips made by A and B respectively,
5. Simplify: log 15 – 2 log 10 + log 60.
write down four inequalities satisfying the given Answer log 9
conditions
6. If m is directly proportional to the square of n and n = 2
(b)(i) On the same axes, draw the graphs of the
when m=1, find the value of m when n = -5.
inequalities and shade the unwanted regions
Answer (m = 6.25)
(ii) Find all the possible number of trips made by each
7. Find the point of intersection of the lines 3x + 2y = 6
truck so that the transport cost is minimised. and x + y = 4.
Answer Answer (x, y) = (-2, 6)
40x + 60y ≥ 480 ...................... (i)
8. The diagram below shows the net of a right triangular
30,000x + 45,000y ≤ 600,000 ... (ii)
pyramid.
x ≤ 12 .................................... (iii)
y ≤ 2x ..................................... (iii)
a
6cm
(b)(i) Possible number of trips made by each truck is (x,
y) = (12, 0), (9, 2), (6, 4) and (3, 6)
b 8cm
8cm

c
Find the lengths marked a, b, and c.
Answer( a = 10 cm, b = 6cm, c = 10)
9. Given that S(-2, 6) and T(3, 3) are two points, find the
1
coordinates of R if OR = 4OS + 3 OT and O is the
origin.
Answer (R(-7, 25))
10. A cylinder has a radius of 3 cm and a height of 5 cm.
Find the area of the curved surface.
Answer ( 94.286 cm2 (3 d.p)

SECTION B
11. (a) Hellen won shs42 million in a lotary. She shared
the money with her parents in the ratio 5:2
respectively. Find how much money she gave her
parents.

458
(b) Without using mathematical table or a calculator, Answers
1
(b)(i) It broke down when he was 15km away from
simplify 64 1
3

Kampala
27 3
(ii) 6km/hr
(c) Given that 15 = 3.873, 6 = 2.450 and 10 = (iii) Ojok’s average speed ~ 16.4 km/hr
3.162, without using a calculator, evaluate to two (iv The two men met at around 10:07 am
significant figures 3  2 . (v) Both men met at a distance of approximately
5 2 14.25km from Kampala.
Answer (a) 12 million; (b)  3 (c) 7 (2 s.f)
4 15. In the figure below, OA = a and OB = b. 3OB = 2BD. P
12. A group of students was asked what game they play. is a point on AD such that. OC = 3CE = 3AP.
It was found out that 20 play Rugby (R), 30 play A P D
Soccer (S) and 15 play Basket ball (B). 6 play both
Rugby and Soccer, 4 play both Soccer and Basket
ball and 5 play both Rugby and Basket ball. The
number of students who play Soccer only is equal to a
twice the number of students who play Rugby only. b
All the students play at least one of the games. O
C E
(a) Represent the above information on a Venn
diagram. (a) Express the following vectors in terms of
(b) Find the number of students: vectors a and b:
(i) who play all the three games. (i) AD.
(ii) in the group. (ii) AP.
(c) If a student is chosen at random from the group, find (b) Show that AD : OE  3: 8
the probability that the student plays at least two 1 1
games. Answers 2 (ii) = 52 Answers (AD = 5b  2a  AP  5b  2a 
2 3
11 16. Paul and Mary invested shs600,000 each in a savings
(c) P(A student plays at least two games)  society for 2 years. Paul opted for simple interest while
52
13. (a) Given that f(x) = 4x – 3, find Mary opted for compound interest. Both interests were
(i) f(2), at 12% per annum.
(a) Find the interest earned by each of them
(ii) f-1(x),
(b) Who earned more interest and by how much?
(iii) f-1(-1)
Answer
(b) Given that g(x) = x2 + 1 and h(x) = x – 3,
(a) For Paul; = Shs 144,000
find the value of x for which gh(x) = hg(x). For Mary; = Shs. 152,640
x3 1 (b) Mary earned more interest by Shs 8,640
Answer: (a)(i) 5; (ii) , , (b) x = 2
4 2
14. Lugazi is 45 km from Kampala. Kintu set off at 0815 17. The figure below shows a right pyramid on a
hours from Kampala riding a bicycle at 15 km/hr. Kintu’s rectangular base ABCD.
bicycle broke down at 0915 hours and he was delayed for V Calculate the:
45 minutes. He then walked back to Kampala and reached (a) Volume of the
pyramid
at 1230 hours. Ojok set off at 0915 hours from Kampala,
riding a bicycle and reached Lugazi at 1200 hours. (b) Angle between AV
(a) On the same axes, draw the graphs showing the and the base ABCD.
24cm

journeys of Kintu and Ojok. (c) Angle between the


(b) Use your graph in (a) to find: D planes ADV and
C
(i) How far from Kampala Kintu was when his BCV.
bicycle broke down. O 12cm
(ii) The speed at which Kintu walked back to
Kampala. A 16cm B
(iii) Ojok’s average speed. Answer
(iv) The time when the two men met. (a) 1536 cm3. (b) θ = 67.38° (2 d.p.)
(v) The distance from Kampala where the two men (c) α = 36.87 (2 d.p)
met.

459

You might also like